AAP PREP 2010

630
2010 PREP SA on CD-ROM Question: 1 You are counseling the mother of a 3-month-old breastfed infant whose family has been urging her to introduce cereals to her baby’s diet. She asks your advice. Of the following, the MOST likely outcome of introducing solid foods at this age is to A. accelerate the development of oral-motor skills B. help the infant sleep through the night C. increase the risk of food allergies D. increase the risk of gastroesophageal reflux E. increase the risk of gastrointestinal infections Copyright © 2010 by the American Academy of Pediatrics page 1

Transcript of AAP PREP 2010

Page 1: AAP PREP 2010

2010 PREP SA on CD-ROM

Question: 1

You are counseling the mother of a 3-month-old breastfed infant whose family has been urgingher to introduce cereals to her baby’s diet. She asks your advice.

Of the following, the MOST likely outcome of introducing solid foods at this age is to

A. accelerate the development of oral-motor skills

B. help the infant sleep through the night

C. increase the risk of food allergies

D. increase the risk of gastroesophageal reflux

E. increase the risk of gastrointestinal infections

Copyright © 2010 by the American Academy of Pediatrics page 1

Page 2: AAP PREP 2010

2010 PREP SA on CD-ROM

Preferred Response: ECritique: 1

The most likely consequence of early (before 6 months of age) feeding of complementaryfoods such as cereals to breastfed infants is an increased likelihood of gastrointestinal infection.The direct relationship between early complementary feedings and the incidence of diarrhealillness is based on several case-control studies. In one investigation from Belarus, a large groupof infants who were exclusively breastfed for more than 6 months was compared with a groupreceiving a mixed diet of human milk plus solids, with solids introduced between 3 and 6 monthsof age. Exclusively breastfed infants had a significantly reduced risk of one or moregastrointestinal illnesses. Furthermore, other observations suggest that this effect may beenhanced with greater duration and exclusivity of breastfeeding. However, prior studies havefailed to show any clear risk reduction in the prevalence of upper and lower respiratory tractillnesses, asthma, and otitis media among exclusively breastfed infants compared with infantswho received a mixed diet of human milk and solids.

No available evidence supports the hypothesis that the introduction of solid foods eitheraccelerates the development of oral-motor skills or helps infants to sleep through the night.

Data concerning the effect of early introduction of solids on the development of allergies areconflicting. The Belarus study found no reduction in risk for atopic eczema in exclusivelybreastfed infants; a Finnish investigation showed a reduced eczema risk at 1 year but not at 5years of age in a similar group. Although the Finnish study demonstrated a small reduction in anyatopic condition for exclusively breastfed infants, the results were not statistically significant.Evidence also failed to demonstrate that early solid food introduction was associated with anincreased incidence of positive skin prick tests.

Results of obesity studies also are inconclusive. In exclusively breastfed infants, solid foodintroduction prior to 6 months of age generally is associated with reduced human milk intakewithout accelerated weight gain. However, formula-fed infants may be encouraged to consumethe same amount of formula, even after complementary feedings are introduced. This practicemay lead to increased calorie consumption and excessive weight gain.

Gastroesophageal reflux (GER) is the result of transient relaxations of the lower esophagealsphincter. Studies using intraesophageal pH probe monitoring data have shown that the refluxindex (RI) (percent time that esophageal pH is less than 4) is significantly greater in infants (RImean upper limit of normal: ~12) than in older individuals (mean: ~6). The addition of solids to thediet does not influence the time to resolution of clinical GER during infancy, although thefrequency and severity of symptomatic reflux episodes may be reduced, at least in part, bythickening feedings or increasing solid consumption in appropriately aged infants.

The appropriate timing for introducing solid foods to the infant diet depends on developmentof both neuromuscular function and gastrointestinal maturation. The American Academy ofPediatrics supports exclusive breastfeeding for the first 6 postnatal months. However, from adevelopmental perspective, term infants often are capable of accepting solids (complementaryfoods) between 4 and 6 months of age. Maturational readiness to tolerate complementaryfeedings is indicated by loss of the extrusion reflex (usually by 4 months) and by the ability toswallow non-liquid foods. The most obvious risk posed by solid food consumption prior toreaching these developmental milestones is that failure to achieve oropharyngeal coordinationfor consuming solids may lead to aspiration.

Copyright © 2010 by the American Academy of Pediatrics page 2

Page 3: AAP PREP 2010

2010 PREP SA on CD-ROM

References:

Braganza SF, Adam HF. In brief: gastroesophageal reflux. Pediatr Rev. 2005;26:304-305.Available at: http://pedsinreview.aappublications.org/cgi/content/full/26/8/304

Fein SB, Labiner-Wolfe J, Scanlon KS, Grummer-Strawn LM. Selected complementary feedingpractices and their association with maternal education. Pediatrics. 2008;122:S91-S97.Available at:http://pediatrics.aappublications.org/cgi/reprint/122/Supplement_2/S91?maxtoshow=&HITS=10&hits=10&RESULTFORMAT=&fulltext=Selected+complementary+feeding+practices+and+their+association+with+maternal+education&searchid=1&FIRSTINDEX=0&sortspec=relevance&resourcetype=HWCIT

Kleinman RE. Complementary feeding. In: Pediatric Nutrition Handbook. 5th ed. Elk GroveVillage, Ill: American Academy of Pediatrics; 2004:103-115

Kramer MS, Chalmers B, Hodnett ED, et al. Promotion of Breastfeeding Intervention Trial(PROBIT): a randomized trial in the Republic of Belarus. JAMA. 2001;285:413-420. Available at:http://jama.ama-assn.org/cgi/content/full/285/4/413

Copyright © 2010 by the American Academy of Pediatrics page 3

Page 4: AAP PREP 2010

2010 PREP SA on CD-ROM

Question: 2

You are called to labor and delivery to attend the vaginal delivery of a 37 weeks’ gestation maleto a 24-year-old primiparous mother. She reports that her membranes ruptured 36 hours ago.She is afebrile.

Of the following, the maternal condition that is MOST likely to require antibiotic therapy for thisneonate is

A. chorioamnionitis

B. diabetes mellitus

C. group B streptococcal colonization

D. preeclampsia

E. urinary tract infection in the first trimester

Copyright © 2010 by the American Academy of Pediatrics page 4

Page 5: AAP PREP 2010

2010 PREP SA on CD-ROM

Preferred Response: ACritique: 2

The mother described in the vignette has premature rupture of the membranes, whichoccurred 36 hours prior to her delivery. Premature rupture of the membranes is defined asrupturing of the amniotic sac not followed by the onset of labor within 8 hours. Preterm ruptureof membranes is defined as rupturing of the amniotic sac before 37 completed weeks ofgestation. Prolonged rupture of membranes (PROM) before delivery of the fetus increases therisk for early-onset neonatal sepsis (EONS); PROM of 18 hours or more is consideredsignificant.

Both term and preterm infants are at risk for EONS associated with PROM. However, thepreterm infant delivered after PROM has a greater risk for infection than the term infant due toimmature innate immune function (eg, immature phagocytic and neutrophil function) and potentiallack of passively acquired transplacental immunity (prior to 32 weeks’ gestation). Two maternalconditions can increase the risk for EONS in the face of PROM: chorioamnionitis and maternalgroup B streptococcal (GBS) colonization. The former confers a risk for fetal infection that maynot be treated adequately with maternal antibiotics. Hence, the newborn may continue to havepartially treated bacteremia, pneumonia, or meningitis and require treatment for presumed sepsis.The latter is of import because infants born before 36 weeks’ completed gestation are moresusceptible to GBS infection, especially in the face of PROM or chorioamnionitis. A sepsisevaluation and empiric treatment with a penicillin and an aminoglycoside is recommended for thepreterm infant following PROM.

Maternal GBS colonization status is of most importance in deciding whether to evaluate andtreat a preterm but not term infant unless the term infant has symptoms of infection. Maternaldiabetes does not affect the treatment of an infant with antibiotics. Preeclampsia does notconfer risk to the fetus or newborn for acquiring an infection in the face of PROM, but it may beassociated with leukopenia, especially in the very low-birthweight newborn, and may have anegative impact on the newborn’s ability to clear infection. A first-trimester maternal urinary tractinfection is of little import because it is remote from delivery. However, in a GBS-positivepregnant woman who experiences a urinary tract infection that is due to GBS, the potentialinoculum is greater in the neonate, which affects the evaluation and presumptive treatment ofthe newborn.

References:

American Academy of Pediatrics. Group B streptococcal infections. In: Pickering LK, Baker CJ,Kimberlin DW, Long SS, eds. Red Book: 2009 Report of the Committee on Infectious Diseases.28th ed. Elk Grove Village, Ill: American Academy of Pediatrics; 2009:628-634

Centers for Disease Control and Prevention. Guidelines for GBS: Recommendations. 2008.Available at: http://www.cdc.gov/groupbstrep/guidelines/recommendations.htm

Puopolo KM, Madoff LC, Eichenwald EC. Early-onset group B streptococcal disease in the era ofmaternal screening. Pediatrics. 2005;115:1240-1246. Available at:http://pediatrics.aappublications.org/cgi/content/full/115/5/1240

Copyright © 2010 by the American Academy of Pediatrics page 5

Page 6: AAP PREP 2010

2010 PREP SA on CD-ROM

Schrag S, Gorwitz R, Fultz-Butts K, Schuchat A. Prevention of perinatal group B streptococcaldisease. Revised guidelines from CDC. MMWR Recomm Rep. 2002;51(RR-11):1-22. Availableat: http://www.cdc.gov/mmwr/preview/mmwrhtml/rr5111a1.htm

Thilo EH, Rosenberg AA. The newborn infant. In: Hay WW Jr, Levin MJ, Sondheimer JM,Deterding RR, eds. CURRENT Diagnosis & Treatment: Pediatrics. 19th ed. New York, NY: TheMcGraw-Hill Companies, Inc; 2009:Chapter 1. Available for subscription at:http://www.accessmedicine.com/content.aspx?aID=3396500

Copyright © 2010 by the American Academy of Pediatrics page 6

Page 7: AAP PREP 2010

2010 PREP SA on CD-ROM

Question: 3

A mother brings her 9-year-old boy to your clinic because he has been complaining of beingtired in physical education class at school for the past few months. When you ask him about hissymptoms, he reports having trouble catching his breath after he runs. Past medical history isnegative, and a review of systems reveals only a cough that occurs primarily at night severaltimes a month. He has grown well, and findings on physical examination are normal.

Of the following, the MOST likely reason for his exercise intolerance is

A. cystic fibrosis

B. exercise-induced asthma

C. iron deficiency anemia

D. vocal cord dysfunction

E. Wolff-Parkinson-White syndrome

Copyright © 2010 by the American Academy of Pediatrics page 7

Page 8: AAP PREP 2010

2010 PREP SA on CD-ROM

Preferred Response: BCritique: 3

Exercise intolerance is the failure to tolerate physical exercise at a level that would beexpected for a person’s age and condition, such as described for the boy in the vignette. For thechild, it is important to determine whether exercise intolerance is due to a primary pulmonary orextrapulmonary cause. Pulmonary causes include asthma, cystic fibrosis, and acute and chronicinfections of the lung. Among the extrapulmonary causes of exercise intolerance are cardiacdisorders such as congestive heart failure, neuromuscular disorders such as musculardystrophy, anemia, and deconditioning.

Exercise intolerance is measured primarily by the maximal oxygen consumption test, andexamining the components of maximal oxygen consumption can be useful in understanding thereasons for exercise intolerance associated with various disease states. The Fick equation formaximal oxygen consumption is:

VO2max = SVmax x HRmax x (CaO2 — CvO2)max

where VO2 = oxygen consumption, SV=stroke volume, HR=heart rate, CaO2=oxygen

content of arterial blood, and CvO2=oxygen content of mixed venous blood. A sedentarylifestyle and certain cardiac diseases such as congestive heart failure and cyanotic heartdisease can cause a decrease in stroke volume. Diseases such as asthma, cystic fibrosis,anemia, and vocal cord dysfunction lower the oxygen content of arterial blood. States causingmuscle weakness, such as muscular dystrophy or general deconditioning, can result indecreased oxygen use by the tissues. Alterations in any of these components can lead todecreased maximal oxygen consumption and exercise intolerance.

The shortness of breath after running and a nighttime cough described for the boy in thevignette make exercise-induced bronchoconstriction (EIB), also called exercise-induced asthma,the most likely diagnosis. Children who have EIB generally experience shortness of breath,chest tightness, and cough approximately 10 to 15 minutes after beginning exercise.

Administration of a short-acting beta2 agonist or inhaled cromolyn sodium prior to exercising canhelp to prevent the symptoms. For patients who have poorly controlled asthma and experienceEIB, the most appropriate management is the use of inhaled corticosteroids and possibly othermaintenance medications to control overall asthma symptoms. If a child who has presumed EIB

fails to respond to pretreatment with beta2 agonists or inhaled cromolyn sodium, otherdiagnoses such as vocal cord dysfunction should be considered.

Vocal cord dysfunction is the paradoxic adduction of the vocal cords during inspiration,causing airway obstruction during exercise. Inspiratory wheezing and throat tightness arecommon symptoms, but cough at night is not. Cystic fibrosis is unlikely in any child who isgrowing well and has no extrapulmonary symptoms. Iron deficiency anemia can cause exercise-induced dyspnea, but the boy’s history is not suggestive of this condition. Wolff-Parkinson-Whitesyndrome causes re-entrant tachycardia; syncope rather than exercise intolerance is the usualclinical manifestation.

References:

Copyright © 2010 by the American Academy of Pediatrics page 8

Page 9: AAP PREP 2010

2010 PREP SA on CD-ROM

McColley SA. Extrapulmonary diseases with pulmonary manifestations. In: Kliegman RM,Behrman RE, Jenson HB, Stanton BF, eds. Nelson Textbook of Pediatrics. 18th ed. Philadelphia,Pa: Saunders Elsevier; 2007:1846-1847

O’Byrne PM. Exercise-induced bronchoconstriction. UpToDate Online 16.3. 2008. Available forsubscription at:http://www.utdol.com/online/content/topic.do?topicKey=asthma/13974&selectedTitle=1~43&source=search_result630

Owens S, Gutin B. Exercise intolerance. Pediatr Rev. 2000;21:6-9. Available at:http://pedsinreview.aappublications.org/cgi/content/full/21/1/6

Copyright © 2010 by the American Academy of Pediatrics page 9

Page 10: AAP PREP 2010

2010 PREP SA on CD-ROM

Question: 4

You are seeing a 1-year-old patient in your clinic for a health supervision visit. You explain therecommended screening tests for this visit to the medical student who accompanies you.

Of the following, the MOST appropriate recommended screening test at this visit is

A. blood lead concentration by fingerstick

B. blood lead concentration by venipuncture

C. complete blood count with differential count

D. serum ferritin

E. serum iron

Copyright © 2010 by the American Academy of Pediatrics page 10

Page 11: AAP PREP 2010

2010 PREP SA on CD-ROM

Preferred Response: ACritique: 4

The diagnosis of lead poisoning or increased lead absorption depends on the measurementof blood lead concentrations. In the 1990s, both the American Academy of Pediatrics and theCenters for Disease Control and Prevention recommended universal blood lead screening of 1-and 2-year-old children, but because of the substantial decrease in the prevalence of elevatedblood lead concentrations, the criteria for screening are changing in many communities. Thus, itmay be helpful to contact your local health department to determine if children in your area are atrisk for environmental lead exposure.

Although blood lead concentration can be measured most accurately from a sample obtainedby venipuncture, a capillary specimen obtained by fingerstick is the most appropriate screeningtest for the toddler described in the vignette. The specimen must be obtained carefully to avoidcontamination from lead on the skin. Capillary specimen values greater than 10 mcg/dL (0.5mcmol/L) must be confirmed by a venous sample because of the possibility of skin contamination

causing a false-positive result. Although obtaining a complete blood count with smear and measuring serum ferritin and

serum iron may be useful in the diagnosis and management of children who have anemia,including that associated with environmental lead exposure, these tests are not definitive fordetermining exposure to environmental lead. Finally, hair evaluation for lead poisoning is neithersensitive nor specific due to the lack of correlation with blood lead values and should not beused.

References:

American Academy of Pediatrics Committee on Environmental Health. Lead exposure in children:prevention, detection, and management. Pediatrics. 2005;116:1036-1046. Available at:http://pediatrics.aappublications.org/cgi/content/full/116/4/1036

Binns HJ, Campbell C, Brown MJ for the Advisory Committee on Childhood Lead PoisoningPrevention. Interpreting and managing blood lead levels of less than 10 mcg/dL in children andreducing childhood exposure to lead: recommendations of the Centers for Disease Control andPrevention Advisory Committee on Childhood Lead Poisoning Prevention. Pediatrics.2007;120:e1285-e1298. Available at:http://pediatrics.aappublications.org/cgi/content/full/120/5/e1285

Laraque D, Trasande L. Lead poisoning: successes and 21st century challenges. Pediatr Rev.2005;26:435-443. Available at: http://pedsinreview.aappublications.org/cgi/content/full/26/12/435

Rischitelli G, Nygren P, Bougatsos C, Freeman M, Helfand M. Screening for elevated lead levelsin childhood and pregnancy: an updated summary of evidence for the US Preventive ServicesTask Force. Pediatrics. 2006;118:e1867-e1895. Available at:http://pediatrics.aappublications.org/cgi/content/full/118/6/e1867

Yeoh B, Woolfenden S, Wheeler D, Alperstein G, Lanphear B. Household interventions for

Copyright © 2010 by the American Academy of Pediatrics page 11

Page 12: AAP PREP 2010

2010 PREP SA on CD-ROM

prevention of domestic lead exposure in children. Cochrane Database Syst Rev.2008;2:CD006047

Copyright © 2010 by the American Academy of Pediatrics page 12

Page 13: AAP PREP 2010

2010 PREP SA on CD-ROM

Question: 5

During the health supervision visit for a 14-year-old boy, you note that his body mass index(BMI) is at the 95th percentile. At last year’s health supervision visit, his BMI was at the 85thpercentile.

Of the following, the complication for which this boy is at the HIGHEST risk is

A. atrial fibrillation

B. hypothyroidism

C. narcolepsy

D. social isolation

E. steatorrhea

Copyright © 2010 by the American Academy of Pediatrics page 13

Page 14: AAP PREP 2010

2010 PREP SA on CD-ROM

Preferred Response: DCritique: 5

Childhood obesity is one of the most common chronic conditions of childhood. It is believedto be strongly associated with a number of morbidities, including type 2 diabetes mellitus,hypertension, the metabolic syndrome, and psychosocial conditions. For individual children, theimmediate psychosocial effects of social isolation, discrimination, and peer problems mayaccompany childhood obesity. Obese adolescents have been reported to have lower self-esteem as well as increased rates of sadness, social isolation, loneliness, and nervousness.Several studies have demonstrated the relationship between obesity, as measured by bodymass index (BMI) and the psychosocial development of children.

Adolescent obesity is a strong predictor of adult obesity, and adult obesity has beenassociated with depression, especially in women. Studies also have suggested an associationbetween depression in adolescence and higher BMI in adulthood. Whether depression leads toobesity or obesity causes depression is unclear. In contrast, there is a strong associationbetween lower self-esteem and higher BMI across the elementary school years. In manychildren, the presence of an increased BMI and obesity precedes low self-esteem, suggesting acausal relationship. Accordingly, prevention and management strategies for children who areoverweight and obese should be undertaken early to minimize the impact on self-esteem and theother important psychosocial aspects of healthy development.

The boy described in the vignette, who meets the diagnostic criteria for obesity (BMI >95thpercentile), may be at increased risk for obstructive apnea and resulting effects on the rightventricle, but he is not necessarily at a greater risk for atrial arrhythmias such as atrial fibrillation.Similarly, he may have poor sleeping habits, but obesity does not lead to narcolepsy. Althoughhypothyroidism can be associated with weight gain, no evidence suggests that obesity leads tohypothyroidism. Finally, obesity is not known to be a cause of steatorrhea.

References:

Goodman E, Whitaker RC. A prospective study of the role of depression in the development andpersistence of adolescent obesity. Pediatrics. 2002;110:497-504. Available at:http://pediatrics.aappublications.org/cgi/content/full/110/3/497

Hesketh K, Wake M, Waters E. Body mass index and parent-reported self-esteem in elementaryschool children: evidence for a causal relationship. Int J Obes Relat Metab Disord.2004;28:1233-1237. Available at: http://www.nature.com/ijo/journal/v28/n10/full/0802624a.html

Ludwig DS. Childhood obesity–the shape of things to come. N Engl J Med. 2007;357:2325-2327.Available at: http://content.nejm.org/cgi/content/full/357/23/2325

Stunkard AJ, Wadden TA. Psychological aspects of severe obesity. Am J Clin Nutr. 1992;55(2suppl):524S-532S. Available at: http://www.ajcn.org/cgi/reprint/55/2/524S

Copyright © 2010 by the American Academy of Pediatrics page 14

Page 15: AAP PREP 2010

2010 PREP SA on CD-ROM

Question: 6

A mother brings in her otherwise healthy 4-year-old son because of problems walking. He hasnot wanted to walk since this morning and only stands on tip-toe or crawls. He had an upperrespiratory tract illness with a fever 1 week ago. On physical examination, the boy appearswell, but he complains of leg pains bilaterally. He reports no back pain. His calf muscles aretender to palpation, but joints are not warm, red, or swollen. Patellar and ankle jerk reflexes arepresent bilaterally, and there is no clonus.

Of the following, the MOST appropriate initial diagnostic procedure is

A. antinuclear antibody measurement

B. lumbar puncture

C. magnetic resonance imaging of the spine

D. nerve conduction studies/electromyography

E. serum creatine kinase measurement

Copyright © 2010 by the American Academy of Pediatrics page 15

Page 16: AAP PREP 2010

2010 PREP SA on CD-ROM

Preferred Response: ECritique: 6

A child who has an acute or subacute gait disturbance represents a medical emergency.Diagnostic evaluation must be thorough, considering causes at all neuroanatomic levels:brain/cerebrum, cerebellum, brainstem, spinal cord, root, nerve, junction, and muscle. Promptidentification of hydrocephalus, spinal cord lesions, or Guillain-Barré syndrome can preventlifelong neurologic damage or save a life.

The boy described in the vignette has symptoms only in his legs, with both weakness andleg muscle tenderness. Such findings typically localize to muscle and suggest the diagnosis ofacute myositis. An elevated serum creatine kinase value (sometimes to more than 1,000 units/L)confirms the diagnosis. In children, acute myositis most often occurs after respiratory infectionssuch as influenza. The condition is self-limited, rhabdomyolysis is unlikely, and no treatment isneeded.

A spinal cord process that necessitates neuroimaging of the spine is very unlikely for thisboy. Findings suggestive of such a diagnosis include flaccid reflexes (initially), loss of boweland bladder function, and sensory deficits with a sensory level. A lumbar puncture is used toassess for acute inflammatory demyelinating polyneuropathy, also known as Guillain-Barrésyndrome. The absence of back pain and the preservation of reflexes make this diagnosisunlikely. Nerve conduction studies/electromyography often are helpful when neuropathy issuspected in a child who has motor and sensory deficits plus loss of reflexes. The boy has nohistory or examination finding to support a rheumatologic disease that requires antinuclearantibody measurement.

References:

Compeyrot-Lacassagne S, Feldman BM. Inflammatory myopathies in children. Pediatr Clin NorthAm. 2005;52:493-520. Abstract available at: http://www.ncbi.nlm.nih.gov/pubmed/15820377

Mackay MT, Kornberg AJ, Shield LK, Dennett X. Benign acute childhood myositis: laboratory andclinical features. Neurology. 1999;53:2127-2131. Abstract available at:http://www.ncbi.nlm.nih.gov/pubmed/10599793

Millichap JG. Benign acute myositis and influenza viral infection. AAP Grand Rounds. 2000;3:32.Available for subscription at: http://aapgrandrounds.aappublications.org/cgi/content/full/3/3/32

Copyright © 2010 by the American Academy of Pediatrics page 16

Page 17: AAP PREP 2010

2010 PREP SA on CD-ROM

Question: 7

You are called to the neonatal intensive care unit to evaluate a newly admitted 34-weekgestational age male infant who has respiratory distress. When you arrive, the baby is receivingoxygen supplementation by hood. You note that the baby’s weight, length, and headcircumference are all below the 10th percentile. He has excess hair over his forehead,shoulders, and back (Item Q7). In addition, he is very irritable, despite correction of his oxygensaturation to 95%.

Of the following, this infant’s unusual findings are MOST likely related to prenatal exposure to

A. alcohol

B. cocaine

C. marijuana

D. methamphetamine

E. tobacco

Copyright © 2010 by the American Academy of Pediatrics page 17

Page 18: AAP PREP 2010

2010 PREP SA on CD-ROM

Preferred Response: ACritique: 7

The infant described in the vignette has features consistent with fetal alcohol spectrumdisorders (FASDs). FASDs are characterized by a range of recognizable outcomes in infantsexposed to alcohol prenatally, the most severe of which is fetal alcohol syndrome (FAS). FASincludes the presence of specific facial anomalies, such as short palpebral fissures, thinvermilion border of the upper lip, and smooth philtrum, as well as evidence of pre- or postnatalgrowth restriction (height or weight <10th percentile) and findings consistent with abnormalbrain growth (head circumference <10th percentile) or brain development (structural brainanomalies). Maternal alcohol exposure need not be confirmed to make a diagnosis of FAS, butother syndromes and conditions that have overlapping features should be ruled out. Othercategories of FASD include partial FAS with or without confirmed maternal alcohol exposure,alcohol-related birth defects, and alcohol-related neurodevelopmental disorder. Newbornsaffected by FAS frequently are irritable and tremulous, and although these symptoms suggestneonatal withdrawal, they can continue for months. Infants also can be unusually hirsute(ethnicity always must be considered when judging hirsutism), and this feature typicallydissipates over the first 6 postnatal months (Item C7). Although there is no well-characterizedneonatal alcohol withdrawal syndrome, the physician should be alert to signs of drugwithdrawal when FASD is suspected due to the frequent concomitant use of alcohol and drugs.

Despite numerous publications describing deleterious effects of cocaine on the developingembryo and fetus, the impact of prenatal cocaine exposure remains uncertain. It is generallyaccepted that cocaine use in pregnancy increases the likelihood of placental abruption, andthere is an increased incidence of sudden infant death syndrome in exposed infants. There alsomay be an increased risk for genitourinary and limb anomalies. There is no generally agreed-upon "cocaine syndrome." Because cocaine often is used in combination with other drugs,cigarettes, and alcohol, it can be difficult to discern what fetal abnormalities are cocaine-related.

Marijuana use in pregnancy is not known to be associated with an increased risk for birthdefects, dysmorphic features, or developmental delay in exposed offspring.

Methamphetamines have not been shown to increase the risk for birth defects in exposedinfants, although decreased birthweight has been reported in some exposed infants. A neonatalwithdrawal syndrome that includes abnormal sleep patterns, tremulousness, poor feeding, andincreased tone frequently is described. Concern has been raised for neurodevelopmentalproblems in later years, but further investigation is needed.

The effects of maternal smoking on pregnancy outcome continue to be an active area ofstudy. Cigarette smoking is associated with an increased risk for miscarriage, reduced fetalweight, and abnormal placentation. There may be an increased risk for facial clefting, butcigarette smoking is not otherwise associated with major congenital anomalies.

References:

Cigarette smoking (tobacco). Teris. Available for subscription at:http://depts.Washington.edu/terisweb/teris

Cigarette smoking. Reprotox®. Available for subscription at: http://www.reprotox.org

Copyright © 2010 by the American Academy of Pediatrics page 18

Page 19: AAP PREP 2010

2010 PREP SA on CD-ROM

Cocaine. Reprotox®. Available for subscription at: http://www.reprotox.org

Cocaine. Teris. Available for subscription at: http://depts.Washington.edu/terisweb/teris/

Hoyme HE, May PA, Kalberg WO, et al. A practical clinical approach to diagnosis of fetal alcoholspectrum disorders: clarification of the 1996 Institute of Medicine criteria. Pediatrics.2005;115:39-47. Available at: http://pediatrics.aappublications.org/cgi/content/full/115/1/39

Copyright © 2010 by the American Academy of Pediatrics page 19

Page 20: AAP PREP 2010

2010 PREP SA on CD-ROM

Question: 8

A 15-year-old girl presents to the emergency department with right upper quadrant pain for 2days that is severe enough to keep her out of school. Her appetite is decreased and she hasnausea but no vomiting or diarrhea. She has mild discomfort with urination but no vaginaldischarge. The only medication she is taking is combined oral contraceptive pills. Her last

menstrual period was heavier that usual. Laboratory tests reveal:

•White blood cell count, 7.4x103/mcL (7.4x109/L) with 64% segmented neutrophils and

26% lymphocytes

•Total bilirubin, 0.4 mg/dL (6.9 mcmol/L)

•Alanine aminotransferase, 14 units/L

•Aspartate aminotransferase, 16 units/LHer urine has 7 white blood cells per high-power field. Abdominal ultrasonography reveals anormal liver, spleen, gallbladder, and kidneys.

Of the following, the MOST likely diagnosis is

A. cholecystitis

B. Fitz-Hugh-Curtis syndrome

C. hepatitis A infection

D. infectious mononucleosis

E. pyelonephritis

Copyright © 2010 by the American Academy of Pediatrics page 20

Page 21: AAP PREP 2010

2010 PREP SA on CD-ROM

Preferred Response: BCritique: 8

Both the American Medical Association’s Guidelines for Adolescent Preventive Services andBright Futures recommend that all adolescents should be asked annually about involvement insexual behaviors that may result in unintended pregnancy and sexually transmitted infections(STIs), including human immunodeficiency virus infection. In addition to annual screening, thepossibility of a pregnancy or an STI should be considered at every visit with an adolescent, andthe last menstrual period should be documented.

The laboratory results for this girl rule out hepatitis, including that caused by mononucleosis,and biliary tract obstruction. Fitz-Hugh-Curtis syndrome or perihepatitis presents as right upperquadrant pain that results from inflammation of the liver capsule from ascending pelvic infection.Although typically associated with salpingitis, it can exist without other signs of pelvicinflammatory disease and may mimic other abdominal emergencies. The absence of fever andthe location of pain for this girl make pyelonephritis unlikely. Pyuria raises the possibility ofurethritis, which commonly occurs with Neisseria gonorrhoeae and Chlamydia trachomatisinfections. C trachomatis can cause inflammation of the genital tract without the classicsymptoms and signs of pelvic inflammatory disease. Often, heavier menstrual flow may be theonly symptom.

References:

Burstein GR, Murray PJ. Diagnosis and management of sexually transmitted disease pathogensamong adolescents. Pediatr Rev. 2003;24:75-82. Available at:http://pedsinreview.aappublications.org/cgi/content/full/24/3/75

Elster AB, Kuznets NJ, eds. American Medical Association Guidelines for AdolescentPreventive Services (GAPS): Recommendations Monograph. Chicago, Ill: American MedicalAssociation; 1997. Available at: http://www.ama-assn.org/ama/upload/mm/39/gapsmono.pdf

Hagan JF Jr, Shaw JS, Duncan P. Bright Futures: Guidelines for Health Supervision of Infants,Children and Adolescents. 3rd ed. Elk Grove, Ill: American Academy of Pediatrics; 2008

Hammerschlag MR. Chlamydia trachomatis and Chlamydia pneumoniae infections in childrenand adolescents. Pediatr Rev. 2004;25:43-51. Available at:http://pedsinreview.aappublications.org/cgi/content/full/25/2/43

Copyright © 2010 by the American Academy of Pediatrics page 21

Page 22: AAP PREP 2010

2010 PREP SA on CD-ROM

Question: 9

You are evaluating a 4-year-old boy in the emergency department for septic shock. On physicalexamination, his heart rate is 140 beats/minute, respiratory rate is 30 breaths/minute, and bloodpressure is 65/40 mm Hg.

Of the following, the MOST appropriate next step is administration of

A. 5 mL/kg of 25% albumin

B. 5 mL/kg of 3% normal saline

C. 10 mL/kg of 5% albumin

D. 20 mL/kg of 0.45% normal saline

E. 20 mL/kg of 0.9% normal saline

Copyright © 2010 by the American Academy of Pediatrics page 22

Page 23: AAP PREP 2010

2010 PREP SA on CD-ROM

Preferred Response: ECritique: 9

The boy described in the vignette is exhibiting signs and symptoms of septic shock, which isdefined as the presence of sepsis and cardiovascular organ dysfunction. Septic shock is amedical emergency that requires prompt recognition and treatment.

One of the most important factors in lowering the mortality associated with septic shock isearly and aggressive fluid resuscitation, defined as isotonic fluid boluses of 20 mL/kg titratedaccording to clinical assessment of adequacy of cardiac output, such as heart rate, urineoutput, and level of consciousness. Often, 60 mL/kg is needed for the initial resuscitation of achild who has septic shock.

The choice of fluid (crystalloid versus colloid) for use in shock resuscitation has been thesubject of much debate, with little evidence to support the superiority of one over the other. Inthis scenario, 20 mL/kg of normal saline is preferred over 10 mL/kg of 5% albumin solely due tothe larger volume administered. Initial fluid boluses should be 20 mL/kg, and often 60 mL/kg ormore is required in the first hour of shock resuscitation.

The decision to use 5% albumin or normal saline often is dependent on institutionalpreferences. Both fluids are isotonic and increase intravascular volume, although a greatervolume of saline is needed acutely to achieve the same effect. When endothelial integrity isaltered, albumin can leak into the interstitium and increase edema formation. In addition, albumin isconsiderably more expensive than normal saline, can produce hypocalcemia, and has a smallrisk of allergic reaction. In a large double-blind study comparing the use of 4% albumin withnormal saline for fluid resuscitation of nearly 7,000 adult patients in the intensive care unit,investigators were unable to demonstrate statistical differences in mortality, length of intensivecare unit and hospital stay, days of mechanical ventilation, or need for renal replacementtherapy. A post-hoc analysis demonstrated an association with increased mortality in thosepatients who had traumatic brain injury and were treated with 4% albumin.

Although research is ongoing on the use of smaller amounts of hypertonic solutions forhemorrhagic shock, there is little research to date on its use in septic shock. Therefore, neitherthe administration of 3% normal saline nor 25% albumin is indicated. A 0.45% normal salinesolution is a hypotonic fluid that would not be used for initial resuscitation.

References:

Brierly J, Carcillo JA, Choong J, et al. Clinical practice parameters for hemodynamic support ofpediatric and neonatal septic shock: 2007 update from the American College of Critical CareMedicine. Crit Care Med. 2009;37:666-688

Dellinger RP, Levy MM, Carlet JM, et al. Surviving Sepsis Campaign: international guidelines formanagement of severe sepsis and septic shock: 2008. Crit Care Med. 2008;36:296-327.Abstract available at: http://www.ncbi.nlm.nih.gov/pubmed/18158437

Finfer S, Bellomo R, Boyce N. French J, Myburgh J, Norton R; SAFE Study Investigators. Acomparison of albumin and saline for fluid resuscitation in the intensive care unit. N Engl J Med.2004;350:2247—2256. Available at: http://content.nejm.org/cgi/content/full/350/22/2247

Copyright © 2010 by the American Academy of Pediatrics page 23

Page 24: AAP PREP 2010

2010 PREP SA on CD-ROM

Gupta M. Saline or albumin in the ICU. AAP Grand Rounds. 2004;12:16-17

McKiernan MA, Lieberman SA. Circulatory shock in children: an overview. Pediatr Rev.2005;26:451-460. Available at: http://pedsinreview.aappublications.org/cgi/content/full/26/12/451

SAFE Study Investigators; et al. Saline or albumin for fluid resuscitation in patients with traumaticbrain injury. N Engl J Med. 2007;357:874-884. Available at:http://content.nejm.org/cgi/content/full/357/9/874

Copyright © 2010 by the American Academy of Pediatrics page 24

Page 25: AAP PREP 2010

2010 PREP SA on CD-ROM

Question: 10

You are examining a 6-year-old girl at her annual health supervision visit. She has SexualMaturity Rating 3 pubic hair, but no axillary hair. Her mother notes that the child has had an adultbody odor for about 6 months. You cannot detect breast tissue, and she does not haveclitoromegaly. You examine her growth chart for height (Item Q10).

Of the following, the MOST important test to perform at this time is

A. bone age radiography

B. magnetic resonance imaging of the head

C. pelvic ultrasonography

D. serum estradiol measurement

E. serum testosterone measurement

Copyright © 2010 by the American Academy of Pediatrics page 25

Page 26: AAP PREP 2010

2010 PREP SA on CD-ROM

Preferred Response: ACritique: 10

Bone age radiographs can offer useful information about the adult height potential of healthychildren who have bone ages of more than 7 years. In addition, a bone age that is advanced, incontrast to chronologic age, can offer some understanding of the length of time or the severityof exposure to sex steroids, particularly estrogen, which is responsible for epiphysealadvancement. Similarly, a bone age that is delayed in relation to chronologic age can offer someinference as to the length of time of endocrine deficiency disorders such as hypothyroidism orchronic nutritional disorders such as celiac disease. However, bone age radiographs do notpermit accurate height predictions in children who have bone ages less than 7 years and cannotoffer appropriate height predictions for children who have abnormal growth patterns because ofbone disorders or other growth disorders. Studies have shown that in children who haveisolated premature pubic or axillary hair development (premature pubarche or adrenarche), abone age radiograph that is more than 1 year advanced compared with chronologic ageincreases the possibility of a serious underlying disorder such as late-onset congenital adrenalhyperplasia.

The girl described in the vignette has premature adrenarche or pubarche, the earlyappearance of the physical signs of adrenal puberty. Her growth chart reveals a smallacceleration in height, which is consistent with this diagnosis (Item C10). She has no evidenceof either ovarian puberty (thelarche or breast tissue) or exposure to potent androgens(clitoromegaly). An increase in adrenal production of weak androgen precursors(dehydroepiandrosterone [DHEA] and dehydroepiandrosterone sulfate [DHEA-s]) can beidentified in most children between 4 and 6 years of age. However, the earliest signs of adrenalpuberty (adult body odor, pubic hair) usually occur at the same time or a few months afterthelarche. Rare children may have higher DHEA and DHEA-s values before the age of 7 years ormay be particularly sensitive to low circulating concentrations of these hormones. Such children,including the girl in the vignette, come to medical attention because of early adult body odor andpubic and/or axillary hair.

The most likely worrisome possibility in the differential diagnosis for children who havepremature adrenarche is late-onset congenital adrenal hyperplasia. A bone age radiographreading within 1 year of the child’s actual age effectively rules out the diagnosis. Magneticresonance imaging of the head might be indicated in a female child who has early true sexualprecocity (activation of the hypothalamic gonadotropic axis with thelarche), but there is noindication for such a study in this child. Pelvic ultrasonography is indicated if there is suspicionabout an androgen-producing ovarian tumor, but the slow progression and lack of clitoromegalymakes this very unlikely for this girl. Measures of serum estradiol and testosterone are notuseful in this situation. The child’s lack of thelarche or clitoromegaly indicates low estradiol andtestosterone concentrations, respectively, and standard commercial assays do not provideaccurate results at the low concentrations of estrogen and testosterone found in early puberty.Her growth at a higher percentile is typical of girls who will be slightly early maturers and arehaving the mid-childhood growth spurt, sometimes attributed to adrenarche.

References:

Copyright © 2010 by the American Academy of Pediatrics page 26

Page 27: AAP PREP 2010

2010 PREP SA on CD-ROM

Ibánez L, Jiménez R, de Zegher F. Early puberty-menarche after precocious pubarche: relationto prenatal growth. Pediatrics. 2006;117:117-121. Available at:http://pediatrics.aappublications.org/cgi/content/full/117/1/117

Kaplowitz P. Clinical characteristics of 104 children referred for evaluation of precociouspuberty. J Clin Endocrinol Metab. 2004;89:3644-3650. Available at:http://jcem.endojournals.org/cgi/content/full/89/8/3644

Kaplowitz PB. Precocious puberty. eMedicine Specialties, Pediatrics: General Medicine,Endocrinology. 2007. Available at: http://emedicine.com/ped/TOPIC1882.HTM

Muir A. Precocious puberty. Pediatr Rev. 2006;27:373-381. Available at:http://pedsinreview.aappublications.org/cgi/content/full/27/10/373

Copyright © 2010 by the American Academy of Pediatrics page 27

Page 28: AAP PREP 2010

2010 PREP SA on CD-ROM

Question: 11

You are examining a 4-year, 11-month-old child during her health supervision visit. Her motherhas been home with her full time since birth. The friendly, happy child tells you about her trip tothe zoo last week. She asks if you like animals and counts out loud 10 different animals shesaw. When given a crayon, she is able to write her name stating the letters as she writes them.Her mother expresses concern about her beginning kindergarten next month because she justmeets the age cut-off for kindergarten entry. She inquires about the factors that will helpdetermine if her child is ready to attend school.

Of the following, the MOST appropriate response is that the girl should

A. be able to maintain attention during story time for at least 20 to 25 minutes

B. be able to state her birthday and address

C. be able to tolerate separations from her parent for several hours at a time

D. identify upper and lower case letters

E. undergo psychoeducational testing to assess kindergarten readiness

Copyright © 2010 by the American Academy of Pediatrics page 28

Page 29: AAP PREP 2010

2010 PREP SA on CD-ROM

Preferred Response: CCritique: 11

Children are socially ready to attend school when they are able to separate from theirparents for several hours at a time. They are expected to pay attention to the teacher, follow theclassroom routine, play well with others, and take turns. During classroom group-basedactivities, the young child should be able to pay attention to the teacher for 10 to 15 minutes, sitquietly, and listen to the teacher without bothering peers or disrupting the activity. Children alsoshould be able to relate personal experiences and tell stories.

Children entering kindergarten should know color names, be able to count to 10, retell astory, identify some printed letters, and print their names. Knowing their addresses and birthdates and being able to identify both upper and lower case letters is expected of childrenentering first grade. Because the child described in the vignette has the prerequisite skills forkindergarten, a psychoeducational evaluation is not indicated at this time.

References:

High PC and the Committee on Early Childhood, Adoption, and Dependent Care and Council onSchool Health. School readiness. Pediatrics. 2008;121:e1008-e1015. Available at:http://pediatrics.aappublications.org/cgi/content/full/121/4/e1008

Kaplan-Sanoff M. School readiness. In: Parker S, Zukerman B, Augustyn M, eds. Developmentaland Behavioral Pediatrics: A Handbook for Primary Care. 2nd ed. Philadelphia, Pa: LippincottWilliams & Wilkins; 2005:285-288

Copyright © 2010 by the American Academy of Pediatrics page 29

Page 30: AAP PREP 2010

2010 PREP SA on CD-ROM

Question: 12

A family comes to your office for consultation regarding a 3-week trip to India they are planningto take in 3 months. The children, a 9-year-old boy and a 7-month-old girl, are well, and theirimmunizations are up to date.

Of the following, the MOST appropriate prophylaxis to provide in preparation for travel is

A. chloroquine for both children

B. hepatitis A vaccination for both children

C. measles vaccination for the girl

D. polio vaccination for the boy

E. typhoid vaccine for both children

Copyright © 2010 by the American Academy of Pediatrics page 30

Page 31: AAP PREP 2010

2010 PREP SA on CD-ROM

Preferred Response: CCritique: 12

Protection against infectious diseases is an important issue in preparing children and adultsfor international travel. Clinicians can obtain specific knowledge of available vaccines andprophylaxis for certain conditions from the American Academy of Pediatrics 2009 Report of the

Committee on Infectious Diseases (Red Book®) and the travelers’ health site of the Centers forDisease Control and Prevention. Travel to India involves a potentially increased exposure tomalaria, hepatitis A, measles, polio, and Salmonella typhi. However, there are otherconsiderations in recommending various preventive measures for travelers.

Measles may be encountered more commonly in many parts of the world, including India.Accordingly, measles vaccine is recommended for 6- to 11-month-old children, and the 7-month-old girl in the vignette should be given a dose of measles vaccine. She still will require twodoses of measles-containing vaccine after 1 year of age because the immune response may besuboptimal at her young age. If the 9-year-old boy is up to date on immunizations, he requires noadditional measles vaccination.

Although exposure to malaria is a concern on a prolonged trip to India, resistance tochloroquine is a major concern in this region, as it is in all of South and Southeast Asia, sub-Saharan Africa, and tropical areas of South America. Available agents for resistant malariaprophylaxis in infants and children include atovaquone/proguanil and mefloquine. Doxycyclinecan be used in children older than 8 years of age.

Hepatitis A is a concern, but hepatitis A vaccine is not approved in children younger than 1year of age. Intramuscular immunoglobulin is recommended for children younger than 1 year ofage, as the baby in the vignette, traveling to an endemic area. The boy should receive his firstdose of hepatitis A vaccine at least 2 to 4 weeks before departure if he has not been immunizedpreviously, with completion of the two-dose series 6 to 12 months later.

Although polio exposure may be a concern, if both children are up to date in theirvaccination series, no additional polio vaccine is indicated. Finally, typhoid vaccine might beindicated for a trip to India that lasts longer than 2 weeks, but neither of the two licensedvaccines is indicated in children younger than 2 years of age.

References:

American Academy of Pediatrics. International travel. In: Pickering LK, Baker CJ, Kimberlin DW,Long SS, eds. Red Book: 2009 Report of the Committee on Infectious Diseases. 28th ed. ElkGrove Village, Ill: American Academy of Pediatrics; 2009:98-104

Centers for Disease control and Prevention. Travelers’ Health Web site. Available at:http://wwwn.cdc.gov/travel/default.aspx

Centers for Disease Control and Prevention (CDC). Update: measles—United States,January–July 2008. MMWR Morbid Mortal Wkly Rep. 2008;57:893-896. Available at:http://www.cdc.gov/mmwr/preview/mmwrhtml/mm5733a1.htm

Copyright © 2010 by the American Academy of Pediatrics page 31

Page 32: AAP PREP 2010

2010 PREP SA on CD-ROM

Question: 13

You are evaluating a 2-year-old girl who was adopted from an orphanage in Eastern Europe.She has had a pruritic rash since she was brought to the United States 3 weeks ago. Accordingto the mother, the rash is so pruritic that the girl must wear socks on her hands at night toprevent her from scratching. Physical examination demonstrates multiple 2- to 3-mmerythematous papules and vesicles around her waist, in her inguinal folds, on her neck, and onthe palms and soles (Item Q13). No other focal findings are evident on physical examination.

Of the following, the MOST appropriate agent with which to treat this patient is

A. acyclovir orally

B. hydrocortisone topically

C. hydroxyzine orally

D. permethrin topically

E. prednisone orally

Copyright © 2010 by the American Academy of Pediatrics page 32

Page 33: AAP PREP 2010

2010 PREP SA on CD-ROM

Preferred Response: DCritique: 13

Scabies is a common disorder caused by infestation with the mite Sarcoptes scabiei.Scabies is contracted by prolonged close personal contact with an infected person, usually insituations such as families with school-age children or individuals living in close quarters, suchas the orphanage described for the girl in the vignette.

Permethrin, an insecticide that has been available since 1989, is a safe and effectivetreatment for scabies. Permethrin acts by disrupting the sodium channel current, resulting indelayed repolarization, paralysis, and death of the parasite. It is effective during all stages of thelife cycle of the parasite. Because of its excellent safety profile, 5% permethrin cream is the first-line drug for the treatment of scabies, especially among patients who have neurologic disordersand infants and young children. Lindane, an agent that was used in the past, no longer isrecommended. The 1% solution of permethrin used to treat head lice has too low of aconcentration to treat scabies effectively.

Permethrin has a low potential for toxicity but occasionally may cause redness of the skin,burning, and stinging with application and has been associated with rash and diarrhea. It is notrecommended for use in infants younger than 2 months of age or for pregnant women.

Acyclovir is an antiviral agent used to treat herpesvirus infections and is not effective in thetreatment of scabies. Topical hydrocortisone and oral hydroxyzine can be used to alleviate theitching, and oral prednisone may decrease the inflammation of scabies, but none of these agentsis used to treat the infection.

References:

American Academy of Pediatrics. Scabies. In: Pickering LK, Baker CJ, Kimberlin DW, Long SS,eds. Red Book: 2009 Report of the Committee on Infectious Diseases. 28th ed. Elk GroveVillage, Ill: American Academy of Pediatrics; 2009:589-591

Downs A. Comparing antiscabies treatments. Arch Dermatol. 1997;133:526

Schultz MW, Gomez M, Hansen RC, et al. Comparative study of 5% permethrin cream and 1%lindane lotion for the treatment of scabies. Arch Dermatol. 1990;126:167-170. Abstract availableat: http://www.ncbi.nlm.nih.gov/pubmed/1689135

Strong M, Johnstone P. Interventions for treating scabies. Cochrane Database Syst Rev.2007;3:CD000320. Available at:http://www.mrw.interscience.wiley.com/cochrane/clsysrev/articles/CD000320/frame.html

Copyright © 2010 by the American Academy of Pediatrics page 33

Page 34: AAP PREP 2010

2010 PREP SA on CD-ROM

Question: 14

An 18-month-old girl who has a 2-day history of vomiting with reduced oral intake presents tothe clinic with a 24-hour history of nonbloody diarrhea. She was previously well. Upon furtherquestioning, her mother reports a reduced number of wet diapers prior to the onset of diarrhea.She states that the girl is having four to five loose stools per day. You estimate the girl to be 5%dehydrated.

Of the following, the MOST likely additional examination finding is

A. bounding peripheral pulses

B. capillary refill of 4 seconds

C. hypotension

D. periorbital edema

E. tachycardia

Copyright © 2010 by the American Academy of Pediatrics page 34

Page 35: AAP PREP 2010

2010 PREP SA on CD-ROM

Preferred Response: ECritique: 14

Causes of dehydration in the pediatric patient include fluid losses from the gastrointestinaltract (vomiting or diarrhea) and less commonly, fluid losses due to excessive urinary production,as can occur in children who have urinary concentrating defects (due to renal dysplasia ordiuretics).

Total body water constitutes approximately 60% of the body weight in children. Two thirdsof the body fluid is contained within the intracellular compartment and one third within theextracellular compartment. The extracellular fluid compartment is divided further into theinterstitial compartment (75%) and the intravascular compartment (25%).

Dehydration affects both the intracellular and extracellular compartments, but most physicalsigns and symptoms result from a reduction of intravascular volume (IVV). Clinically, the IVV canbe expressed as the effective circulating blood volume (ECBV), which represents the volumeand pressure providing perfusion to the tissues. A patient who has a low IVV has a low ECBV,but a patient who has a high IVV or normal IVV also may have a low ECBV if cardiacdysfunction also is present.

Children who have gastroenteritis and mild-to-moderate dehydration, such as the childdescribed in the vignette, can maintain cardiac output and blood pressure by increasing heartrate and effective myocardial contractility. Therefore, tachycardia is the most common clinicalfinding in this setting. Hypotension is sign of severe dehydration. Other signs in more advanceddehydration (approaching 10%) are decreased skin turgor, bounding pulses, and decreasedcapillary refill. Periorbital edema is not expected in the clinical setting of mild dehydration.

References:

Armon K, Stephenson T, MacFaul R, Eccleston P, Werneke U. An evidence and consensusbased guideline for acute diarrhoea management. Arch Dis Child. 2001;85:132-142. Abstractavailable at: http://www.ncbi.nlm.nih.gov/pubmed/11466188

Boineau FG, Lewy JE. Estimation of parenteral fluid requirements. Pediatr Clin North Am.1990;37:257-264. Abstract available at: http://www.ncbi.nlm.nih.gov/pubmed/2184395

Gorelick MH, Shaw KN, Murphy KO. Validity and reliability of clinical signs in the diagnosis ofdehydration in children. Pediatrics. 1997;99;e6. Available at:http://pediatrics.aappublications.org/cgi/content/full/99/5/e6

Hill LL. Body composition, normal electrolyte concentrations, and the maintenance of normalvolume, tonicity, and acid-base metabolism. Pediatr Clin North Am. 1990;37:241-256. Abstractavailable at: http://www.ncbi.nlm.nih.gov/pubmed/2184394

Rose BD, Post TW. Regulation of the effective circulating volume. In: Clinical Physiology of Acid-base and Electrolyte Disorders. 5th ed. New York, NY: McGraw-Hill Medical Publishing Division;2001:258-284

Copyright © 2010 by the American Academy of Pediatrics page 35

Page 36: AAP PREP 2010

2010 PREP SA on CD-ROM

Question: 15

A 10-year-old boy presents to the clinic complaining of tongue and mouth itching within a fewminutes after eating apples. His mother states that he has not experienced these symptoms withother foods, but they occur every time he eats a fresh apple. He denies systemic symptoms,and the oral symptoms resolve within a few minutes. Other than allergic rhinitis in the springmonths, he is healthy.

Of the following, you are MOST likely to advise his mother that

A. allergy skin testing to fresh apples probably will have negative results

B. cooking the apple will not alter its allergenicity

C. her son should avoid eating all fruits

D. her son should avoid milk products

E. her son’s symptoms are related to his allergic rhinitis

Copyright © 2010 by the American Academy of Pediatrics page 36

Page 37: AAP PREP 2010

2010 PREP SA on CD-ROM

Preferred Response: ECritique: 15

The boy described in the vignette is exhibiting a common form of food allergy called foodpollen syndrome or oral allergy syndrome (OAS). OAS is seen in 30% to 40% of children whohave allergic rhinitis. Certain foods contain proteins that are similar to airborne allergens, andpatients who are allergic to an aeroallergen are at risk of developing reactions to the cross-reacting food protein (Item C15).

In most cases, symptoms are isolated to the oropharynx, where food comes in contact witha mucosal surface, and include lip, tongue, and oral mucosal pruritus; tingling; and occasionallyangioedema. Interestingly, because these food proteins are heat-labile, cooking the food (eg,apple pie) negates its antigenic properties. Although symptoms typically are mild, there arereports of severe reactions. In one recent review involving 1,361 patients who had OAS, 8.7%experienced systemic symptoms outside the gastrointestinal tract, 3% experienced symptomsother than oral symptoms, and 1.7% experienced anaphylactic shock.

Because OAS is relatively specific to particular cross-reacting food(s), patients do not needto avoid other fruits or vegetables to which they have not experienced reactions. Avoidance ofunrelated foods (eg, milk, eggs) is not recommended unless the history suggests a previousreaction. The decision to avoid causative foods can be based on the severity of reaction.Referral to an allergist typically is reserved for situations when skin testing is desired or if thechild has experienced systemic symptoms. Skin testing is performed using a commercial extractor the fresh fruit or vegetable. When using fresh food, the sensitivity of skin testing with ahistory of reproducible reactions is close to 90%, while the negative predictive value is morethan 90%. The skin prick device is pressed into the food and then pressed in the skin (so-called"prick-prick" skin test).

Other immunoglobulin (Ig) E food reactions include atopic dermatitis, eosinophilic esophagitis,and specific food allergy. In the United States, 85% of specific food allergies are due to egg,milk, wheat, soy, peanuts, tree nuts, fish, and shellfish. Most children who have IgE foodallergies react to only one or two causative foods, although children who have tree nut allergy,atopic dermatitis, and eosinophilic esophagitis often have IgE-mediated reactions to multiplefoods.

References:

Hyams JS. Food allergy (food hypersensitivity). In: Kliegman RM, Behrman RE, Jenson HB,Stanton BF, eds. Nelson Textbook of Pediatrics. 18th ed. Philadelphia, Pa: Saunders Elsevier;2007:1585-1586

Ma S, Sicherer S, Nowak-Wegrzyn A. A survey on the management of pollen-food syndrome inallergy practices. J Allergy Clin Immunol. 2003;112:784-788. Abstract available at:http://www.ncbi.nlm.nih.gov/pubmed/14564362

Sampson HA, Leung DYM. Adverse reactions to foods. In: Kliegman RM, Behrman RE, JensonHB, Stanton BF, eds. Nelson Textbook of Pediatrics. 18th ed. Philadelphia, Pa: SaundersElsevier; 2007:986-989

Copyright © 2010 by the American Academy of Pediatrics page 37

Page 38: AAP PREP 2010

2010 PREP SA on CD-ROM

Sicherer SH, Sampson HA. 9. Food allergy. J Allergy Clin Immunol. 2006;117(2 suppl mini-primer):S470-S475. Abstract available at: http://www.ncbi.nlm.nih.gov/pubmed/16455349

Copyright © 2010 by the American Academy of Pediatrics page 38

Page 39: AAP PREP 2010

2010 PREP SA on CD-ROM

Question: 16

A 5-year-old boy who has epilepsy and severe developmental delay is brought to theemergency department because of increasing somnolence over the past 12 hours. His motherreports that his activity level has decreased over the past 2 days, and this morning he wasdifficult to arouse. He has not been otherwise ill and has not had any seizures for the past 6months. His antiepileptic medications include phenobarbital and oxcarbazepine. The dosesrecently were increased. On physical examination, the boy is difficult to arouse and moans topainful stimulation. His heart rate is 70 beats/min, respiratory rate is 18 breaths/min, and bloodpressure is 80/50 mm Hg.

Of the following, the MOST likely additional abnormality on physical examination of this child is

A. dilated pupils

B. hyperactive bowel sounds

C. hyperreflexia

D. hypothermia

E. tremors

Copyright © 2010 by the American Academy of Pediatrics page 39

Page 40: AAP PREP 2010

2010 PREP SA on CD-ROM

Preferred Response: DCritique: 16

The patient described in the vignette is exhibiting signs and symptoms consistent with asedative-hypnotic overdose, including coma, bradycardia, bradypnea, and hypotension.Although most antiepileptic drugs can cause lethargy at high doses, barbiturates (eg,phenobarbital) are particularly sedating. The central nervous system depressant effects ofbarbiturates are primarily caused by drug action on the inhibitory neurotransmitter gammaaminobutyric acid (GABA). Barbiturates both increase GABA activity and directly stimulateGABA receptors. They also competitively inhibit glutamate, an excitatory neurotransmitter, frombinding to receptors.

Other physical findings that may be present in the setting of a sedative/hypnotic overdoseinclude hypothermia, hyporeflexia, hypoactive bowel sounds, and decreased muscular activity.Pupillary light reflex is slowed, but pupillary size usually is normal.

References:

Lafferty KA. Toxicity, barbiturate. eMedicine Specialties, Emergency Medicine, Toxicology.2008. Available at: http://www.emedicine.com/emerg/topic52.htm

Schachter SC. Pharmacology of antiepileptic drugs. UpToDate Online 16.3. 2008. Available at:http://www.utdol.com/online/content/topic.do?topicKey=epil_eeg/5220&selectedTitle=1~150&source=search_result630

Copyright © 2010 by the American Academy of Pediatrics page 40

Page 41: AAP PREP 2010

2010 PREP SA on CD-ROM

Question: 17

During a routine health supervision visit, the mother of a 2½ month-old male infant tells you thatthe baby has been experiencing bloating and flatulence. His diet consists of 5 to 6 oz of a cowmilk-based formula given five times per 24 hours. Because of frequent spitting-up, his motherrecently added rice cereal to each bottle. He has two to three seedy stools per day. On physicalexamination, the baby is alert and vigorous. His length and weight are tracking between the 50thand 75th percentiles. The infant’s mother asks you whether switching to a soy protein-basedformula will help her baby’s "gassiness."

Of the following, the MOST likely the cause of this infant’s symptoms is

A. cow milk protein allergy

B. excessive energy intake

C. incomplete starch digestion

D. lactose malabsorption

E. sucrase-isomaltase deficiency

Copyright © 2010 by the American Academy of Pediatrics page 41

Page 42: AAP PREP 2010

2010 PREP SA on CD-ROM

Preferred Response: CCritique: 17

The infant described in the vignette has been given formula thickened with rice cereal toameliorate spitting-up. Following the introduction of cereal, his mother has noted increased"gassiness." The most likely cause of this symptom is incomplete starch digestion.

Development of the digestive-absorptive function of the gastrointestinal tract is not completeat birth. The newborn can assimilate considerable amounts of complex carbohydrates throughhydrolysis by salivary gland amylase until pancreatic function and small intestinal intraluminalpancreatic amylase activity mature. Nevertheless, until pancreatic maturity is achieved, andcertainly in infants younger than 4 months of age, dietary starches may be hydrolyzedincompletely. As a result, increased amounts of undigested carbohydrate pass into the colon,where bacterial fermentation results in gas production that may cause the symptoms describedfor the infant in the vignette.

A diagnosis of cow milk protein allergy frequently is considered in the differential diagnosisof a variety of diverse gastrointestinal complaints. Symptoms that may be associated with cowmilk protein intolerance include diarrhea, failure to thrive, hypoproteinemia, hematochezia,anemia, and vomiting as well as other cutaneous and systemic manifestations of atopy. Therelationship between infantile colic and cow milk protein allergy remains highly controversial,particularly when fussiness or irritability is the sole complaint. For a thriving infant who developsvague gastrointestinal symptoms after the type of dietary changes described in the vignette,cow milk protein allergy should be considered only after ruling out other, more likely causes,such as incomplete digestion of complex carbohydrates.

It is unlikely that the infant described in the vignette has excessive energy intake becausehis weight gain is not excessive, and thickening of the formula does not appreciably add toenergy intake in an infant who is consuming 25 to 30 oz of formula per day.

Lactase concentrations reach mature values in the small intestine by the 36th week ofgestation in all healthy infants. Congenital or early-onset primary lactose intolerance is anextremely rare condition that is associated with severe diarrhea and inanition. It typicallypresents with voluminous diarrhea soon after the first feedings of human milk or cow milk-basedformula. During infancy and childhood, secondary lactase deficiency may occur as aconsequence of intestinal mucosal damage following a prolonged diarrheal illness, as a result ofother intestinal disorders (eg, celiac disease), or in association with malnutrition.

Sucrase-isomaltase (SI) deficiency is the most common congenital disaccharidasedeficiency. Diarrhea is a virtually universal symptom of SI deficiency and may be associatedwith poor weight gain. Symptoms usually appear in older infants following the introduction ofsucrose-containing foods, particularly fruits and juices. Infants who have SI deficiency also donot tolerate soy or protein hydrolysate formulas because both sucrose and glucose polymersare maldigested and malabsorbed.

References:

Craig WR, Hanlon-Dearman A, Sinclair C, Taback S, Moffatt M. Metoclopramide, thickenedfeedings, and positioning for gastro-oesophageal reflux in children under two years. CochraneDatabase Syst Rev. 2004;3:CD003502. Available at:

Copyright © 2010 by the American Academy of Pediatrics page 42

Page 43: AAP PREP 2010

2010 PREP SA on CD-ROM

http://www.mrw.interscience.wiley.com/cochrane/clsysrev/articles/CD003502/frame.html

Hall RT, Carroll RE. Infant feeding. Pediatr Rev. 2000; 21:191-200. Available at:http://pedsinreview.aappublications.org/cgi/content/full/21/6/191

Montes RG. Carbohydrate malabsorption. In: Rudolph CD, Rudolph AM, Hostetter MK, Lister G,Siegel NJ, eds. Rudolph’s Pediatrics. 21st ed. New York, NY: McGraw-Hill; 2003:1423-1427

Thomas DW, McGilligan K, Eisenberg LD, Lieberman HM, Rissman EM. Infantile colic and type ofmilk feeding. Am J Dis Child. 1987;141:451-453. Abstract available at:http://www.ncbi.nlm.nih.gov/pubmed/3494394

Copyright © 2010 by the American Academy of Pediatrics page 43

Page 44: AAP PREP 2010

2010 PREP SA on CD-ROM

Question: 18

You are asked by your pediatric resident on rounds why you remove umbilical artery cathetersin very low-birthweight infants in your nursery by postnatal day 7.

Of the following, the BEST reason for such removal is to prevent

A. anemia

B. hyperglycemia

C. hypotension

D. sepsis

E. thrombocytosis

Copyright © 2010 by the American Academy of Pediatrics page 44

Page 45: AAP PREP 2010

2010 PREP SA on CD-ROM

Preferred Response: DCritique: 18

Umbilical access to the arterial and venous circulation is a mainstay of early monitoring forthe critically ill newborn. The umbilical artery provides access to the central arterial circulationvia the inferior iliac artery, common iliac artery, and aorta. An umbilical arterial catheter (UAC) isindicated for only two uses: the frequent sampling of arterial blood for blood gas analysis in anewborn who has respiratory distress or the provision of inline transduced arterial bloodpressure monitoring. In either circumstance, the risks of placing and using a UAC must beconsidered.

Among the risks of using a UAC, infection is a significant concern when catheters remain inplace for more than 10 to 14 days. The UAC becomes colonized by commensal staphylococciwithin 24 hours of placement, and such colonization may lead to bloodstream infection due to theimmature immune system of very low-birthweight (VLBW) infants. The risk for infection isincreased when hyperalimentation fluid is administered through a UAC. Removal of the UAC bythe end of the first postnatal week reduces such risk significantly and generally is feasiblebecause most VLBW newborns have improved respiratory status by this time.

Additional risks of inserting and using a UAC include vascular spasm; thrombogenesis;either large-vessel occlusion or embolization of microthrombi in the distal arterial circulation; anddamage to the vascular endothelium or further injury leading to a mycotic aneurysm.Hypoglycemia may occur if the UAC is inserted in the thoracic aorta (between T6 and T10) andfluids that have a high glucose concentration are administered, inducing a hyperinsulinemicresponse by the pancreas.

Anemia, a common problem in critically ill VLBW newborns, does not result from UACinsertion or use, unless blood loss occurs when the line is accessed, which is rare.Hyperglycemia is not a complication of inserting or using a UAC. Hypotension is an indication toinsert a UAC and monitor blood pressure, not a complication. Thrombocytopenia, notthrombocytosis, may occur when a thrombus forms on the end of a UAC or elsewhere in thecirculation.

References:

Coleman MM, Spear ML, Finkelstein M, et al. Short-term use of umbilical artery catheters may notbe associated with increased risk for thrombosis. Pediatrics. 2004;113:770-774. Available at:http://pediatrics.aappublications.org/cgi/content/full/113/4/770

Hermansen MC, Harmansen MG. Intravascular catheter complications in the neonatal intensivecare unit. Clin Perinatol. 2005;32:141-156. Abstract available at:http://www.ncbi.nlm.nih.gov/pubmed/15777826

O’Grady NP, Alexander M, Dellinger EP, et al. Guidelines for the prevention of intravascularcatheter-related infections. The Hospital Infection Control Practices Advisory Committee, Centersfor Disease Control and Prevention. U.S. Pediatrics. 2002;110:e51-e75. Available at:http://pediatrics.aappublications.org/cgi/content/full/110/5/e51

Copyright © 2010 by the American Academy of Pediatrics page 45

Page 46: AAP PREP 2010

2010 PREP SA on CD-ROM

Rodriguez RJ, Martin RJ, Fanaroff AA. Respiratory distress syndrome and its management. In:Martin RJ, Fanaroff AA, Walsh MC, eds. Fanaroff and Martin's Neonatal-Perinatal Medicine:Diseases of the Fetus and Infant. 8th ed. Philadelphia, Pa: Mosby Elsevier; 2006:1097-1107

Wortham BM, Galtatzes CG, Rais-Bahrami K. Umbilical artery catheterization. In: MacDonald MG,Ramasethu J, eds. Atlas of Procedures in Neonatology. 4th ed. Philadelphia, Pa: LippincottWilliams & Wilkins; 2007:157-176

Copyright © 2010 by the American Academy of Pediatrics page 46

Page 47: AAP PREP 2010

2010 PREP SA on CD-ROM

Question: 19

You are evaluating a new patient in your office, a 10-year-old boy who has no significant pastmedical history or illnesses. His weight and height are both at the 75th percentile. On physicalexamination, you notice a mild pectus excavatum but no cardiopulmonary abnormalities. Whenyou ask him about it, he replies, "I’ve always had it, and it doesn’t bother me."

Of the following, the MOST appropriate next step in the management of this problem is

A. electrocardiography

B. exercise stress testing

C. psychological evaluation

D. reassurance

E. surgery consultation

Copyright © 2010 by the American Academy of Pediatrics page 47

Page 48: AAP PREP 2010

2010 PREP SA on CD-ROM

Preferred Response: DCritique: 19

Pectus excavatum is a skeletal abnormality of the chest wall characterized by concavity ofthe anterior chest. It may occur in isolation or can be associated with other disorders, such asMarfan syndrome and Ehlers-Danlos syndrome, and it is more common in boys. The deformityusually is present in infancy, although symptoms do not present until later in life. Many childrenare asymptomatic and are not concerned about the cosmetic appearance of the chest, but somechildren may experience psychological distress about their appearance. Some childrenexperience symptoms such as dyspnea with exertion, chest or rib pain, and decreasedexercise tolerance. Pulmonary function abnormalities, when present, include an obstructivepattern and, less commonly, a restrictive pattern. Cardiac dysfunction due to impaired strokevolume with exercise has been described and is more common in older patients.

Because the boy described in the vignette does not complain of any symptoms and is notpsychologically affected by the cosmetic appearance of his chest, only reassurance andobservation are necessary. Lateral chest radiographs and chest computed tomography scancan demonstrate the deformity, but such tests generally are not necessary in asymptomaticpatients. Similarly, electrocardiography and exercise stress testing are not indicated unlessexercise intolerance or dyspnea upon exertion is present. Psychological evaluation could beconsidered for patients who are very concerned about the cosmetic appearance, andcorrective surgery should be considered for those who have severe psychological distress.Pulmonary function, if impaired, often is not improved with surgical correction.

References:

Aronson DC, Bosgraaf RP, Merz EM, van Steenwijk RP, van Aalderen WM, van Baren R. Lungfunction after the minimal invasive pectus excavatum repair (Nuss procedure). World J Surg.2007;31:1518-1522. Available at:http://www.pubmedcentral.nih.gov/articlerender.fcgi?tool=pubmed&pubmedid=17534548

Boas SR. Skeletal diseases influencing pulmonary function. In: Kliegman RM, Behrman RE,Jenson HB, Stanton BF, eds. Nelson Textbook of Pediatrics. 18th ed. Philadelphia, Pa: SaundersElsevier; 2007:1841-1844

Koumbourlis AC, Stolar CJ. Lung growth and function in children and adolescents with idiopathicpectus excavatum. Pediatr Pulmonol. 2004;38:339-343. Abstract available at:http://www.ncbi.nlm.nih.gov/pubmed/15334513

Rowland T, Moriarty K, Banever G. Effect of pectus excavatum deformity on cardiorespiratoryfitness in adolescent boys. Arch Pediatr Adolesc Med. 2005;159:1069-1073. Available at:http://archpedi.ama-assn.org/cgi/content/full/159/11/1069

Copyright © 2010 by the American Academy of Pediatrics page 48

Page 49: AAP PREP 2010

2010 PREP SA on CD-ROM

Question: 20

A nurse practitioner in your clinic has asked you to review the chart of an 11-year-old boyreferred from school for evaluation of acanthosis nigricans.

Of the following, the physical examination parameter that is MOST likely to be useful in predictingcomorbidities in this patient is

A. blood pressure at the 75th percentile for height

B. body mass index at the 95th percentile for age

C. sum of triceps and subscapular skinfold thickness greater than 90% for age

D. weight at the 50th percentile and height at the 25th percentile for age

E. weight at the 95th percentile and height at the 75th percentile for age

Copyright © 2010 by the American Academy of Pediatrics page 49

Page 50: AAP PREP 2010

2010 PREP SA on CD-ROM

Preferred Response: BCritique: 20

Because acanthosis nigricans is associated with obesity, the young man in the vignette islikely to be overweight and at risk for numerous comorbidities, including hypertension,hyperlipidemia, and the metabolic syndrome.

Although several methods have been used to assess obesity, body mass index (BMI),

defined as weight in kilograms divided by height in meters squared (kg/m2), has become thestandard measure for children, adolescents, and adults. The correlation of BMI with measures ofadiposity is excellent in adults, but slightly less reliable for children, whose BMIs change withage during childhood and adolescence. Of note, boys have less body fat than do girls at thesame BMI. Total body fat correlates with sexual maturational level more than age because assexual maturity progresses, body fat increases. Those who have a higher waist-to-hip ratio atthe same BMI have more body fat.

In 2000, the Centers for Disease Control and Prevention (CDC) released new growthcurves, including BMI curves for boys and girls from ages 2 to 20 years(www.cdc.gov/growthcharts). Obesity has been defined as a BMI greater than the 95thpercentile for age on the 2000 CDC growth curves. Children whose BMIs fall between the 85thand 95th percentiles for age are considered overweight. Children whose BMIs are greater thanthe 99th percentile are defined as severely obese.

Weight-for-height is a less accurate measure and is subject to misinterpretation ofoverweight versus obesity. In the estimation of risk for comorbid conditions, including diabetesor insulin insensitivity, hypertension, hyperlipidemia, or metabolic syndrome, the BMI nowrepresents a standardized measurement. However, high weight-for-height must be used toassess obesity in children younger than 2 years of age for whom BMI norms are not available. Itis unclear whether childhood obesity is an independent risk factor for hyperlipidemia andatherosclerotic heart disease. It is known that the higher the BMI, the greater the risk of leftventricular wall thickness in children who have hypertension and that BMI correlates witharterial wall thickness. Skinfold thickness measurements are accurate in estimating total body fatbut are not currently the standard for determining risk for comorbidities of obesity. In addition,they may be difficult to obtain in the primary care office setting due to issues with interobserverreliability, training of staff in the technique, and the expense of the calipers ($200). However,skinfold thickness measurements may be useful in very athletic, muscular adolescent maleswhose BMIs are artificially elevated by high muscle mass.

All children older than 3 years of age should have blood pressures measured at each healthsupervision visit; children who are overweight should have blood pressures monitored morefrequently. A blood pressure that is at the 75th percentile is within the normal range and lesslikely to be associated with comorbidities than BMI at the 95th percentile.

References:

Barlow SE and the Expert Committee. Expert Committee recommendations regarding theprevention, assessment, and treatment of child and adolescent overweight and obesity:summary report. Pediatrics. 2007;120 (suppl):S164-S192. Available at:http://pediatrics.aappublications.org/cgi/content/full/120/Supplement_4/S164

Copyright © 2010 by the American Academy of Pediatrics page 50

Page 51: AAP PREP 2010

2010 PREP SA on CD-ROM

Feld LG, Corey H. Hypertension in childhood. Pediatr Rev. 2007;28:283-298. Available at:http://pedsinreview.aappublications.org/cgi/content/full/28/8/283

Freedman DS, Kettel Khan L, Mei A, Dietz WH, Srinivasan SR, Berenson GS. Relation ofchildhood height to obesity among adults: the Bogalusa Heart Study. Pediatrics. 2002;109:e23.Available at: http://pediatrics.aappublications.org/cgi/content/full/109/2/e23

Katzmarzyk PT, Srinivasan SR, Chen W, Malina RM, Bouchard C, Berenson GS. Body massindex, waist circumference, and clustering of cardiovascular disease risk factors in a biracialsample of children and adolescents. Pediatrics. 2004;114:e198-e205. Available at:http://pediatrics.aappublications.org/cgi/content/full/114/2/e198

Schneider MB, Brill SR. Obesity in children and adolescents. Pediatr Rev. 2005;26:155-162.Available at: http://pedsinreview.aappublications.org/cgi/content/full/26/5/155

Sorof J, Daniels S. Obesity hypertension in children: a problem of epidemic proportion.Hypertension. 2002;40:441-447. Available at:http://hyper.ahajournals.org/cgi/content/full/40/4/441

Summerbell CD, Waters E, Edmunds LD, Kelly S, Brown T, Campbell KJ. Interventions forpreventing obesity in children. Cochrane Database Syst Rev. 2005;3:CD001871

Whitlock EP, O’Connor EA, Williams SB, Beil TL, Lutz KKW. Effectiveness of Weight ManagementPrograms in Children and Adolescents. Evidence Report/Technology Assessment No. 170.AHRQ Publication No. 08-E014. Rockville, Md: Agency for Healthcare Research and Quality, U.S.Department of Health and Human Services; 2008. Available at:http://www.ncbi.nlm.nih.gov/books/bv.fcgi?rid=hstat1b.chapter.139937

Copyright © 2010 by the American Academy of Pediatrics page 51

Page 52: AAP PREP 2010

2010 PREP SA on CD-ROM

Question: 21

You are demonstrating digital clubbing in a teenage patient at the physical diagnosis course youteach at your local medical school. One of the students asks what condition would predisposean adolescent to this finding.

Of the following, the MOST likely predisposing condition is

A. hypoplastic left heart syndrome after completion of a Fontan procedure

B. pulmonary atresia associated with unrepaired ventricular septal defect

C. tetralogy of Fallot that was repaired in infancy

D. transposition of the great arteries that was repaired at 1 week of age

E. unrepaired atrial septal defect

Copyright © 2010 by the American Academy of Pediatrics page 52

Page 53: AAP PREP 2010

2010 PREP SA on CD-ROM

Preferred Response: BCritique: 21

Hypertrophic pulmonary osteoarthropathy, better known as clubbing of the digits (ItemC21A), can be seen in a variety of entities, the most common of which is cyanotic heart disease,such as pulmonary atresia with an unrepaired ventricular septal defect and collateral pulmonaryblood flow. Other conditions in which it has been described are chronic lung disease, biliarycirrhosis, and infective endocarditis. Finally, clubbing of the digits can be a normal variant,occurring as a familial trait.

Clubbing initially becomes apparent when the angle between the proximal nail and the softtissue of the digit is obliterated or filled in, which can be demonstrated by having the patientplace the distal phalangeal joints together in a "mirrorlike" fashion. For the individual who has noclubbing, the maneuver creates a diamond-shaped space (Item C21B). In contrast, the patientwho has clubbing demonstrates complete occlusion of this space because all aspects of the nailbed and distal soft tissue directly oppose one another. This finding is referred to as a positiveSchamroth sign.

Hypoplastic left heart syndrome is an obstruction of systemic blood flow that typicallypresents in the first several days after birth as the ductus arteriosus and, thus, the route ofsystemic blood flow constricts. Palliation of this complex disorder involves difficult surgicalanastomoses, including the creation of systemic blood flow from the right ventricle through thenative pulmonary valve and artery that has been brought to the aorta. The pulmonary blood flowis delivered through an aortic-pulmonary artery shunt (eg, Blalock-Taussig shunt) or directlyfrom the right ventricle to the pulmonary artery (Sano shunt). The second stage of the palliationusually occurs about midway through the first postnatal year and consists of a superior venacava-to-pulmonary artery passive shunt such as the Glenn operation. Both the first and secondstages of the palliation result in oxygen saturations in the 70% and 80% range, but uponcompletion of the third stage, which brings the inferior vena cava to the pulmonary arteries,saturations reach the 90% range. Clubbing is not expected in affected patients. Similarly,patients who have tetralogy of Fallot may present with or develop cyanosis, but upon completesurgical repair, their oxygen saturations are normal. The same is true of the child who hasrepaired transposition of the great arteries. The atrial septal defect typically is a left-to-rightshunt associated with normal oxygen saturation.

References:

Haddad GG, Green TP. Diagnostic approach to respiratory disorders. In: Kliegman RM, BehrmanRE, Jenson HB, Stanton BF, eds. Nelson Textbook of Pediatrics. 18th Ed. Philadelphia, Pa:Saunders Elsevier; 2007:1731-1732

Karnath B. Digital clubbing: sign of underlying disease. Hospital Physician. 2003;39(9):25-27.Available at: http://www.turner-white.com/pdf/hp_sep03_club.pdf

Copyright © 2010 by the American Academy of Pediatrics page 53

Page 54: AAP PREP 2010

2010 PREP SA on CD-ROM

Question: 22

A 14-year-old boy presents to your office because the side of his face is drooping. His motherstates that he complained yesterday of decreased food taste. Today, while at school, he couldnot use the microscope in science class because he couldn't close his left eye, and his teachernoted that his smile was crooked. Physical examination reveals no abnormalities and no vesiclesin his ears. Mental status on neurologic examination is normal, pupil responses are normal,extraocular movements are full, and there is no nystagmus or reported double vision. He isunable to close his left eye or raise his left eyebrow, has decreased left-side nasolabial folds,and cannot close his mouth to puff out his cheeks (Item Q22). His palate and tongue movementsare normal. Motor examination reveals normal proximal and distal strength in both arms andnormal regular and tandem gait.

Of the following, the MOST appropriate initial diagnostic procedure is

A. blood test for antistreptococcal antibodies

B. brain magnetic resonance imaging

C. edrophonium (Tensilon®) test

D. no further testing

E. noncontrast head computed tomography scan

Copyright © 2010 by the American Academy of Pediatrics page 54

Page 55: AAP PREP 2010

2010 PREP SA on CD-ROM

Preferred Response: DCritique: 22

Acute focal or generalized weakness is a medical emergency requiring a systematic historyand neurologic examination to localize the problem. For acute focal weakness, the problem canlocalize to the brain, brainstem, spinal cord, anterior horn cell, root, nerve, junction, or muscle.Often, the physical examination can localize the problem to one of these levels. In the case ofacute unilateral facial weakness, as described for the boy in the vignette, the typical differentialdiagnosis is acute facial nerve palsy (ie, Bell palsy) or a more rostral disease process of thebrainstem or brain (cerebrum, motor cortex) such as a stroke. The key diagnostic point for facialweakness is whether the weakness involves the entire side of the face or the face below theforehead. A 7th nerve palsy affects all the innervated muscles, weakening or paralyzing theentire hemi-face from forehead to chin. A lesion above the facial nerve nucleus typicallyweakens the face below the forehead.

The boy described in the vignette has full left-sided facial weakness, including the musclesin his forehead. Such findings localize to the facial nerve, and in this clinical setting,neuroimaging is not revealing. Accordingly, no further testing is required. When the examinationlocalizes a problem involving facial weakness to the brain or brainstem, brain magneticresonance imaging (MRI) or, if MRI is not available quickly, noncontrast head computedtomography should be obtained.

Facial weakness due to an acute brain process, such as a left middle cerebral artery stroke,usually presents with involvement of both the contralateral right face and the right hand. Thebrainstem, specifically the pons, is the source of the facial nerve, and brainstem diseases canproduce full hemi-facial weakness. However, due to the close proximity of other brainstemnuclei, a brainstem lesion affecting the left face also should affect other functions, including theleft 6th nerve, which abducts the left eye. Often, sensory and motor findings on the oppositeside of the body, the so-called "crossed signs," indicate brainstem disease.

The edrophonium/Tensilon® test involves administration of this acetylcholinesterase inhibitorto increase acetylcholine at the neuromuscular junction and reverse weakness. The test is usedfor diagnosis of myasthenia gravis. Myasthenia gravis typically produces bilateral fatiguingweakness, particularly ptosis, as well as weakness in other cranial nerves or generalizedweakness.

Facial nerve palsy can be caused by a variety of infectious agents, but no specificdiagnostic testing is indicated in most cases. However, in regions where Lyme disease isendemic or exposure is possible, testing for Lyme disease may be indicated. Assessment ofantistreptococcal antibodies is not helpful because streptococci do not cause facial nerve palsy.

The American Academy of Neurology practice parameter states that oral steroids probablyare beneficial and acyclovir possibly is beneficial for treatment of facial nerve palsy. Pediatricstudies and reviews have concluded that evidence is insufficient to recommend steroids forchildren. However, many clinicians recommend administering a short course of oral prednisonefor Bell palsy.

References:

Ashtekar CS, Joishy M, Joshi R. Best evidence topic report. Do we need to give steroids in

Copyright © 2010 by the American Academy of Pediatrics page 55

Page 56: AAP PREP 2010

2010 PREP SA on CD-ROM

children with Bell's palsy? Emerg Med J. 2005;22:505-507. Abstract available at:http://www.ncbi.nlm.nih.gov/pubmed/15983089

Grogan PM, Gronseth GS. Practice parameter: steroids, acyclovir, and surgery for Bell's palsy(an evidence-based review): report of the Quality Standards Subcommittee of the AmericanAcademy of Neurology. Neurology. 2001;56:830-836. Available at:http://www.neurology.org/cgi/content/full/56/7/830

Sarnat HB. Bell palsy. In: Kliegman RM, Behrman RE, Jenson HB, Stanton BF, eds. NelsonTextbook of Pediatrics. 18th ed. Philadelphia, Pa: Saunders Elsevier; 2007:2566-2568

Singhi P, Jain V. Bell's palsy in children. Semin Pediatr Neurol. 2003;10:289-297. Abstractavailable at: http://www.ncbi.nlm.nih.gov/pubmed/14992461

Copyright © 2010 by the American Academy of Pediatrics page 56

Page 57: AAP PREP 2010

2010 PREP SA on CD-ROM

Question: 23

A 4-year-old girl in your practice has moderate persistent asthma. The child’s mother tells youthat she and her husband would like a big family, and she asks if there is anything they can doto reduce the risk of asthma to their future children.

Of the following, increased asthma risk in the offspring is MOST associated with prenatalexposure to maternal

A. alcohol binging

B. barbiturate use

C. benzene exposure

D. cigarette smoking

E. cocaine sniffing

Copyright © 2010 by the American Academy of Pediatrics page 57

Page 58: AAP PREP 2010

2010 PREP SA on CD-ROM

Preferred Response: DCritique: 23

A growing body of epidemiologic data show various negative outcomes for individualsexposed prenatally to maternal cigarette smoking. The bulk of the information comes from birthregistries or retrospective studies, which show associations between maternal cigarettesmoking and miscarriage, fetal growth restriction, preterm birth, and low birthweight. Astatistically significant increase in cleft lip +/- cleft palate was shown in at least one study thathad large sample sizes, but maternal cigarette smoking is not otherwise reliably associated withcongenital anomalies. Strong evidence demonstrates an increased risk for sudden infant deathsyndrome among exposed children, possibly due to altered autonomic function. Multiple studiesexamining the association between in utero cigarette smoke exposure (without postnatalenvironmental tobacco smoke exposure) and childhood respiratory disorders demonstrate asignificantly increased prevalence of physician-diagnosed asthma and wheezing.

Alcohol binging places the exposed embryo/fetus at greater risk for fetal alcohol spectrumdisorders than daily moderate alcohol intake. However, the incidence of asthma is not increasedin these individuals.

Prenatal barbiturate exposure is associated with birth defects in 10% to 20% of exposedindividuals. Anomalies include midface hypoplasia, cleft lip +/- cleft palate, and heart defects aswell as pre- and postnatal growth deficiency. Asthma risk is not increased.

Prenatal exposure to benzene occurs most commonly in a setting where industrial solventsare used. Benzene has not been found to increase the risk of congenital anomalies in exposedembryos/fetuses, and it is not associated with an increased asthma risk.

Fetal and childhood abnormalities associated with maternal cocaine use during pregnancyare difficult to study because cocaine often is used concomitantly with tobacco, other streetdrugs, and alcohol. An increased incidence of placental abruption and sudden infant deathsyndrome has been associated with prenatal cocaine exposure, and genitourinary anomaliesand limb amputations have been reported. Cocaine alone, however, is not associated withincreased asthma risk in prenatally exposed individuals.

References:

Benzene. Reprotox®. Available for subscription at: http://www.reprotox.org

Benzene. Teris. Available for subscription at: http://depts.Washington.edu/terisweb/teris/

Cigarette smoking (tobacco). Teris. Available for subscription at:http://depts.Washington.edu/terisweb/teris/

Cigarette smoking. Reprotox®. Available for subscription at: http://www.reprotox.org

Cocaine. Reprotox®. Available for subscription at: http://www.reprotox.org

Cocaine. Teris. Available for subscription at: http://depts.Washington.edu/terisweb/teris/

Copyright © 2010 by the American Academy of Pediatrics page 58

Page 59: AAP PREP 2010

2010 PREP SA on CD-ROM

Higgins S. Smoking in pregnancy. Curr Opin Obstet Gynecol. 2002;14:145-151. Abstractavailable at: http://www.ncbi.nlm.nih.gov/pubmed/11914691

Hoyme HE, May PA, Kalberg WO, et al. A practical clinical approach to diagnosis of fetal alcoholspectrum disorders: clarification of the 1996 Institute of Medicine criteria. Pediatrics.2005;115:39-47. Available at: http://pediatrics.aappublications.org/cgi/content/full/115/1/39

Phenobarbital. Reprotox®. Available for subscription at: http://www.reprotox.org

Phenobarbital. Teris. Available for subscription at: http://depts.Washington.edu/terisweb/teris/

Copyright © 2010 by the American Academy of Pediatrics page 59

Page 60: AAP PREP 2010

2010 PREP SA on CD-ROM

Question: 24

You are examining an asymptomatic adolescent male for his annual health supervision visit.Genital examination reveals genitalia at Sexual Maturity Rating 5 and a nontender mass in his leftscrotum that extends from the inguinal canal to the upper pole of the testis (Item Q24). The massdecreases in size when he lies down.

Of the following, the MOST likely diagnosis is

A. direct inguinal hernia

B. hydrocele

C. spermatocele

D. testicular tumor

E. varicocele

Copyright © 2010 by the American Academy of Pediatrics page 60

Page 61: AAP PREP 2010

2010 PREP SA on CD-ROM

Preferred Response: ECritique: 24

Varicosity of the pampiniform plexus of veins in the scrotum usually develops slowly andmay be asymptomatic or cause a heavy sensation in the scrotum. Varicoceles are more commonin males ages 15 to 25 years and are seen most often on the left side of the scrotum. This isattributed to increased pressure in the left spermatic vein as it drains directly into the left renalvein. In contrast, the right spermatic vein drains into the inferior vena cava in the pelvis. Onphysical examination, varicoceles vary in size, can extend from the testis to the inguinal canal,and feel like a "bag of worms" (Item C24). As described for the boy in the vignette, they increasein size with standing or a valsalva maneuver and reduce spontaneously when the patient liesdown. If a varicocele develops suddenly, does not reduce in the supine position, or is on theright side, a cause for obstruction of the spermatic vein should be sought.

Inguinal hernias usually present as painless, intermittent groin masses that appear withstraining and usually reduce spontaneously. They may increase in size and become obstructed,resulting in a painful scrotal mass. A hydrocele represents fluid within the tunica vaginalis thatsurrounds the testis rather than being distinct from it. A hydrocele transilluminates, whichdistinguishes it from a solid mass. Spermatoceles are cystic lesions within the spermatic cordthat are above and distinct from the testis and transilluminate. Testicular tumors are painlesssolid masses within the testicle that do not transilluminate.

References:

Adelman WP, Joffe A. Consultation with the specialist: testicular masses/cancer. Pediatr Rev.2005;26:341-344. Available at: http://pedsinreview.aappublications.org/cgi/content/full/26/9/341

Diamond DA. Adolescent varicocele. Curr Opin Urol. 2007;17:263-267. Abstract available at:http://www.ncbi.nlm.nih.gov/pubmed/17558270

Kass EJ. Adolescent varicocele. Pediatr Clin North Am. 2001;48:1559-1569. Abstract availableat: http://www.ncbi.nlm.nih.gov/pubmed/11732130

Kumanov P, Robeva RN, Tomova A. Adolescent varicocele: who is at risk? Pediatrics.2008;121:e53-e57. Available at: http://pediatrics.aappublications.org/cgi/content/full/121/1/e53

Copyright © 2010 by the American Academy of Pediatrics page 61

Page 62: AAP PREP 2010

2010 PREP SA on CD-ROM

Question: 25

You are called to evaluate a 16-year-old girl who was the unrestrained driver in a motor vehiclecrash. She reportedly hit the steering wheel but maintained consciousness and now complainsof chest pain. On physical examination, she has a heart rate of 110 beats/min, blood pressure of120/80 mm Hg, and a respiratory rate of 30 breaths/min. Her oxygen saturation by pulseoximetry is 85% while receiving 8 L/min oxygen via a nonrebreathing face mask. As youobserve her breathing pattern, you notice that her right chest moves inward with eachinspiration. Her chest radiograph demonstrates several significant right-sided findings, includingmultiple fractures of the 7th, 8th, and 9th ribs; a small apical pneumothorax; and a diffuseopacification consistent with a pulmonary contusion.

Of the following, the MOST appropriate next step is

A. administration of 10 mL/kg of 0.9% normal saline

B. administration of furosemide

C. endotracheal intubation

D. observation

E. surgical fixation of the rib fractures

Copyright © 2010 by the American Academy of Pediatrics page 62

Page 63: AAP PREP 2010

2010 PREP SA on CD-ROM

Preferred Response: CCritique: 25

The pulmonary contusion, multiple rib fractures, and pneumothorax described for the girl inthe vignette are consistent with significant thoracic trauma. On physical examination, she has aparadoxic respiratory pattern, as evidenced by inward movement of her right chest duringinspiration. This is a result of her multiple rib fractures producing an isolated, unstable area thatdoes not move in the appropriate direction during respiratory efforts, the so-called "flail chest."The diagnosis of flail chest is primarily clinical. It is less common in young children due toelasticity of the thoracic skeleton, which results in traumatic forces being transmitted morereadily to internal organs.

Respiratory distress, as described for this patient, must be addressed urgently. Initialtreatment of flail chest generally consists of intubation, mechanical ventilation, and adequate paincontrol. The associated hypoxemia is due to the underlying pulmonary contusion and is unlikelyto improve with a 10-mL/kg fluid bolus, administration of a diuretic, or simple observation.Surgical fixation of the flail segment 24 to 36 hours after injury is a relatively new technique thathas shown initial promise in improving pulmonary function and cosmetic appearance, but it is notpart of initial management.

Thoracic injuries account for 5% to 8% of pediatric trauma cases, with mortality rates of 7%to 15%, second only to the mortality of head injuries. Blunt trauma to the chest can produce avariety of injuries, including cardiac tamponade, aortic arch dissection, rib fractures, pulmonarycontusion, hemo- and pneumothorax, and diaphragmatic rupture. Patients in whom thoracicinjuries are suspected due to the mechanism of injury (eg, motor vehicle or bicycle crashes,falls) or signs and symptoms (eg, respiratory distress, failure to respond to supplementaloxygen, visible wounds to the chest, decreased breath sounds, hyperresonance on chestpercussion, distended neck veins) should undergo radiography of the chest and cervical spine.Focused assessment sonography for trauma (FAST) may be useful to rule out thoracichemorrhage and cardiac tamponade. Computed tomography scan of the chest should beperformed after initial stabilization.

References:

Dayan PS, Klein BL. Acute care of the multiple trauma victim. In Kliegman RM, Behrman RE,Jenson HB, Stanton BF, eds. Nelson Textbook of Pediatrics. 18th ed. Philadelphia, Pa: SaundersElsevier; 2007:431-436

Herrera P, Langer JC. Thoracic trauma in children. In: Mikrogianakis A, Valani R, Cheng A, eds.The Hospital for Sick Children Manual of Pediatric Trauma. Philadelphia, Pa: Lippincott, Williams& Wilkins. 2008:131-144

Copyright © 2010 by the American Academy of Pediatrics page 63

Page 64: AAP PREP 2010

2010 PREP SA on CD-ROM

Question: 26

An anxious mother calls your office to tell you that her 5-year-old daughter has a "high bloodsugar." The family had been visiting the child’s grandparents, and her maternal grandmother,who has type 2 diabetes mellitus, used her glucose meter to measure her granddaughter’s bloodglucose about 90 minutes after the child had a piece of cake and a glass of apple juice. Themeter reading was 253 mg/dL (14.0 mmol/L). The girl has been entirely asymptomatic, withoutnocturia, polyuria, or weight loss. Her mother has been trying to control her daughter’s weightlately, but the girl has a large appetite, and it was hard to control her eating at her grandparents’house. There is no history of type 1 diabetes mellitus in the family.

Of the following, the MOST likely explanation for the child’s glucose reading is

A. a normal blood glucose value 90 minutes after a high-carbohydrate meal

B. maturity onset diabetes of the young

C. postprandial glucose intolerance

D. residual glucose on the fingers

E. type 2 diabetes

Copyright © 2010 by the American Academy of Pediatrics page 64

Page 65: AAP PREP 2010

2010 PREP SA on CD-ROM

Preferred Response: DCritique: 26

Assessment of whole blood glucose values using glucose meters is a useful technique formanagement of diabetes, but methodologic errors occur. One of the most common iscontamination of the fingers with glucose-containing food, which is the most likely explanationfor the reading described for the child in the vignette. Foods such as juices and somecommercial baked goods may contain appreciable amounts of glucose. Causes of falsely lowglucose readings include cold temperature (outdoor winter use of a meter) and outdated teststrips. Causes of falsely low or high glucose readings include failure to calibrate the strips, ifthis is required by the meter, by setting the appropriate code. It is also possible to manipulateblood glucose readings willfully by using diluted blood or testing using a standard control solutionrather than blood.

A healthy little girl should not have a blood glucose reading of 253 mg/dL (14.0 mmol/L) 90minutes after eating. Random glucose concentrations greater than 200 mg/dL (11.1 mmol/L) ontwo separate occasions are diagnostic of diabetes. A child’s reading should not exceed therenal threshold for glucose (180 mg/dL [10.0 mmol/L]) after oral intake for more than a fewminutes.

Maturity onset diabetes of the young (MODY) is a rare autosomal dominant disorder resultingfrom mutations in at least six different genes, most of which are transcription factors that controlthe release of insulin from the beta cell. This group of disorders comprises fewer than 5% of alldiagnoses of diabetes in childhood. Postprandial glucose intolerance could be a marker ofdiabetes or impaired glucose tolerance, but the prevalence of diabetes in 6-year-old children isless than 1 per 1,000. Type 2 diabetes, which results from insulin resistance and a diminishedcapacity for insulin release, typically is seen in adults. This disorder is increasingly recognized inobese children and adolescents, but it would be unusual in a normal weight 6-year-old child.

References:

American Diabetes Association. Diagnosis and classification of diabetes mellitus. DiabetesCare. 2008;31(suppl 1):S55-S60. Available at:http://care.diabetesjournals.org/cgi/content/full/31/Supplement_1/S55

Craig ME, Hattersley A, Donahue K; International Society for Pediatric and Adolescent Diabetes.ISPAD clinical practice consensus guidelines 2006-2007: definition, epidemiology andclassification. Pediatr Diabetes. 2006;7:343-351

Rewers M, Pihoker C, Donaghue K, Hanas R, Swift P, Klingensmith GJ; International Society forPediatric and Adolescent Diabetes (ISBAD). Assessment and monitoring of glycemic control inchildren and adolescents with diabetes. Pediatr Diabetes. 2007;8:408-418

Copyright © 2010 by the American Academy of Pediatrics page 65

Page 66: AAP PREP 2010

2010 PREP SA on CD-ROM

Question: 27

A 13-year-old girl presents with the sudden loss of sight following the violent death of hermother. On physical examination, her pupils are round and equal and constrict briskly to light.When instructed to do so, she is unable to touch the examiner’s hand held in front of her. Thereare no other neurologic findings on examination. Results of head magnetic resonance imagingare normal, and a dilated ophthalmologic evaluation reveals no abnormalities.

Of the following, the MOST likely diagnosis is

A. body dysmorphic disorder

B. conversion disorder

C. hypochondriasis

D. malingering

E. somatic delusions

Copyright © 2010 by the American Academy of Pediatrics page 66

Page 67: AAP PREP 2010

2010 PREP SA on CD-ROM

Preferred Response: BCritique: 27

The girl described in the vignette is displaying a conversion disorder, which simulatesdisease (mostly acute) and is monosymptomatic. Conversion disorder occurs more frequently infemales and is most common in adolescents and young adults. Symptoms are not compatiblewith physiologic mechanisms or anatomy.

Hypochondriasis is concern about a disease or preoccupation with illness. It often occurs inpersons 20 to 30 years of age, equally in males and females, and in individuals who have hadprevious physical disease.

Malingering is diagnosed when an individual presents with false or exaggerated physical orpsychological symptoms. The individual has a motive either to avoid asituation/punishment/responsibility, to obtain compensation, or to retaliate. The presence of adefined goal distinguishes this from other factitious disorders.

The person who has a strong belief or fear that he or she is unattractive or even repulsivedespite having a normal or near-normal appearance is experiencing body dysmorphic disorder.Neither compliments nor reassurance alleviate the person’s fear.

Somatic delusions encompass the belief that something is physically wrong with theindividual. The delusion may involve a medical condition or illness or a perceived deformity, suchas a belief that the person’s heart is melting. This condition differs from hypochondriasis, whichinvolves excessive worries about health that stem from nonreality-based interpretations ofspecific physical signs as being not normal.

References:

Sadock BJ, Sadock VA. Additional conditions that may be a focus of clinical attention. In: Kaplanand Sadock’s Synopsis of Psychiatry: Behavioral Sciences/Clinical Psychiatry. 9th ed.Philadelphia, Pa: Lippincott Williams & Wilkins; 2003:894-900

Sadock BJ, Sadock VA. Somatoform disorders. In: Kaplan and Sadock’s Synopsis of Psychiatry:Behavioral Sciences/Clinical Psychiatry. 9th ed. Philadelphia, Pa: Lippincott Williams & Wilkins;2003:643-660

Copyright © 2010 by the American Academy of Pediatrics page 67

Page 68: AAP PREP 2010

2010 PREP SA on CD-ROM

Question: 28

A 12-year-old boy who has acute lymphoblastic leukemia (ALL) is undergoing reinductionchemotherapy and has an indwelling Broviac catheter. He has received multiple courses ofantibiotics for episodes of fever and neutropenia. He recently completed a 6-week course ofvancomycin for persistent coagulase-negative staphylococcal bacteremia. He is admitted to the

hospital with a temperature of 39.5°C and a white blood cell count of 0.2x103/mcL (0.2x109/L)(0% neutrophils). Blood culture grows gram-positive cocci that are resistant to vancomycin.

Of the following, the MOST likely pathogen on the blood culture is

A. group B Streptococcus

B. Klebsiella pneumoniae

C. Listeria monocytogenes

D. methicillin-resistant Staphylococcus aureus

E. vancomycin-resistant Enterococcus

Copyright © 2010 by the American Academy of Pediatrics page 68

Page 69: AAP PREP 2010

2010 PREP SA on CD-ROM

Preferred Response: ECritique: 28

The boy described in the vignette is immunocompromised, has an indwelling catheter, andhas undergone multiple antibiotic courses, including prolonged exposure to vancomycin.Accordingly, he has several risk factors for colonization and infection with vancomycin-resistant enterococci (VREC).

Enterococci are normal inhabitants of the gastrointestinal tract, with E faecalis and Efaecium accounting for most human infections. They are low-grade pathogens but have beenassociated with bacteremia in neonates. In older children, they may be isolated from intra-abdominal infections (generally in association with polymicrobial infection), device-associatedinfections, and urinary tract infections. In adults, they are also a major agent in infectiveendocarditis.

Traditionally, enterococci have been susceptible to ampicillin and vancomycin. Gentamicinmay offer synergistic benefit. Enterococci are resistant to all cephalosporins. Over the past 20years, enterococcal strains resistant to vancomycin have emerged, with spread occurringprimarily in the hospital setting. Previous antibiotic treatment, especially with vancomycin andcephalosporins, is the most common risk factor for nosocomial acquisition of VREC. Most of thevancomycin-resistant strains are E faecium. VREC strains also are resistant to ampicillin.Treatment of VREC infections may be difficult. Linezolid shows in vitro activity against thestrains and is approved for such treatment in adults. Pediatric safety and efficacy of linezolidhas been demonstrated.

The Society for Healthcare Epidemiology of America and the Hospital Infection ControlPractices Advisory Committee of the Centers for Disease Control and Prevention have publishedguidelines to prevent transmission of multidrug-resistant organisms, including VREC. Preventionof nosocomial transmission of VREC includes contact precautions (gowns and gloves), handhygiene, and, if necessary, cohorting of VREC-colonized and -infected patients.

Methicillin-resistant Staphylococcus aureus (MRSA) infections of central catheters canoccur, and this boy’s recent antibiotic courses plus hospitalizations do increase the risk foracquiring MRSA, but vancomycin-resistant strains are extremely rare to date. Group Bstreptococcal infections are a concern for immunocompromised hosts, but the organism remainssensitive to vancomycin and is seen less commonly than MRSA or VREC in this setting. AlthoughKlebsiella and Listeria infections occur in immunocompromised hosts and are resistant tovancomycin, they are not gram-positive cocci.

References:

American Academy of Pediatrics. Non-group A or B streptococcal and enterococcal infections.In: Pickering LK, Baker CJ, Kimberlin DW, Long SS, eds. Red Book: 2009 Report of the

Committee on Infectious Diseases. 28th ed. Elk Grove Village, Ill: American Academy ofPediatrics: 2009:634-636

Weinstein JW, Anderson DJ. Epidemiology and prevention and control of vancomycin-resistantenterococci. UpToDate Online 16.3. 2007. Available for subscription at:http://www.utdol.com/online/content/topic.do?topicKey=hosp_inf/2484&selectedTitle=1~150&so

Copyright © 2010 by the American Academy of Pediatrics page 69

Page 70: AAP PREP 2010

2010 PREP SA on CD-ROM

urce=search_result

Copyright © 2010 by the American Academy of Pediatrics page 70

Page 71: AAP PREP 2010

2010 PREP SA on CD-ROM

Question: 29

You are speaking to a group of medical students about antifungal agents, their potential uses,and the potential adverse effects associated with their use.

Of the following, the MOST common adverse effect of ketoconazole is

A. arrhythmias

B. headache

C. nausea

D. rash

E. seizures

Copyright © 2010 by the American Academy of Pediatrics page 71

Page 72: AAP PREP 2010

2010 PREP SA on CD-ROM

Preferred Response: CCritique: 29

Ketoconazole is an imidazole antifungal agent that works by inhibiting the biosynthesis ofergosterol and other membrane lipids that compose the fungal cell membrane. Such inhibitionresults in fungal cellular membranes that lack the sterol components, causing increasedpermeability and progressive instability.

Ketoconazole is effective in the treatment of chronic mucocutaneous candidiasis,coccidioidomycosis, histoplasmosis, blastomycosis, and paracoccidioidomycosis innonimmunosuppressed hosts. It is not effective in the treatment of aspergillosis, cryptococcosis,or mucormycosis. Because other antifungals are effective against these pathogens and havefewer adverse effects, ketoconazole no longer is considered the drug of choice.

The most frequent adverse effects of ketoconazole are anorexia, nausea, and vomiting,which occur in about 15% to 29% of patients. Allergic rash, headache, and pruritus are seen in4% to 10% of patients. Mild elevations in liver transaminases occur in 2% to 5% of patientsduring the course of therapy. However, the most serious adverse effect associated withketoconazole therapy is fulminant hepatitis. This occurs rarely, in 1 in 15,000 exposedindividuals. Arrhythmias and seizures have not been associated with the use of this drug.

References:

Cross JT Jr, Hickerson SL, Yamauchi T. Antifungal drugs. Pediatr Rev. 1995;16:123-129.Abstract available at: http://pedsinreview.aappublications.org/cgi/content/abstract/16/4/123

Dismukes WE, Stamm AM, Graybill JR, et al. Treatment of systemic mycoses with ketoconazole:emphasis on toxicity and clinical response in 52 patients. National Institute of Allergy andInfectious Diseases collaborative antifungal study. Ann Intern Med. 1983;98:13-20. Abstractavailable at: http://www.ncbi.nlm.nih.gov/pubmed/6293361

Heel RC, Brogden RN, Carmine A, Morley PA, Speight TM, Avery GS. Ketoconazole: a review ofits therapeutic efficacy in superficial and systemic fungal infections. Drugs. 1982;23:1-36

National Institute of Allergy and Infectious Diseases Mycoses Study Group. Treatment ofblastomycosis and histoplasmosis with ketoconazole. Results of a prospective, randomizedclinical trial. Ann Intern Med. 1985;103:861-872. Abstract available at:http://www.ncbi.nlm.nih.gov/pubmed/2865921

Copyright © 2010 by the American Academy of Pediatrics page 72

Page 73: AAP PREP 2010

2010 PREP SA on CD-ROM

Question: 30

A 14-year-old girl presents with a 2-month history of joint pain that is responding poorly to over-the-counter anti-inflammatory medications. She reports some sores in her mouth and mildswelling around her eyes and ankles. On physical examination, her temperature is 37.0°C, heartrate is 76 beats/min, respiratory rate is 14 breaths/min, and blood pressure is 130/86 mm Hg.She has oral ulcers, mild periorbital and pretibial edema, and mild swelling of her wrists and knee

joints. Laboratory findings include:

•Sodium, 136 mEq/L (136 mmol/L)

•Potassium, 4.8 mEq/L (4.8 mmol/L)

•Chloride, 100 mEq/L (100 mmol/L)

•Bicarbonate, 22 mEq/L (22 mmol/L)

•Blood urea nitrogen, 24.0 mg/dL (8.6 mmol/L)

•Creatinine, 1.3 mg/dL (114.9 mcmol/L)

•Albumin, 2.5 g/dL (25.0 g/L)

•Hemoglobin, 10.1 g/dL (101.0 g/L)

•White blood cell count, 3.0x103/mcL (3.0x109/L)

•Platelet count, 190x103/mcL (190x109/L)

•Urinalysis: 3+ blood, 3+ protein, with 20 to 50 red blood cells/high-power field

•Antinuclear antibody titer: 1:1,280•Anti-double-stranded DNA titer: 1:640

Of the following, the next BEST step in management is to

A. admit the patient for intravenous cyclophosphamide treatment

B. initiate treatment with ibuprofen

C. order a 24-hour urine for protein collection

D. refer the patient for a renal biopsy

E. refer the patient for bone marrow aspiration

Copyright © 2010 by the American Academy of Pediatrics page 73

Page 74: AAP PREP 2010

2010 PREP SA on CD-ROM

Preferred Response: DCritique: 30

The adolescent girl in the vignette meets the diagnostic criteria for systemic lupuserythematosus (SLE). Her renal involvement necessitates an aggressive approach to diagnosisand treatment, but the severity of renal involvement must be determined before aggressivetreatment is initiated.

Renal disease in patients who have SLE usually manifests as an immune complex-mediatedglomerulonephritis (GN), often associated with hypocomplementemia and positive serologictesting for antinuclear antibody (ANA) and anti-double-stranded (ds) DNA. A recentobservational study from Toronto demonstrated that 37% of children have nephritis at diagnosis,46% within 1 year of diagnosis, and 55% in long-term follow-up. The clinical manifestations oflupus nephritis are those typically seen with GN and may include one or more of the following:hematuria, proteinuria, azotemia, hypertension, and edema. Lupus nephritis is categorizedfurther by histologic criteria into the World Health Organization classification system: class I(normal), class II (mesangial proliferative GN), class III (focal proliferative GN), class IV (diffuseproliferative GN), and class V (membranous GN). Because of the need to classify the form ofnephritis prior to the institution of corticosteroids, the standard of care is to obtain a renal biopsyprior to treatment. Some forms of lupus nephritis, including diffuse proliferative nephritis, aretreated with cyclophosphamide as an adjunctive agent, but this medication should not be usedfor renal indications without a kidney biopsy. Results of the renal biopsy can provide bothprognostic and treatment information.

Once the renal disease is classified histologically and initial treatment is instituted, the patientcan be monitored by periodic assessment of urinary protein excretion as well as measurementof serologic markers such as complement components and anti-ds DNA titers. Patients whoexhibit worsening proteinuria, decreasing concentrations of complement components, or risinganti-ds DNA titers require assessment for a disease flare, which may necessitate increasingimmunosuppressive therapy (including corticosteroids).

Because nonsteroidal anti-inflammatory drugs such as ibuprofen have potentialnephrotoxicity, they usually are not administered to children who have lupus nephritis. The 24-hour urine collection typically is not used for quantitating urine protein excretion, which can beassessed accurately with a spot urine protein and creatinine measurement. There is noindication for a bone marrow aspiration in this patient.

References:

Adams A, MacDermott EJ, Lehman TJ. Pharmacotherapy of lupus nephritis in children: arecommended treatment approach. Drugs. 2006;66:1191-1207. Abstract available at:http://www.ncbi.nlm.nih.gov/pubmed/16827597

Hiraki LT, Benseler SM, Tyrrell PN, Hebert D, Harvey E, Silverman ED. Clinical and laboratorycharacteristics and long-term outcome of pediatric systemic lupus erythematosus: a longitudinalstudy. J Pediatr. 2008;152:550-556. Abstract available at:http://www.ncbi.nlm.nih.gov/pubmed/18346514

Copyright © 2010 by the American Academy of Pediatrics page 74

Page 75: AAP PREP 2010

2010 PREP SA on CD-ROM

Question: 31

The mother of a 14-year-old girl who has asthma is concerned that her daughter’s recentsevere exacerbation is due to mold exposure. Their home sustained flood damage last year. Themother provides you with a list of diagnostic tests she found on the Internet, which she believeswill help confirm her daughter’s mold allergy.

Of the following, the MOST appropriate testing to evaluate the girl for possible mold allergy is

A. allergy skin testing

B. applied kinesiology

C. cytotoxic testing

D. provocation-neutralization

E. pulse test

Copyright © 2010 by the American Academy of Pediatrics page 75

Page 76: AAP PREP 2010

2010 PREP SA on CD-ROM

Preferred Response: ACritique: 31

Clinicians should be aware that many unconventional and unproven testing methods are stillpromoted for the diagnosis and management of allergic conditions. For example, appliedkinesiology theorizes that an allergen or irritant substance provokes weakness when the personingests, holds, or even stands in close proximity to the offending agent. To date, no conclusiveevidence supports this theory or test.

The cytotoxic test is an in vitro test involving microscopic evaluation of blood after placing adrop of the offending agent (eg, food, mold) on a slide containing the patient’s blood. Changes inthe appearance, size, shape, and integrity of leukocytes are interpreted as a "positive"response. A thorough United States Food and Drug Administration review has concluded thatthe efficacy of the cytotoxic test is unproven.

The pulse test evaluates whether an increase or decrease in pulse rate follows foodingestion, injection, or sublingual application and is interpreted as an "allergy" if a change occurs.

Finally, provocation-neutralization is a nonstandardized test for food and inhalant allergies. Aseries of dilutions of an allergen are injected in the upper arm until the patient reports subjectivesensations. A progressive series of lower concentrations subsequently are administered untilthe sensation abates. The lower dose is used for injection treatment to build immunologictolerance for the offending trigger. Both the testing and treatment method are unproven andshould be considered similar to placebo therapy.

Finally, Candida immunotherapy or avoidance of yeast ingestion for patients diagnosed withyeast or Candida hypersensitivity is unproven.

The parent in the vignette is concerned for mold allergy. Molds generally are believed tohave three effects: direct infection through ingestion or inhalation, release of toxins or irritantsas mold byproducts, and an immune response (eg, allergic bronchopulmonary aspergillosis,allergic fungal sinusitis, allergic rhinitis, asthma). Significant water damage, leaking watersources, or visible mold growth represent real health concerns. Commercial testing for mold canprovide some information, but quantifiable amounts are found in 80% of homes in the UnitedStates. Thus, the presence of mold does not necessarily indicate causation. For example,Alternaria alternata is linked to fatal asthma but is primarily an outdoor aeroallergen. Allergy skinprick testing or blood testing can aid in detecting immunoglobulin E-mediated responses to moldsand other allergens and is reasonable as part of determining the cause of this adolescent’sasthma exacerbations.

As a result of reviewing this information, do you intend to make a change in practiceto provide better patient care?Yes No

References:

Bush RK, Portnoy JM, Saxon A, Terr AI, Wood RA. The medical effects of mold exposure. JAllergy Clin Immunol. 2006;117:326-333. Abstract available at:http://www.ncbi.nlm.nih.gov/pubmed/16514772

Copyright © 2010 by the American Academy of Pediatrics page 76

Page 77: AAP PREP 2010

2010 PREP SA on CD-ROM

Bush RK, Portnoy JM. The role and abatement of fungal allergens in allergic diseases. J AllergyClin Immunol. 2001;107(3 suppl):S430-S440. Abstract available at:http://www.ncbi.nlm.nih.gov/pubmed/11242604

Matsui EC, Eggleson PA. Immunotherapy for allergic disease. In: Leung DYM, Sampson HA, GehaRS, Szefler SJ, eds. Pediatric Allergy Principles and Practice. St. Louis, Mo: Mosby Elsevier;2003:277-285

Nelson HS. Immunotherapy for inhalant allergens. In: Adkinson NF Jr, Yunginger JW, Busse WW,Bochner BS, Holgate ST, Simons FER, eds. Middleton’s Allergy Principles and Practice. 6th ed.Philadelphia, Pa: Mosby Elsevier; 2003:1455-1473

O’Connor GT. Allergen avoidance in asthma: what do we do now? J Allergy Clin Immunol.2005;116:26-30. Abstract available at: http://www.ncbi.nlm.nih.gov/pubmed/15990768

Terr AI. Unconventional theories and unproved methods in allergy. In: Adkinson NF Jr, YungingerJW, Busse WW, Bochner BS, Holgate ST, Simons FER, eds. Middleton’s Allergy Principles andPractice. 6th ed. Philadelphia, Pa: Mosby Elsevier; 2003:1711-1728

Copyright © 2010 by the American Academy of Pediatrics page 77

Page 78: AAP PREP 2010

2010 PREP SA on CD-ROM

Question: 32

The parents of a 12-year-old boy bring him to the emergency department after finding himunresponsive in bed when they tried to wake him for school. They report that he has had norecent illnesses and was in his usual state of health when he went to bed last night. Of note, hehas enuresis, treated with imipramine. On physical examination, he is responsive only to pain,his heart rate is 120 beats/min, respiratory rate is 6 breaths/min, and blood pressure is 60/40 mmHg. His pupils are 6 mm, equal, and sluggishly reactive. All other findings are within normalparameters. He is endotracheally intubated, ventilated with 100% oxygen, placed on a cardiacmonitor, and given a 20-mL/kg bolus of normal saline. Electrocardiography demonstrates sinustachycardia, PR interval of 130 msec, and QRS duration of 140 msec.

Of the following, the next MOST appropriate step is to

A. administer adenosine

B. administer amiodarone

C. administer sodium bicarbonate

D. begin external pacing

E. perform synchronized cardioversion at 0.5 J/kg

Copyright © 2010 by the American Academy of Pediatrics page 78

Page 79: AAP PREP 2010

2010 PREP SA on CD-ROM

Preferred Response: CCritique: 32

The coma, respiratory depression, tachycardia, hypotension, and dilated pupils describedfor the patient in the vignette are consistent with an acute tricyclic antidepressant (TCA)overdose. Among the additional signs that might be observed are seizures, dysrhythmias, andother anticholinergic features such as dry mouth, hyperthermia, urinary retention, flushed skin,and agitation. Tricyclic antidepressants cause these clinical features by inhibiting a variety ofneurotransmitter receptors, including muscarinic acetylcholine, alpha-1-adrenergic, gammaaminobutyric acid (GABA), and histamine receptors as well as cardiac fast sodium channels.

The most serious toxicities from TCA overdose affect the heart and central nervous system(CNS). Because of the TCA effect on fast sodium channels, conduction velocity is decreased. Inaddition, repolarization duration and absolute refractory periods are prolonged. These effects,coupled with alpha-1-adrenergic antagonism, are responsible for the hypotension andconduction delays that are seen commonly. CNS excitation and seizures or depression may berelated to effects on GABA or histamine receptors.

Initial management of a TCA overdose begins with ensuring a patent airway and restoringadequate oxygenation, ventilation, and perfusion. Intubation and mechanical ventilation often arenecessary, as is fluid resuscitation with boluses of normal saline. Seizures are treated withbenzodiazepines. Alpha-adrenergic pressors (eg, norepinephrine) may be required to treatrefractory hypotension. Decontamination should be performed with activated charcoal. Althoughacetaminophen and aspirin concentrations should be measured, especially in intentionalingestions to evaluate for possible coingestants, measurement of TCA concentrations is notclinically useful.

The single most useful diagnostic and prognostic test in the setting of a TCA overdose iselectrocardiography. In a study from 1985, toxicologists found that a QRS duration of greaterthan 100 msec predicted seizures in 34% and dysrhythmias in 14% of patients who had TCAoverdoses. The QRS widening is related to fast sodium channel blockade caused by direct TCAeffects and exacerbated by acidemia. These effects can be overcome by the administration ofsodium bicarbonate boluses. Sodium bicarbonate should be administered until the QRS durationis less than 100 msec. The exact mechanism for this effect is unknown.

Adenosine and synchronized cardioversion are treatments for supraventricular tachycardia.Amiodarone is a drug of choice for ventricular arrhythmias. External pacing is appropriatetreatment for refractory, symptomatic bradycardia.

References:

Boehnert MT, Lovejoy FH Jr. Value of QRS duration versus the serum drug level in predictingseizures and ventricular arrhythmias after an acute overdose of tricyclic antidepressants. NEngl J Med. 1985;313:474-479. Abstract available at:http://www.ncbi.nlm.nih.gov/pubmed/4022081

Hutchinson MD, Traub SJ. Tricyclic antidepressant poisoning. UpToDate Online 16.3. 2008.Available at:http://www.utdol.com/online/content/topic.do?topicKey=ad_tox/10025&selectedTitle=1~150&sou

Copyright © 2010 by the American Academy of Pediatrics page 79

Page 80: AAP PREP 2010

2010 PREP SA on CD-ROM

rce=search_result630

Jacob J. Toxicity, antidepressant. In: eMedicine Specialties, Emergency medicine, Toxicology.2008. Available at: http://www.emedicine.com/emerg/topic37.htm

Woolf AD, Erdman AR, Nelson LS, et al. Tricyclic antidepressant poisoning: an evidence-basedconsensus guideline for out-of-hospital management. Clin Toxicol (Phila). 2007;45:203-233.Brief summary available at:http://guidelines.gov/summary/summary.aspx?doc_id=9906&nbr=005302&string=tricyclic+AND+antidepressant+AND+overdose

Copyright © 2010 by the American Academy of Pediatrics page 80

Page 81: AAP PREP 2010

2010 PREP SA on CD-ROM

Question: 33

You are evaluating a 2-year-old daughter of strict vegan parents. Her birthweight at term was3.5 kg. Since weaning at 12 months of age, the child’s diet has included a homemade,macrobiotic-based formula. In your office today, the girl’s weight is 11.2 kg.

Of the following, the child’s diet MOST likely is deficient in

A. essential amino acids

B. linoleic acid

C. vitamin A

D. vitamin B12

E. vitamin C

Copyright © 2010 by the American Academy of Pediatrics page 81

Page 82: AAP PREP 2010

2010 PREP SA on CD-ROM

Preferred Response: DCritique: 33

Strict vegan diets include foods that come solely from plant sources. Such diets generallycontain adequate amounts of vitamins A and C as well as essential fatty acids (including linoleic

acid). However, a strict vegan diet instituted after weaning contains very little vitamin B12, anutrient primarily found in meats, eggs, and dairy products, unless supplements are provided.

Breastfed infants of vegan mothers may develop vitamin B12 deficiency, but only if maternalstores are low. Although the vitamin composition of human milk is directly related to dietary intakeof vitamins A, C, D, and the B group, studies comparing the vitamin content of human milk fromvegan compared with nonvegan mothers have not demonstrated any significant micronutrientdifferences. Commercial soy-based formulas are alternatives for vegan mothers who do notbreastfeed. In most cases, therefore, the greatest potential nutritional risks for both breastfedand soy formula-fed infants of vegan parents occur after weaning. This is particularly the casewhen a homemade weaning formula is given. Conversely, commercially available soy milks aresupplemented with vitamins.

Studies in both the United States and the United Kingdom have shown that vegan childrenexhibit small but significant differences in growth variables (height and weight percentiles)compared with children eating mixed diets. This observation most likely is the consequence ofgroup differences in total energy consumption, although other studies have demonstrated thatthe calcium and zinc content of the vegan diet also may be low, indicating the requirement forsupplementation. Conversely, the essential amino acid and total protein intake of vegan childrenhas been shown to be adequate to support normal growth. Despite the lower mean height andweight of vegan children compared with children eating mixed diets, a large British study foundno evidence of growth failure (weight or height less than the 5th percentile) in vegan children,and no between-group differences were noted in terms of muscle strength and overall health.

A routine health assessment of any child should include a careful dietary history. Informationabout specific cultural or family customs permits identification of patients at nutritional risk andaids the clinician in determining whether caregivers require education regarding appropriatenutrition for growing children. For example, in industrialized countries, vitamin D deficiencyrickets is an emerging nutritional problem. Such deficiency is particularly prevalent for dark-skinned infants living in temperate or northern climates, those whose cultural/religious customsmay include extensive covering of body surfaces, and in infants and children who receive littledirect sunlight exposure (ie, less than 30 minutes to the face and hands three times per week).

References:

Graham EA. Economic, racial and cultural influences on the growth and maturation of children.Pediatr Rev. 2005;26:290-294. Available at:http://pedsinreview.aappublications.org/cgi/content/full/26/8/290

Hebbelinck M, Clarys P, De Malsche A. Growth, development, and physical fitness of Flemishvegetarian children, adolescents, and young adults. Am J Clin Nutr. 1999;70(3 suppl):579S-585S. Available at: http://www.ajcn.org/cgi/content/full/70/3/579S

Copyright © 2010 by the American Academy of Pediatrics page 82

Page 83: AAP PREP 2010

2010 PREP SA on CD-ROM

Joiner TA, Foster C, Shope T. The many faces of vitamin D deficiency rickets. Pediatr Rev.2000;21:296-302. Available at: http://pedsinreview.aappublications.org/cgi/content/full/21/9/296

Kramer MS, Guo T, Platt RW, et al. Breastfeeding and infant growth: biology or bias? Pediatrics.2002;110:343-347. Available at: http://pediatrics.aappublications.org/cgi/content/full/110/2/343

Moilanen BC. In brief: vegan diets in infants, children, and adolescents. Pediatr Rev. 2004;25:174-176. Available at: http://pedsinreview.aappublications.org/cgi/content/full/25/5/174

Sanders TA. Vegetarian diets and children. Pediatr Clin North Am. 1995;42:955-965. Abstractavailable at: http://www.ncbi.nlm.nih.gov/pubmed/7610022

Copyright © 2010 by the American Academy of Pediatrics page 83

Page 84: AAP PREP 2010

2010 PREP SA on CD-ROM

Question: 34

You are called to the newborn nursery to examine a 4-hour-old term infant delivered to a motherwho had polyhydramnios. The infant’s Apgar scores were 7 and 8 at 1 and 5 minutes,respectively. The nurse reports that the infant requires frequent oropharyngeal suctioning anddisplays cyanosis when suctioned. She placed a pulse oximeter on the infant and reportsfrequent episodes of desaturation from 94% to 70% on room air during the cyanotic episodes.Physical examination reveals an appropriately grown infant who has substantial oral secretions,scattered rales on auscultation, normal S1 and S2 heart sounds, no heart murmur, normal bowelsounds, and no abdominal distention.

Of the following, the MOST important next step is to

A. insert a feeding tube

B. insert an umbilical arterial catheter

C. measure bedside blood glucose concentration

D. order emergent echocardiography

E. order renal ultrasonography

Copyright © 2010 by the American Academy of Pediatrics page 84

Page 85: AAP PREP 2010

2010 PREP SA on CD-ROM

Preferred Response: ACritique: 34

The newborn described in the vignette displays the classic clinical signs of atracheoesophageal fistula (TEF) with accompanying esophageal atresia. Attempting to insert anorogastric (OG) feeding tube is the most important next step in evaluating a newborn in whomthis diagnosis is suspected. The inability to insert the OG tube confirms esophageal atresia,which can be verified by seeing the tube coiled in the proximal esophageal pouch on chestradiography (Item C34). In the presence of esophageal atresia, TEF is confirmed by radiographicexamination of the abdomen revealing air in the lower gastrointestinal tract, which could onlyreach that locale via a fistula between the upper airway and the esophagus distal to any atreticportion.

The intermittent cyanosis exhibited by the newborn may be due to pooling of oral secretionsin the hypopharynx, airway obstruction, aspiration of oral secretions through the larynx into thetrachea (because the infant cannot swallow these secretions), pneumonitis, or hypoxiaresulting from the reflux of gastric contents via the TEF into the tracheobronchial tree. Thematernal history of polyhydramnios is a clue to likely swallowing dysfunction or gastrointestinaltract obstruction. The physical examination and judicious use of diagnostic imaging tools such asplain films and ultrasonography to assess for the presence of vertebral anomalies, anorectalstenosis or atresia, structural heart disease, renal anomalies, and limb anomalies is important indetermining if the esophageal atresia or TEF are isolated defects or part of the VACTERLassociation, which occurs in one third of infants who have esophageal atresia.

TEF and esophageal atresia is a surgical emergency that requires early evaluation forsurgical ligation of the TEF to protect the airway. Until surgery is performed, vigilantoropharyngeal suctioning is required, and the newborn’s head should be kept elevated. Somenewborns may require tracheal intubation and assisted ventilation. Early insertion of agastrostomy tube for gastrointestinal decompression and subsequent feeding until such time asthe esophagus can be used also is common. Surgical anastomosis of the proximal and distalesophagus may be accomplished as a later procedure.

Measuring bedside glucose concentration is important in newborns who have respiratorydistress but should follow evaluation of airway obstruction in the newborn in the vignette.Echocardiography can help in the evaluation of the newborn for structural heart disease, but thedocumented normal room air saturation of 94% indicates no fixed cardiac shunt or cyanoticlesion, making this test less imperative at this time. Renal ultrasonography is indicated ifVACTERL association is suspected but requires an initial diagnosis of TEF. An umbilical arterialcatheter may or may not be indicated, depending on the degree of respiratory distress.

References:

Goyal A, Jones MO, Couriel JM, Losty PD. Oesophageal atresia and tracheo-oesophageal fistula.Arch Dis Child Fetal Neonatal Ed. 2006;91:F381-F384. Abstract available at:http://www.ncbi.nlm.nih.gov/pubmed/16923940

Keckler SJ, St Peter SD, Valusek PA, et al. VACTERL anomalies in patients with esophagealatresia: an updated delineation of the spectrum and review of the literature. Pediatr Surg Int.

Copyright © 2010 by the American Academy of Pediatrics page 85

Page 86: AAP PREP 2010

2010 PREP SA on CD-ROM

2007;23:309-313. Abstract available at: http://www.ncbi.nlm.nih.gov/pubmed/17377826

Kulayalat NA, Narchi H. Index of suspicion: case 5. Pediatr. Rev. 2000;21:20-28. Available at:http://pedsinreview.aappublications.org/cgi/content/full/21/1/20

Magnuson D, Parry RL, Chwals WJ. Selected thoracic gastrointestinal anomalies. In: Martin RJ,Fanaroff AA, Walsh MC, eds. Fanaroff and Martin's Neonatal-Perinatal Medicine: Diseases ofthe Fetus and Infant. 8th ed. Philadelphia, Pa: Mosby Elsevier; 2006:1373-1380

Nakayama DK. Esophageal atresia and tracheoesophageal fistula. In: Nakayama DK, Bose CL,Chescheir NC, Valley R, eds. Critical Care of the Surgical Newborn. Armonk, NY: FuturaPublishing Company, Inc; 1997:227-249

Thilo EH, Rosenberg AA, The newborn infant. In: Hay WW Jr, Levin MJ, Sondheimer JM,Deterding RR, eds. CURRENT Diagnosis & Treatment: Pediatrics. 19th ed. New York, NY: TheMcGraw-Hill Companies; 2009:Chapter 1. Available for subscription at:http://www.accessmedicine.com/content.aspx?aID=3396500

Copyright © 2010 by the American Academy of Pediatrics page 86

Page 87: AAP PREP 2010

2010 PREP SA on CD-ROM

Question: 35

You are working in the newborn nursery when the nurse asks you to evaluate a girl who hasjust been admitted. According to her records, her mother had good prenatal care and results ofprenatal laboratory evaluations were normal. The infant is vigorous and pink. Findings onphysical examination are normal except for a reddish-purple patch over her right forehead,eyelid and cheek (Item Q35).

Of the following, the condition that is MOST likely to be associated with this skin lesion is

A. Kasabach-Merritt syndrome

B. neurofibromatosis type 1

C. Osler-Weber-Rendu disease

D. Sturge-Weber syndrome

E. tuberous sclerosis complex

Copyright © 2010 by the American Academy of Pediatrics page 87

Page 88: AAP PREP 2010

2010 PREP SA on CD-ROM

Preferred Response: DCritique: 35

Vascular lesions in the newborn may be characterized as benign vascular neoplasms suchas hemangiomas or as vascular malformations such as salmon patches (also called a nevussimplex) or port wine stains (PWSs). PWSs occur commonly and usually are not associated withunderlying disorders, but approximately 5% to 10% of infants who have PWSs in the distributionof the first branch of the trigeminal nerve (V1), as described for the infant in the vignette, haveSturge-Weber syndrome (SWS). SWS is associated with seizures and meningeal and cerebralcortex abnormalities ipsilateral to the PWS, and the presence of a PWS in the V1 region isincluded in the diagnostic criteria for the syndrome. Eye involvement, most commonly glaucoma,also is a feature of SWS occurring in 60% of patients.

Large atypical-appearing hemangiomas (actually hemangioendotheliomas or tuftedangiomas) and thrombocytopenia are seen with Kasabach-Merritt syndrome (Item C35A). Theprimary cutaneous manifestations of neurofibromatosis type 1 are café au lait macules, whichmay be present at birth, and neurofibromas (Item C35B), which typically appear during childhoodor early adolescence. Osler-Weber-Rendu disease (hereditary hemorrhagic telangiectasia) ischaracterized by recurrent epistaxis and cutaneous telangiectases that develop later in life.Tuberous sclerosis complex is a neurocutaneous syndrome; the most common skin lesions arehypopigmented macules (ash leaf macules), angiofibromas (adenoma sebaceum) (Item C35C),and shagreen patches. Other skin lesions include periungual fibromas and facial plaques.

PWSs can be treated effectively with pulsed dye laser. Laser treatment may result incomplete resolution or significant lightening of the lesion, and the degree of effectiveness isrelated to the location of the lesion (Item C35D). Results of studies are conflicting, but lesions inthe periorbital area and central forehead generally respond better than do lesions in the midfaceor on the limbs. Other factors associated with improved results include lesion size less than 20cm and age at onset of therapy. Risks of treatment are small and include atrophy, hypertrophy,and hyperpigmentation of the treated area. PWSs recur in many patients after treatment.

References:

Jasim ZF, Handley JM. Treatment of pulsed dye laser-resistant port wine stain birthmarks. J AmAcad Dermatol. 2007;57:677-682. Abstract available at:http://www.ncbi.nlm.nih.gov/pubmed/17658196

Pielop JA. Vascular lesions in the newborn. UpToDate Online 16.3. 2008. Available forsubscription at:http://www.utdol.com/online/content/topic.do?topicKey=ped_derm/4439&selectedTitle=9~150&source=search_result630

Stier MF, Glick SA, Hirsch RJ. Laser treatment of pediatric vascular lesions: port wine stains andhemangiomas. J Am Acad Dermatol. 2008;58:261-285. Abstract available at:http://www.ncbi.nlm.nih.gov/pubmed/18068263

Copyright © 2010 by the American Academy of Pediatrics page 88

Page 89: AAP PREP 2010

2010 PREP SA on CD-ROM

Question: 36

You are the physician at a summer camp for special needs children. An adolescent who is ajunior counselor comes to the infirmary after being stung by a bee. Physical examination revealsswelling, pain, erythema, and tenderness in a well-circumscribed area on the right upper arm.On close examination, you see a small foreign body in a punctum.

Of the following, the MOST appropriate means of removing the stinger is to

A. apply duct tape and rapidly pull it off

B. pass a needle subcutaneously alongside the stinger

C. probe for the stinger with tweezers

D. scrape the stinger from the skin gently with a tongue blade

E. wait for the stinger to extrude by itself

Copyright © 2010 by the American Academy of Pediatrics page 89

Page 90: AAP PREP 2010

2010 PREP SA on CD-ROM

Preferred Response: DCritique: 36

The honeybee is unique among hymenoptera species in that it may leave a stingerembedded in skin; wasps, hornets, and yellow jackets do not. The stinger should be removed toavoid continued envenomation from the accompanying venom sac.

Simple scraping away of the stinger can avoid rupture of the venom sac. In a medical setting(such as a camp infirmary), a tongue blade may be available, but in other settings, the blunthandle of a spoon or other such instrument may be used. The area should be cleansed withsoap and water after removal of the stinger. Because allergic reactions are more likely to occurwhen the stinger is not removed promptly, tweezers may be used if nothing is available toscrape out the stinger. However, use of tweezers in a pincer motion may release more venom.Unnecessary probing using blunt or sharp instruments should be avoided if the stinger cannotbe extracted readily. In recent years, a variety of methods, including home remedies using

common household product adhesives such as duct tape and Superglue®, have becomepopular for removal of transdermal foreign bodies and superficial lesions such as cactus spinesand common warts. There is no evidence to suggest their efficacy in removing bee stingers.Waiting for the stinger to extrude increases the risk for allergic reaction.

References:

Booker GM, Adam HM. In brief: insect stings. Pediatr Rev. 2005;26:388-389. Available at:http://pedsinreview.aappublications.org/cgi/content/full/26/10/388

Mendez E, Sicklick MJ. In brief: hymenoptera reactions. Pediatr Rev. 1995;16:355-356. Abstractavailable at: http://pedsinreview.aappublications.org/cgi/content/abstract/16/9/355

Vankawala HH, Park R. Bee and hymenoptera stings. eMedicine Specialties, EmergencyMedicine, Environmental. 2008. Available at: http://emedicine.medscape.com/article/768764-overview

Visscher PK, Vetter RS, Camazine S. Removing bee stings. Lancet. 1996;348:301—302. Abstractavailable at:http://www.ncbi.nlm.nih.gov/pubmed/8709689?ordinalpos=6&itool=EntrezSystem2.PEntrez.Pubmed.Pubmed_ResultsPanel.Pubmed_DefaultReportPanel.Pubmed_RVDocSum

Copyright © 2010 by the American Academy of Pediatrics page 90

Page 91: AAP PREP 2010

2010 PREP SA on CD-ROM

Question: 37

You are treating a 4-month-old infant who was born with tetralogy of Fallot. Her mother bringsher to the clinic because she has had diarrhea and fever since the previous evening. Onphysical examination, the infant is irritable and has cyanosis and a heart rate of 180 beats/min.

Of the following, the finding that is MOST consistent with a tetralogy spell is

A. clubbing of the digits

B. hepatomegaly

C. inability to hear a murmur

D. oxygen saturation of 75% in room air

E. S3 gallop rhythm

Copyright © 2010 by the American Academy of Pediatrics page 91

Page 92: AAP PREP 2010

2010 PREP SA on CD-ROM

Preferred Response: CCritique: 37

Tetralogy of Fallot (TOF) is the most common form of cyanotic congenital heart disease, withan incidence of approximately 0.2 in 1,000 live births and accounting for 9% of all congenitalheart disease. The four components of TOF are right ventricular outflow/pulmonary stenosis,ventricular septal defect (VSD), overriding aorta, and right ventricular hypertrophy (Item C37).The primary lesion is underdevelopment of the pulmonary infundibulum, which has led some torefer to this disease as "monology of Fallot" because all aspects of the tetrad result from thislesion. The result of underdevelopment of the pulmonary infundibulum is deviation of theinfundibular septum anteriorly and superiorly, bringing it into the right ventricular outflow tract.This leads to obstructed right ventricular outflow and the commonly seen underdevelopment ofthe pulmonary valve and pulmonary arteries caused by diminished blood flow through thesestructures. The underdeveloped pulmonary infundibulum also creates a VSD, which is almostuniversally large and of the malalignment type. The defect resulting from anterior malalignment ofthe infundibulum allows the aorta to "override" the ventricular septum. Finally, right ventricularhypertrophy results from exposure to systemic pressures (large VSD and pulmonary stenosis).

Most patients who have TOF do not present with cyanosis in the newborn period, but rathercome to medical attention because of a harsh systolic murmur. The murmur results frominfundibular stenosis and pulmonary stenosis, not from the VSD. The second heart sound issingle. Because the degree of pulmonary blood flow obstruction can vary among patients, thedegree of systemic oxygen desaturation ranges from mild to severe. Children who have mildobstruction may appear "pink," and those who have severe pulmonary stenosis havesignificantly reduced pulmonary blood flow and an increase in right-to-left shunting across theVSD into the aorta, leading to more pronounced cyanosis. Furthermore, as pulmonary blood flowdecreases with tight pulmonary stenosis, pulmonary venous return to the left atrium decreases,resulting in less highly saturated blood leaving the left ventricle and entering the aorta.Conversely, mild pulmonary stenosis is associated with more pulmonary blood flow, less right-to-left intracardiac shunting, and less systemic desaturation. In the mildest cases, there is left-to-right shunting across the VSD and near-normal or normal systemic saturation.

A decreased or absent murmur signifies diminished pulmonary blood flow, as occurs in thecyanotic spell or tetralogy spell. Such spells are marked by distress, crying, inconsolability,hyperpnea, and increasing cyanosis, as described for the infant in the vignette. They frequentlyoccur in the morning or at times of dehydration (eg, fever, gastroenteritis). If not treated quickly,cyanotic spells can lead to serious morbidity and even death.

Treatment of cyanotic spells centers on increasing pulmonary blood flow, which isaccomplished by several means. The first step is to alter the ratio of relative resistance ofpulmonary and systemic beds. Increasing the systemic vascular resistance relative to thepulmonary vascular resistance decreases the right-to-left shunt at the VSD and can beaccomplished by placing the patient in a knee-to-chest position or by squatting in older children.Pharmacologic augmentation of the systemic vascular resistance can be achieved withintravenous phenylephrine. Therapy also includes the use of sedation with morphine, whichsuppresses the sensation of suffocation and can relieve the patient’s fear. The use of high-flowoxygen, which dilates pulmonary vasculature, constricts systemic vasculature and increases

Po2 of pulmonary venous return, and generous intravascular fluid administration to increase

Copyright © 2010 by the American Academy of Pediatrics page 92

Page 93: AAP PREP 2010

2010 PREP SA on CD-ROM

preload are important therapies for the patient experiencing a tetralogy spell. Clubbing of the digits can be seen in cyanotic heart disease as well as a variety of other

entities, but it is not typical in patients younger than 1 year of age and its presence is notassociated with a tetralogy spell. Hepatomegaly is uncommon in the infant who has TOF; itspresence suggests right heart failure. Diminished oxygen saturation is a component of atetralogy spell, although the physical findings and condition of the patient, not the oxygensaturation, define the spell. Finally, an S3 gallop rhythm can be heard in the patient who hasmyocardial failure but is not expected in a patient who has TOF, particularly with the pronouncedtachycardia described for the patient in the vignette.

References:

Doyle TP, Kavanaugh-McHugh A, Graham TP. Tetralogy of Fallot and pulmonary atresia withventricular septal defect. In: Moller JH, Hoffman JIE, eds. Pediatric Cardiovascular Medicine.Philadelphia, Pa: Churchill Livingstone; 2000:391-408

Neches WH, Park SC, Ettedgui JA. Tetralogy of Fallot and tetralogy of Fallot with pulmonaryatresia. In: Garson A Jr, Bricker JT, Fisher DJ, Neish SR, eds. The Science and Practice ofPediatric Cardiology. 2nd ed. Baltimore, Md: Williams & Wilkins; 1998:1383-1411

Copyright © 2010 by the American Academy of Pediatrics page 93

Page 94: AAP PREP 2010

2010 PREP SA on CD-ROM

Question: 38

During the health supervision visit for a healthy 4-month-old boy, you note that his headcircumference is 46 cm (>98th percentile) and his length and weight are at the 50th percentile.He has mild frontal bossing and widely split cranial sutures. The fontanelle is flat. Arm and legmovements, tone, and reflexes are normal. In reviewing prior growth parameters, you note thathis head circumference was at the 75th percentile at birth and the 90th percentile at 2 months.

Of the following, the MOST helpful next diagnostic procedure is

A. electroencephalography

B. head ultrasonography

C. lumbar puncture with manometry

D. plain radiography of the skull

E. three-dimensional head computed tomography scan

Copyright © 2010 by the American Academy of Pediatrics page 94

Page 95: AAP PREP 2010

2010 PREP SA on CD-ROM

Preferred Response: BCritique: 38

The boy described in the vignette is healthy, with no abnormalities of neurodevelopment, andhis neurologic examination reveals no changes in mental status or evidence of focal brainabnormalities or increased intracranial pressure. However, assuming measurements areaccurate, he has steadily crossed head circumference percentiles since birth. Neuroimaging isimportant to determine the reason for his crossing of percentiles. The differential diagnosisincludes hydrocephalus, arachnoid cyst, parenchymal brain lesions, subdural hematomas, andneurodegenerative diseases. Although the most helpful high-resolution image in this case isbrain magnetic resonance imaging, sedation usually is required for such imaging in a child of thisage. In contrast, head ultrasonography through the fontanelle can be performed quickly and withless risk and cost because sedation is not needed. The results can be used in planning referralto neurosurgery or neurology. Some conditions, such as an arachnoid cyst, may not requireurgent intervention, and observation suffices, with magnetic resonance imaging scheduled for alater date.

Because the child does not have seizures, electroencephalography is not indicated. Lumbarpuncture to rule out elevated intracranial pressure is not needed because the sutures are open.Therefore, in a chronic process, pressure will not rise to a level that is dangerous or requiresmeasurement. Studies of bone are not helpful. Radiography of the skull will not provideinformation that affects management. Three-dimensional computed tomography scan is helpfulfor assessing craniosynostosis if the child’s head shape is abnormal and fused ridges at thecranial sutures are palpable.

References:

Haslam RHA. Neurologic evaluation. In: Kliegman RM, Behrman RE, Jenson HB, Stanton BF, eds.Nelson Textbook of Pediatrics. 18th ed. Philadelphia, Pa: Saunders Elsevier; 2007:2433-2442

Koenigsberg RA, Bianco BA, Faro SH, et al. Neuroimaging. In: Goetz C, ed. Textbook of ClinicalNeurology. 3rd ed. Philadelphia, Pa: Saunders Elsevier; 2007:427-466

Piatt JH Jr. Recognizing neurosurgical conditions in the pediatrician's office. Pediatr Clin NorthAm. 2004;51:237-270. Abstract available at: http://www.ncbi.nlm.nih.gov/pubmed/15062671

Stoll BJ. The newborn. In: Kliegman RM, Behrman RE, Jenson HB, Stanton BF, eds. NelsonTextbook of Pediatrics. Philadelphia, Pa: Saunders Elsevier; 2007: 675-682

Copyright © 2010 by the American Academy of Pediatrics page 95

Page 96: AAP PREP 2010

2010 PREP SA on CD-ROM

Question: 39

You are called to examine a newborn girl who has multiple congenital anomalies. On physicalexamination, you notice several "punched-out" scalp ulcers (Item Q39A), bilateral cleft lip (ItemQ39B) and palate, postaxial polydactyly (extra digit on the ring finger side) of the hands (ItemQ39C), and a small omphalocele.

Of the following, this infant’s karyotype MOST likely is

A. 45,X

B. 45,X/47,XXX

C. 47,XX+13

D. 47,XX+18

E. 47,XX+21

Copyright © 2010 by the American Academy of Pediatrics page 96

Page 97: AAP PREP 2010

2010 PREP SA on CD-ROM

Preferred Response: CCritique: 39

The newborn described in the vignette has a 47,XX+13 karyotype consistent with trisomy13. Her unusual features of "punched-out" scalp lesions (also known as "aplasia cutis"),bilateral cleft lip and palate, and polydactyly are present in at least 50% of newborns who havethis diagnosis. Other common anomalies in affected individuals include holoprosencephaly(incomplete septation of the frontal lobes), microcephaly, and cardiac defects (80%). Trisomy 13has a poor prognosis, with approximately 50% of affected individuals dying by 2 weeks of ageand 90% dying by 1 year.

Approximately 50% of individuals who have Turner syndrome have a 45,X karyotype; theremainder have a mosaic karyotype that includes a cell line consistent with Turner syndrome,such as 45,X/47,XXX, or a karyotype with 46 chromosomes wherein one of the X chromosomesis aberrant (eg, ring X, isochromosome Xq). Affected newborns may exhibit dysmorphisms ormay appear completely normal. Unusual features include webbed neck with low posteriorhairline, broad chest with widely spaced nipples, narrow and hyperconvex nails, and cardiacdefects, most commonly bicuspid aortic valve.

Individuals who have trisomy 18 usually have a 47,XX(or XY)+18 chromosome complement.Characteristic features include intrauterine growth restriction, prominent occiput, small facialfeatures, clenched hands with overlapping of the second finger over the third and the fifthfinger over the fourth, and hypertonia. Trisomy 18 has a poor prognosis; approximately 50% ofaffected individuals die by 2 weeks of age and 90% die by 1 year.

47,XX(or XY)+21 is the most common karyotype seen in individuals who have Downsyndrome. Affected newborns typically exhibit midface hypoplasia with epicanthal folds,upslanting palpebral fissures, small ears with overfolded pinnae, redundant nuchal skin, andhypotonia. Many affected individuals have fifth finger clinodactyly (in-curving) (Item C39), andalmost 50% have a single transverse palmar crease. Approximately 45% of affected individualshave congenital heart defects.

References:

Carey JC. Trisomy 18 and trisomy 13 syndromes. In: Cassidy SB, Allanson JE, eds. Managementof Genetic Syndromes. 2nd ed. Hoboken, NJ: Wiley-Liss; 2005:555-568

Jones KL. Down syndrome. In: Smith’s Recognizable Patterns of Human Malformation. 6th ed.Philadelphia, Pa: Elsevier Saunders; 2006:7-12

Jones KL. Trisomy 13. In: Smith’s Recognizable Patterns of Human Malformation. 6th ed.Philadelphia, Pa: Elsevier Saunders; 2006:18-21

Jones KL. Trisomy 18. In: Smith’s Recognizable Patterns of Human Malformation. 6th ed.Philadelphia, Pa: Elsevier Saunders; 2006:13-17

Sybert VP. Turner syndrome. In: Cassidy SB, Allanson JE, eds. Management of GeneticSyndromes. 2nd ed. Hoboken, NJ: Wiley-Liss; 2005:589-605

Copyright © 2010 by the American Academy of Pediatrics page 97

Page 98: AAP PREP 2010

2010 PREP SA on CD-ROM

Question: 40

A 15-year-old girl is concerned about irregular menses and acne. Menarche was at age 11years and 9 months, and she remembers developing pubic hair around age 7 years. On physical

examination, her vital signs are normal and her body mass index is 32.3 kg/m2. She has facialcomedonal and papular acne as well as mild darkening of the skin of her neck (Item Q40) andaxilla. You also note hypopigmented, narrow stretch marks on her abdomen and hair in a lineardistribution from her umbilicus to the pubic symphysis and on the upper inner surface of herthighs. She is at Sexual Maturity Rating 5, and her clitoral diameter is 2 mm.

Of the following, the MOST likely diagnosis is

A. Cushing syndrome

B. hypothyroidism

C. metabolic syndrome

D. physiologic anovulation

E. polycystic ovarian syndrome

Copyright © 2010 by the American Academy of Pediatrics page 98

Page 99: AAP PREP 2010

2010 PREP SA on CD-ROM

Preferred Response: ECritique: 40

The presence of acanthosis nigricans combined with obesity (body mass index >30 kg/m2),acne, and some increase in body hair described for the girl in the vignette as well as irregularmenses 3 years after menarche suggests the need for further evaluation for polycystic ovariansyndrome (PCOS). The diagnosis of PCOS, using the 2003 Rotterdam criteria, requires, inaddition to exclusion of related conditions, the presence of two of the following three criteria: 1)oligo- or anovulation, 2) clinical or biochemical signs of hyperandrogenism, and 3) polycysticovaries. Oligo- or anovulation presents as irregular menses, and hyperandrogenism may presentas acne, increased body hair, and rarely, clitorimegaly (a transverse clitoral diameter greaterthan 3 mm). The severity of hirsutism may be assessed using the Ferriman-Gallwey Scoringsystem. A score ranging from 0 (no hair) to 4 (frankly virile [extensive hair growth]) is assessedfor each of nine body areas most sensitive to androgens. These sites include the upper lip, chin,chest, abdomen, suprapubic region, arms, thighs, upper back, and lower back. A score of 8 ormore is considered significant and suggestive of increased androgen concentrations. Theseverity of acne and hirsutism, however, may not correlate well with the concentrations ofandrogens because the response of the androgen-dependent follicle to androgen excess variesconsiderably between and within persons. Therefore, total and free testosterone measurementmay be supportive of this diagnosis.

A number of risk factors for PCOS have been outlined at various stages of development.One of these factors is premature adrenarche, which is the appearance of pubic hair beforeage 8 years without other evidence of puberty. Whether peripubertal obesity predisposes toPCOS remains to be determined. Those who have risk factors for insulin resistance such asacanthosis nigricans or a family history of type 2 diabetes and cardiovascular disease may be atincreased risk for PCOS. Acanthosis nigricans is a velvety hyperpigmentation and thickening ofthe skin on the nape of the neck, axilla, and other body folds. It is a nonspecific sign of insulinresistance.

A number of disorders may be considered in the differential diagnosis of PCOS but are notassociated with signs of androgen excess. Patients who have hypothyroidism may beoverweight and have menstrual disturbances, but they typically have other symptoms, includinghair loss, constipation, and dry skin. A common symptom of Cushing syndrome is sudden weightgain. In addition, affected patients have signs or symptoms of cortisol excess such as muscleweakness; facial rounding and plethora; easy bruising; and multiple wide, purplish striae on theabdomen, not the narrow hypopigmented type exhibited by the patient in the vignette. In additionto central obesity and high blood pressure, patients who have metabolic syndrome haveelevated fasting glucose and triglyceride values and decreased high-density lipoproteincholesterol values. Metabolic syndrome is common in those who have PCOS, and such patientsshould be screened regularly for metabolic syndrome. Physiological anovulation becomes lesslikely as an explanation for irregular menses 3 years after menarche.

References:

Hassan A, Gordon CM. Polycystic ovary syndrome update in adolescence. Curr Opin Pediatr.2007;19:389-397. Abstract available at:

Copyright © 2010 by the American Academy of Pediatrics page 99

Page 100: AAP PREP 2010

2010 PREP SA on CD-ROM

http://www.ncbi.nlm.nih.gov/pubmed/17630601?ordinalpos=10&itool=EntrezSystem2.PEntrez.Pubmed.Pubmed_ResultsPanel.Pubmed_DefaultReportPanel.Pubmed_RVDocSum

Rosenfeld RL. Clinical practice. Hirsutism. N Engl J Med. 2005;353:2578-2588. Extract availableat: http://content.nejm.org/cgi/content/extract/353/24/2578

Rosenfield RL. Clinical review: identifying children at risk for polycystic ovary syndrome. J ClinEndocrinol Metab. 2007;92:787-796. Available at:http://jcem.endojournals.org/cgi/content/full/92/3/787

Schneider MB, Brill SR. Obesity in children and adolescents. Pediatr Rev. 2005;26:155-162.Available at: http://pedsinreview.aappublications.org/cgi/content/full/26/5/155

Wornham WL. Complementary and alternative medicine for gynecology patients. In: Emans SJH,Laufer MR, Goldstein DP, eds. Pediatric and Adolescent Gynecology. 5th ed. Philadelphia, Pa:Lippincott Williams & Wilkins, 2005:976-987

Copyright © 2010 by the American Academy of Pediatrics page 100

Page 101: AAP PREP 2010

2010 PREP SA on CD-ROM

Question: 41

You are called to the pediatric ward to evaluate a 10-month-old infant awaiting liver transplantwho began vomiting blood after coughing. She has biliary atresia and a failed Kasai procedure.On physical examination, the thin infant has jaundice and a protuberant abdomen with visibleveins (Item Q41). Her heart rate is 150 beats/minute, respiratory rate is 30 breaths/minute, andblood pressure is 65/40 mm Hg. A moderate amount of bright red blood is on her bed sheetsnear the head of her bed.

Of the following, the MOST likely cause of her bleeding is

A. anterior epistaxis

B. esophageal varices

C. gastric stress ulcer

D. infectious enterocolitis

E. Meckel diverticulum

Copyright © 2010 by the American Academy of Pediatrics page 101

Page 102: AAP PREP 2010

2010 PREP SA on CD-ROM

Preferred Response: BCritique: 41

Gastrointestinal (GI) bleeding is a common pediatric problem, but life-threatening GIhemorrhage is relatively rare. The location of acute bleeding (above or below the ligament ofTreitz) can be determined initially by inserting a nasogastric tube. If blood is recovered, thesource is the upper GI tract. Common causes of hematemesis (vomiting of bright red blood)include swallowed blood (eg, from epistaxis, tonsillectomy), esophagitis, Mallory-Weiss tears,reactive gastritis (eg, "stress ulcer" due to alcohol, nonsteroidal anti-inflammatory drugs, criticalillness, radiation), peptic ulcer, and esophageal varices (due to portal hypertension).

The child described in the vignette has significant liver disease caused by underlying biliaryatresia with resultant portal hypertension, as evidenced by the dilated collateral veins seen onher abdominal wall. Portal hypertension, defined as portal pressures greater than 10 mm Hg, iscaused by obstruction to portal blood flow due to intra- or extrahepatic causes. Collateralvessels often form to shunt blood to the systemic circulation, but portal flow remains high, as doportal pressures. Collateral vessels are seen commonly in the esophagus; they may dilate andrupture as the result of chronically elevated pressures. In addition, sudden rises in pressure,such as seen with coughing, can produce acute rupture.

Although epistaxis and gastric stress ulcers can cause hematemesis, esophageal varicesare a much more likely cause of this child’s bleeding because of her underlying clinical condition.Infectious enterocolitis and Meckel diverticulum are both causes of lower GI bleeding andtypically present as either hematochezia (passage of bright red blood per rectum) or melena(passage of black, tarry stools).

References:

Boyle JT. Gastrointestinal bleeding in infants and children. Pediatr Rev. 2008;29:39-52. Availableat: http://pedsinreview.aappublications.org/cgi/content/full/29/2/39

Mehta R, Shanley TP. Gastrointestinal bleeding. In: Wheeler DS, Wong HR, Shanley TP, eds.Pediatric Critical Care Medicine: Basic Science and Clniical Evidence. New York, NY:Springer-Verlag London Limited; 2007:1013-1021

Suchy FJ. Portal hypertension and varices. In: Kliegman RM, Behrman RE, Jenson HB, StantonBF, eds. Nelson Textbook of Pediatrics. 18th ed. Philadelphia, Pa: Saunders Elsevier; 2007:1709-1711

Copyright © 2010 by the American Academy of Pediatrics page 102

Page 103: AAP PREP 2010

2010 PREP SA on CD-ROM

Question: 42

At her initial health supervision visit with you, a 13-year-old girl confides that she has just movedto the United States to join her father because her family could not afford treatment to make hergrow taller in Guatemala. Her father tells you that the doctors diagnosed a chromosome problemwhen she was 8 years old, handing you a letter from a doctor in Guatemala stating that she hasTurner syndrome. On physical examination, the girl is 130 cm in height, and her weight isappropriate for height. She has Sexual Maturity Rating 3 pubic hair but no breast tissue. She hasa high-arched palate and a triangular-shaped face with a low posterior hairline. You tell the girland her father that treatment with growth hormone may be possible, but certain tests must beundertaken to determine if the growth hormone will be effective.

Of the following, the MOST helpful test in determining the usefulness of growth hormone is

A. bone age radiography

B. pelvic ultrasonography

C. serum estradiol measurement

D. serum follicle-stimulating hormone measurement

E. serum growth hormone measurement

Copyright © 2010 by the American Academy of Pediatrics page 103

Page 104: AAP PREP 2010

2010 PREP SA on CD-ROM

Preferred Response: ACritique: 42

The girl described in the vignette has features of Turner syndrome, including significantshort stature. Growth hormone has been shown to increase linear growth rate and adult heightin girls who have Turner syndrome as well as in many other conditions associated with shortstature. The average increase in adult height afforded by treatment is about 1/3 of an inch or alittle less than 1 cm for each year of treatment. Therefore, the most important determinant of thelong-term growth response to growth hormone is physiologic maturation, as reflected in thebone age radiograph. For example, a 13-year-old girl who has a bone age of 11 years wouldhave several more years of growth left compared with a 13-year-old girl who has a bone age of13 years.

A girl who has Turner syndrome generally has streak ovaries and an immature uterus,which is evident on pelvic ultrasonography, but development of the ovaries and uterus isimportant for linear growth only if there is estrogen production, which advances epiphysealmaturation. Measurement of serum estradiol is not useful for the girl described in the vignettebecause she shows no evidence of estrogen effect, and most girls who have Turner syndromedo not release much estrogen from their poorly functioning ovarian remnants. In addition, it isdifficult to measure low circulating concentrations of estradiol accurately. Measurement ofserum follicle-stimulating hormone (FSH) should confirm that this girl has ovarian failure becausevalues usually are elevated in girls who have Turner syndrome. FSH is the pituitary hormone thatdirectly stimulates follicular function, and past the usual age at puberty, values rise if there isovarian failure, just as they do at menopause. Rarely do girls who have Turner syndrome havefunctioning ovaries and normal FSH concentrations. Elevated FSH values are common in girlswho have Turner syndrome after the age of 11 years. Because girls who have Turnersyndrome are not deficient in growth hormone, assessment of the capacity of growth hormonerelease is not necessary. Further, a single growth hormone value rarely is useful in diagnosingthe cause of short stature in children because growth hormone is released in response tovarious stimuli, and unstimulated concentrations are low or not measurable in the blood.

References:

Bannink EM, Raat H, Mulder PG, de Muinck Keizer-Schrama SM. Quality of life after growthhormone therapy and induced puberty in women with Turner syndrome. J Pediatr. 2006;148:95-101. Abstract available at:http://www.ncbi.nlm.nih.gov/pubmed/16423606?ordinalpos=74&itool=EntrezSystem2.PEntrez.Pubmed.Pubmed_ResultsPanel.Pubmed_DefaultReportPanel.Pubmed_RVDocSum

Baxter L, Bryant J, Cave CB, Milne R. Recombinant growth hormone for children andadolescents with Turner syndrome. Cochrane Database Syst Rev. 2007;1:CD003887. Availableat: http://www.mrw.interscience.wiley.com/cochrane/clsysrev/articles/CD003887/frame.html

Loscalzo ML. Turner syndrome. Pediatr Rev. 2008;29:219-227. Available at:http://pedsinreview.aappublications.org/cgi/content/full/29/7/219

Copyright © 2010 by the American Academy of Pediatrics page 104

Page 105: AAP PREP 2010

2010 PREP SA on CD-ROM

Sybert VP, McCauley E. Turner's syndrome. N Engl J Med. 2004;351:1227-1238. Available at:http://content.nejm.org/cgi/content/full/351/12/1227

Copyright © 2010 by the American Academy of Pediatrics page 105

Page 106: AAP PREP 2010

2010 PREP SA on CD-ROM

Question: 43

A 10-year-old girl is having difficulty completing her schoolwork in class without making errors.She is capable of doing the work with extended time but not within time limits. She is trying veryhard to keep up with her classmates, but she works very slowly. Her parents report that she isable to do her homework but requires a lot of time. Her parents bring you a copy of her schooltesting results, including both the 10 subtest scores and the 4 index scores of the WechslerIntelligence Scale for Children-Fourth Edition.

Of the following, the score that is MOST likely to be affected for this girl is the

A. full-scale intelligence quotient

B. perceptual reasoning factor

C. processing speed factor

D. verbal comprehension factor

E. working memory factor

Copyright © 2010 by the American Academy of Pediatrics page 106

Page 107: AAP PREP 2010

2010 PREP SA on CD-ROM

Preferred Response: CCritique: 43

The Wechsler Intelligence Scale for Children (WISC-IV) is used to assess a child’s mentalability in comparison with the abilities of other children of the same age via a numerical scorereferred to as the full-scale intelligence quotient (IQ). The scores on a cognitive test are used topredict how a person will function academically.

The WISC-IV consists of 15 subtests: 10 core subtests and 5 additional optional subtests.The subtests are grouped into four composite scales known as factor scores. The individualfactor scores provide more detailed information regarding the child’s mental ability than does thefull-scale IQ score. The verbal comprehension factor assesses skills such as verbal knowledgeand how a person uses verbal skills in novel situations. The perceptual reasoning factorevaluates the ability to reason and organize material that is seen without the use of words. Theworking memory factor is based on the ability to remember information and either to manipulate itor use it to perform calculations. The processing speed factor assesses the speed ofprocessing information.

The girl described in the vignette is unable to complete tasks within time limits, indicating thatshe has difficulty with the speed of processing simple visual information without making amistake. Therefore, the score that is most likely to be affected for her is the processing speedfactor.

Factor scores on the WISC-IV should be fairly similar. A substantial difference betweenscores suggests a greater likelihood of a learning or cognitive disability.

References:

Braaten EB, Norman D. Intelligence (IQ) testing. Pediatr Rev. 2006;27:403-408. Available at:http://pedsinreview.aappublications.org/cgi/content/full/27/11/403

Wechsler D. Manual for the Wechsler Intelligence Scale for Children-Revised. New York, NY:Psychological Corporation; 1974

Copyright © 2010 by the American Academy of Pediatrics page 107

Page 108: AAP PREP 2010

2010 PREP SA on CD-ROM

Question: 44

A 7 year-old-girl presents to your office with a 1-day history of a temperature of 38.9°C. Notablefindings from her past medical history include static encephalopathy, seizure disorder, andrecurrent urinary tract infections. She is receiving intermittent straight catheterization andtrimethoprim-sulfamethoxazole prophylaxis. Her medications also include phenytoin, albuterol vianebulizer, ipratropium, and ranitidine. Urinalysis reveals more than 100 white blood cells per high-power field and is positive for leukocyte esterase and nitrites.

Of the following, the BEST option for oral empiric therapy pending culture results is

A. amoxicillin

B. azithromycin

C. ciprofloxacin

D. nitrofurantoin

E. trimethoprim-sulfamethoxazole

Copyright © 2010 by the American Academy of Pediatrics page 108

Page 109: AAP PREP 2010

2010 PREP SA on CD-ROM

Preferred Response: CCritique: 44

The history of recurrent urinary tract infections (UTIs) and development of a UTI whilereceiving trimethoprim-sulfamethoxazole (TMP-SMX) described for the girl in the vignette raiseconcern for a resistant pathogen, making ciprofloxacin, a fluoroquinolone, the best option fortherapy, pending culture results and sensitivity testing. Azithromycin is not indicated fortreatment of UTIs. Amoxicillin and nitrofurantoin may have roles in treatment of a simple UTI butare not adequate in the possible presence of a resistant pathogen. The development of this UTIwhile the patient is receiving TMP-SMX prophylaxis suggests that the infecting organism is

resistant to this agent. Fluoroquinolones are derivatives of the urinary tract agent nalidixic acid that have a very

broad antimicrobial spectrum, excellent oral absorption, and relatively few adverse effects.Studies in juvenile animals demonstrated arthropathy, which initially limited their investigation anduse in children. Subsequent trials and analyses of uncontrolled use of fluoroquinolones inpediatrics have documented no increased incidence of arthropathy. However, fluoroquinoloneuse to date generally has been associated with rapid development of resistant organisms. Themost recent recommendations of the Committee on Infectious Diseases of the AmericanAcademy of Pediatrics suggest that fluoroquinolone use in pediatrics be restricted to situationsin which the pathogen is multidrug-resistant and there is no safe and effective alternative orwhen parenteral therapy is not feasible and there is no effective alternative oral agent. Suchsituations might include UTIs caused by multidrug-resistant gram-negative rods, includingPseudomonas aeruginosa; gastrointestinal and respiratory tract infections caused by resistantgram-negative organisms; and chronic or acute osteomyelitis caused by P aeruginosa. Inaddition, a fluoroquinolone may be indicated for treatment or prevention of anthrax and fortreatment of mycobacterial infection with sensitive strains.

Clinicians also must be aware of potential drug interactions that may occur whenfluoroquinolones are used. Antacids containing aluminum, magnesium, or calcium as well aswarfarin may decrease the absorption of fluoroquinolones. Fluoroquinolones may increasecaffeine concentrations or the anticoagulant effects of warfarin. Nonsteroidal anti-inflammatorydrugs may potentiate the central nervous system effects of fluoroquinolones and should beused cautiously in patients receiving fluoroquinolone therapy. Fluoroquinolones also can inhibitpotassium channels in cardiac tissue and increase the risk for arrhythmias associated with aprolonged QT interval. Finally, they may cause hypo- or hyperglycemia when administered topatients receiving an antidiabetes agent or insulin.

As a result of reviewing this information, do you intend to make a change in practiceto provide better patient care?Yes No

References:

Committee on Infectious Diseases. The use of systemic fluoroquinolones. Pediatrics.2006;118:1287-1292. Available at:http://pediatrics.aappublications.org/cgi/content/full/118/3/1287

Copyright © 2010 by the American Academy of Pediatrics page 109

Page 110: AAP PREP 2010

2010 PREP SA on CD-ROM

Hooper DC. Fluoroquinolones. UpToDate Online 16.3. 2008. Available for subscription at:http://www.utdol.com/online/content/topic.do?topicKey=antibiot/8621&selectedTitle=1~150&source=search_result

Oliphant CM, Green GM. Quinolones: a comprehensive review. Am Fam Physician. 2002;65:455-464. Available at: http://www.aafp.org/afp/20020201/455.html

Copyright © 2010 by the American Academy of Pediatrics page 110

Page 111: AAP PREP 2010

2010 PREP SA on CD-ROM

Question: 45

You are evaluating a 5-year-old boy who has acquired immunodeficiency syndrome and whoserecent CD4 percentage is 18%. He presents today with a 5-day history of a temperature to38.9°C, increased work of breathing, and cough. His mother states that he is normally veryactive but over the past 2 days has been having a problem catching his breath just walking tothe bathroom. Physical examination shows a tired-appearing boy in mild respiratory distress. Hehas a temperature of 38.4°C, respiratory rate of 28 breaths/min, some mild nasal flaring,moderate intercostal retractions, and scattered crackles at the lung bases bilaterally. Oxygen

saturation by pulse oximetry on room air is 88%, and an arterial blood gas reveals a Pao2 of 60mm Hg. A chest radiograph demonstrates bilateral perihilar infiltrates (Item Q45).

Of the following, the MOST appropriate antimicrobial agent to start is

A. azithromycin

B. cefotaxime

C. clindamycin

D. trimethoprim-sulfamethoxazole

E. vancomycin

Copyright © 2010 by the American Academy of Pediatrics page 111

Page 112: AAP PREP 2010

2010 PREP SA on CD-ROM

Preferred Response: DCritique: 45

The child described in the vignette presents with classic signs and symptoms ofPneumocystis jiroveci (formerly Pneumocystis carinii) pneumonia. This pathogen is animportant cause of pulmonary infections in immunocompromised patients. Characteristic signsand symptoms include dyspnea at rest, tachypnea, nonproductive cough, fever, and hypoxiawith an increased oxygen requirement. The intensity of the signs and symptoms can vary, andonset may be acute and fulminant. Chest radiographs frequently demonstrate diffuse bilateralinterstitial or alveolar disease (Item C45). Pneumocystis pneumonia is diagnosed definitively bydemonstrating the presence of the organism in lung tissue or respiratory secretions. Themortality rate in immunocompromised patients ranges from 5% to 40% if treated and approaches100% if untreated. Treatment should not be delayed until a definitive diagnosis is established inan immunocompromised patient who has a clinically compatible illness.

Trimethoprim-sulfamethoxazole is the drug of choice for therapy of a Pneumocystisinfection. It also is used as prophylaxis for children whose CD4 percentage is less than 15%.Azithromycin, cefotaxime, clindamycin, and vancomycin have no activity against P jiroveci.Patients who are unable to tolerate trimethoprim-sulfamethoxazole or who have severe diseaseand do not respond to trimethoprim-sulfamethoxazole after 5 to 7 days may be treated withintravenous pentamidine as an alternate agent.

In pediatrics, the sulfonamide agents alone have several uses. They may be used in thetreatment of acute urinary tract infections, inflammatory bowel diseases, burns, umbilical cordcare, vaginitis, nongonococcal urethritis due to Chlamydia, toxoplasmosis, and bacterialconjunctivitis. Trimethoprim-sulfamethoxazole remains the drug of choice for susceptible isolatesin the treatment of urinary tract infections; prevention and treatment of P jiroveci infections inimmunocompromised patients; and gastroenteritis due to Salmonella, Shigella, and Isosporabelli. It also can be used in the treatment of upper respiratory tract infections (eg, otitis media,sinusitis, pneumonitis) due to sensitive organisms, although it is not the first-line antimicrobialagent for these infections. Trimethoprim-sulfamethoxazole also is effective in the treatment ofinfections caused by Stenotrophomonas maltophilia, Burkholderia cepacia, Brucella, andmethicillin-resistant Staphylococcus aureus.

References:

American Academy of Pediatrics. Pneumocystis jiroveci infections. In: Pickering LK, Baker CJ,Kimberlin DW, Long SS, eds. Red Book: 2009 Report of the Committee on Infectious Diseases.28th ed. Elk Grove Village, Ill: American Academy of Pediatrics; 2009:536-540

Burchett SK, Pizzo PA. HIV infection in infants, children, and adolescents. Pediatr Rev.2003;24:186-194. Available at: http://pedsinreview.aappublications.org/cgi/content/full/24/6/186

Smith CL, Powell KR. Review of the sulfonamides and trimethoprim. Pediatr Rev. 2000;21:368-371. Available at: http://pedsinreview.aappublications.org/cgi/content/full/21/11/368

Walzer PD, Smulian AG. Pneumocystis species. In: Mandell GL, Bennett JE, Dolan R, eds.

Copyright © 2010 by the American Academy of Pediatrics page 112

Page 113: AAP PREP 2010

2010 PREP SA on CD-ROM

Mandell, Douglas, and Bennett’s Principles and Practice of Infectious Diseases. 6th ed.Philadelphia, Pa: Elsevier Churchill Livingstone; 2005:3080-3094

Zinner SH, Mayer KH. Sulfonamides and trimethoprim. In: Mandell GL, Bennett JE, Dolan R, eds.Mandell, Douglas, and Bennett’s Principles and Practice of Infectious Diseases. 6th ed.Philadelphia, Pa: Elsevier Churchill Livingstone; 2005:440-450

Copyright © 2010 by the American Academy of Pediatrics page 113

Page 114: AAP PREP 2010

2010 PREP SA on CD-ROM

Question: 46

A 14-year-old boy presents for a sports physical examination prior to participating in football. Heis required to undergo a urinalysis as part of the evaluation. His temperature is 37.1°C, heart rateis 74 beats/min, respiratory rate is 14 breaths/min, and blood pressure is 114/68 mm Hg. Findingson his physical examination are unremarkable. His urinalysis demonstrates a urine specificgravity of 1.025, pH of 6.5, 3+ protein, and no blood. Microscopy findings are negative.

Of the following, the MOST appropriate next step is to

A. collect a first-morning urine specimen to measure protein and creatinine concentrations

B. measure serum albumin concentration and obtain renal function studies

C. obtain a 24-hour urine collection to measure protein and creatinine concentrations

D. quantitate the proteinuria with a random urine collection to measure protein and creatinineconcentrations

E. repeat the urinalysis in 1 year because of the lack of coexisting hematuria

Copyright © 2010 by the American Academy of Pediatrics page 114

Page 115: AAP PREP 2010

2010 PREP SA on CD-ROM

Preferred Response: ACritique: 46

The adolescent described in the vignette has asymptomatic, isolated proteinuria without anysigns or symptoms of a systemic disease. Furthermore, he has a normal blood pressure andlacks any associated hematuria. These pertinent negative features suggest that he is unlikely tohave severe renal disease.

Proteinuria typically is detected on urinalysis by the urine dipstick, which screens foralbuminuria. The reaction of urinary albumin tetrabromphenol blue impregnated within the dipsticktest results in a color change to green. The result can be false-negative if the urine is dilute(urine specific gravity <1.015) or false-positive if the urine is alkaline or highly concentrated(urine specific gravity >1.030). Finally, the dipstick test does not detect other low-molecular

weight proteins such as beta2-microglobulin.The most likely diagnosis for an adolescent who has asymptomatic, isolated proteinuria is

postural or orthostatic proteinuria, which occurs in 2% to 5% of children of this age and isbelieved to have an excellent prognosis. The amount of urinary protein excreted in the supineposition is normal, but that excreted in the upright position may be as much as tenfold higher thannormal. The mechanism is unclear but appears to be a subtle glomerular defect that isexacerbated by a tendency for protein filtration in the upright position. The upright position isbelieved to cause venous pooling in the legs, followed by renal vein congestion and increasedefferent arteriolar resistance and the subsequent tendency to protein filtration. If orthostaticproteinuria is confirmed, an annual urinalysis is recommended to monitor for progression ofproteinuria. If proteinuria has resolved at the next annual visit, further follow-up is not required.

A random urine sample in the clinical setting can be used to diagnose orthostatic proteinuriaby documenting elevated urinary protein excretion. Serologic testing, renal function tests, serumalbumin concentrations, and quantitative measurement of urine protein excretion usually are notperformed until orthostatic proteinuria has been excluded. Because of the efficacy of easilyobtained random urine samples, 24-hour urine collections are not recommended for theevaluation of proteinuria in the pediatric patient. Finally, patients who have nonorthostaticproteinuria should be followed relatively closely, with a repeat urine check in 1 to 2 months tomonitor for the development of hematuria or worsening of proteinuria, which signals progressionof the underlying disorder that may warrant a renal biopsy.

References:

Bergstein JM. A practical approach to proteinuria. Pediatr Nephrol. 1999;13:697-700. Abstractavailable at: http://www.ncbi.nlm.nih.gov/pubmed/10502130

Devarajan P. Mechanisms of orthostatic proteinuria: lessons from a transplant donor. J Am SocNephrol. 1993;4:36-39. Available at: http://jasn.asnjournals.org/cgi/reprint/4/1/36

Hogg RJ, Portman RJ, Milliner D, Lemley KV, Eddy A, Ingelfinger J. Evaluation and management ofproteinuria and nephrotic syndrome in children: recommendations from a pediatric nephrologypanel established at the National Kidney Foundation Conference on Proteinuria, Albuminuria,Risk, Assessment, Detection, and Elimination (PARADE). Pediatrics. 2000;105:1242-1249.

Copyright © 2010 by the American Academy of Pediatrics page 115

Page 116: AAP PREP 2010

2010 PREP SA on CD-ROM

Available at: http://pediatrics.aappublications.org/cgi/content/full/105/6/1242

Vehaskari VM, Rapola J. Isolated proteinuria: analysis of a school-age population. J Pediatr.1982;101:661-668. Abstract available at: http://www.ncbi.nlm.nih.gov/pubmed/7131137

Copyright © 2010 by the American Academy of Pediatrics page 116

Page 117: AAP PREP 2010

2010 PREP SA on CD-ROM

Question: 47

An 18-year-old girl presents with a 12-month history of severe nasal congestion and anosmia.She was diagnosed with allergic rhinitis at age 13 and has been receiving allergenimmunotherapy for the past 3 years. Despite allergy shots, allergy medication (oral antihistamineand nasal corticosteroid), and two 21-day courses of antibiotics, her symptoms have persisted.She describes her rhinorrhea as thick and "peanut buttery." On physical examination, her heightand weight are at the 75th percentile for age and she has bilateral nasal polyps. The remainderof the examination results are normal. Computed tomography scan of her sinuses showscomplete unilateral opacification of the right maxillary sinus (Item Q47).

Of the following, the MOST likely diagnosis is

A. allergic fungal sinusitis

B. allergic rhinitis

C. chronic bacterial sinusitis

D. cystic fibrosis

E. primary ciliary dyskinesia

Copyright © 2010 by the American Academy of Pediatrics page 117

Page 118: AAP PREP 2010

2010 PREP SA on CD-ROM

Preferred Response: ACritique: 47

Sinusitis is an infection of the nasal passages and sinuses. In the past, sinusitis wasclassified as acute (less than 4 weeks in duration), subacute (4 to 12 weeks), and chronic(more than 12 weeks). Recent nomenclature has altered this classification to include chronicsinusitis with and without nasal polyposis. The finding of nasal polyps for the girl described inthe vignette should raise the suspicion for cystic fibrosis. However, there are other, morecommon causes of nasal polyposis (Item C47), and the teenager lacks any other clinical featuresconsistent with cystic fibrosis (eg, poor weight gain, gastrointestinal symptoms, clubbing,pansinusitis, recurrent pneumonias).

The finding of unilateral sinus disease, "peanut buttery" mucin, nasal polyps, and a history ofatopy makes allergic fungal sinusitis (AFS) the most likely diagnosis for this girl. This conditionaccounts for 10% to 15% of chronic rhinosinusitis in adolescents and young adults. The exactmechanism is unknown, but exposure to a particular fungus (usually Bipolaris or Curvularia)results in local inflammation, eosinophilic tissue infiltration, and mucus production. The fiveprimary characteristics of AFS are:

–Production of eosinophilic-containing, noninvasive fungal hyphae

–Nasal polyposis

–Characteristic radiographic findings (unilateral sinus disease)

–Immunocompetence

–Immunoglobulin (Ig) E to fungi, as determined by skin testing or serum-specific IgEThe management of AFS involves oral corticosteroids after surgical debridement, but

recurrences are frequent. Uncomplicated allergic rhinitis rarely results in polyps or thick mucin. Further, allergic rhinitis

should respond promptly to usual therapies, such as oral antihistamines, nasal steroids, andallergen immunotherapy. Chronic bacterial sinusitis can present identically to allergic fungalsinusitis. Although polyps may be present in bacterial sinusitis, improvement is expected with aprolonged course of antibiotics. Ciliary dyskinesia represents a defect in the dynein arm or radialspokes of cilia. Patients typically present with recurrent otitis media and pneumonia. Situsinversus is seen in approximately 50% of patients and is termed Kartagener syndrome. Patientsexperiencing recurrent sinusitis or chronic sinusitis should be evaluated for possible underlyingrisk factors.

References:

Mazur LJ, Kim J, American Academy of Pediatrics Committee on Environmental Health. Spectrumof noninfectious health effects from molds. Pediatrics. 2006;118:e1909-e1926.http://pediatrics.aappublications.org/cgi/content/full/118/6/e1909

Meltzer EO, Jamilos DL, Hadley JA, et al; American Academy of Allergy, Asthma and Immunology(AAAAI); American Academy of Otolayngic Allergy (AAOA); American Academy ofOtolaryngology–Head and Neck Surgery (AAO-HNS); American College of Allergy, Asthma and

Copyright © 2010 by the American Academy of Pediatrics page 118

Page 119: AAP PREP 2010

2010 PREP SA on CD-ROM

Immunology (ACAAI); American Rhinologic Society (ARS). Rhinosinusitis: establishing definitionsfor clinical research and patient care. J Allergy Clin Immunol. 2004;114:155-212

Pappas DE, Hendley JO. Sinusitis. In: Kliegman RM, Behrman RE, Jenson HB, Stanton BF, eds.Nelson Textbook of Pediatrics. 18th ed. Philadelphia, Pa: Saunders Elsevier; 2007:1749-1752

Taylor A, Adam HM. In brief: sinusitis. Pediatr Rev. 2006;27:395-397. Available at:http://pedsinreview.aappublications.org/cgi/content/full/27/10/395

Copyright © 2010 by the American Academy of Pediatrics page 119

Page 120: AAP PREP 2010

2010 PREP SA on CD-ROM

Question: 48

The mother of one of your patients calls frantically because she just found her 2-year-olddaughter with an open bottle of prenatal vitamins and several of the tablets in her mouth. Thechild is acting normally. The mother reports that the label says there is 30 mg of elemental ironper tablet and five tablets are missing from the bottle she picked up at the pharmacy thismorning. Her daughter weighs 25 lb.

Of the following, the MOST appropriate advice to give the mother is to

A. bring the child to office in the morning for assessment of serum iron concentration

B. give the child activated charcoal

C. give the child syrup of ipecac

D. observe the child at home for symptoms

E. take the child to the nearest emergency department

Copyright © 2010 by the American Academy of Pediatrics page 120

Page 121: AAP PREP 2010

2010 PREP SA on CD-ROM

Preferred Response: DCritique: 48

Iron overdoses continue to be common among children younger than 6 years of age. Most ofthese ingestions are unintentional and do not result in significant toxicity. However, iron ingestioncan cause fatalities, especially in the setting of an intentional ingestion or exposure to an adultpreparation.

Management of iron ingestion begins with determining, if possible, how much elemental ironwas ingested and if the patient is symptomatic. Any child who is symptomatic within 6 hoursafter ingesting iron, regardless of the estimated dose, or if the dose is unknown, should bebrought to medical attention. In the asymptomatic child, ingestions of less than 40 mg/kg ofelemental iron are not significant, and the patient can be observed at home. The mother in thevignette can be reassured that because her child ingested 13.2 mg/kg of elemental iron (<40mg/kg), she is unlikely to develop toxicity and can be observed at home. If she remainsasymptomatic for more than 6 hours, no further evaluation or treatment is necessary. If shedevelops symptoms, she should be taken to an emergency department for evaluation andtreatment.

For the patient who has a significant ingestion, laboratory evaluation should includemeasurement of a serum iron concentration within 4 hours of ingestion, serum electrolyte andaminotransferase determinations, a complete blood count, and coagulation tests. Laboratoryindicators of a potentially significant ingestion include serum iron concentration greater than 350

mcg/dL (62.7 mcmol/L), white blood cell count greater than 15.0x103/mcL (15.0x109/L), andserum glucose values greater than 150 mg/dL (8.3 mmol/L). In a symptomatic patient, abdominalradiography should be used to look for the presence of retained tablets in the gastrointestinaltract. If present, gastrointestinal decontamination using whole-bowel irrigation is indicated.Patients who have severe symptoms, anion gap acidosis, serum iron concentrations of greaterthan 500 mcg/dL (89.5 mcmol/L), or a significant number of pills visible on abdominal radiographyshould be treated with deferoxamine to chelate free circulating iron. Neither activated charcoal,which adsorbs iron poorly, nor syrup of ipecac is indicated for decontamination.

Iron is both a corrosive and a cellular toxin. The clinical phases of significant iron toxicity areattributable directly to these two mechanisms. Phase 1 (gastrointestinal phase) occurs between30 minutes to 6 hours after ingestion and includes vomiting, diarrhea, abdominal pain, andhematemesis or melena. Such signs and symptoms are caused by the agent’s corrosive effectson the gastrointestinal mucosa. Phase 2 (latent phase) occurs 6 to 12 hours after ingestion butcan last as long as 24 hours. Patients often are asymptomatic during this time while free iron istaken up into the reticuloendothelial organs. Phase 3 (shock, metabolic acidosis, hepatic failurephase) may be seen as early as 6 to 12 hours after ingestion and is the result of mitochondrialdysfunction and cell death. Phase 4 (bowel obstruction phase) occurs 2 to 8 weeks after theacute ingestion and results from gastrointestinal tract scarring following iron-induced corrosivedamage.

References:

Liebelt EL, Kronfol R. Acute iron poisoning. UpToDate Online 16.3. 2008. Available at:http://www.utdol.com/online/content/topic.do?topicKey=ped_tox/4912&selectedTitle=1~150&sou

Copyright © 2010 by the American Academy of Pediatrics page 121

Page 122: AAP PREP 2010

2010 PREP SA on CD-ROM

rce=search_result630

Manoguerra AS, Erdman AR, Booze LL, et al. Iron ingestion: an evidence-based consensusguideline for out-of-hospital management. Clin Toxicol (Phila). 2005;43:553-570. Abstractavailable at: http://www.ncbi.nlm.nih.gov/pubmed/16255338

Spanierman C. Toxicity, iron. eMedicine Specialties, Emergency Medicine, Toxicology. 2007.Available at: http://www.emedicine.com/emerg/topic285.htm

Copyright © 2010 by the American Academy of Pediatrics page 122

Page 123: AAP PREP 2010

2010 PREP SA on CD-ROM

Question: 49

The mother of a 5-month-old boy has come to your office seeking nutritional advice. Sheexclusively breastfed the infant for the first 4 months, then weaned the baby to a standard, cowmilk protein-based infant formula. One week after weaning, she noted that the baby "strainedwith stool." Because of her concerns regarding the development of constipation, the motherswitched him to a formula containing 2 mg/L iron.

Of the following, the MOST important dietary recommendation for this infant is to

A. add pureed vegetables to the diet

B. change to a cow milk protein-based formula containing 12 mg/L iron

C. change to a soy protein-based formula

D. continue the present regimen and supplement with 4 oz/day diluted apple juice

E. substitute oatmeal for rice cereal in the diet

Copyright © 2010 by the American Academy of Pediatrics page 123

Page 124: AAP PREP 2010

2010 PREP SA on CD-ROM

Preferred Response: BCritique: 49

For the infant described in the vignette, the most appropriate dietary recommendation is toreturn the infant to a formula fortified with 12 mg/L iron. Parents frequently use low-ironformulas in the false belief that iron-supplemented formulas induce constipation. However, thechange to a low iron-containing formula not only fails to alter the stooling pattern but can placethe infant at risk for iron deficiency.

Although the optimal iron content of infant formula has been debated, all available dataindicate that iron supplementation is essential for formula-fed infants to prevent a deficiencystate. For nursing infants, the lactoferrin content of human milk greatly enhances ironbioavailability and absorption, despite the milk’s low iron content. Therefore, during the first fewmonths, no added iron is required. After 6 months, however, breastfed infants are at risk foriron deficiency, unless dietary supplements are provided. For infants weaned to the bottle, thismay be accomplished by using a standard iron-fortified formula. For infants who continue tonurse after 6 months, iron-fortified stage 1 baby foods and cereals meet the requirement.

Supplementing human milk with formula or changing formulas are common responses to awide range of nonspecific complaints. In otherwise healthy and thriving infants, availableevidence neither supports this practice nor recommends the consumption of solids for otherthan nutritionally indicated purposes.

In situations such as that described in the vignette, caregivers often try a soy formula,usually on the advice of friends or family. In fact, if constipation is the perceived issue, soyformulas actually may exacerbate the problem. Proprietary soy preparations contain eithersucrose or glucose polymers as the constituent carbohydrate. Such di- or polysaccharides maybe absorbed more efficiently compared with lactose, particularly in young infants, resulting in afirmer stool. From a nutritional perspective, all commercial soy formulas are iron-fortified. Suchfortification is required for soy-based formulas because soy protein products contain phytates,which bind iron and other minerals intraluminally.

The consumption of poorly absorbed carbohydrates (eg, sorbitol in fruit juice) or fiber (fruits,vegetables, cereals) may aid in alleviating true constipation, but the infant described in thevignette is not suffering from constipation, and little direct evidence supports such dietaryinterventions. In addition, supplementing the infant’s diet with fruit juice may lead to excessiveweight gain due to increased intake of sugars.

Parents should be reassured that straining with the passage of a soft stool is commonduring the first few postnatal months. Discomfort prior to and during defecation results from theinfant’s inability to coordinate pelvic floor relaxation with the Valsalva maneuver and tostraighten the anal canal when lying down. Unless straining is accompanied by passage of hardstools, intervention is not indicated and does not ameliorate the problem.

References:

Abi-Hanna A, Lake AM. Constipation and encopresis in childhood. Pediatr Rev. 1998;19:23-31.Available at: http://pedsinreview.aappublications.org/cgi/content/full/19/1/23

American Academy of Pediatrics, Committee on Nutrition. Iron fortification of infant formulas.

Copyright © 2010 by the American Academy of Pediatrics page 124

Page 125: AAP PREP 2010

2010 PREP SA on CD-ROM

Pediatrics. 1999;104:119-123. Available at:http://pediatrics.aappublications.org/cgi/content/full/104/1/119

Dee DL, Sharma AJ, Cogswell ME, Grummer-Strawn LM, Fein SB, Scanlon KS. Sources ofsupplemental iron among breastfed infants during the first year of life. Pediatrics. 2008;122:S98-S104. Available at: http://pediatrics.aappublications.org/cgi/content/full/122/Supplement_2/S98

Hall RT, Carroll RE. Infant feeding. Pediatr Rev. 2000; 21:191-200. Available at:http://pedsinreview.aappublications.org/cgi/content/full/21/6/191

McQueen DA. In brief: the nutritional adequacy of mineral content of formulas. Pediatr Rev.1997;18:67-69. Available at: http://pedsinreview.aappublications.org/cgi/content/full/18/2/67

Pizarro F, Yip R, Dallman PR, Olivares M, Hertrampf E, Walter T. Iron status with different infantfeeding regimens: relevance to screening and prevention of iron deficiency. J Pediatr.1991;118:687-692. Abstract available at: http://www.ncbi.nlm.nih.gov/pubmed/2019922

Scariati PD, Grummer-Strawn LM, Fein SB, Yip R. Risk of diarrhea related to iron content ofinfant formula: lack of evidence to support the use of low-iron formula as a supplement forbreastfed infants. Pediatrics. 1997;99:e2. Available at:http://pediatrics.aappublications.org/cgi/content/full/99/3/e2

Copyright © 2010 by the American Academy of Pediatrics page 125

Page 126: AAP PREP 2010

2010 PREP SA on CD-ROM

Question: 50

A 4,050-g infant is delivered at 36 weeks’ gestation to a 28-year-old multiparous woman whohas poorly controlled diabetes mellitus. The vaginal delivery was not complicated by prolongedrupture of membranes, maternal fever, or perinatal asphyxia. You are called to examine the baby

at 1 hour of age because of decreasing bedside blood glucose values:

•Delivery room, 40.0 mg/dL (2.22 mmol/L)

•Newborn nursery at 15 minutes of age, 30.0 mg/dL (1.67 mmol/L)

•Newborn nursery at 45 minutes of age, 20.0 mg/dL (1.11 mmol/L)During physical examination, the infant is flexed and crying and has room air saturations of 92%by pulse oximetry. No heart murmur is audible, and breath sounds are equal in both lung fields.The abdomen is soft, and bowel sounds are normal.

Of the following, the BEST explanation for hypoglycemia in this baby is

A. intrauterine growth restriction

B. maternal diabetes mellitus

C. neonatal hypocalcemia

D. neonatal respiratory distress

E. neonatal sepsis

Copyright © 2010 by the American Academy of Pediatrics page 126

Page 127: AAP PREP 2010

2010 PREP SA on CD-ROM

Preferred Response: BCritique: 50

The newborn described in the vignette is large for gestational age (LGA), was born at 36weeks’ gestation, and has macrosomia, common fetal consequences of poorly controlledmaternal diabetes mellitus. Hypoglycemia in this newborn is due to a state of hyperinsulinemiathat developed in utero due to excess amounts of nutrients, including carbohydrates that werepresented to the fetus via the uteroplacental circulation.

Fetal blood glucose is 60% to 70% that of the mother. When the fetus is exposed tohyperglycemia, the fetal pancreas reacts with insulin secretion sufficient to regulate glucose intothe normal range. Increased insulin concentrations promote fetal growth and contribute to thedevelopment of visceromegaly, increased somatic growth, and an accrual of white fat. Whenthe umbilical circulation is interrupted at birth, the supply of glucose substrate to the newbornceases, yet the relative hyperinsulinemia persists for hours, and the pancreatic islet cellhyperresponsiveness to a glucose stimulus may not subside for a number of days.

Newborns who experience intrauterine growth restriction are small for gestational age andmay have insulin resistance. Neonatal hypocalcemia is seen more frequently in infants ofdiabetic mothers than other infants, but it does not cause hypoglycemia. The newborn in thevignette did not exhibit respiratory distress, although infants of diabetic mothers are at a greaterrisk for this condition at any given gestational age, and the work of breathing conceivably couldcontribute to hypoglycemia. Finally, sepsis, which is not an issue for this infant, may result inneonatal hypoglycemia due to hypermetabolism.

References:

Schwartz R, Teramo KA. Effects of diabetic pregnancy on the fetus and newborn. SeminPerinatol. 2000;24:120-135. Abstract available at:http://www.ncbi.nlm.nih.gov/pubmed/10805168

Thilo EH, Rosenberg AA. The newborn infant. In: Hay WW Jr, Levin MJ, Sondheimer JM,Deterding RR, eds. CURRENT Diagnosis & Treatment: Pediatrics. 19th ed. New York, NY: TheMcGraw-Hill Companies; 2009:Chapter 1. Available for subscription at:http://www.accessmedicine.com/content.aspx?aID=3396500

Copyright © 2010 by the American Academy of Pediatrics page 127

Page 128: AAP PREP 2010

2010 PREP SA on CD-ROM

Question: 51

As you are examining a newborn boy in the nursery, you notice that his testicles are not in thescrotum, and you are unable to palpate them in the inguinal canal. You tell his parents that youwill be following this closely because of the risk of infertility and malignancy and that surgicalcorrection may be necessary if the testicles do not descend into the scrotum.

Of the following, the MOST appropriate age at which surgical correction should take place if thetesticles do not descend is

A. 1 month

B. 2 months

C. 1 year

D. 2 years

E. 5 years

Copyright © 2010 by the American Academy of Pediatrics page 128

Page 129: AAP PREP 2010

2010 PREP SA on CD-ROM

Preferred Response: CCritique: 51

Cryptorchidism, the failure of one or both testes to descend into the scrotum duringdevelopment, occurs in approximately 5% of term males. It is more common in preterm infants.Ten percent of cases are bilateral, and cryptorchid testes may be either completely absent orundescended. Undescended testes may lie anywhere along the normal track of descent,including the inguinal canal and abdomen, or they may be ectopic and lie in another position,such as the femoral canal or suprapubic area. A variety of mechanical and hormonal factors areresponsible for the normal descent of testes, and disruption in any of them, including changes inabdominal pressure and deficient concentrations of gonadotropins or androgens, can causecryptorchidism. Most testes that are undescended at birth descend into the scrotum within thefirst few postnatal months.

In phenotypically male infants who have bilateral unpalpable testes at birth, the mostcommon diagnosis is undescended testes. However, other entities should be considered,including anorchia (ie, absent testes), congenital adrenal hyperplasia resulting in virilization of agenetic female, or that the infant is a true hermaphrodite. If indicated, a hormonal evaluationshould be performed. Radiologic tests, including ultrasonography and computed tomographyscan, may not help to discern the presence or absence of testes, so diagnostic laparoscopymay be considered. The examiner also should determine whether testes are truly undescendedor merely retractile. Retractile testes can be brought into the scrotum using a gentle sweepingtechnique. Approximately one third of boys who have retractile testes in infancy develop anacquired undescended testicle in middle to late childhood, indicating the need for a physicalexamination once or twice a year to monitor for this development.

Undescended testes are associated with a risk of testicular malignancy and infertility,torsion (which may be difficult to detect if the testicle is intra-abdominal), and psychologicaldisturbances. The risk of malignancy can be reduced but not eliminated by surgical correction(orchiopexy). However, bringing the testicle into the scrotum allows for better monitoring for thedevelopment of testicular masses. The risk of infertility is decreased with orchiopexy, andstudies have shown that fertility is improved by earlier correction. The current recommendationis for orchiopexy to take place by 1 year of age if testes have not spontaneously descended bythat age; some experts recommend orchiopexy at 6 months of age because descent after thattime is extremely rare.

References:

Adelman WP, Joffe A. Consultation with the specialist: testicular masses/cancer. Pediatr Rev.2005;26:341-344. Available at: http://pedsinreview.aappublications.org/cgi/content/full/26/9/341

Cooper CS, Docimo SG. Undescended testes (cryptorchidism). UpToDate Online 16.3. 2008.Available for subscription at:http://www.utdol.com/online/content/topic.do?topicKey=gen_pedi/16379&selectedTitle=1~53&source=search_result630

Elder JS. Disorders and anomalies of the scrotal contents. In: Kliegman RM, Behrman RE, Jenson

Copyright © 2010 by the American Academy of Pediatrics page 129

Page 130: AAP PREP 2010

2010 PREP SA on CD-ROM

HB, Stanton BF, eds. Nelson Textbook of Pediatrics. 18th ed. Philadelphia, Pa: SaundersElsevier; 2007:2260-2265

Haynes JH. Inguinal and scrotal disorders. Surg Clin North Am. 2006;86:371-381

Copyright © 2010 by the American Academy of Pediatrics page 130

Page 131: AAP PREP 2010

2010 PREP SA on CD-ROM

Question: 52

A 5-day-old boy who was born at 38 weeks’ gestation by dates, presents for his first visit afterhospital discharge. He appears clinically jaundiced and has a transcutaneous bilirubinmeasurement of 16.0 mg/dL (273.7 mcmol/L). He is exclusively breastfed. His birthweight was2,900 g and today’s weight is 2,800 g. According to his mother, his older brother was born atterm and had no problems with jaundice. Both the mother’s and baby’s blood types are O+, andresults of a direct Coombs test are negative. You obtain a blood sample, which confirms thebilirubin value.

Of the following, the MOST appropriate approach to managing this infant’s hyperbilirubinemia isto

A. initiate intravenous fluids in the hospital

B. initiate exposure to sunlight in the home

C. initiate phototherapy

D. interrupt breastfeeding for 12 to 24 hours

E. supplement breastfeeding with formula at home

Copyright © 2010 by the American Academy of Pediatrics page 131

Page 132: AAP PREP 2010

2010 PREP SA on CD-ROM

Preferred Response: ECritique: 52

The infant described in the vignette has breast milk jaundice (BMJ), which is defined asindirect hyperbilirubinemia in a breastfed newborn that develops after the first 4 to 7 daysfollowing birth, persists longer than physiologic jaundice, and has no other identifiable cause.

For healthy term infants who have BMJ and bilirubin values of 12 to 17 mg/dL (205 to 291

mcmol/L), the following options may be considered:•Ensure breastfeeding 8 to 12 times per day and remeasure the serum bilirubin in 12 to 24

hours, reassuring the mother about the relatively benign nature of BMJ. If needed,breastfeeding also can be supported with manual or electric pumps or other techniques

that assist the mother in maximizing the infant’s intake of breast milk.

•Continue breastfeeding and supplement with formula.

•Temporarily interrupt breastfeeding.Temporary interruption of breastfeeding for 12 to 24 hours usually results in a rapid

decrease in bilirubin concentrations but is only recommended for otherwise healthy term infantswhose serum bilirubin values are at least 20 mg/dL (342 mcmol/L). To maintain milk productionand minimize discomfort for the mother, breasts should be pumped and the milk stored for lateruse.

Phototherapy is not indicated for the healthy term infant who has uncomplicated jaundiceand a bilirubin valve of 16.0 mg/dL (273.7 mcmol/L). Use of sunlight is not recommendedbecause of reliability and safety issues involved in exposing an unclothed newborn to sunlight.

Use of supplemental intravenous fluids is invasive and generally unnecessary in the infantwho is otherwise healthy, well hydrated, and responding to therapy.

References:

American Academy of Pediatrics Section on Breastfeeding. Breastfeeding and the use of humanmilk. Pediatrics. 2005;115:496-506. Available at:http://pediatrics.aappublications.org/cgi/content/full/115/2/496

American Academy of Pediatrics Subcommittee on Hyperbilirubinemia. Clinical practice guideline:management of hyperbilirubinemia in the newborn infant 35 or more weeks of gestation.Pediatrics. 2004;114:297-316. Available at:http://pediatrics.aappublications.org/cgi/content/full/114/1/297

Engle WA, Tomashek KM, Wallman C, and the Committee on Fetus and Newborn. "Late-preterm"infants: a population at risk. Pediatrics. 2007;120:1390-1401. Available at:http://pediatrics.aappublications.org/cgi/content/full/120/6/1390

Gartner LM, Herrarias CT, Sebring RH. Practice patterns in neonatal hyperbilirubinemia.Pediatrics. 1998;101:25-31. Available at:http://pediatrics.aappublications.org/cgi/content/full/101/1/25

Copyright © 2010 by the American Academy of Pediatrics page 132

Page 133: AAP PREP 2010

2010 PREP SA on CD-ROM

Ip S, Chung M, Raman G, et al. Breastfeeding and Maternal and Infant Health Outcomes inDeveloped Countries. Evidence Assessment No. 153. AHRQ Publication No. 07-E007. Rockville,Md: Agency for Healthcare Research and Quality; April 2007. Available at:http://www.ahrq.gov/downloads/pub/evidence/pdf/brfout/brfout.pdf

Maisels MJ. Neonatal jaundice. Pediatr Rev. 2006;27:443-454. Available at:http://pedsinreview.aappublications.org/cgi/content/full/27/12/443

Mills JF, Tudehope D. Fibreoptic phototherapy for neonatal jaundice. Cochrane Database SystRev. 2001;1:CD002060

Copyright © 2010 by the American Academy of Pediatrics page 133

Page 134: AAP PREP 2010

2010 PREP SA on CD-ROM

Question: 53

You are working in an urgent care clinic when an 18-year-old boy is brought in by his motherbecause of slurred speech over the past hour. She explains that he has unrepaired cyanoticcongenital heart disease.

Of the following, the finding that is MOST likely to be related to the symptoms reported for thispatient is

A. hypoglycemia

B. hypothyroidism

C. iron deficiency

D. polycythemia

E. Wolff-Parkinson-White syndrome

Copyright © 2010 by the American Academy of Pediatrics page 134

Page 135: AAP PREP 2010

2010 PREP SA on CD-ROM

Preferred Response: CCritique: 53

Cerebrovascular accidents can occur in patients affected by congenital heart disease as aresult of perioperative surgical complications, hemodynamic abnormalities, intracranial abscess,or endocarditis. Patients who have chronic cyanotic heart disease, such as the boy described inthe vignette, are at additional risk for cerebrovascular accident due to paradoxic emboli and arelative anemia.

Paradoxic emboli result from the right-to-left intracardiac shunting of blood such that anembolus that originates in the systemic venous system avoids the filtering function of thepulmonary vasculature and crosses from the right to the left side in the heart, thereby gainingaccess to the systemic circulation, including the cerebral circulation. Stroke from relative anemiamay result from complex interactions of iron-deficient blood in the cerebral circulation, includingthe decreased oxygen-carrying capacity of iron-deficient blood. In addition, the microcytesformed under conditions of relative anemia are less deformable than those formed in an iron-richenvironment, and such "structural" erythrocyte changes may be associated with an increasedviscosity despite the lower hemoglobin concentration. The higher viscosity leads to moreresistance to flow, and the altered flow in the cerebral microvasculature can result incerebrovascular accident.

Hypoglycemia might lead to abnormal speech, both in terms of content and patterns, but thiswould not be expected to occur over the course of 1 hour in the absence of other symptoms.Hypothyroidism would not be expected to present with slurring of speech. Polycythemia can beassociated with increased viscosity and, therefore, decreased flow through small cerebralvessels, as mentioned previously. However, iron deficiency in the presence of polycythemiaposes a greater risk for stroke. Finally, Wolff-Parkinson-White syndrome would be expected topresent with palpitations, diaphoresis, or chest pain, but not slurring of speech in isolation.

References:

Oechslin E. Hematological management of the cyanotic adult with congenital heart disease. Int JCardiol. 2004;97(suppl 1):109—115

Spence MS, Balaratnam MS, Gatzoulis MA. Clinical update: cyanotic adult congenital heartdisease. Lancet. 2007;370:1530-1532

Copyright © 2010 by the American Academy of Pediatrics page 135

Page 136: AAP PREP 2010

2010 PREP SA on CD-ROM

Question: 54

A 5-year-old boy presents for evaluation of problems with impulsivity and hyperactivity. Histeacher has asked if he might have attention-deficit/hyperactivity disorder (ADHD). According tothe boy’s mother, he has had progressive behavioral problems and is having troubleconstructing and building with toys. No behavioral problems were noted in preschool. The familyhistory is negative for ADHD and mood disorders. The mother reports no disruptive changes athome or school. On physical examination, you note normal mental status and cranial nerves,crossed adduction when eliciting patellar reflexes, and sustained clonus at both ankles.

Of the following, the MOST appropriate next step is

A. observation with re-evaluation in 6 months

B. referral to neurology for diagnostic evaluation

C. referral to physical therapy for gait spasticity

D. referral to psychiatry for medication consultation

E. referral to psychology for psychoeducational evaluation

Copyright © 2010 by the American Academy of Pediatrics page 136

Page 137: AAP PREP 2010

2010 PREP SA on CD-ROM

Preferred Response: BCritique: 54

The most common diagnosis in school-age children who have disruptive behavior is ADHD.However, physicians must be vigilant when children present with cognitive, emotional, orbehavioral problems for the rare cases when such symptoms are due to cerebral pathology.Abnormalities evident on the motor examination can be a clue to such a secondary cause.

The boy described in the vignette has two "upper motor neuron signs" on his neurologicexamination. The crossed adduction sign means that tapping one patellar reflex leads toadduction of the contralateral leg. Similarly, sustained ankle clonus is abnormal. This boyrequires a prompt referral for specialty consultation with neurology. If such consultation cannotbe obtained readily, the case should be discussed by phone with a neurologist, who mayrecommend initial neuroimaging, preferably brain magnetic resonance imaging.

Referral to physical therapy, psychology, and psychiatry should be deferred until theneurodiagnostic issues have been addressed. The child need not be sent to the emergencydepartment, but clinical observation is not appropriate in the setting of abnormal neurologicexamination findings coupled with cognitive or behavioral changes.

The boy in the vignette has the childhood cerebral form of adrenoleukodystrophy, ademyelinating condition with mean age of onset of 7 years that manifests initially with poorexecutive function and emotional lability after a period of normal motor and cognitivedevelopment. It progresses to more severe visual, cognitive, and motor problems involvingspasticity and incoordination. A vegetative state and death follow. The disease is X-linked,caused by a mutation in the ABCD1 gene that results in defective oxidation of long-chain fattyacids in peroxisomes. Very long-chain fatty acids accumulate in myelin, adrenal glands, andelsewhere. Phenotypes can be milder, with adrenal insufficiency in adrenomyeloneuropathy andspastic paraplegia.

References:

Johnston MV. Neurodegenerative disorders of childhood. In: Kliegman RM, Behrman RE, JensonHB, Stanton BF, eds. Nelson Textbook of Pediatrics. 18th ed. Philadelphia, Pa: SaundersElsevier; 2007:2499-2504

Moser HW, Raymond GV, Dubey P. Adrenoleukodystrophy: new approaches to aneurodegenerative disease. JAMA. 2005;294:3131-3134. Abstract available at:http://www.ncbi.nlm.nih.gov/pubmed/16380594

Copyright © 2010 by the American Academy of Pediatrics page 137

Page 138: AAP PREP 2010

2010 PREP SA on CD-ROM

Question: 55

A couple comes to you for counseling following the delivery of a stillborn fetus that had multipleanomalies 1 year ago. They would like to have another baby, but they are concerned about thepossibility of recurrence. The couple declined autopsy after the fetus was delivered, but theybring you photos for review. You note a cleft extending diagonally across the face (Item Q55A),a constriction ring around one arm (Item Q55B), and multiple digital amputations on the ipsilateralhand with attached strands of tissue (Item Q55C).

Of the following, the MOST accurate statement regarding this condition is that it stems from anabnormality that usually occurs

A. at conception

B. in association with amniocentesis

C. in association with chorionic villus sampling

D. in the first trimester

E. in the second or third trimester

Copyright © 2010 by the American Academy of Pediatrics page 138

Page 139: AAP PREP 2010

2010 PREP SA on CD-ROM

Preferred Response: DCritique: 55

The infant described in the vignette has amniotic band sequence (also known as amnioticrupture sequence and amniotic band disruption sequence); the strands of tissue attached to thehand with digital amputations are important clues to the diagnosis. Amniotic band sequencerefers to the fetal anomalies that can occur as a result of early amnion rupture. Once thisdiagnosis is established, it is appropriate to tell the parents that this is a sporadic occurrence,and recurrence risk for future pregnancies is negligible.

Although multiple theories have been proposed about the cause of this phenomenon, one ofthe most widely held is that it is due to early, primary rupture of the amnion before 12 weeks’gestation. Prior to 12 weeks’ gestation, the amnion and chorion are separate membranes, makingthe amnion more susceptible to rupture. Fetal entrapment can occur any time throughout theremainder of the pregnancy. Following rupture, strands of amnion can wrap around developingfetal structures, leading to constriction and reduced distal blood flow that results in hypoplasia,syndactyly, or amputation. The tissue strands also can transect the body and head, resulting inabdominal wall defects and facial clefts, respectively. Furthermore, amniotic bands may causechanges, such as limitation of movement, that lead to unusual fetal position and result in fetaldeformation. It is important to look for evidence of amniotic bands in the newborn who has limbdeficiencies; they often are washed away with the first bath. Examination of the placenta alsomay allow for confirmation of amniotic rupture.

There are rare reports of amniotic bands occurring in association with maternal trauma suchas a bad fall within the first 12 weeks of pregnancy, resulting in amniotic rupture. Amnioticbands are not associated with procedures such as chorionic villus sampling or amniocentesis.

References:

Bamforth JS. Amniotic band sequence: Streeter hypothesis revisited. Birth Defects Orig ArticSer. 1993;29:279-289

Bianchi DW, Crombleholme TM, D’Alton ME. Amniotic band syndrome. In: Fetology. New York,NY: McGraw-Hill Medical Publishing Division; 2000:761-770

Jones KL. Amnion rupture sequence. In: Smith’s Recognizable Patterns of HumanMalformation. 6th ed. Philadelphia, Pa: Elsevier Saunders; 2006:732-735

Levy PA, Adam HM. In brief: amniotic bands. Pediatr Rev. 1998;19:249. Available at:http://pedsinreview.aappublications.org/cgi/content/full/19/7/249

Copyright © 2010 by the American Academy of Pediatrics page 139

Page 140: AAP PREP 2010

2010 PREP SA on CD-ROM

Question: 56

In researching a presentation you are making at your son’s high school, you learn thatunintentional injuries are the leading cause of death in the adolescent age group. You plan toaddress this issue in your discussion of preventive care.

Of the following, the LEADING cause of death from unintentional injuries in adolescents is

A. automobile crashes

B. bicycle injuries

C. drowning

D. firearms

E. sports injuries

Copyright © 2010 by the American Academy of Pediatrics page 140

Page 141: AAP PREP 2010

2010 PREP SA on CD-ROM

Preferred Response: ACritique: 56

Adolescence is a time of exploration and risk-taking, especially during the middle adolescentstage of psychological development. Certain risky behaviors may result in fatal consequences.The three leading causes of death in the 11- to 21-year-old age group are vehicular injuries,homicide, and suicide. The word "injury" is preferred over "accidents" because the wordaccident implies that the event is not preventable.

Health-care professionals should direct their anticipatory guidance for adolescents toencouraging behaviors that promote safety and injury prevention. In the area of automobilesafety, parents should be encouraged to enforce a policy of not drinking or using drugs whiledriving and avoiding distractions, such as the use of cell phones. In addition, they shouldfamiliarize themselves with the Graduated Driver License Law, if any, in their state and requiretheir children to adhere to the law when they are either a driver or passenger in a motor vehicle.

Exposure to violence increases the risk for homicide, aggressive behaviors, and mentalhealth issues. All adolescents should be screened for violence exposure to identify those inneed of further intervention.

Clinicians should ask parents and adolescents about the presence of firearms in the homeand discuss measures to enhance safe storage. Preventing injuries during sports activities isthe shared responsibility of adolescents, parents, physical education teachers, and coaches.Using protective equipment (eg, bicycle helmets), limiting the duration of repetitive activities,setting an appropriate pace, and refraining from the use of ergogenic aids are all importantissues to address.

References:

Committee on Injury, Violence, and Poison Prevention and Committee on Adolescence, AmericanAcademy of Pediatrics. Policy statement: the teen driver. Pediatrics. 2006;118:2570-2581.Available at: http://pediatrics.aappublications.org/cgi/content/full/118/6/2570

Gutgesell ME Payne N. Issues of adolescent psychological development in the 21st century.Pediatr Rev. 2004;25:79-85. Available at:http://pedsinreview.aappublications.org/cgi/content/full/25/3/79

Hagan JF Jr, Shaw JS, Duncan P. Bright futures visits: middle childhood (5 to 10 years): 9 and10 year visits. In: Bright Futures: Guidelines for Health Supervision of Infants, Children, andAdolescents. 3rd ed. Elk Grove, Ill: American Academy of Pediatrics; 2008:499-514

Hazen H, Schlozman S, Beresin E. Adolescent psychological development: a review. PediatrRev. 2008;29:161-168. Available at:http://pedsinreview.aappublications.org/cgi/content/full/29/5/161

Johnston BD, Rivara FP. Injury control: new challenges. Pediatr Rev. 2003;24:111-118. Availableat: http://pedsinreview.aappublications.org/cgi/content/full/24/4/111

Copyright © 2010 by the American Academy of Pediatrics page 141

Page 142: AAP PREP 2010

2010 PREP SA on CD-ROM

Question: 57

You are evaluating a 2-year-old girl in your office who suffered a near-drowning accident at 1year of age that left her ventilator-dependent. She receives feeding via a gastrostomy tube andhas a seizure disorder that is well-controlled by medications. Her mother reports that the childsometimes seems to choke after her tube feeding and that she has developed a fever and anincreased oxygen requirement. On physical examination, the girl’s tracheostomy tube site isclean and dry, her temperature is 39.0°C, her heart rate is 140 beats/min, and her respiratoryrate is 48 breaths/min. You obtain a chest radiograph (Item Q57).

Of the following, the MOST likely diagnosis is

A. aspiration pneumonia

B. congenital lobar emphysema

C. pneumothorax

D. pulmonary hemorrhage

E. viral pneumonia

Copyright © 2010 by the American Academy of Pediatrics page 142

Page 143: AAP PREP 2010

2010 PREP SA on CD-ROM

Preferred Response: ACritique: 57

The care of the technology-dependent child represents a considerable challenge for familiesand clinicians. The exact number of such children is unknown, but recent estimates suggest thatthey represent up to 20% of patient discharges from tertiary children’s hospitals.

Anatomic abnormalities, impaired neurologic status, loss of normal oromotor coordination,gastroesophageal reflux, and the presence of devices such as nasoenteric tubes andtracheostomies can lead to recurrent aspiration of gastric or upper airway secretions amongsuch children. Aspiration can lead to a variety of respiratory problems, including wheezing,coughing, apnea, laryngospasm, and recurrent pneumonias.

Tracheostomy tubes, even cuffed tubes, do not prevent aspiration and, in fact, mayincrease the risk in some patients due to desensitization of the larynx and loss of protectivereflexes in combination with uncoordinated laryngeal closure. The child described in the vignettehas a history of coughing with feedings and now has signs of an acute respiratory infectionthat include fever, tachypnea, and increased respiratory rate. Her chest radiographdemonstrates a diffuse right upper lobe infiltrate in a location consistent with aspirationpneumonia and no evidence of a pneumothorax (Item C57A).

Congenital lobar emphysemas typically present with respiratory distress by 6 months ofage. Radiographic findings include hyperinflation of the affected lobe with resultant atelectasisof adjacent lobes and shifting of the mediastinum away from the lesion (Item C57B). Chestradiography in viral infection typically shows hyperinflation with bilateral interstitial infiltrates andperibronchial cuffing, and pulmonary hemorrhage is characterized by diffuse consolidation witha "ground glass" appearance.

References:

Columbo JL. Chronic recurrent aspiration. In: Kliegman RM, Behrman RE, Jenson HB, Stanton BF,eds. Nelson Textbook of Pediatrics. 18th ed. Philadelphia, Pa: Saunders Elsevier; 2007:1790-1791

Mohan P. In brief: aspiration in infants and children. Pediatr Rev. 2002;23:330-331. Available at:http://pedsinreview.aappublications.org/cgi/content/full/23/9/330

Sherman JM, Davis S, Albamonte-Petrick S, et al. Care of the child with a chronic tracheostomy.This official statement of the American Thoracic Society was adopted by the ATS board ofdirectors, July 1999. Am J Respir Crit Care Med. 2000;161:297-308. Available at:http://ajrccm.atsjournals.org/cgi/content/full/161/1/297

Copyright © 2010 by the American Academy of Pediatrics page 143

Page 144: AAP PREP 2010

2010 PREP SA on CD-ROM

Question: 58

Of the following growth curves (Item Q58A, Item Q58B, Item Q58C, Item Q58D, Item Q58E), theone MOST likely to be associated with familial short stature in a boy who had a birthweight of3.3 kg is

A. Item Q58A

B. Item Q58B

C. Item Q58C

D. Item Q58D

E. Item Q58E

Copyright © 2010 by the American Academy of Pediatrics page 144

Page 145: AAP PREP 2010

2010 PREP SA on CD-ROM

Preferred Response: ACritique: 58

Children who are born relatively large but are destined to have short stature as adultsbecause they come from short families (familial short stature) generally show a shift in growthpercentiles so that by the time they are 2 years of age, they are growing at a steady rate andtheir height percentile is appropriate for their family. They mature at a normal time and achieveshort normal adult stature after reaching full maturation, as in growth chart A. Some affectedchildren have idiopathic short stature and some may have a known single gene mutation leadingto short stature.

Growth charts B, C, and D show the progress of children who have growth attenuation orarrest occurring or persisting past the second year. Such children likely have serious underlyingillnesses interfering with linear growth. An examination of weight for age might be helpful inassessing the cause of the growth attenuation. For example, a child who has celiac diseasewould be underweight and often experience weight loss before slowing in growth, while a childwho has hypothyroidism would have a normal weight or be overweight for age, but havemarked growth attenuation.

Growth chart E shows a continuation of growth with a growth spurt after other boys havereached adult height. A period of slowdown or attenuation in growth rate is documented justbefore the pubertal growth spurt, which may be relatively prolonged if puberty is late. Thispattern is seen in delayed adolescence, and it can be associated with relative short statureduring childhood and a normal adult height.

References:

Clark PA. Constitutional growth delay. eMedicine Specialties, Pediatrics: General Medicine,Endocrinology. 2007. Available at: http://www.emedicine.com/ped/topic472.htm

Daniel M, Kleis L, Cemeroglu AP. Etiology of failure to thrive in infants and toddlers referred to apediatric endocrinology outpatient clinic. Clin Pediatr. 2008;47:762-765. Abstract available at:http://www.ncbi.nlm.nih.gov/pubmed/18467671

Ferry RJ Jr. Short stature. eMedicine Specialties, Pediatrics: General Medicine, Endocrinology.2007. Available at: http://www.emedicine.com/ped/topic2087.htm

Rogol AD. Diagnostic approach to short stature. UpToDate Online 16.3. 2008. Available forsubscription at: http://www.utdol.com/online/content/topic.do?topicKey=pediendo/2375

Copyright © 2010 by the American Academy of Pediatrics page 145

Page 146: AAP PREP 2010

2010 PREP SA on CD-ROM

Question: 59

The parents of a 2-year-old boy are concerned because they are having trouble managing hisbehavior. His language and social development are age-appropriate, but he frequently goes tohis closet and throws his clothes on the floor. He also enjoys throwing food at the dinner tableand would rather run around the dining room than sit and eat his food. His parents ask youradvice on how best to manage his behavior.

Of the following, the MOST appropriate response is to

A. evaluate the child for attention-deficit/hyperactivity disorder and counsel the parents aboutthe disorder

B. explain about setting limits and realistic behavioral expectations for toddlers

C. explain how to set up a behavior system using a token economy

D. reassure them that his behavior will improve with time and schedule a follow-up appointmentin 6 months

E. refer the boy for behavioral therapy

Copyright © 2010 by the American Academy of Pediatrics page 146

Page 147: AAP PREP 2010

2010 PREP SA on CD-ROM

Preferred Response: BCritique: 59

The parents described in the vignette need to set limits for their toddler’s behavior and haveage-appropriate expectations for him. An often-recommended disciplinary strategy is time-out,which refers to time out from negative behavior. This is an effective strategy for age 1 year toearly adolescence. However, to be effective, the parent also must provide "time in," that is, shortnonverbal physical contact on a frequent basis when the child is engaging in acceptablebehavior. When a child engages in an undesirable behavior, the parent should withdraw allattention, which is referred to as extinction. One difficulty with this procedure is that initially theintensity of the behavior may increase, a phenomenon referred to as extinction burst. However,if the parent can persevere, the undesirable behavior will diminish.

If a parent has difficulty using an abrupt extinction procedure, planned ignoring may beeffective. With this approach, the parents gradually ignore the child’s behavior, a process thattends to take longer but does not lead to an intensive increase in the undesirable behavior.

The child described in the vignette is too young to participate in a token economy. Thissystem provides a method of keeping track of a child’s appropriate and inappropriate behaviors.A chip system, in which the child earns a chip for positive behavior, may be used for childrenages 3 to 7 years. A simple exchange system may be appropriate for children 5 to 12 years ofage. The parents in the vignette should be given guidance about how to manage their child’sbehavior and not reassured that the situation will improve. If the previously noted initialapproaches are not effective, the family can be referred for behavioral therapy. Evaluation forattention-deficit/hyperactivity disorder is not appropriate for a child of this age.

References:

Christophersen ER. Behavioral management: theory and practice. In: Parker S, Zukerman B,Augustyn M, eds. Developmental and Behavioral Pediatrics: A Handbook for Primary Care. 2nded. Philadelphia, Pa: Lippincott Williams & Wilkins; 2005:55-60

Christophersen ER. Learning, discipline, and compliance in children. In: Pediatric Compliance: AGuide for the Primary Care Physician (Critical Issues in Developmental and BehavioralPediatrics). New York, NY: Plenum Medical Book Company; 1994:5-46

Copyright © 2010 by the American Academy of Pediatrics page 147

Page 148: AAP PREP 2010

2010 PREP SA on CD-ROM

Question: 60

A 4-year-old boy who has acute myelogenous leukemia is admitted for the treatment of feverand neutropenia. He has a Broviac catheter in place. His temperature on admission is 39.3°C and

absolute neutrophil count (ANC) is less than 0.1x103/mcL (0.1x109/L). No focus of infection isapparent on physical examination. After blood cultures are obtained, he is begun on treatmentwith piperacillin/tazobactam and gentamicin. Five days later, the cultures remain negative, ANC

continues to be less than 0.1x103/mcL (0.1x109/L), and his daily maximum temperaturecontinues to be greater than 39.3°C.

Of the following, the MOST appropriate management at this point is to

A. add amphotericin B to the antibiotic regimen

B. administer granulocyte transfusions

C. change the antibiotic regimen to meropenem and amikacin

D. continue the present antibiotic regimen

E. stop the antibiotics and obtain another culture

Copyright © 2010 by the American Academy of Pediatrics page 148

Page 149: AAP PREP 2010

2010 PREP SA on CD-ROM

Preferred Response: ACritique: 60

Oncology patients who have persistent fever and neutropenia (Absolute Neutrophil Count <

0.5x103/mcL [0.5x109/L]), such as the boy described in the vignette, are at increased risk ofinvasive fungal infection. Although initial management begins with antibacterial therapy, if afocus of infection is not identified, the child remains febrile and neutropenic, and the culturesremain negative, it is appropriate to begin empiric therapy with amphotericin B after 5 to 7 days.

Granulocyte transfusions have very limited indications and are not administered routinely tochildren who have fever and neutropenia. Rarely, they may be considered in severely ill patientswho have neutropenia and fungal or gram-negative sepsis. Although consideration of aresistant bacterial infection is reasonable for this patient, in the absence of an isolate, the risk offungal infection is considered greater and, thus, the addition of antifungal therapy isrecommended. After 5 to 7 days of persistent fever and neutropenia, the risk of untreatedinfection warrants the addition of amphotericin B, not just continuing the same antibiotics.Similarly, although it might be logical to consider stopping antibiotics and obtaining anotherculture, the risk of invasive infection is sufficiently great that such an action is not consideredsafe.

Amphotericin B is active against a broad array of fungi, including Candida, Aspergillus,Zygomycetes, Histoplasma, and Coccoides immitis. The drug is administered in one daily doseof 0.5 to 1 mg/kg per day over several hours or longer. Primary toxicities include febrilereactions, hypokalemia, and nephrotoxicity. Liposomal preparations of amphotericin are equallyefficacious and less nephrotoxic than the standard product, but their increased costs limit theiruse to select patients who have renal dysfunction.

References:

American Academy of Pediatrics. Antifungal drugs for systemic fungal infections. In: PickeringLK, Baker CJ, Kimberlin DW, Long SS, eds. Red Book: 2009 Report of the Committee onInfectious Diseases. 28th ed. Elk Grove Village, Ill: American Academy of Pediatrics; 2009:765-767

Drew RH. Pharmacology of amphotericin B. UpToDate Online 16.3. 2008. Available forsubscription at: www.utdol.com/online/content/topic.do?topicKey=antibiot/4619&view=print

Hughes WT, Armstrong D, Bodey GP, et al. IDSA Guidelines: 2002 guidelines for the use ofantimicrobial agents in neutropenic patients with cancer. Clin Infect Dis. 2002;34:730-751.Available at: http://www.journals.uchicago.edu/doi/full/10.1086/339215

Copyright © 2010 by the American Academy of Pediatrics page 149

Page 150: AAP PREP 2010

2010 PREP SA on CD-ROM

Question: 61

A 15-year-old girl presents for evaluation of vaginal discharge, mild lower abdominal cramping,and pruritus of her vulvar area that began 3 days ago. She is sexually active with one partner,and her last encounter was several weeks ago. She denies having fever, pain with urination, orother symptoms. On pelvic examination, you observe a copious amount of thin, whitish-yellowvaginal discharge, and her vaginal walls and cervix appear erythematous and inflamed. Herabdomen is soft and nontender, and there is no cervical motion tenderness or adnexal masses.Laboratory evaluation of the vaginal discharge demonstrates a pH of 6.0. Wet mount of thedischarge shows a moderate number of white blood cells and a few epithelial cells with a fewmotile organisms.

Of the following, the MOST appropriate antibiotic for the treatment of this patient is

A. albendazole

B. clindamycin

C. doxycycline

D. fluconazole

E. metronidazole

Copyright © 2010 by the American Academy of Pediatrics page 150

Page 151: AAP PREP 2010

2010 PREP SA on CD-ROM

Preferred Response: ECritique: 61

Metronidazole is a nitroimidazole drug that is bactericidal. It is effective against mostinfections involving anaerobic bacteria, many microaerophilic bacteria, Clostridium,Trichomonas vaginalis, Treponema pallidum, Gardnerella vaginalis, oral spirochetes,Helicobacter pylori, Campylobacter fetus, and certain other parasitic infections. It also has beenuseful in several types of bowel bacterial overgrowth syndromes such as complications ofjejunoileal bypass for obesity and dysfunction of the continent ileostomy and for the preventionof intrahepatic cholestasis associated with parenteral nutrition.

The patient described in the vignette has classic signs, symptoms, and laboratory findings ofa T vaginalis infection. T vaginalis is a flagellated protozoan that is the second most commonsexually transmitted infection in the United States. Diagnosis usually is established by presentingsigns and symptoms and examination of a wet-mount preparation of the vaginal discharge. Aclassic finding is the presence of motile organisms that have flagella.

Albendazole is an antiparasitic drug effective against intestinal nematodes, Enterobiusvermicularis, cutaneous and visceral larval migrans, and other parasitic infections, but it has noactivity against T vaginalis. Clindamycin and doxycycline are antibiotic agents and fluconazole isan antifungal agent, all of which have no activity against T vaginalis.

References:

American Academy of Pediatrics. Trichomonas vaginalis infections (trichomoniasis). In:Pickering LK, Baker CJ, Kimberlin DW, Long SS, eds. Red Book: 2009 Report of the Committeeon Infectious Diseases. 28th ed. Elk Grove Village, Ill: American Academy of Pediatrics;2009:674-675

Rosenblatt JE, Edson RS. Metronidazole. Mayo Clin Proc. 1983;58:154-157. Abstract availableat: http://www.ncbi.nlm.nih.gov/pubmed/6600804

Salvatore M, Meyers B. Metronidazole. In: Mandell GL, Bennett JE, Dolan R, eds. Mandell,Douglas, and Bennett’s Principles and Practice of Infectious Diseases. 6th ed. Philadelphia, Pa:Elsevier Churchill Livingstone; 2005:388-396

Copyright © 2010 by the American Academy of Pediatrics page 151

Page 152: AAP PREP 2010

2010 PREP SA on CD-ROM

Question: 62

A 10-year-old girl presents with cola-colored urine and mild swelling of her legs that she initiallynoticed 12 hours ago. Her mother reports that she had a sore throat 10 days ago, and thechange in urine color occurred yesterday evening. There is no history of trauma, and the patientdenies flank pain, frequency, urgency, dysuria, or passing clots in the urine. On physicalexamination, her blood pressure is 144/90 mm Hg, and she has mild swelling of the face andlower extremities.

Of the following, the MOST appropriate next step is

A. echocardiography

B. renal and bladder ultrasonography

C. serum creatinine measurement

D. throat swab for rapid streptococcal antigen

E. urine culture

Copyright © 2010 by the American Academy of Pediatrics page 152

Page 153: AAP PREP 2010

2010 PREP SA on CD-ROM

Preferred Response: CCritique: 62

Children who have acute glomerulonephritis (AGN) typically present with hematuria,proteinuria, and one or more of the following: hypertension, azotemia, and edema. The classicpresentation of the "nephritic syndrome" includes painless gross hematuria (cola-colored urinewithout clots), a history of decreased urine output, hypertension, and facial or pretibial swelling,as described for the girl in the vignette.

The initial laboratory evaluation of a patient who has AGN begins with a urinalysis thatincludes microscopic examination. The dipstick component demonstrates the presence of bloodand protein, and the microscopic component reveals the presence of red blood cells, somewhite blood cells (as seen with inflammation), and possibly red blood cell casts (Item C62A). Redblood cell casts are seen best on a freshly spun urine specimen, which can be performed bythe clinician, but may not be reported by the average laboratory unless specifically requested. Ifthe urinalysis results are consistent with AGN, renal function tests should be obtained alongwith serum electrolyte measurements. Additional serologic tests should focus on measurementof complement components, antinuclear antibody (ANA), and anti-double-stranded (ds) DNAantibody.

Measurement of complement component 3 (C3) allows classification of the AGN ashypocomplementemic (low C3) or normocomplementemic (normal C3). Children who havehypocomplementemic AGN are presumed to have postinfectious AGN initially.Membranoproliferative glomerulonephritis (MPGN) is also a consideration because two thirds ofcases are hypocomplementemic at presentation, sometimes with an accompanying low C4.Lupus nephritis is also a consideration and is usually associated with positive ANA and anti-dsDNA findings. The differential diagnosis for normocomplementemic AGN includes renal-limiteddiseases such as immunoglobulin A (IgA) glomerulonephritis, MPGN, and Alport syndrome;systemic diseases such as Henoch-Schönlein purpura; and antineutrophil cytoplasmic antibody-associated diseases such as Wegener granulomatosis and microscopic polyangiitis (Item C62B).

It is essential that the clinician assess renal function. If the serum creatinine value iselevated, a nephrology consultation should be sought immediately to ensure that the child doesnot have a rapidly progressive glomerulonephritis that warrants renal biopsy and urgenttreatment. Management also should focus on control of the hypertension, which usually can beaccomplished by restriction of fluid and sodium intake. Fluids should be restricted to one half totwo thirds maintenance. A vasodilator also may be warranted. Hospitalization sometimes isrequired to manage the hypertension or monitor the renal function in those who have azotemia.Although hypoalbuminemia may be present, albumin infusions generally are not required, and ifconsidered, should be provided only with the input of a pediatric nephrologist because ofconcerns about potential development of pulmonary edema with albumin infusion.

The role of echocardiography is to assess for left ventricular hypertrophy (LVH) caused byhypertension. Its use in the clinical setting of AGN is limited because LVH is very unlikely in acutehypertension. Renal ultrasonography is helpful in the child who has gross or microscopichematuria when looking for a renal cyst, stone, or mass. In the setting of AGN, its use is limited.A throat swab for streptococcal antigen may help determine if the patient has group AStreptococcus within the pharynx, but the pathogen may not be present in someone who hasacute poststreptococcal GN at presentation or in those who have other causes of AGN. Urine

Copyright © 2010 by the American Academy of Pediatrics page 153

Page 154: AAP PREP 2010

2010 PREP SA on CD-ROM

culture is helpful to rule out a urinary tract infection, but this patient’s lack of urinary symptoms inthe presence of "cola-colored urine" without clots make an infectious cause unlikely.

References:

Massengill SF. Hematuria. Pediatr Rev. 2008;29:342-348. Available at:http://pedsinreview.aappublications.org/cgi/content/full/29/10/342

Vehaskari VM, Aviles DH. Acute glomerulonephritis and rapidly progressive glomerulonephritis.In: Kher KK, Schnaper HW, Makker SP, eds. Clinical Pediatric Nephrology. 2nd ed. London,England: Informa Healthcare; 2007:145-154

Copyright © 2010 by the American Academy of Pediatrics page 154

Page 155: AAP PREP 2010

2010 PREP SA on CD-ROM

Question: 63

The parents of a 10-year-old boy who has a peanut and tree nut food allergy ask your adviceon the treatment of food allergy reactions at school. They describe a scenario that occurred lastyear when their son started itching diffusely and having difficulty breathing during lunchtimeafter inadvertently eating some of his friend’s chocolate candy bar that contained peanuts. Athis current school, the child is allowed to carry his own self-injectable epinephrine. His currentweight is 90 lb (41 kg).

Of the following, the BEST advice for the child, if a similar situation occurs, is to

A. have the school call emergency services (911), who should evaluate and administerepinephrine if needed

B. have the school nurse observe the child for 10 to 15 minutes while calling his parents

C. immediately administer 0.15 mg of self-injectable epinephrine

D. immediately administer 0.30 mg of self-injectable epinephrine

E. take an oral antihistamine immediately

Copyright © 2010 by the American Academy of Pediatrics page 155

Page 156: AAP PREP 2010

2010 PREP SA on CD-ROM

Preferred Response: DCritique: 63

The boy described in the vignette experienced an anaphylactic reaction, a potentially life-threatening event. In children, the most commonly identified causes for anaphylaxis are food,insects, drugs, latex, and vaccines. Food allergy is the most common cause of anaphylaxis inthe home or school setting and accounts for an estimated 50% of all pediatric cases annually.Some 85% to 90% of allergic reactions to food in children are due to milk, egg, soy, wheat,peanuts, tree nuts, fish, and shellfish. Peanuts and tree nuts account for most cases of fatalanaphylaxis from foods in the United States.

Recently, a panel of experts published a set of clinical criteria for diagnosing anaphylaxis(Item C63).

The skin and respiratory system are the most commonly affected systems in cases of foodallergy-induced anaphylaxis, as described for the boy in the vignette. Fatal anaphylaxis almostalways is due to airway edema and subsequent respiratory failure.

For a person experiencing anaphylaxis, epinephrine should be administered immediately andwithout delay. Observation of the child while calling his parents wastes precious time in thissituation. In the school setting, self-injectable intramuscular epinephrine is used. Other methodsof delivery, used primarily in the hospital setting, include intravenous, intraosseous, and via anendotracheal tube. Current epinephrine injectors are available in two strengths: 0.15 mg and0.30 mg. The child in the vignette, who weighs more than 30 kg, should be given the 0.30-mgdose, preferably in the lateral thigh. Antihistamines may decrease pruritus or flushing, but theireffect has a slow onset, and they are not recommended as the initial treatment for anaphylaxis.Because some children may require additional doses of epinephrine and observation,emergency services should be called, but waiting for them to arrive to make a decision regardingthe initial dose of epinephrine is not recommended.

Caregivers of children who have experienced food-induced anaphylaxis should haveepinephrine readily available, understand the indications for its use, have a written action plan,and understand the proper technique for use of self-injectable epinephrine devices.

References:

Joint Task Force on Practice Parameters; American Academy of Allergy, Asthma andImmunology; American College of Allergy, Asthma and Immunology; Joint Council of Allergy,Asthma and Immunology. The diagnosis and management of anaphylaxis: an updated practiceparameter. J Allergy Clin Immunol. 2005;115(3 suppl 1):S483-S523

Kreet CA, Wood RA. Food allergy and anaphylaxis. Immunol Allergy Clin North Am. 2007;27;193-212. Abstract available on: http://www.ncbi.nlm.nih.gov/pubmed/17493498

Lieberman P, Decker W, Camargo CA Jr, Oconnor RO, Oppenheimer J, Simons FE. SAFE: amultidisciplinary approach to anaphylaxis education in the emergency department. Ann AllergyAsthma Immunol. 2007;98:519-523. Abstract available on:http://www.ncbi.nlm.nih.gov/pubmed/17601263

Copyright © 2010 by the American Academy of Pediatrics page 156

Page 157: AAP PREP 2010

2010 PREP SA on CD-ROM

Sampson HA, Leung DYM. Anaphylaxis. In: Kliegman RM, Behrman RE, Jenson HB, Stanton BF,eds. Nelson Textbook of Pediatrics. 18th ed. Philadelphia, Pa: Saunders Elsevier; 2007:983-984

Sampson HA, Muñoz Furlong A, Campbell RL, et al. Second symposium on the definition andmanagement of anaphylaxis: summary report-Second National Institute of Allergy and InfectiousDisease/Food Allergy and Anaphylaxis Network symposium. J Allergy Clin Immunol.2006;117:391-397. Abstract available on: http://www.ncbi.nlm.nih.gov/pubmed/16461139

Waibel KH. Anaphylaxis. Pediatr Rev. 2008;29:255-263. Available at:http://pedsinreview.aappublications.org/cgi/content/full/29/8/255

Copyright © 2010 by the American Academy of Pediatrics page 157

Page 158: AAP PREP 2010

2010 PREP SA on CD-ROM

Question: 64

A father brings his 2-year-old son to the emergency department in status epilepticus. He reportsthat the boy spent several hours in the garage with him while he was repairing the car. Onquestioning, the father states that over the course of the afternoon the child seemed sleepierthan usual, then became lethargic, vomited, and seemed like he was "drunk." On the way to thehospital he began having seizures. In the emergency department, the boy is given a dose oflorazepam to stop the seizure and is endotracheally intubated because of respiratory

depression. His initial laboratory results are:

•Sodium, 138 mEq/L (138 mmol/L)

•Potassium, 4.9 mEq/L (4.9 mmol/L)

•Chloride, 100 mEq/L (100 mmol/L)

•Bicarbonate, 6 mEq/L (6 mmol/L)

•Glucose, 120 mg/dL (6.7 mmol/L)

•Blood urea nitrogen, 10 mg/dL (3.6 mmol/L)

•Calcium, 5.5 mEq/L (5.5 mmol/L)•Serum osmolality, 335 mOsm/kg (335 mmol/kg)

Of the following, the MOST likely cause of this child’s clinical condition is ingestion of

A. ethylene glycol

B. gasoline

C. motor oil

D. organophosphate insecticide

E. turpentine

Copyright © 2010 by the American Academy of Pediatrics page 158

Page 159: AAP PREP 2010

2010 PREP SA on CD-ROM

Preferred Response: ACritique: 64

The progressive lethargy, ataxia, seizures, anion gap metabolic acidosis of 30 mEq/L (ItemC64A), and osmolar gap of 53 mmol/L (Item C64B) described for the boy in the vignette arehighly suggestive of alcohol poisoning. This is a particular diagnostic possibility because the boymay have had access in the garage to such potential toxic alcohols as ethylene glycol(antifreeze) and methanol (windshield wiper fluid). The hypocalcemia suggests ethylene glycolexposure because the metabolism of ethylene glycol uses the patient’s calcium stores to createcalcium oxalate, which is excreted in the urine as crystals (Item C64C). Other findings inethylene glycol poisoning may include flank pain, hematuria, and acute renal failure.

Rapid diagnosis is critical for a patient who has symptomatic ethylene glycol poisoningbecause delay in treatment can lead to renal damage, cerebral herniation, multiple organ systemfailure, and death. Often, initial treatment is based on clinical suspicion before alcohol values areavailable. Indirect laboratory evidence of alcohol toxicity includes an anion gap acidosis and anosmolar gap greater than 10 mmol/L. Initial treatment involves stabilization of vital functions,administration of sodium bicarbonate to correct acidosis, and administration of the antidotefomepizole (or ethanol, if fomepizole is unavailable). Fomepizole inhibits alcohol dehydrogenase,which metabolizes the nontoxic parent alcohols into their toxic byproducts. Because alcoholsare absorbed so quickly from the gastric mucosa, there is little role for gastrointestinaldecontamination. Hemodialysis is indicated for severe poisonings.

Many household products are toxic and frequently accessible to young children. Gasolineand turpentine are volatile hydrocarbons that cause pulmonary injury after aspiration. Motor oilalso is a hydrocarbon, but because of its high viscosity and low volatility, it poses little risk foraspiration or toxicity. Organophosphate insecticides inhibit acetylcholinesterase and cause acholinergic crisis manifested by bradycardia, hypersalivation, bronchorrhea, diarrhea, andmuscle weakness.

References:

Keyes DC. Toxicity, ethylene glycol. eMedicine Specialties, Emergency Medicine, Toxicology.2007. Available at http://www.emedicine.com/emerg/topic177.htm

Sharman M, Sarnaik AP. Approach to the child with metabolic acidosis. UpToDate Online 16.3.2008. Available at:http://www.utdol.com/online/content/topic.do?topicKey=pedineph/16228&selectedTitle=2~150&source=search_result

Sivilotti MLA, Winchester JF. Methanol and ethylene glycol poisoning. UpToDate Online 16.3.2008. Available at:http://www.utdol.com/online/content/topic.do?topicKey=ad_tox/8204&selectedTitle=2~105&source=search_result

Copyright © 2010 by the American Academy of Pediatrics page 159

Page 160: AAP PREP 2010

2010 PREP SA on CD-ROM

Question: 65

A 3-year-old boy who has Noonan syndrome presents with diarrhea and poor weight gain ofseveral months’ duration. One of your cardiology colleagues is treating the boy for pulmonaryvalvular stenosis. On physical examination, you note abdominal distention and pitting edema ofboth lower extremities (Item Q65) and the presacral area.

Of the following, the test MOST likely to identify the cause of this boy’s problem is

A. abdominal ultrasonography

B. echocardiography

C. fecal alpha-1-antitrypsin measurement

D. percutaneous liver biopsy

E. serum albumin measurement

Copyright © 2010 by the American Academy of Pediatrics page 160

Page 161: AAP PREP 2010

2010 PREP SA on CD-ROM

Preferred Response: CCritique: 65

Congenital cardiac disease, including atrioventricular canal defects and right-sided heartlesions, frequently is associated with Noonan syndrome. The valvular pulmonic stenosisdescribed for the boy in the vignette can lead to increased right heart pressures and may resultin heart failure. Subsequent elevations in lymphatic pressure cause both hepatic congestion andintestinal lymphangiectasis, leading to "weeping" of protein into the intestinal lumen. Excessiveintestinal protein loss results in hypoproteinemia and edema. In addition to such secondarylymphangiectasia, other lymphatic abnormalities, including primary intestinal lymphangiectasia (ie,without associated heart disease), have been identified in up to 20% of children affected byNoonan syndrome.

The diagnosis of such primary and secondary protein-losing enteropathies may beestablished readily by measuring fecal concentrations of alpha-1-antitrypsin, a stable andnondietary protein that is a sensitive and specific marker of intestinal protein loss.

Typically, clinical signs of hypoproteinemia may appear when the serum albumin measuresless than 2.5 g/dL (25 g/L). Although this measurement can identify hypoproteinemia, it cannotdifferentiate among the varied causes of protein loss, which may involve disorders associatedwith increased urinary losses, hepatic synthesis, systemic catabolism with increased albumindegradation, gastrointestinal secretion, inflammation, and lymphatic obstruction. In the absenceof signs of liver failure or evidence of proteinuria, gastrointestinal protein loss should besuspected. A protein-losing enteropathy not only may result from cardiovascular dysfunctionwith secondary intestinal lymphangiectasia, but its source may be a gastrointestinalmalabsorptive or inflammatory state. Abdominal ultrasonography, echocardiography,percutaneous liver biopsy, and serum albumin measurement may be useful in defining specificconditions associated with hypoproteinemia but are not useful in establishing its cause.

As stated previously, protein loss across the intestinal mucosa can result from a widevariety of both gastrointestinal and nongastrointestinal illnesses. Hypoalbuminemia and edemadevelop when enteric losses exceed hepatic albumin synthetic capability. The list of disordersthat may be associated with a protein-losing enteropathy is large (Item C65). Secondarylymphangiectasia is a well-described complication of both right- and left-sided heart failure andcan be manifested both by intestinal and pulmonary protein loss. The latter may lead to pleuraleffusion and chylothorax and can occur with or without an associated protein-losingenteropathy. The most common clinical scenario leading to this problem is following the Fontanprocedure for hypoplastic left heart syndrome. However, any cardiac lesion or cardiac surgicalprocedure that results in elevated right-sided pressures, including pulmonary valvular stenosis(a common cardiac lesion in Noonan syndrome), may be the precipitating factor.

For most of the disorders associated with protein-losing enteropathy, gastrointestinal loss isthe sole contributor to the hypoproteinemic state, although some are characterized by lymphaticobstruction and protein losses at other sites. Thus, primary intestinal lymphangiectasia often isassociated with thoracic duct obstruction leading to chylothorax as well as with peripherallymphedema. Intestinal protein excretion, in the absence of lymphatic losses elsewhere, canresult from mechanical obstruction of lymphatic channels (eg, malrotation, malignancy) anddiverse gastrointestinal and vascular inflammatory conditions. One interesting yet uncommondisorder in childhood is hypertrophic rugal gastropathy or Ménétrier disease. This generally

Copyright © 2010 by the American Academy of Pediatrics page 161

Page 162: AAP PREP 2010

2010 PREP SA on CD-ROM

benign, self-limited illness requires endoscopy and gastric biopsy for diagnosis, althoughfindings of giant rugal folds on upper gastrointestinal radiographic series may be suggestive.The disorder has been linked to infection with cytomegalovirus, but other autoimmune and toxicinsults have been proposed.

Copyright © 2010 by the American Academy of Pediatrics page 162

Page 163: AAP PREP 2010

2010 PREP SA on CD-ROM

Question: 66

A 30-year-old mother who has a history of opiate addiction, which was managed withmethadone throughout her pregnancy, asks when you plan to discharge her term newborn,who weighs 2,450 g.

Of the following, the BEST response is at a postnatal age of

A. 24 hours

B. 36 hours

C. 48 hours

D. 5 days

E. 10 days

Copyright © 2010 by the American Academy of Pediatrics page 163

Page 164: AAP PREP 2010

2010 PREP SA on CD-ROM

Preferred Response: DCritique: 66

Maternal opiate use places the newborn at risk for neonatal opiate withdrawal syndrome,often referred to as neonatal abstinence syndrome (NAS). The syndrome is characterized bysigns noted in Item C66 and may not become apparent until 5 days after birth. For methadone-treated mothers, a higher daily methadone dose may be associated with a greater likelihood ofthe newborn experiencing NAS.

The infant described in the vignette is term, but is small for gestational age, weighing only2,450 g, which is defined as low birthweight (LBW). Early discharge (<48 hours after birth) isnot recommended for a LBW infant, and some inquiry should be made into the reason for LBWstatus. Maternal use of illicit drugs, alcohol, and tobacco can be associated with poor prenatalcare and inadequate maternal and fetal weight gain throughout the pregnancy. Unfortunately,pregnant women receiving methadone may be noncompliant with methadone therapy andcontinue to use illicit drugs during their pregnancy.

The infant described in the vignette probably can be discharged at 5 days if symptoms ofNAS have not appeared. Because NAS may not develop until after 48 hours, discharge at orbefore 48 hours is not appropriate. Because symptoms of NAS appear by 5 days after birth,delaying discharge of an asymptomatic infant until day 10 is not indicated.

References:

Dashe JS, Sheffield JS, Olscher DA, Todd SJ, Jackson GL, Wendel GD. Relationship betweenmaternal methadone dosage and neonatal withdrawal. Obstet Gynecol. 2002;100:1244-1249.Abstract available at: http://www.ncbi.nlm.nih.gov/pubmed/12468169

Ebner N, Rohrmeister K, Winklbaur B, et al. Management of neonatal abstinence syndrome inneonates born to opioid maintained women. Drug Alcohol Depend. 2007;87:131-138. Abstractavailable at: http://www.ncbi.nlm.nih.gov/pubmed/17000060

Jansson LM, Choo R, Velez ML, et al. Methadone maintenance and breastfeeding in the neonatalperiod. Pediatrics. 2008;121:106-114. Available at:http://pediatrics.aappublications.org/cgi/content/full/121/1/106

Kuschel C. Managing drug withdrawal in the newborn infant. Semin Fetal Neonatal Med.2007;12:127-133. Abstract available at: http://www.ncbi.nlm.nih.gov/pubmed/17321815

Schempf AH. Illicit drug use and neonatal outcomes: a critical review. Obstet Gynecol Surv.2007;62:749-757. Abstract available at: http://www.ncbi.nlm.nih.gov/pubmed/17925048

Copyright © 2010 by the American Academy of Pediatrics page 164

Page 165: AAP PREP 2010

2010 PREP SA on CD-ROM

Question: 67

You are giving a lecture to medical students on dermatologic conditions in childhood. You tellthem they should advise parents to use sunscreen on their children and that surveillance of theskin is important at all health supervision visits to screen for the presence of pigmented nevi.

Of the following, the type of nevus that has the HIGHEST risk for the development of melanomais the

A. common blue nevus

B. giant congenital nevus

C. halo nevus

D. nevus of Ota

E. Spitz nevus

Copyright © 2010 by the American Academy of Pediatrics page 165

Page 166: AAP PREP 2010

2010 PREP SA on CD-ROM

Preferred Response: BCritique: 67

Cutaneous nevi are common in childhood and adolescence, and most are benign lesions thathave no risk of malignancy. However, some nevi are associated with an increased risk ofmalignant transformation to melanoma, and these should be monitored carefully. Congenitalmelanocytic nevi, which are present at birth and classified by size, are at increased risk for thedevelopment of melanoma. Giant congenital nevi (Item C67A), which are larger than 20 cm, havethe greatest risk at approximately 5% to 15%. Such nevi may cover the entire trunk and containirregular pigmentation or hair. Small (<2 cm) and intermediate (2 to 20 cm) congenital nevi are atmuch lower risk of malignancy.

The common blue nevus is a smooth, dome-shaped papule or nodule that usually is acquiredlater in life (Item C67B). Malignant transformation is unusual but can occur. A halo nevus typicallyis seen on the back and has a characteristic ring of hypopigmentation surrounding an involutingacquired nevus (Item C67C). The malignant potential for this type of nevus is very small. Thenevus of Ota is a brown, black, or blue patch that has a speckled appearance. It is mostcommon in females, Asians, and African Americans; malignant change is rare. A Spitz nevus isa pink or red, smooth, dome-shaped papule or nodule (Item C67D) that usually has a benigncourse. However, its histologic features may be confused with those of malignant melanoma.

Risk factors for the development of malignant melanoma include a family history ofmelanoma, the presence of multiple atypical nevi, excessive sun exposure, and fair complexion.All children should wear sunscreen during daytime periods of outdoor activity, and sunexposure between the hours of 10 am and 3 pm should be avoided, if possible. Signs ofmalignant melanoma are asymmetry of the lesion, irregularity in color or borders, and size largerthan 6 mm. Other features suggesting malignancy include subcutaneous nodules, pruritus, andbleeding. The presence of any of these characteristics should prompt a timely consultation witha dermatologist for further evaluation.

References:

Chamlin SL. Shedding light on moles, melanoma, and the sun. Contemp Pediatr. 2002;19:102-112

Morelli JG. Cutaneous nevi. In: Kliegman RM, Behrman RE, Jenson HB, Stanton BF, eds. NelsonTextbook of Pediatrics. 18th ed. Philadelphia, Pa: Saunders Elsevier; 2007:2674-2679

Copyright © 2010 by the American Academy of Pediatrics page 166

Page 167: AAP PREP 2010

2010 PREP SA on CD-ROM

Question: 68

A parent requests referral to a urologist for her 7-year-old son because of his bedwettingproblem. He has no urinary incontinence during the day and is otherwise healthy. Physicalexamination reveals normal growth parameters and normal genitalia with a circumcised phallus.Dipstick urinalysis results are normal, with a specific gravity of 1.025.

Of the following, the MOST appropriate next step is

A. a complete blood count

B. psychological evaluation

C. reassurance and a follow-up visit in 6 months

D. referral to a urologist

E. renal/bladder ultrasonography

Copyright © 2010 by the American Academy of Pediatrics page 167

Page 168: AAP PREP 2010

2010 PREP SA on CD-ROM

Preferred Response: CCritique: 68

Nocturnal enuresis is defined as the involuntary passage of urine during sleep in childrenolder than 5 years of age and occurs in approximately 15% of children at age 5 and 1% of teensat age 15. There is a male predominance and often a positive family history.

For the child who experiences no daytime wetting or dysuria and who has normal urinalysisresults, such as the boy described in the vignette, the likelihood of renal pathology is low.Therefore, reassurance and a follow-up visit in 6 months are sufficient to address his problem.No other studies are indicated.

Children who have both daytime and nighttime wetting (diurnal enuresis) beyond the age of6 years; enuresis associated with encopresis; or symptoms of dysuria, frequency, or difficultyinitiating urination require more evaluation and intervention.

A complete blood count is not helpful in diagnosing or treating enuresis. Urologic referral andimaging studies are not helpful unless there are symptoms or signs suggestive of structuralrenal disease (eg, recurrent infection, dysfunctional voiding). A careful physical examinationshould be performed to assess genitourinary anatomy, exclude trauma from sexual abuse,detect lumbosacral spine anomalies, and exclude bladder dilatation or abdominal mass.

Psychological evaluation is unnecessary in uncomplicated enuresis in the developmentallynormal and well-adapted child younger than 8 years of age. Older children and children forwhom nocturnal enuresis is related to social dysfunction, family stress, or poor self-esteem maybenefit from therapy to target those effects. The use of a bedwetting alarm has the highest rateof success in young children.

As a result of reviewing this information, do you intend to make a change in practiceto provide better patient care?Yes No

References:

Austin PF, Ritchey ML. Dysfunctional voiding. Pediatr Rev. 2000;21:336-341. Available at:http://pedsinreview.aappublications.org/cgi/content/full/21/10/336

Glazener CMA, Evans JHC. Simple behavioural and physical interventions for nocturnal enuresisin children. Cochrane Database Syst Rev. 2004;2: CD003637. Available at:http://www.mrw.interscience.wiley.com/cochrane/clsysrev/articles/CD003637/frame.html

Hjalmas K, Arnold T, Bower W, et al. Nocturnal enuresis: an international evidence basedmanagement strategy. J Urol. 2004;171:2545-2561

Lawless MR, McElderry DH. Nocturnal enuresis: current concepts. Pediatr Rev. 2001;22:399-407. Available at: http://pedsinreview.aappublications.org/cgi/content/full/22/12/399

Robson WLM, Leung AKC, Van Howe R. Primary and secondary nocturnal enuresis: similaritiesin presentation Pediatrics. 2005;115:956-959. Available at:

Copyright © 2010 by the American Academy of Pediatrics page 168

Page 169: AAP PREP 2010

2010 PREP SA on CD-ROM

http://pediatrics.aappublications.org/cgi/content/full/115/4/956

Copyright © 2010 by the American Academy of Pediatrics page 169

Page 170: AAP PREP 2010

2010 PREP SA on CD-ROM

Question: 69

A family has just relocated to your community, and you are evaluating their 12-year-old son forthe first time this afternoon. Family history reveals that the boy’s father and grandmother hadpremature cardiovascular disease. The boy’s parents are concerned about his risk of heartdisease.

Of the following, the MOST important next step in this child’s evaluation is

A. echocardiography

B. electrocardiography

C. fasting lipoprotein analysis

D. random cholesterol measurement

E. referral to the cardiology clinic

Copyright © 2010 by the American Academy of Pediatrics page 170

Page 171: AAP PREP 2010

2010 PREP SA on CD-ROM

Preferred Response: CCritique: 69

In recent years, an increasing body of literature has indicated that atherosclerotic diseaseand its effect on the cardiovascular system are progressive processes that begin during earlychildhood. Research has demonstrated that the complex process of acquired cardiovasculardisease is the result of genetic predisposition to disease susceptibility along with factors suchas diet, physical activity, and other comorbidities.

In adults, the strongest risk factors for the development of cardiovascular disease include ahigh concentration of low-density lipoprotein, a low concentration of high-density lipoprotein,elevated blood pressure, type 1 or 2 diabetes mellitus, cigarette smoking, and obesity. Researchin children and adolescents has shown that some of these risk factors may be present in earlychildhood. It is imperative, therefore, for pediatricians to take proactive roles in stressing theimportance of healthy cardiovascular lifestyles and identifying children at risk for cardiovasculardisease.

The importance of the history, especially the family history, cannot be overemphasizedbecause the clinical manifestations of hypercholesterolemia are variable and may not bephysically present until later in childhood, adolescence, or even adulthood. Some children whohave homozygous familial hypercholesterolemia may demonstrate cutaneous or tendinousxanthomas, but often these findings are not apparent until early adulthood. As a result, somechildren who have significant hypercholesterolemia may have normal findings on physicalexamination.

The American Academy of Pediatrics has adopted the recommendation that childrenundergo cholesterol screening when there is a family history of premature cardiovasculardisease, such as for the boy in the vignette, or when there is a family history of high bloodconcentrations of cholesterol. It is also important to screen children for whom the family historyis not known if there is a history of other risk factors for cardiovascular disease such asobesity, hypertension, or diabetes mellitus. Accordingly, the boy in the vignette should undergo ascreening test for lipoproteins that includes cholesterol, high-density lipoproteins, and low-density lipoproteins in the fasting state.

Random cholesterol screening may provide important information, but taken in isolation, willnot offer as much information as a fasting lipoprotein panel. Referral to a specialized clinic suchas cardiology or endocrinology may be indicated in some patients, but this should be consideredonly after more complete information is obtained from the diagnostic evaluation. Neitherechocardiography nor electrocardiography is indicated for this patient at this time, and neither isused as a screening test for cardiovascular risk factors in children.

References:

Daniels SR, Greer FR and the Committee on Nutrition. Lipid screening and cardiovascular healthin childhood. Pediatrics. 2008;122:198-208. Available at:http://pediatrics.aappublications.org/cgi/content/full/122/1/198

Kwiterovich PO Jr. Recognition and management of dyslipidemia in children and adolescents. JClin Endocrinol Metab. 2008;93:4200-4209. Abstract available at:

Copyright © 2010 by the American Academy of Pediatrics page 171

Page 172: AAP PREP 2010

2010 PREP SA on CD-ROM

http://www.ncbi.nlm.nih.gov/pubmed/18697860

Lambert M, Lupien P-J, Gagné C, et al. Treatment of familial hypercholesterolemia in children andadolescents: effect of lovastatin. Pediatrics. 1996;97:619-628. Abstract available at:http://pediatrics.aappublications.org/cgi/content/abstract/97/5/619

Copyright © 2010 by the American Academy of Pediatrics page 172

Page 173: AAP PREP 2010

2010 PREP SA on CD-ROM

Question: 70

A 15-year-old boy presents to the emergency department after an apparent seizure. He had asudden arrest of normal activity at school, with posturing on the left side, eye deviation, and lossof consciousness for about 1 minute, followed by confusion. On physical examination, he is fullyoriented and answers questions appropriately. He is afebrile. Results of cranial nerveexamination, motor examination, and gait evaluation are normal.

Of the following, the procedure that is MOST likely to establish the cause of the seizure is

A. brain magnetic resonance imaging with contrast

B. electroencephalography

C. lumbar puncture

D. noncontrast head computed tomography scan

E. urine toxicology screen

Copyright © 2010 by the American Academy of Pediatrics page 173

Page 174: AAP PREP 2010

2010 PREP SA on CD-ROM

Preferred Response: ACritique: 70

The boy described in the vignette has had a first unprovoked seizure. The semiology of theseizure included some focality, with posturing on the left side, indicating that this was likely afocal, or partial, seizure. In the emergency department, after resolution of the postictalconfusion, his mental status and neurologic examination results are normal.

The most important diagnostic test for determining the cause of one or more focal-onsetseizures is a brain magnetic resonance imaging (MRI) with contrast. Although head computedtomography (CT) scan often is the initial neuroimaging performed in the emergency department,MRI is more sensitive to lesions that can be missed on CT scan (Item C70). Rather thanduplicating costly neuroimaging studies, if the child is clinically healthy and the parents arereliable, the clinician may safely defer an emergent head CT scan and schedule the moresensitive brain MRI as a follow-up test.

Urine toxicology screening should be considered in the emergency department after a firstunprovoked seizure, but the lack of confusion or encephalopathy reported for the boy combinedwith his rapid return to a normal mental status argues against a drug-induced seizure.

A lumbar puncture should be obtained in a child who has a partial seizure, fever, andconfusion to evaluate for encephalitides, particularly herpes encephalitis. Because thisadolescent is afebrile and has a clear sensorium, cerebrospinal fluid studies and urgentneuroimaging need not be obtained.

Electroencephalography (EEG) can be helpful for determining an epilepsy syndromediagnosis. For example, juvenile myoclonic epilepsy can present at this age and is characterizedby myoclonus, generalized tonic-clonic seizures, and sometimes absence seizures, withgeneralized epileptiform discharges on EEG. However, epilepsy is a clinical diagnosis made aftertwo unprovoked seizures. In the setting of a single focal seizure, the initial emphasis should beon determining whether a treatable focal lesion caused the seizure, for which EEG is unlikely tobe helpful.

References:

Hirtz D, Ashwal S, Berg A, et al. Practice parameter: evaluating a first nonfebrile seizure inchildren: report of the quality standards subcommittee of the American Academy of Neurology,the Child Neurology Society, and the American Epilepsy Society. Neurology. 2000;55:616-623.Available at: http://www.neurology.org/cgi/content/full/55/5/616

Koenigsberg RA, Bianco BA, Faro SH, et al. Neuroimaging. In: Goetz C, ed. Textbook of ClinicalNeurology. 3rd ed. Saunders Elsevier; 2007:427-466

Major P, Thiele EA. Seizures in children: determining the variation. Pediatr Rev. 2007;28:363-371.Available at: http://pedsinreview.aappublications.org/cgi/content/full/28/10/363

Major P, Thiele EA. Seizures in children: laboratory diagnosis and management. Pediatr Rev.2007;28:405-414. Available at: http://pedsinreview.aappublications.org/cgi/content/full/28/11/405

Copyright © 2010 by the American Academy of Pediatrics page 174

Page 175: AAP PREP 2010

2010 PREP SA on CD-ROM

Resta M, Palma M, Dicuonzo F, et al. Imaging studies in partial epilepsy in children andadolescents. Epilepsia. 1994;35:1187-1193. Abstract available at:http://www.ncbi.nlm.nih.gov/pubmed/7988509

Copyright © 2010 by the American Academy of Pediatrics page 175

Page 176: AAP PREP 2010

2010 PREP SA on CD-ROM

Question: 71

You are asked to consult on a baby in the neonatal intensive care unit who has severerespiratory distress and unusual physical features. The baby is receiving maximum ventilatorysupport, but his oxygen saturation is in the 80s. His nose is deviated to one side (Item Q71A), hisears are overfolded, and his hands and feet are unusually positioned (Item Q71B). You reviewhis chest radiographs and note bilateral pulmonary hypoplasia.

Of the following, the test that is likely to be MOST helpful in determining the cause of this infant’sabnormalities is

A. chromosome analysis

B. echocardiography

C. head magnetic resonance imaging

D. renal ultrasonography

E. toxicology screen

Copyright © 2010 by the American Academy of Pediatrics page 176

Page 177: AAP PREP 2010

2010 PREP SA on CD-ROM

Preferred Response: DCritique: 71

In 1946, Potter described the clinical features associated with bilateral renal agenesis.Subsequently, fetuses and infants that had similar findings were referred to as having "Pottersyndrome." More recently, however, it has been recognized that the constellation of featuresassociated with Potter syndrome actually are caused by a cascade of effects due primarily tooligohydramnios, and the term "oligohydramnios sequence" has come into favor.

The term "oligohydramnios tetrad" sometimes is used to refer to the four primary clinicalfeatures associated with a prolonged reduction in amniotic fluid: abnormal, "pugilistic" facies;lung hypoplasia; limb deformation; and renal agenesis or, variably, fetal growth deficiency. Infact, renal agenesis is not necessary for oligohydramnios sequence to occur; any circumstancethat leads to significant, longstanding oligohydramnios, such as renal dysplasia with reducedurinary output, posterior urethral valve, and chronic leakage of amniotic fluid, can be causative.Reduction in amniotic fluid volume before, or beginning at the middle, of the second trimesterinterferes with normal fetal "breathing," resulting in lung hypoplasia. Affected newborns cannotbe ventilated adequately, and pulmonary insufficiency usually is the cause of death. Reducedamniotic fluid also leads to fetal constraint that, in turn, causes facial deformities (compressionof the nose, overfolding of the ears) and unusual positioning (deformation) of the limbs.

The physical findings described for the infant in the vignette are consistent witholigohydramnios sequence, and the most appropriate next step in management is evaluation ofthe urinary tract by renal ultrasonography. If the imaging indicates renal agenesis, supportivecare can be offered. If a urinary tract outflow obstruction is present, the clinician must determinewhether surgical correction would offer benefit.

It is important to consider that this infant could have a chromosome abnormality underlyinghis anomalies and might have other birth defects affecting the heart or brain. However, absentor dysplastic kidneys or other genitourinary tract anomaly is likely to be the direct cause of hispresentation. In addition, he may have been exposed to substances prenatally that haveaffected his genitourinary system, such as cocaine or angiotensin-converting enzyme inhibitors,but such knowledge has no impact on immediate medical management.

References:

Jones KL. Oligohydramnios sequence. In: Smith’s Recognizable Patterns of HumanMalformation. 6th ed. Philadelphia, Pa: Elsevier Saunders; 2006:726-727

Potter EL. Bilateral renal agenesis. J Pediatr. 1946;29:68

Thomas IT, Smith DW. Oligohydramnios, cause of nonrenal features of Potter’s syndrome,including pulmonary hypoplasia. J Pediatr 1974;84:811-814

Copyright © 2010 by the American Academy of Pediatrics page 177

Page 178: AAP PREP 2010

2010 PREP SA on CD-ROM

Question: 72

During a health supervision visit, an adolescent girl asks about birth control options. You discussthe issues of personal choice, compliance, confidentiality, and contraceptive efficacy.

Of the following, the birth control method that is the MOST effective when used as directed is

A. combined oral contraceptive pills

B. depomedroxyprogesterone acetate

C. latex condoms

D. levonorgestrel intrauterine device

E. vaginal ring

Copyright © 2010 by the American Academy of Pediatrics page 178

Page 179: AAP PREP 2010

2010 PREP SA on CD-ROM

Preferred Response: DCritique: 72

Contraceptive efficacy usually is expressed as the percentage of women experiencing anunintended pregnancy during the first year of use. Among couples who initiate use of a method(not necessarily for the first time) and who use it perfectly (both consistently and correctly),effectiveness is expressed by the percentage who experience an accidental pregnancy duringthe first year if they do not stop use for any other reason. The effectiveness with typical useusually is lower than for perfect use for most methods (Item C72). In addition to nonadherence toa recommended regimen, a number of women discontinue a method within the first year.

The use of intrauterine devices (IUDs) in adolescents has been re-evaluated, and the newerIUDS are considered effective and safe (without increased risk of pelvic inflammatory diseaseor infertility). Their effectiveness (0.2%) approaches that of sterilization. Another effective(0.05%) long-acting method is the single-rod progesterone-only device for subcutaneousimplantation. Although the initial cost for these two methods is high, when averaged over the 5years of effectiveness for IUDs and 3 years for the implantable device, the cost is lower thanother hormonal methods. In addition to the initial cost, the other disadvantage is an initial period ofirregular bleeding that can last for a few months with both the IUD and the single rod device.

Combined hormonal contraception is available in various delivery forms: pills, a patch, and anintravaginal ring. They are all highly effective with perfect use (0.3%), but difficulties withadherence and continuation make them less effective in typical use (8%). For example, only 68%of women, on average, are still using these methods 1 year after initiation.Depomedroxyprogesterone acetate (DMPA) injection every 3 months is as effective ascombined hormonal methods with perfect use (0.3%) but has higher effectiveness with typicaluse (3%). Compliance only requires that the adolescent return to the office four times a year foran injection.

A new approach to increasing effectiveness of DMPA and combined hormonalcontraceptives is to begin the use of these methods at the time of the office visit. Previously,users were instructed to wait for their menstrual periods to begin using the pill to be sure theywere not pregnant. Studies indicate that this is not necessary if the pregnancy test result isnegative and the patient is asymptomatic at the time of the visit. One study among DMPA usersdemonstrated that this approach was associated with a decrease in the unintended pregnancyrate.

Female and male condoms are less effective than hormonal methods when used as solemethods of contraception with both perfect use (5% and 2%, respectively) and typical use(21% and 15%, respectively). However, adjunctive condom use should be encouraged toprevent sexually transmitted infections.

References:

American College of Obstetricians and Gynecologists. ACOG Committee Opinion No. 392,December 2007. Intrauterine device and adolescents. Obstet Gynecol. 2007;110:1493-1495.Abstract available at: http://www.ncbi.nlm.nih.gov/pubmed/18055754

Committee on Adolescence, American Academy of Pediatrics. Policy statement: contraception

Copyright © 2010 by the American Academy of Pediatrics page 179

Page 180: AAP PREP 2010

2010 PREP SA on CD-ROM

and adolescents. Pediatrics. 2007;120:1135-1148. Available at:http://pediatrics.aappublications.org/cgi/content/full/120/5/1135

Lopez LM, Newmann SJ, Grimes DA, Nanda K, Schulz KF. Immediate start of hormonalcontraceptives for contraception. Cochrane Database Syst Rev. 2008:2:CD006260

Reproductive Health and Research, World Health Organization. Medical Eligibility Criteria forContraceptive Use. 3rd ed. Geneva, Switzerland: World Health Organization; 2004. Available at:www.who.int/reproductive-health/publications/mec/

Trussell J. Contraceptive efficacy. In: Hatcher RA, Trussell J, Nelson AL, Cates W Jr, StewartFH, Kowal D. Contraceptive Technology. 19th ed. New York, NY: Ardent Media, Inc; 2008:747-756

Copyright © 2010 by the American Academy of Pediatrics page 180

Page 181: AAP PREP 2010

2010 PREP SA on CD-ROM

Question: 73

You admit an endotracheally intubated 6-month-old boy who has trisomy 21 and an unbalancedatrioventricular septal canal defect to the pediatric intensive care unit following cardiaccatheterization. The child’s parents report no recent fevers. Cardiac catheterization wasperformed in anticipation of upcoming surgery. On physical examination, you hear decreasedaeration over his upper lung fields bilaterally. You obtain an initial chest radiograph (Item Q73A).You ask the respiratory therapist to increase the tidal volume, and breath sounds improvethroughout his lung fields. Two hours later, you obtain a follow-up chest radiograph (Item Q73B).

Of the following, the MOST likely cause of the findings on the initial chest radiograph is

A. atelectasis

B. left mainstem bronchus intubation

C. pleural effusion

D. pneumonia

E. pneumothorax

Copyright © 2010 by the American Academy of Pediatrics page 181

Page 182: AAP PREP 2010

2010 PREP SA on CD-ROM

Preferred Response: ACritique: 73

The initial radiograph for the infant described in the vignette is highly consistent withatelectasis (Item C73A), and the subsequent image documents marked improvement in a matterof hours with increased respiratory support (Item C73B). Atelectasis is defined as theincomplete expansion or complete collapse of lung tissue and is derived from the Greek wordsateles and ektasis, which mean "incomplete expansion." Atelectasis occurs via three

mechanisms: 1)Compression of airways or lung tissue (such as by tumors, cardiomegaly, vascular

rings, or pneumothorax) 2)Resorption of oxygen in nonventilated but perfused areas of the lung (as in obstructive

diseases, foreign body aspiration, and mucus plugging) 3)Loss of adequate alveolar surface tension due to disturbances in surfactant (as in

neonatal and acute respiratory distress syndromes, pneumonias, and near-drowning) In general, younger patients are more prone to atelectasis, and the condition may be present

in up to 15% of children admitted to the pediatric intensive care unit. The upper lobes areaffected primarily in children, with right upper lobe predominance. Radiographic findings typicallyconsist of airspace consolidation, loss of volume, and displacement of fissures. Massiveatelectasis may be associated with elevation of the diaphragm and shifting of cardiac andmediastinal structures to the affected side. Clinical signs usually consist of tachypnea, cough,oxygen desaturation, and decreased breath sounds over the affected area with crackles.Contrary to traditional teachings, there is no strong evidence that atelectasis is associated withfever.

Although pneumonia also produces airspace consolidation, there is not usually anassociated volume loss, and the appearance of the radiograph does not improve as rapidly aswas observed for the infant in the vignette. Intubation of the left mainstem bronchus may resultin atelectasis, typically involving the right lung due to lack of ventilation. However, there is noevidence of a left mainstem intubation on the patient’s radiograph. In addition, mainstemintubations are typically right-sided due to the sharper angle from which the left mainstembronchus arises from the trachea. The infant’s radiographs show no evidence of apneumothorax or pleural effusion.

References:

Chase MA, Wheeler DS. Disorders of the pediatric chest. In: Wheeler DS, Wong HR, Shanley TP,eds. Pediatric Critical Care Medicine: Basic Science and Clinical Evidence. New York, NY:Springer-Verlag London Limited; 2007:361-375

Engoren M. Lack of association between atelectasis and fever. Chest. 1995;107:81-84.Abstract available at: http://www.ncbi.nlm.nih.gov/pubmed/7813318

Rozenfeld R. Atelectasis. In: Kliegman RM, Behrman RE, Jenson HB, Stanton BF, eds. NelsonTextbook of Pediatrics. 18th ed. Philadelphia, Pa: Saunders Elsevier; 2007:1830-1831

Copyright © 2010 by the American Academy of Pediatrics page 182

Page 183: AAP PREP 2010

2010 PREP SA on CD-ROM

Question: 74

The parents of a 3-year-old boy are concerned because he is the same size as 2-year-oldchildren in his preschool playgroup. Both of the parents are healthy. The father is 5 ft 3 in (160.0cm) in height and the mother is 4 ft 10 in (147.3 cm) in height. They recognize that their child maybe short because they are not tall, but they want to be sure that there is no other problem.

Of the following, the BEST indicator that the boy is following his genetic growth pattern is

A. a bone age radiograph that is normal for age

B. his height at the 3rd percentile for age

C. normal weight for height

D. steady linear growth at 3 cm/year

E. steady linear growth at 5 cm/year

Copyright © 2010 by the American Academy of Pediatrics page 183

Page 184: AAP PREP 2010

2010 PREP SA on CD-ROM

Preferred Response: ECritique: 74

Familial short stature is a diagnosis of exclusion that is defined by the presence of shortparents and an otherwise normal short child. It often is called idiopathic short stature becausefamilial short stature may have known etiologies. Eventually, genetic diagnoses should bedetermined to explain all the differences in height among families, but at present, except forrelatively unusual short stature conditions, scientists do not have the capacity to make a geneticdiagnosis. Children who have familial short stature reach a specific growth centile in the first 2years after birth, which they then follow in the normal manner until reaching adult height. At age3, a growth rate of 5 cm/year is considered normal, whereas a growth rate of 3 cm/year is morethan 3 standard deviations below the mean for growth rate for age and more suggestive of anorganic disorder causing short stature.

Prediction of adult height is based upon the reading of bone age radiographs after the age of6 or 7 years. Before that time, bone age height predictions are not useful; height predictions canbest be made by assessment of midparental height. Height at the 3rd percentile is a good signthat a child will have a reasonably normal height as an adult, but does not give information aboutage at puberty, which can limit or extend the period of active growth because of early or lateepiphyseal fusion. A child who has normal weight for height is less likely to have an underlyingserious organic disorder to explain short stature, but this finding in itself offers littleprognostically. The best predictor is continued good growth for age.

References:

Daniel M, Kleis L, Cemeroglu AP. Etiology of failure to thrive in infants and toddlers referred to apediatric endocrinology outpatient clinic. Clin Pediatr. 2008;47:762-765. Abstract available at:http://www.ncbi.nlm.nih.gov/pubmed/18467671

Ferry RJ Jr. Short stature. eMedicine Specialties, Pediatrics: General Medicine, Endocrinology.2007. Available at: http://www.emedicine.com/ped/topic2087.htm

Rogol AD. Diagnostic approach to short stature. UpToDate Online 16.3. 2008. Available forsubscription at:http://www.utdol.com/online/content/topic.do?topicKey=pediendo/2375

Copyright © 2010 by the American Academy of Pediatrics page 184

Page 185: AAP PREP 2010

2010 PREP SA on CD-ROM

Question: 75

During the health supervision visit for an 18-month-old boy, his parents express concern that heis vocalizing but not saying any real words. He is holding a small piece of string that he movesback and forth repeatedly. When you call his name, he does not respond. You point to the light inthe room and say "look," but he continues to look at the string with a sideways glance. You tryto get him to look at you, but he avoids eye contact.

Of the following, the MOST likely diagnosis for this boy is

A. Asperger disorder

B. autistic disorder

C. expressive/receptive language disorder

D. obsessive-compulsive disorder

E. Rett syndrome

Copyright © 2010 by the American Academy of Pediatrics page 185

Page 186: AAP PREP 2010

2010 PREP SA on CD-ROM

Preferred Response: BCritique: 75

The child described in the vignette shows clinical features of an autistic disorder, aheterogeneous neurodevelopmental disorder (see Table 1http://pediatrics.aappublications.org/cgi/content/full/120/5/1183 for complete diagnostic criteria).Affected individuals have impairments in three specific areas: reciprocal social interactions,verbal and nonverbal communication, and range of activities or interests. The clinicalpresentation is specific to the child, with differing degrees of impairment in each of the threecore symptom areas.

The hallmark of autism is abnormal social interactions. Children lack the ability to shareinterests with others (joint attention skills) using verbal or nonverbal communication. Theycommonly show weakness in eye contact. Their interaction may range from aloofness and anunawareness of other people to having varied or odd interaction with others. Languagedevelopment commonly is delayed, and children may have immediate or delayed echolalia,unusual intonation, and repetitive speech. Children who have autism may engage in repetitiveplay and show little imaginative play. They may focus on sensory aspects of objects or developobsessions about unusual objects (stop signs, elevators). They often have difficulty handlingtransitions and may engage in repetitive hand or body movements. Many affected children havecognitive impairment. Children who have subthreshold clinical features (some but not all of thefeatures) may receive the diagnosis of pervasive development disorder-not otherwise specifiedwithin the autism spectrum.

Red flags of development that warrant further evaluation for possible autism include:

•No babbling by 9 months

•No gesturing by 12 months

•No single words by 16 months

•No functional nonecholalic 2-word phrases by 24 months

•Any loss of language or social skills at any ageNeither a diagnosis of expressive language disorder nor obsessive-compulsive disorder

would account for the impaired social engagement exhibited by the boy in the vignette.Individuals who have Asperger syndrome (Asperger disorder in DSM-IV TR) have impairments insocial interaction and restricted interests and activities, but they have relatively preservedcognitive and language functioning, in contrast to the delays in developing language reported forthe child in the vignette. Rett syndrome almost exclusively affects females and presents with aslowing of motor development between 6 and 18 months of age. Between ages 1 and 4 years,the child exhibits a decline in social interactions, cognitive abilities, purposeful hand movements,and speech.

As a result of reviewing this information, do you intend to make a change in practiceto provide better patient care?Yes No

References:

Copyright © 2010 by the American Academy of Pediatrics page 186

Page 187: AAP PREP 2010

2010 PREP SA on CD-ROM

Chawarska K, Volkmar FR. Autism in infancy and early childhood. In: Volkmar FR, Paul R, Klin A,Cohen DJ, eds. Handbook of Autism and Pervasive Developmental Disorders: Volume 1:Diagnosis, Development, Neurobiology, and Behavior. 3rd ed. Hoboken, NJ: John Wiley & Sons,Inc; 2005:223-246

Duby JC, Johnson CP. Universal screening for autism spectrum disorders: a snapshot within thebig picture. Pediatr Ann. 2009;38:36-41

Johnson CP, Myers SM and the Council on Children with Disabilities. Identification and evaluationof children with autism spectrum disorders. Pediatrics. 2007;120:1183-1251. Available at:http://pediatrics.aappublications.org/cgi/content/full/120/5/1183

Johnson CP, Myers SM. Overview of the AAP autism spectrum disorders toolkit and guidelines: apediatrician’s roadmap to the latest ASD guidelines. Contemp Pediatr. 2008 October: 43-67

Klin A, McPartland J, Volkmar FR. Asperger syndrome. In: Volkmar FR, Paul R, Klin A, Cohen, DJ,eds. Handbook of Autism and Pervasive Developmental Disorders: Volume 1: Diagnosis,Development, Neurobiology, and Behavior. 3rd ed. Hoboken, NJ: John Wiley & Sons, Inc;2005:88-125

National Center on Birth Defects and Developmental Disabilities. Learn the Signs. Act Early.Available at: http://www.cdc.gov/ncbddd/autism/actearly/

Van Acker R, Loncola JA, Van Acker EY. Rett syndrome: a pervasive developmental disorder.In: Volkmar FR, Paul R, Klin A, Cohen DJ, eds. Handbook of Autism and PervasiveDevelopmental Disorders: Volume 1: Diagnosis, Development, Neurobiology, and Behavior.3rd ed. Hoboken, NJ: John Wiley & Sons, Inc: 2005:126-164

Copyright © 2010 by the American Academy of Pediatrics page 187

Page 188: AAP PREP 2010

2010 PREP SA on CD-ROM

Question: 76

A 4-year-old boy presents to your office for evaluation of a fluctuant axillary lymph node thathas been present for 3 weeks. He was seen at an urgent care clinic 2 weeks ago, where hewas prescribed cephalexin, but his condition has not improved. You ascertain no pertinentfindings in his past medical history. He attends preschool 5 half-days per week. The family has a12-year-old cat, a 3-month-old kitten, and an iguana. On physical examination, the child appearsin no acute distress and has a temperature of 39.0°C. There are no unusual findings except fora 4-cm fluctuant right axillary lymph node (Item Q76A) with overlying erythema and a small redpapule above the right wrist (Item Q76B).

Of the following, the test MOST likely to confirm the diagnosis is a(n)

A. antistreptolysin O titer

B. Bartonella henselae serology

C. culture of lymph node aspirate

D. Toxoplasma serology

E. tuberculin skin test

Copyright © 2010 by the American Academy of Pediatrics page 188

Page 189: AAP PREP 2010

2010 PREP SA on CD-ROM

Preferred Response: BCritique: 76

Lymphadenopathy can be caused by bacteria (eg, Staphylococcus aureus or group AStreptococcus), mycobacteria, fungi, and viruses. In certain instances, malignancies (eg,lymphoma or leukemia) also are part of the differential diagnosis. However, infection is morelikely in a child who has fever and an erythematous, fluctuant lymph node. A history of pet oranimal exposures, travel, and ill contacts may help guide further evaluation of the child who haslymphadenitis.

The history of a kitten at home combined with the finding of a fluctuant axillary adenitis and adistal papule described for the boy in the vignette are most suggestive of cat-scratch disease.Bartonella henselae, the etiologic agent of cat-scratch disease, commonly colonizes kittensyounger than 1 year of age; it is less frequent in older cats. Specific immunoglobulin G and Mantibodies against B henselae may help confirm the diagnosis. If a lymph node biopsy wereperformed, a Warthin-Starry silver stain also may confirm the diagnosis, but biopsy generally isnot indicated in uncomplicated disease. Cat-scratch adenitis is self-limited in the otherwisehealthy host, although it may take months to resolve. B henselae demonstrates in vitro sensitivityto a number of antibiotics, and for immunocompromised hosts who may experience moredisseminated disease, treatment with azithromycin, erythromycin, trimethoprim-sulfamethoxazole, rifampin, or gentamicin may be beneficial.

Toxoplasma gondii is an intracellular parasite that undergoes sexual maturation in cats andcan be found in the feces of 30% to 40% of domestic cats. Human infection, however, mostoften is acquired through ingestion of raw or undercooked contaminated meat or contact withsoil contaminated with Toxoplasma cysts. Daily changing of cat litter boxes prevents maturationof cysts and infection from this source. The risk of Toxoplasma infection from direct exposureto domestic cats is not established, but likely is low. Furthermore, Toxoplasma infection in ahealthy host is generally asymptomatic. If adenopathy occurs, it is generally bilateral, nontenderand not associated with fever or fluctuance. Cervical rather than axillary nodes are morecommonly affected. The infection generally is diagnosed by specific serologic testing.

Aspiration of a fluctuant lymph node may relieve discomfort and can help in identifyingbacterial pathogens, but B henselae serology is the preferred diagnostic test for cat-scratchdisease for this boy. An antistreptolysin O titer may give evidence of a recent group Astreptococcal infection, but it is not a primary diagnostic test for acute streptococcal infection. Atuberculin skin test may be reactive in tuberculous adenitis or weakly reactive (5 to 10 mm ofinduration) in atypical mycobacterial adenitis. Mycobacterial adenitis generally is cervical (ItemC76) and would not account for the papule above the child’s wrist. Iguanas and other lizardsmay transmit Salmonella, but that is not a cause of lymphadenitis, as described for this boy.

Zoonoses, animal diseases that are transmitted to people, are a consideration in people whohave pets. Children are at greatest risk because they are likely to have close contact with theirpets. Dogs and cats are the most common pet animals, but lizards, birds, and other exotic petsalso can transmit infections. Thus, a history of animal exposure(s) may be helpful, as in thiscase. Zoonotic infections may be transmitted by animal bites or saliva (eg, Pasteurella multocidafrom cats and dogs, rabies), fecal transmission (eg, Salmonella, Campylobacter, parasites),direct contact (eg, ringworm), or respiratory secretions (eg, Coxiella burnetii, Bordetellabronchiseptica).

Copyright © 2010 by the American Academy of Pediatrics page 189

Page 190: AAP PREP 2010

2010 PREP SA on CD-ROM

Children who attend child care may be at increased risk of exposure to a number ofpathogens, including group A Streptococcus and community-acquired methicillin-resistantStaphylococcus aureus. They also have higher rates of respiratory viral infections and resistantpathogens associated with otitis media.

References:

American Academy of Pediatrics. Appendix VII. Diseases transmitted by animals (zoonoses). In:Pickering LK, Baker CJ, Kimberlin DW, Long SS, eds. Red Book: 2009 Report of the Committeeon Infectious Diseases. 28th ed. Elk Grove Village, Ill: American Academy of Pediatrics;2009:864-870

American Academy of Pediatrics. Cat-scratch disease (Bartonella henselae). In: Pickering LK,Baker CJ, Kimberlin DW, Long SS, eds. Red Book: 2009 Report of the Committee on InfectiousDiseases. 28th ed. Elk Grove Village, Ill: American Academy of Pediatrics; 2009:249-250

American Academy of Pediatrics. Children in out-of-home child care. In: Pickering LK, Baker CJ,Kimberlin DW, Long SS, eds. Red Book: 2009 Report of the Committee on Infectious Diseases.28th ed. Elk Grove Village, Ill: American Academy of Pediatrics; 2009:124-140

Friedmann AM. Evaluation and management of lymphadenopathy in children. Pediatr Rev.2008;29:53-60. Available at: http://pedsinreview.aappublications.org/cgi/content/full/29/2/53

Kotton CN. Zoonoses from pets other than dogs and cats. UpToDate Online 16.3. 2008.Available for subscription at:www.utdol.com/online/content/topic.do?topicKey=oth_bact/5044&view=print

Copyright © 2010 by the American Academy of Pediatrics page 190

Page 191: AAP PREP 2010

2010 PREP SA on CD-ROM

Question: 77

You are evaluating a 4-year-old boy for a 3-day history of temperature to 40.0°C, diarrhea,abdominal pain, and listlessness. His mother states that they returned 1 week ago from Pakistan,where the family spent 1 month visiting relatives in both urban and rural areas. The childreceived no prophylactic travel vaccines or medications prior to the trip. On physicalexamination, the tired-appearing boy has a temperature of 39.5°C; diffuse, moderate abdominaltenderness; and a spleen that is palpable 2 cm below the left costal margin. A complete blood

count shows a peripheral white blood cell count of 14.0x103/mcL (14.0x109/L), hemoglobin of

12 mg/dL (120 g/L), and platelet count of 350x103/mcL (350x109/L).

Of the following, the MOST likely pathogen causing this patient’s symptoms is

A. Entamoeba histolytica

B. hepatitis A virus

C. hepatitis B virus

D. Plasmodium falciparum

E. Salmonella typhi

Copyright © 2010 by the American Academy of Pediatrics page 191

Page 192: AAP PREP 2010

2010 PREP SA on CD-ROM

Preferred Response: ECritique: 77

Obtaining a travel history is a very important part of the evaluation of an ill patient. Such ahistory may help the physician either broaden or narrow the differential diagnosis of an illness,depending on the potential diseases to which the patient might have been exposed. This isespecially important for patients who have traveled to other areas of the world, such as the boyin the vignette, where there are multiple endemic diseases.

One of the greatest risks to travelers in many parts of the developing world is malaria, andappropriate prophylactic drugs must be taken and mosquito protection measures used whengoing to an area in which malaria transmission occurs. Hepatitis A is highly endemic in manycountries and is one of the most common diseases to which a traveler may be exposed.Hepatitis B is highly endemic in Asia, Africa, and parts of South America. Travelers to tropicaland subtropical areas of the world, in addition to being at risk for exposure to malaria, also are atrisk for exposure to dengue fever, leptospirosis, and other vector-borne pathogens. Typhoidfever is endemic in most of the developing world, where poor sanitation and food and watercontamination are common. This is especially true in South America, India, and Pakistan, whereattack rates in travelers are the highest worldwide. Tuberculosis is endemic worldwide,especially in developing countries, and there is a risk for travelers to be exposed to this illness.Other diseases to which a traveler may be exposed include yellow fever (South America andAfrica); meningococcal disease (sub-Saharan Africa; Middle East and Saudi Arabia, especiallyMecca or Medina; other developing countries); Japanese encephalitis (Southeast Asia, ruralareas of China, eastern Russia, Indian subcontinent), rabies (highly endemic in rural areas ofdeveloping countries and South America), cholera (South America, India, Southeast Asia, Africa,Middle East); respiratory tract viruses, including severe acute respiratory syndrome (SARS) andavian influenza; parasites; and polio.

The history of travel to Pakistan and the clinical presentation described for the patient in thevignette are most consistent with typhoid fever caused by Salmonella typhi. The patient mayhave been exposed to all of the other organisms listed during his trip to Pakistan, but hispresentation is not consistent with their disease manifestations. Disease caused by Entamoebahistolytica has a gradual onset over a 3-week period, the manifestations of hepatitis A normallyare asymptomatic or very nonspecific in this age group, the patient has no history of exposureto the risk factors for hepatitis B and the average incubation period is 3 months, and he is notmanifesting typical symptoms for malaria (high fever, chills, rigors, sweats, and headache).

References:

American Academy of Pediatrics. International travel. In: Pickering LK, Baker CJ, Kimberlin DW,Long SS, eds. Red Book: 2009 Report of the Committee on Infectious Diseases. 28th ed. ElkGrove Village, Ill: American Academy of Pediatrics; 2009:98-104

Freeman DO. Protection of travelers. In: Mandell GL, Bennett JE, Dolan R, eds. Mandell, Douglas,and Bennett’s Principles and Practice of Infectious Diseases. 6th ed. Philadelphia, Pa: ElsevierChurchill Livingstone;2005:3637-3646

Copyright © 2010 by the American Academy of Pediatrics page 192

Page 193: AAP PREP 2010

2010 PREP SA on CD-ROM

Wilson ME, Weld LH, Boggild A, et al. Fever in returning travelers: results from the GeoSentinelSurveillance Network. Clin Infect Dis. 2007;44:1560-1568. Available at:http://www.journals.uchicago.edu/doi/full/10.1086/518173?cookieSet=1

Copyright © 2010 by the American Academy of Pediatrics page 193

Page 194: AAP PREP 2010

2010 PREP SA on CD-ROM

Question: 78

You are evaluating a 14-year-old girl during a health supervision visit. On physical examination,her weight is at the 50th percentile, height is at the 75th percentile, temperature is 36.8°C, pulseis 72 beats/min, respiratory rate is 14 breaths/min, and blood pressure is 160/96 mm Hg.

Laboratory evaluation reveals:

•Normal urinalysis findings

•Sodium, 140 mEq/L (140 mmol/L)

•Potassium, 3.2 mEq/L (3.2 mmol/L)

•Chloride, 100 mEq/L (100 mmol/L)

•Bicarbonate, 28 mEq/L (28 mmol/L)

•Blood urea nitrogen, 14.0 mg/dL (5.0 mmol/L)

•Creatinine, 0.8 mg/dL (70.7 mcmol/L)Renal ultrasonography shows a normal right kidney that is 10.5 cm in length and a left kidneythat is 7.0 cm in length (greater than 2 standard deviations below the mean for age). Bothkidneys are of normal echotexture and have no cysts, calculi, or masses. Echocardiographyreveals moderate left ventricular hypertrophy but an otherwise structurally normal heart.

Of the following, the MOST likely cause for this patient’s elevated blood pressure is

A. catecholamine excess

B. fluid overload

C. increased cardiac output

D. increased systemic vascular resistance

E. transient ("white coat") hypertension

Copyright © 2010 by the American Academy of Pediatrics page 194

Page 195: AAP PREP 2010

2010 PREP SA on CD-ROM

Preferred Response: DCritique: 78

Hypertension in children is defined as mean systolic or diastolic blood pressure greater thanthe 95th percentile on repeat (>3) measurements, based on pediatric blood pressure norms forage, sex, and height percentile. The general rule is: the younger the child and the higher theblood pressure, the more likely that hypertension is due to an underlying cause. In addition,secondary hypertension is more common in children than adults.

The patient described in the vignette has marked hypertension associated with ahypokalemic, metabolic alkalosis. The hypokalemic, metabolic alkalosis likely is explained via anactivated renin-angiotensin-aldosterone system (RAAS). In the setting of diminished renal bloodflow, renin production is increased. Through a series of steps, angiotensin II and aldosteroneconcentrations increase. Aldosterone causes increased absorption of sodium in the distal tubulein exchange for potassium and hydrogen, with resultant hypokalemia and metabolic alkalosis.

An additional clinical feature of the patient in the vignette is renal asymmetry. Taken togetherwith the hypokalemic, metabolic alkalosis, the most likely explanation is unilateral renal arterystenosis. Activation of the RAAS results in elevated concentrations of angiotensin II, a potentvasoconstrictor, which causes high systemic vascular resistance. Although affected patientsmay have a component of volume overload from increased aldosterone, this is not thepredominant effect. Other mechanisms of hypertension include increased cardiac output fromeither increased stroke volume (as seen in volume overload) or increased heart rate (fromsympathetic stimulation, as in hyperthyroidism or catecholamine excess). Because the patient inthe vignette does not have tachycardia, these mechanisms are not likely to be the cause of herhypertension.

"White coat" hypertension describes elevated blood pressure in the clinic but not in theambulatory (home) setting. This patient’s markedly elevated blood pressure and laboratoryfindings are inconsistent with white coat hypertension.

References:

Flynn JT. Management of hypertension in children and adolescents. In: Kher KK, Schnaper HW,Makker SP, eds. Clinical Pediatric Nephrology. 2nd ed. London, England: Informa Healthcare;2007:481-492

McNiece KL, Portman RJ. Hypertension: epidemiology and evaluation. In: Kher KK, Schnaper HW,Makker SP, eds. Clinical Pediatric Nephrology. 2nd ed. London, England: Informa Healthcare;2007:461-480

National High Blood Pressure Education Program Working Group on High Blood Pressure inChildren and Adolescents. The fourth report on the diagnosis, evaluation, and treatment of highblood pressure in children and adolescents. Pediatrics. 2004;114:555—576. Available at:http://pediatrics.aappublications.org/cgi/content/full/114/2/S2/555

Rosenbaum DM, Korngold E, Teele RL. Sonographic assessment of renal length in normalchildren. AJR Am J Roentgenol. 1984;142:467-469. Available at:

Copyright © 2010 by the American Academy of Pediatrics page 195

Page 196: AAP PREP 2010

2010 PREP SA on CD-ROM

http://www.ajronline.org/cgi/reprint/142/3/467

Tullus K, Brennan E, Hamilton G, et al. Renovascular hypertension in children. Lancet.2008;371:1453-1463. Abstract available at: http://www.ncbi.nlm.nih.gov/pubmed/18440428

Copyright © 2010 by the American Academy of Pediatrics page 196

Page 197: AAP PREP 2010

2010 PREP SA on CD-ROM

Question: 79

You are evaluating a 2-month-old boy at his scheduled health supervision visit. His mother isconcerned that her son has had clear rhinorrhea for the past 3 days that has madebreastfeeding more difficult. On physical examination, the infant is alert, does not appear ill, andis afebrile. The only finding of note is clear rhinorrhea on nasal examination. The mother askshow she can decrease his nasal discharge.

Of the following, the BEST advice is to administer

A. oral antibiotic

B. over-the-counter antihistamine

C. over-the-counter nasal decongestant

D. over-the-counter oral decongestant

E. saline nasal drops with bulb suctioning

Copyright © 2010 by the American Academy of Pediatrics page 197

Page 198: AAP PREP 2010

2010 PREP SA on CD-ROM

Preferred Response: ECritique: 79

Management of an upper respiratory tract infection (URI) in an infant or child is intended toprovide temporary relief of symptoms, specifically nasal congestion, rhinorrhea, and coughing.For the boy described in the vignette, supportive care that includes saline nasal drops and bulbsuctioning is appropriate.

Common nasal decongestants include pseudoephedrine, phenylephrine, and oxymetazoline.Phenylpropanoloamine was removed from the market due to its association with cardiomyopathyand intracranial hemorrhage. Oxymetazoline is an effective nasal decongestant but generally isnot used in children younger than 6 years of age and can cause rhinitis medicamentosa(rebound congestion) with prolonged usage. This can be dangerous in infants 6 months of ageor younger because they are dependent on nasal airflow for respiration.

When used appropriately, oral decongestants or oral antihistamines are useful and welltolerated in children older than 6 years of age. However, for children younger than 2 years,reported adverse effects have included tachyarrythmias, insomnia, hyperactivity, ataxia,agitated psychosis, hallucinations, and even death. In September 2007, the United States Foodand Drug Administration (FDA) conducted a survey of over-the-counter (OTC) cough and coldmedications. A total of 54 children died from 1969 through 2006 after being administredmedicines containing diphenhydramine, brompheniramine, and chlorpheniramine. Most of thedeaths occurred in children younger than 2 years of age. This finding resulted in a voluntaryremoval of OTC medicines labeled for cold and cough for children younger than age 2 years. Anadditional public health advisory was released by the FDA in 2008, warning parents to avoidgiving OTC cold and cough medicines to children younger than 2 years of age because ofpotentially "serious and life-threatening side effects."

Antibiotics are not indicated for this infant due to the short duration of symptoms (3 days)and because the child has no fever and does not appear ill. The diagnosis of acute bacterialsinusitis as a complication of a viral URI generally is reserved for patients experiencingpersistent symptoms for more than 7 to 10 days.

As a result of reviewing this information, do you intend to make a change in practiceto provide better patient care?Yes No

References:

Centers for Disease Control and Prevention. Infant deaths associated with cough and coldmedications–Two states, 2005. MMWR Morb Mortal Wkly Rep. 2007;56:1-4. Available at:http://www.cdc.gov/mmwr/preview/mmwrhtml/mm5601a1.htm

Kelly LF. Pediatric cough and cold preparations. Pediatr Rev. 2004;25:115-123. Available at:http://pedsinreview.aappublications.org/cgi/content/full/25/4/115

Schaefer MK, Shehab N, Cohen AL, Budnitz DS. Adverse events from cough and coldmedications in children. Pediatrics. 2008;121:783-787. Available at:

Copyright © 2010 by the American Academy of Pediatrics page 198

Page 199: AAP PREP 2010

2010 PREP SA on CD-ROM

http://pediatrics.aappublications.org/cgi/content/abstract/121/4/783

van Velzen AG, van Riel AJ, Hunault C, van Riemsdijk TE, de Vries I, Meulenbelt J. A case seriesof xylometazoline overdose in children. Clin Toxicol. 2007;45:290-294. Abstract available at:http://www.ncbi.nlm.nih.gov/pubmed/17453884

Vernacchio L, Kelly JP, Kaufman DW, Mitchell AA. Cough and cold medication use by USChildren, 1999-2006: results from the Slone survey. Pediatrics. 2008;122:e323-e329. Availableat: http://pediatrics.aappublications.org/cgi/content/full/122/2/e323

Wallace DV, Dykewicz MS, Bernstein DI, et al; Joint Task Force on Practice; American Academyof Allergy, Asthma & Immunology; American College of Allergy, Asthma and Immunology; JointCouncil on Allergy, Asthma and Immunology. The diagnosis and management of rhinitis: anupdated practice parameter. J Allergy Clin Immunol. 2008;122(s suppl):S1-S84

Copyright © 2010 by the American Academy of Pediatrics page 199

Page 200: AAP PREP 2010

2010 PREP SA on CD-ROM

Question: 80

A 16-year-old baseball player is brought to the emergency department after being hit in the lefteye by a pitched ball. He complains of severe pain and decreased vision in his left eye. His heartrate is 120 beats/min, respiratory rate is 20 breaths/min, and blood pressure is 140/86 mm Hg.Physical examination reveals an agitated boy who has marked swelling around his left orbit butno palpable deformities along the orbital ridge. His extraocular movements are intact bilaterally.His right pupil is 3 mm and briskly reactive, but his left pupil is 5 mm and sluggishly reactive.Closer inspection of the left eye reveals findings shown in Item Q80.

Of the following, the MOST appropriate next step is to

A. administer antibiotic eyedrops

B. administer ibuprofen for analgesia

C. instruct the boy to lay flat in bed

D. place an eye shield over the left eye

E. sedate the boy with ketamine

Copyright © 2010 by the American Academy of Pediatrics page 200

Page 201: AAP PREP 2010

2010 PREP SA on CD-ROM

Preferred Response: DCritique: 80

The boy described in the vignette has a hyphema due to blunt globe trauma. A hyphema isthe collection of blood in the anterior chamber of the eye between the iris and the cornea. Ittypically results from damage to the limbal vessels and iris or ciliary tissue following blunt orpenetrating globe injury. Clinically, affected patients complain of visual disturbances ranging fromblurred vision to complete vision loss, photophobia, and eye pain. Nausea, vomiting and lethargyalso may accompany this injury. Physical examination following eye trauma should includeassessment of visual acuity; inspection of the lids, lacrimal duct, and cornea; and evaluation ofpupillary responses and extraocular movements. Patients who have a hyphema demonstratediminished visual acuity in the affected eye and, in most cases, a layer of blood in the inferioraspect of the anterior chamber behind the cornea (Item C80). Slitlamp examination may benecessary to view smaller, microhyphemas.

A hyphema is an ophthalmologic emergency because of the risk for vision-compromisingcomplications. Specifically, early ophthalmologic intervention is directed at preventing rebleeding,which can result in permanent corneal staining, and increased intraocular pressure, which cancause optic nerve damage. Patients at greatest risk of complications are those who havebleeding diatheses, sickle hemoglobinopathies, and large hyphemas.

While awaiting ophthalmologic consultation, the initial management of a hyphema includesplacement of a protective eye shield (not a pressure patch) and recumbent positioning in a dimlylit environment, with the head elevated 30 degrees to facilitate inferior layering of the blood.Direct pressure on the eye should be avoided. No medications should be instilled into the eyeuntil the patient is examined by an ophthalmologist. Vomiting should be treated to preventintraocular pressure increases. Pain is managed with acetaminophen or narcotics; non-steroidalanti-inflammatory analgesics are relatively contraindicated because of their potential effects onplatelet function. Sedation rarely is necessary, although if required, ketamine is contraindicatedbecause it increases intraocular pressure.

References:

Andreoli CM, Gardiner MF. Traumatic hyphema: clinical features and management. UpToDateOnline 16.3. 2008. Available at:http://www.utdol.com/online/content/topic.do?topicKey=ped_trau/9543&selectedTitle=1~17&source=search_result

Shepard JD Jr. Hyphema. eMedicine Specialties, Ophthalmology, Anterior Chamber. 2006.Available at: http://www.emedicine.com/oph/TOPIC765.HTM

Copyright © 2010 by the American Academy of Pediatrics page 201

Page 202: AAP PREP 2010

2010 PREP SA on CD-ROM

Question: 81

You are asked to write intravenous nutrition orders for a 5-year-old, 20-kg boy, who wasadmitted to the hospital for treatment of presumed idiopathic pancreatitis. Based on the child’sclinical status, you expect him to take nothing by mouth for 5 to 7 days.

Of the following intravenous regimens, the one that is MOST appropriate is

A. central venous alimentation with 20% dextrose and 1.5% amino acids

B. central venous alimentation with 20% dextrose, 1.5% amino acids, and 20 g/day lipid

C. peripheral venous infusion with 10% dextrose and 3.0% amino acids

D. peripheral venous infusion with 10% dextrose, 1.5% amino acids, and 10 g/day lipid

E. peripheral venous infusion with 15% dextrose and 2.5% amino acids

Copyright © 2010 by the American Academy of Pediatrics page 202

Page 203: AAP PREP 2010

2010 PREP SA on CD-ROM

Preferred Response: DCritique: 81

Decisions regarding nutrition support of the hospitalized patient must be based on the

answers to three critical questions:

1.Can the gastrointestinal (GI) tract be used, either partially or totally?

2.If parenteral nutrition is required, for how long will it be used?3.Based upon the answers to questions 1 and 2, how may the patient’s nutrient and fluid

requirements best be met?The child described in the vignette, who has acute pancreatitis, is expected to require

parenteral nutrition for 5 to 7 days. The parenteral route of administration is chosen both toreplace fluids lost due to pancreatic inflammation and to promote "pancreatic rest" by decreasingthe cephalic, gastric, and intestinal phases of pancreatic secretion. Because the length ofparenteral infusion is estimated to be for 7 or fewer days, a peripheral vein is the indicated routeof administration. Peripheral intravenous nutrition comprising 10% dextrose, 1.5% amino acids,and 10 g/day lipid is best for supporting this child’s basal energy and protein needs andameliorating the potential catabolic state during recovery.

Parenteral nutrition can be a lifesaving intervention, providing essential fluids and nutrientsfor the patient in whom the GI tract cannot be used fully to meet nutritional needs for extendedperiods. Parenteral nutrition administered via central venous catheter generally should bereserved for two situations. The first is when no oral/enteral nutrition may be given for morethan 7 days because of either medical or postsurgical conditions. The second is when partialoral/enteral feedings, in conjunction with peripheral intravenous nutrition, either cannot meetnutrient needs or will be required for extended periods, thus presenting problems of prolongedperipheral venous access. Central venous support also may be required in some children whenvenous access problems make peripheral infusions difficult to maintain, either because of patientage or clinical condition. Whenever possible, the GI tract should be used to deliver nutrients,even if only a portion of total daily requirements for energy and protein are met. Infants deprivedof oral feeding stimulation for long periods may lose the desire or the ability to feed. Intraluminalnutrients are essential for maintaining intestinal function. Prolonged absence of oral/enteralfeedings also may compromise the mucosal barrier and increase the risk for bacteria andbacterial products in the intestine entering the blood.

Although a comprehensive review of fluid and macro- and micronutrient needs for infantsand children is beyond the scope of this critique, daily requirements for a child weighing 20 kg ormore who is receiving parenteral nutrition in the hospital include:

1.Fluid = 1,500 mL + 20 mL/kg for more than 20 kg2.Energy = 30 to 75 kcal/kg as dextrose (3.4 kcal/g) (~10 to 20 g/kg)3.Lipids = 1 to 3 g/kg4.Amino Acids = 1 to 2 g/kg

For the patient receiving total intravenous nutrition via central venous catheter, dextroseconcentrations generally are in the range of 20%. However, solutions containing more than12.5% dextrose are highly sclerosing to small veins and should be avoided in all peripheralinfusions. In most cases, a 10% dextrose solution is employed for peripheral vein infusions.

For the 20-kg boy described in the vignette, total fluid volume initially should be in the range

Copyright © 2010 by the American Academy of Pediatrics page 203

Page 204: AAP PREP 2010

2010 PREP SA on CD-ROM

of 1,500 mL per 24 hours. An infusate containing 10% dextrose provides 150 g of dextrose tothis child, which is equal to 510 kcal or approximately 25 kcal/kg per day. This amount of energyis less than calculated requirements and reflects the limits posed by infusing a 10% dextrosesolution. However, based on patient tolerance, the infusion rate may be increased gradually toenhance energy intake. A 1.5% amino acid infusion administered at the stated rate shouldprovide approximately 1 g/kg protein. Adding significantly greater amounts to the initial infusate(eg, 3.0% amino acids) may result in excessive protein consumption if the rate is increased.Excessive protein intake potentially can increase the risk for hyperammonemia and azotemia dueto protein catabolism.

Lipid infusions always should commence at a lower-than-targeted amount to allow formetabolic adaptation to intravenous fat and to prevent hypertriglyceridemia. Indeed, for short-term administration, 0.5 g/kg per day of intravenous lipids should be sufficient to preventdeficiency of essential fatty acids. The decision to increase lipids beyond this level depends onseveral factors, including the need to increase energy intake in a patient who has compromisednutritional status and any fluid volume limitations.

Some have stated that intravenous lipid should be avoided in the patient who haspancreatitis because hyperlipidemia is a known risk factor for pancreatitis. However, availableevidence demonstrates that appropriately monitored, lipid-based intravenous nutrition regimensare well tolerated in affected patients.

References:

Kleinman RE. Parenteral nutrition. In: Pediatric Nutrition Handbook. 5th ed. Elk Grove Village, Ill:American Academy of Pediatrics; 2004:369-389

Pietzak MM, Thomas DW. Pancreatitis in childhood. Pediatr Rev. 2000;21:406-412. Available at:http://pedsinreview.aappublications.org/cgi/content/full/21/12/406

Shulman RJ, Phillips S. Parenteral nutrition indications, administration, and monitoring. In: BakerSS, Baker RD, Davis AM, eds. Pediatric Nutrition Support. Sudbury, Mass: Jones and BartlettPublishers; 2006:273-286

Silberman H, Dixon NP, Eisenberg D. The safety and efficacy of a lipid-based system ofparenteral nutrition in acute pancreatitis. Am J Gastroenterol. 1982;77:494-497. Abstractavailable at: http://www.ncbi.nlm.nih.gov/pubmed/6178287

Copyright © 2010 by the American Academy of Pediatrics page 204

Page 205: AAP PREP 2010

2010 PREP SA on CD-ROM

Question: 82

A 25-year-old woman who had a recent urine drug screen that was positive for amphetaminesdelivers a 2,200-g infant at 38 weeks’ gestation. When you examine the baby at 4 hours of age,he is irritable and difficult to console. However, he becomes calm when swaddled and placed inan environment with low light and minimal noise.

Of the following, a TRUE statement about amphetamine use in pregnancy is that it

A. is associated with an increase in fetal anomalies

B. often is associated with polydrug use

C. results in central nervous system depression

D. results in fetal macrosomia

E. results in a neonatal withdrawal syndrome

Copyright © 2010 by the American Academy of Pediatrics page 205

Page 206: AAP PREP 2010

2010 PREP SA on CD-ROM

Preferred Response: BCritique: 82

The newborn described in the vignette is small for gestational age (SGA) and of lowbirthweight (LBW) (<2.5 kg). He is irritable and may be described as easily stressed oroverstimulated with environmental stimuli and having poor state control. These characteristicsare common in newborns exposed to amphetamines in utero. Many women who useamphetamines also use tobacco, alcohol, and illicit medications. Accordingly, polydrug use maycontribute to some characteristics seen in the newborn whose mother used amphetamines.Although no amphetamine withdrawal syndrome has been described and there is no pattern offetal anomalies attributable to maternal amphetamine ingestion, the direct effects of

amphetamines (which are central nervous system stimulants) are manifested as:1)Fetal growth restriction (due to vascular effects on the uteroplacental vasculature and

general impaired maternal health and nutritional status)

2)Neonatal agitation, irritability, and hypersensitivity to environmental stimuli3)Potential developmental and cognitive impairment in early childhood

References:

Smith LM, Lagasse LL, Derauf C, et al. Prenatal methamphetamine use and neonatalneurobehavioral outcome. Neurotoxicol Teratol. 2008;30:20-28. Available at:http://www.pubmedcentral.nih.gov/articlerender.fcgi?tool=pubmed&pubmedid=18031987

Smith LM, LaGasse LL, Derauf C, et al. The Infant Development, Environment, and LifestyleStudy: effects of prenatal methamphetamine exposure, polydrug exposure, and poverty onintrauterine growth. Pediatrics. 2006;118:1149-1156. Available at:http://pediatrics.aappublications.org/cgi/content/full/118/3/1149

Copyright © 2010 by the American Academy of Pediatrics page 206

Page 207: AAP PREP 2010

2010 PREP SA on CD-ROM

Question: 83

A 12-year-old girl comes to your clinic for her health supervision visit. She has been healthy inthe past and is doing well in special education classes at school. Her only complaint at this visitis the development of "bad acne" over the last year. She has tried over-the-counter benzoylperoxide without relief. Physical examination reveals multiple 1- to 2-mm reddish-brown, firmpapules over her nose and cheeks and extending up to the lower eyelids (Item Q83).

Of the following, the MOST likely underlying cause of this finding is

A. acne vulgaris

B. neurofibromatosis type 1

C. periorificial dermatitis

D. systemic lupus erythematosus

E. tuberous sclerosis complex

Copyright © 2010 by the American Academy of Pediatrics page 207

Page 208: AAP PREP 2010

2010 PREP SA on CD-ROM

Preferred Response: ECritique: 83

The girl described in the vignette has skin findings consistent with angiofibromas, previouslytermed adenoma sebaceum. Angiofibromas are small, firm papules that may be flesh-colored,reddish pink, or brown (Item C83A). Typically they are found over the nose and cheeks, but theymay appear elsewhere on the face. Over time, they enlarge to polypoid growths, but at earlystages, they can be mistaken for acne. Angiofibromas are one of the dermatologic features oftuberous sclerosis complex. Other skin findings in this disease are shagreen patches (yellowishraised plaques) (Item C83B), ash-leaf macules (hypopigmented macules) (Item C83C), andperiungual fibromas (Item C83D). Neurologic manifestations in tuberous sclerosis complex rangefrom epilepsy and severe intellectual disability to mild intellectual disability or normal intelligencewithout seizures.

Several types of lesions are associated with acne vulgaris, including open and closedcomedones, papules, pustules, and nodules, which may be seen anywhere on the face, chest,and back. The skin findings in neurofibromatosis type 1 are café au lait macules, axillaryfreckling (Item C83E), and neurofibromas. Periorificial dermatitis occurs primarily in young womenand is characterized by discrete erythematous papules and pustules in the nasolabial folds andchin (Item C83F). The cause is unclear, but it may be due to external irritants. Systemic lupuserythematosus can be associated with several skin abnormalities but the two most common aremalar erythema extending across the nose and cheeks and discoid lesions, which are well-circumscribed plaques colored red to purplish.

References:

Krowchuk DP. Managing adolescent acne: a guide for pediatricians. Pediatr Rev. 2005;26:250-261. Available at: http://pedsinreview.aappublications.org/cgi/content/full/26/7/250

Morelli JG. Hypopigmented lesions. In: Kliegman RM, Behrman RE, Jenson HB, Stanton BF, eds.Nelson Textbook of Pediatrics. 18th ed. Philadelphia, Pa: Saunders Elsevier; 2007:2682-2685

Copyright © 2010 by the American Academy of Pediatrics page 208

Page 209: AAP PREP 2010

2010 PREP SA on CD-ROM

Question: 84

A 13-year-old girl who plays soccer presents with a temperature to 38.9°C for 2 days,dysphagia, malaise, and nausea. She has had no cough or rhinorrhea. Physical examinationreveals erythema of the tonsils with petechial hemorrhages, petechiae on the soft palate, mildenlargement of the cervical lymph nodes, and vague discomfort in the epigastric area. Onevaluation, you palpate a spleen tip. You obtain a rapid streptococcal antigen test and a throatculture. The rapid streptococcal antigen test is negative and you recommend antipyretics andrest. She returns to the clinic 48 hours later because her symptoms have not improved. Thethroat culture is negative, but the girl reports continued fever, increasing malaise, and somevomiting. Findings on the physical examination have not changed, and there are no signs ofserious bacterial infection or dehydration.

Of the following, the MOST efficacious next test for this patient is

A. complete blood count with differential count and platelet measurement

B. Epstein-Barr virus titers

C. rapid influenza test

D. spot test for infectious mononucleosis

E. viral culture for adenovirus

Copyright © 2010 by the American Academy of Pediatrics page 209

Page 210: AAP PREP 2010

2010 PREP SA on CD-ROM

Preferred Response: DCritique: 84

The girl described in the vignette has symptoms and signs of pharyngitis, but the rapidstreptococcal antigen test is negative. Accurate diagnosis and management of pharyngitis inchildren prevents unnecessary antibiotic usage for viral disease. The judicious use of antibioticsin children who have pharyngitis begins with administration of rapid streptococcal antigen testsor throat culture to confirm streptococcal disease prior to antibiotic administration. Empiricantibiotics rarely are justified, even in a child who is clinically ill. The Centor criteria forstreptococcal pharyngitis include fever, sore throat, tender cervical lymphadenopathy, and theabsence of cough. The presence of exudates is not particularly helpful because up to 70% ofcases of exudative pharyngitis are due to viral infection. Petechiae on the soft palate (Item C84)may be a sign of group A streptococcal pharyngitis, but macules, vesicles, and other lesions ofthe palate also are seen in Epstein-Barr virus, coxsackievirus, and adenoviral infections.Adenovirus causes up to 25% of exudative pharyngitis, often in association with conjunctivitis.Enteroviral infections are notorious for causing pharyngitis, gastrointestinal complaints, andvesicular lesions of the pharynx, often including the palate. Arcanobacterium haemolyticum is abacterial cause of pharyngitis that produces symptoms that are indistinguishable from thosecaused by group A Streptococcus. This pathogen is seen primarily in adolescents.

The viral pharyngitis associated with Epstein-Barr virus infection may be accompanied byprolonged symptoms and risk factors for more serious illness, including hepatosplenomegaly,hepatitis, or significant lymphadenopathy. Abdominal pain may be seen with streptococcaldisease, Epstein-Barr virus infection, and a variety of other illnesses. Other illnesses that mustbe excluded include Kawasaki disease in the child who has fever and oral mucous membranechanges, especially with rash and adenopathy, as well as adenovirus and common rhinovirusesand influenza.

For the girl described in the vignette, the heterophile antibody rapid mononucleosis spot testis likely to be helpful because it is widely available and results can be obtained quickly. If resultsare negative and clinical suspicion for this infection remains high, Epstein-Barr virus titers maybe obtained. Standard adenovirus culture does not yield rapid results, but newer methodologies(shell vial culture) may produce results in approximately 2 days. However, this test may not bewidely available in community settings. Of note, the tonsils may harbor adenovirus for monthsafter an infection, leading to confusion with prior illnesses. A haemolyticum may be culturedwith conventional throat culture, but the laboratory must be informed of suspicion for thisorganism because it may take longer to grow. Although a rapid influenza test may beconsidered, especially during flu season, the absence of cough or coryza in this patient makes itless likely. Complete blood count with differential count and platelet measurement arenonspecific and nondiagnostic tests. Thus, they are not as helpful in determining the cause ofpharyngitis as the mononucleosis spot test.

References:

American Academy of Pediatrics. Enterovirus (nonpoliovirus) infections (group A and BCoxsackieviruses, echoviruses, and numbered enteroviruses). In: Pickering LK, Baker CJ,Kimberlin DW, Long SS, eds. Red Book: 2009 Report of the Committee on Infectious Diseases.

Copyright © 2010 by the American Academy of Pediatrics page 210

Page 211: AAP PREP 2010

2010 PREP SA on CD-ROM

28th ed. Elk Grove Village, Ill: American Academy of Pediatrics; 2009:287-288

American Academy of Pediatrics. Epstein-Barr virus infections (infectious mononucleosis). In:Pickering LK, Baker CJ, Kimberlin DW, Long SS, eds. Red Book: 2009 Report of the Committeeon Infectious Diseases. 28th ed. Elk Grove Village, Ill: American Academy of Pediatrics;2009:289-292

American Academy of Pediatrics. Group A streptococcal infections. In: Pickering LK, Baker CJ,Kimberlin DW, Long SS, eds. Red Book: 2009 Report of the Committee on Infectious Diseases.28th ed. Elk Grove Village, Ill: American Academy of Pediatrics; 2009:616-628

Arnold SR, Straus SE. Interventions to improve antibiotic prescribing practices in ambulatorycare. Cochrane Database Syst Rev. 2005;4: CD003539. Available at:http://www.mrw.interscience.wiley.com/cochrane/clsysrev/articles/CD003539/frame.html

Caddle S Adam HM. In brief: enteroviruses. Pediatr Rev. 2003;24:358-359. Available at:http://pedsinreview.aappublications.org/cgi/content/full/24/10/358

Gross CW, Harrison SE. Tonsils and adenoids. Pediatr Rev. 2000;21:75-78. Available at:http://pedsinreview.aappublications.org/cgi/content/full/21/3/75

Jaggi P, Shulman ST. Group A streptococcal infections. Pediatr Rev. 2006;27:99-105. Availableat: http://pedsinreview.aappublications.org/cgi/content/full/27/3/99

Junker AK. Epstein-Barr virus. Pediatr Rev. 2005;26:79-85. Available at:http://pedsinreview.aappublications.org/cgi/content/full/26/3/79

Langley JM. Adenoviruses. Pediatr Rev. 2005;26:244-249. Available at:http://pedsinreview.aappublications.org/cgi/content/full/26/7/244

Linder JA, Bates DW, Lee GM, Finkelstein JA. Antibiotic treatment of children with sore throat.JAMA. 2005;294:2315-2322. Available at: http://jama.ama-assn.org/cgi/content/full/294/18/2315

Copyright © 2010 by the American Academy of Pediatrics page 211

Page 212: AAP PREP 2010

2010 PREP SA on CD-ROM

Question: 85

You are seeing a 2-week-old girl in your office for a health supervision visit. Her parents reportthat she is eating well and has good weight gain. On physical examination, you note a strongright brachial pulse, but you cannot feel pulses in the right or left femoral region. As you explainthe diagnosis to the parents, they ask you about long-term complications following repair of hercondition.

Of the following, the MOST likely long-term complication for this child is

A. frequent pulmonary infections

B. hypertension

C. neurodevelopmental delay

D. poor exercise performance

E. renal dysfunction

Copyright © 2010 by the American Academy of Pediatrics page 212

Page 213: AAP PREP 2010

2010 PREP SA on CD-ROM

Preferred Response: BCritique: 85

The girl described in the vignette has the classic physical findings of coarctation of theaorta: an easily palpable pulse in the right arm (blood flow origin proximal to the obstruction) andan absent pulse in the lower extremities (blood flow origin distal to the obstruction). Coarctationof the aorta refers to an anatomic obstruction or narrowing in the aorta (Item C85) that can belocalized as a ridge of tissue, formed as a discrete ring of tissue, or collarlike with length forminga segment of aortic hypoplasia. Less-than-normal blood flow through the aortic arch during fetallife may result in hypoplasia of the arch and promote the likelihood of coarctation developing,which forms the basis for the association between aortic stenosis (and other left heartobstructions) and coarctation.

The incidence of coarctation is approximately 1 in 2,300 live births, making it one of the mostcommon types of congenital heart disease encountered by the pediatrician. It occurs withgreater frequency in females who have Turner syndrome (45,X), in whom the incidence may beas high as 15%. Patients who have coarctation have a high incidence of associated congenitalheart disease, the most common of which are a patent ductus arteriosus, bicuspid aortic valve,and mitral valve abnormalities.

Physical examination in the patient who has coarctation usually reveals a discrepant pulsequality between the right radial and the femoral or dorsalis pedis. Patients also may come toattention with hypertension noted on examination. A systolic ejection murmur of low intensity isaudible at the base and axilla and left interscapular region and usually is loudest over the back.Neonates who have significant coarctation may present with signs and symptoms of congestiveheart failure and inadequate perfusion of the gut and lower body. Ultimately, affected patientscan present in cardiogenic shock because the left ventricle is unable to pump against theafterload imposed by the coarctation.

Coarctation that presents in the symptomatic neonate should be repaired surgically.However, even with aggressive and excellent surgical repair, recoarctation can occur as thechild grows. In addition, patients who undergo surgical repair of aortic coarctation have a higherincidence of hypertension at long-term follow-up and should be followed closely for thiscomplication. In contrast to the long-term risk for hypertension, patients who have undergone anuncomplicated neonatal repair of aortic coarctation do not have an increased rate of pulmonaryinfections, neurodevelopmental delay, or poor exercise performance. Although congenital heartdisease can be associated with renal abnormalities in some cases, routine coarctation and itsrepair are not associated with the long-term complication of renal dysfunction.

References:

Hager A, Kanz S, Kaemmerer H, Schreiber C, Hess J. Coarctation Long-term Assessment(COALA): significance of arterial hypertension in a cohort of 404 patients up to 27 years aftersurgical repair of isolated coarctation of the aorta, even in the absence of restenosis andprosthetic material. J Thorac Cardiovasc Surg. 2007;134:738-745.

Toro-Salazar OH, Steinberger J, Thomas W, Rocchini AP, Carpenter B, Moller JH. Long-termfollow-up of patients after coarctation of the aorta repair. Am J Cardiol. 2002;89:541-547.

Copyright © 2010 by the American Academy of Pediatrics page 213

Page 214: AAP PREP 2010

2010 PREP SA on CD-ROM

Abstract available at: http://www.ncbi.nlm.nih.gov/pubmed/11867038

Copyright © 2010 by the American Academy of Pediatrics page 214

Page 215: AAP PREP 2010

2010 PREP SA on CD-ROM

Question: 86

A 6-year-old girl presents to the emergency department with a facial rash, fatigue, and musclepain. She has been previously healthy. On physical examination, you note an erythematous rashover her cheeks (Item Q86A) and eyelids and papules over her knuckles (Item Q86B). Hermuscles are tender to palpation. With encouragement, her grip strength is normal, but herstrength seems diminished on shoulder abduction. She has difficulty standing from a seatedposition.

Of the following, the MOST likely cause for this child's symptoms is

A. dermatomyositis

B. Duchenne muscular dystrophy

C. polymyositis

D. systemic lupus erythematosus

E. viral myositis

Copyright © 2010 by the American Academy of Pediatrics page 215

Page 216: AAP PREP 2010

2010 PREP SA on CD-ROM

Preferred Response: ACritique: 86

The skin and muscle findings described for the child in the vignette are consistent withjuvenile dermatomyositis, an inflammatory myopathy. The initial symptom usually is the rash, butweakness, possibly unrecognized by the parent, may already be present. Periorbital violaceouserythema (heliotrope rash) may appear as a mask over the nasal bridge, cheeks, and ears (ItemC86A). Gottron papules may be observed over the dorsal interphalangeal joints (Item C86B). Aviolaceous discoloration, sometimes with scaling, may appear over the extensor surfaces of theelbows and knees (Item C86C). Weakness is symmetric and proximal, affecting tasks such ashair combing or arising from the floor or chair. Muscles may be tender. As with other musclediseases, cardiac involvement should be suspected and the appropriate assessment undertakenearly. Serum concentrations of muscle-derived enzymes such as creatine kinase may be normalinitially. The initial erythrocyte sedimentation rate also usually is normal, but antinuclear antibodywith a speckled pattern is typical. Lymphopenia and changes in white cell subtypes may indicatedisease activity as well, and fat-suppression magnetic resonance imaging of the muscle ishighly sensitive for disease activity. Treatment with steroids, methotrexate, and otherimmunosuppressive regimens may be needed.

Polymyositis and systemic lupus erythematosus-associated myopathies are far less likelycauses of the girl’s condition because of her dermatologic findings. Polymyositis generally is farless common in children and does not present with this rash. Similarly, although viral myositis ismore common than dermatomyositis, that condition is not characterized by the type of rashdescribed. Duchenne muscular dystrophy is a chronic X-linked disease affecting muscle thatpresents with proximal weakness in boys of this age but does not involve a rash.

References:

Compeyrot-Lacassagne S, Feldman BM. Inflammatory myopathies in children. Pediatr Clin NorthAm. 2005;52:493-520. Abstract available at: http://www.ncbi.nlm.nih.gov/pubmed/15820377

Feldman BM, Rider LG, Reed AM, Pachman LM. Juvenile dermatomyositis and other idiopathicinflammatory myopathies of childhood. Lancet. 2008;371:2201-2212. Abstract available at:http://www.ncbi.nlm.nih.gov/pubmed/18586175

Greenberg SA. Inflammatory myopathies: evaluation and management. Semin Neurol.2008;28:241-249. Abstract available at: http://www.ncbi.nlm.nih.gov/pubmed/18351525

Pachman L. Juvenile dermatomyositis. In: Kliegman RM, Behrman RE, Jenson HB, Stanton BF,eds. Nelson Textbook of Pediatrics. 18th ed. Philadelphia, Pa: Saunders Elsevier; 2007:1019-1023

Copyright © 2010 by the American Academy of Pediatrics page 216

Page 217: AAP PREP 2010

2010 PREP SA on CD-ROM

Question: 87

You care for a 5-year-old girl who recently received a diagnosis of neurofibromatosis type 1(NF1). Her parents tell you that they have read that NF1 is associated with an increased risk forcancers, and they ask you for more information.

Of the following, the MOST accurate statement regarding cancers associated with NF1 is that

A. leukemia is an unlikely cancer type

B. Lisch nodules predispose to tumors of the eye

C. optic glioma most commonly presents at the onset of puberty

D. pheochromocytoma is common in early childhood

E. plexiform neurofibromas may show malignant transformation

Copyright © 2010 by the American Academy of Pediatrics page 217

Page 218: AAP PREP 2010

2010 PREP SA on CD-ROM

Preferred Response: ECritique: 87

Neurofibromatosis type 1 (NF1) is among the most common genetic conditions affectinghumans, occurring in 1 in 3,000 births worldwide, regardless of ethnicity or geographic location.NF1 is caused by mutations in the NF1 gene on chromosome 17; this gene encodes the proteinneurofibromin, which acts as a tumor suppressor by downregulating the ras signal transductionpathway. NF1 is an autosomal dominant condition, and 50% of cases are due to spontaneousgene mutations. Mutations in NF1 result in the production of cells that have only half the normalamount of intracellular neurofibromin. The subsequent somatic mutation of the normal NF1 geneleads to tumor formation in affected tissues.

NF1 is a highly variable condition, even within families. Diagnosis is based on the finding ofat least two features from the list of National Institutes of Health diagnostic criteria for NF1 (ItemC87A). Using these criteria, approximately 95% of affected individuals can be diagnosed by age11 years.

Although most of the tumors associated with NF1 are benign (eg, cutaneous andsubcutaneous neurofibromas), affected individuals are at increased risk for malignancy(approximately 35% to 37% lifetime risk versus approximately 30% risk in the generalpopulation). Plexiform neurofibromas, which occur in about 25% of individuals who have NF1,are a special type of neurofibroma that may exist largely beneath the skin’s surface (Item C87B)or entirely internally. Plexiform neurofibromas undergo malignant transformation toneurofibrosarcomas in 10% to 15% of patients who have them. Signs of malignanttransformation include the sudden onset of rapid tumor growth and pain at the site. Expeditiousmedical evaluation is warranted in such circumstances.

Lisch nodules are hamartomas that resemble freckles and are located in the iris stroma (ItemC87C). They can be appreciated in light-colored eyes with the use of a direct ophthalmoscope; aslitlamp is helpful for detecting them in darkly pigmented irises. Although not pathognomonic forNF1, the presence of two or more is a diagnostic criterion for the condition. They are benign andhave no impact on vision.

Optic glioma occurs in approximately 15% of individuals who have NF1 and typicallydevelops before 6 years of age. Of note, optic glioma may be discovered at an older age uponhead imaging performed for any purpose, but if it is not present by age 6 years, it typically doesnot occur. These benign tumors may cause problems due to their location along the optic nerves(Item C87D).

Myeloproliferative and myelodysplastic leukemias are among the malignancies associatedwith NF1. Individuals who have NF1 have an increased incidence of pheochromocytoma, whichtypically occurs in adulthood. Individuals who have NF1 do not appear to be at increased risk fordeveloping the common cancers affecting the breast, colon, and prostate.

References:

Diagnosis of NF1. Children’s Tumor Foundation. Available at:https://www.ctf.org/neurofibromatosis-type-1/diagnosis-of-nf1.html

Friedman JM. Neurofibromatosis 1. GeneReviews. 2007. Available at:

Copyright © 2010 by the American Academy of Pediatrics page 218

Page 219: AAP PREP 2010

2010 PREP SA on CD-ROM

http://www.ncbi.nlm.nih.gov/bookshelf/br.fcgi?book=gene&part=nf1

Hersh JH and Committee on Genetics. Health supervision for children with neurofibromatosis.Pediatrics. 2008;121:633-642. Available at:http://pediatrics.aappublications.org/cgi/content/full/121/3/633

Viskochil D. Neurofibromatosis type 1. In: Cassidy SB, Allanson JE, eds. Management of GeneticSyndromes. 2nd ed. Hoboken, NJ: Wiley-Liss; 2005:369-384

Copyright © 2010 by the American Academy of Pediatrics page 219

Page 220: AAP PREP 2010

2010 PREP SA on CD-ROM

Question: 88

A 15-year-old girl complains of vaginal pruritus and a discharge that has worsened over thepast 2 weeks. Past medical history reveals a recent urinary tract infection that was treated withan antibiotic. She says she has a monogamous relationship with her boyfriend, so they do notuse condoms, and he has no symptoms. Physical examination reveals normal-appearing externalgenitalia and a discharge visible at her introitus. On speculum examination, she has a frothydischarge in her vagina and a normal-appearing cervix. Results of her bimanual examination arenormal. You obtain a normal saline wet mount of the discharge (Item Q88).

Of the following, the MOST important next step, in addition to prescribing medications, is to

A. discuss treatment for the boyfriend

B. encourage the practice of douching

C. repeat the urine culture

D. notify the public health department

E. obtain pelvic ultrasonography

Copyright © 2010 by the American Academy of Pediatrics page 220

Page 221: AAP PREP 2010

2010 PREP SA on CD-ROM

Preferred Response: ACritique: 88

Vaginitis in the adolescent, unlike the prepubertal child, usually has a specific cause. Avaginal discharge may be the presenting symptom for cervicitis or a vaginitis and usually isrelated to a sexually transmitted infectious agent. The characteristics of the discharge for thepatient described in the vignette and the results of the microscopic examination of vaginalsecretions suggest that she has a Trichomonas vaginalis infection.

A physiologic discharge, which is the result of increasing estrogen concentrations duringpuberty, is whitish and mucoid and occurs without other symptoms. The characteristics of thedischarge change during the menstrual cycle, becoming more copious and watery at mid-cycleand stickier and scantier in the second half of the cycle. T vaginalis causes a white or yellowfrothy discharge and, occasionally, punctuate hemorrhages on the cervix. It may be associatedwith pruritus. Candida vaginitis results in a milky-to-curdy discharge that also is associated withpruritus. Bacterial vaginosis, which may not be sexually transmitted, results in a malodorous,grayish discharge. Rarely, a foreign body, such as a retained tampon, may be the cause of adischarge that is foul-smelling and bloody. Cervicitis results in a visible discharge at the externalcervical os and typically is caused by Neisseria gonorrhoeae, Chlamydia trachomatis, orherpes simplex virus.

Because most of these conditions are sexually transmitted, an important part of thetreatment plan (Item C88) is the need to treat the patient’s partner. Many sexually transmittedinfections, including Chlamydia, gonorrhea, and syphilis, require notification of the local publichealth department when the laboratory results are obtained, but this is not universally so for Tvaginalis. The practice of douching should be discouraged because it alters the vaginal pH andthe normal flora, increasing the risk of acquiring infections. Pelvic ultrasonography may help inthe staging of pelvic inflammatory disease, but it is not otherwise helpful for diagnosing vaginitisor cervicitis. A change in vaginal pH and the presence of clue cells are two of the diagnosticcriteria for bacterial vaginosis. The presence of white blood cells indicates an infection and iscommon with a trichomonal infection; hyphae indicate a candidal infection.

Cystitis is common after the onset of sexual activity. One previous infection, however, doesnot warrant a follow-up culture.

References:

Centers for Disease Control and Prevention, Workowski KA, Berman SM. Sexually transmitteddiseases treatment guidelines, 2006. MMWR Recomm Rep. 2006;55(RR-11):1-94. Available at:http://www.cdc.gov/mmwr/preview/mmwrhtml/rr5511a1.htm

Freeto JP, Jay MS. "What's really going on down there?" A practical approach to the adolescentwho has gynecologic complaints. Pediatr Clin North Am. 2006;53: 529-545. Abstract availableat: http://www.ncbi.nlm.nih.gov/pubmed/16716795

Martino JL, Vermund SH. Vaginal douching: evidence for risks or benefits to women's health.Epidemiol Rev. 2002;24:109-124. Available at:http://epirev.oxfordjournals.org/cgi/content/full/24/2/109

Copyright © 2010 by the American Academy of Pediatrics page 221

Page 222: AAP PREP 2010

2010 PREP SA on CD-ROM

Woods ER, Emans SJ. Vulvovaginal complaints in the adolescent. In: Emans SJH, Laufer MR,Goldstein DP, eds. Pediatric and Adolescent Gynecology. Philadelphia, Pa: Lippincott Williams &Wilkins; 2005:525-564

Copyright © 2010 by the American Academy of Pediatrics page 222

Page 223: AAP PREP 2010

2010 PREP SA on CD-ROM

Question: 89

You are called to the emergency department to evaluate a 1-year-old girl who was rescuedfrom a house fire by paramedics. She was found unconscious at the scene and had sootaround her nares. On arrival to the emergency department, she is able to open her eyes but stillappears sleepy. Her temperature is 37.0°C, heart rate is 150 beats/min, respiratory rate is 30breaths/min, and blood pressure is 90/60 mm Hg. Her oxygen saturation by pulse oximetry is97% on 100% oxygen administered at 8 L/min via a nonrebreathing facemask. The nurse asks ifyou would like to reduce the oxygen because the girl’s oxygen saturation is greater than 95%.You explain to the nurse that the pulse oximetry findings are unlikely to be reliable in this patient.

Of the following, the MOST likely reason for inaccurate pulse oximetry results in this patient is

A. elevated concentration of carboxyhemoglobin

B. elevated concentration of fetal hemoglobin

C. elevated concentration of methemoglobin

D. the inability of pulse oximetry to discriminate values greater than 90%

E. the presence of hypotension and shock

Copyright © 2010 by the American Academy of Pediatrics page 223

Page 224: AAP PREP 2010

2010 PREP SA on CD-ROM

Preferred Response: ACritique: 89

Pulse oximetry has gained widespread acceptance, often is considered the "fifth vital sign,"and has proven extremely useful in recognizing hypoxemia and decreasing critical airwayevents. Pulse oximetry uses red and infrared light to measure the level of oxygenated anddeoxygenated hemoglobin. The amplitudes of the light signals are measured and mathematicallycalculated to express the percentage of oxygen saturation.

Despite its enormous value, pulse oximetry has not been demonstrated to reduce mortalityand has several key limitations. First, it does not measure ventilation and may falsely reassureclinicians about the adequacy of the patient’s overall respiratory status. In addition, pulseoximetry can yield falsely low saturations in critically ill patients who have vasoconstriction, poorperfusion, hypothermia, or arrhythmias. Inaccurate readings also may result from ambient lightinterference or patient movement, inability to discriminate values below 70% accurately,interference by venous pulsations with tricuspid regurgitation, and the presence of abnormalhemoglobin such as carboxyhemoglobin and methemoglobin. Arterial blood gases with co-oximetry measurements should be obtained when the pulse oximetry reading does not correlatewith clinical findings.

The child described in the vignette does not demonstrate any evidence of shock or impairedcardiac performance. She has been rescued from a house fire and, therefore, is at high risk ofcarbon monoxide poisoning. Pulse oximetry cannot distinguish carboxyhemoglobin fromoxyhemoglobin, resulting in a falsely high reading. Methemoglobinemia usually is caused by drugexposure or congenital enzymatic defects. Because methemoglobin is read as bothoxyhemoglobin and deoxyhemoglobin by pulse oximeters, the saturation reading is generallyaround 85%. Fetal hemoglobin has little effect on the accuracy of pulse oximetry. In addition, thechild in the vignette is 1 year of age and would be expected to have a trivial concentration offetal hemoglobin. Finally, pulse oximetry does discriminate values greater than 90% accurately,

but at higher values (>95%), large fluctuations in Pao2 can occur with only small changes inoxygen saturation.

References:

Keidan I, Gravenstein D, Berkenstadt H, Ziv A, Shavit I, Sidi A. Supplemental oxygencompromises the use of pulse oximetry for detection of apnea and hypoventilation duringsedation in simulated pediatric patients. Pediatrics. 2008;122:293-298. Available at:http://pediatrics.aappublications.org/cgi/content/full/122/2/293

Mack E. Focus on diagnosis: co-oximetry. Pediatr Rev. 2007;28:73-74. Available at:http://pedsinreview.aappublications.org/cgi/content/full/28/2/73

Marcum J, Newth CJL. Respiratory monitoring. In: Wheeler DS, Wong HR, Shanley TP, eds.Pediatric Critical Care Medicine: Basic Science and Clinical Evidence. New York, NY:Springer-Verlag London Limited; 2007:376-389

Copyright © 2010 by the American Academy of Pediatrics page 224

Page 225: AAP PREP 2010

2010 PREP SA on CD-ROM

Question: 90

A 14-year-old boy is worried because he is so much smaller than his peers. You review hisprevious growth records (Item Q90).

Of the following, the factor that MOST strongly suggests that the boy has constitutional growthdelay is that

A. he has a normal sense of smell

B. he has Sexual Maturity Rating 3 pubic hair

C. he was small for gestational age

D. his current weight is at the 50th percentile for age

E. his mother reached menarche at 15 years of age

Copyright © 2010 by the American Academy of Pediatrics page 225

Page 226: AAP PREP 2010

2010 PREP SA on CD-ROM

Preferred Response: ECritique: 90

Children who have constitutional delay commonly have a strong family history of such delay,and in some cases, genes associated with hypothalamic hypogonadism have been identified inaffected families. However, in most recent studies, no genes to explain constitutional delay havebeen found, although the pattern of growth is clearly heritable and as common in females as inmales. Therefore, a boy whose mother experienced late menarche might be expected to havedelayed puberty. Such a boy might have continued growth at a prepubertal growth rate or evenrather marked growth attenuation (slowed growth rate before puberty) at the age when otherboys are entering puberty, followed by a catch-up in height after late puberty, with normal lineargrowth sometimes continuing into the third decade of life. When there is growth attenuation, it isimportant to rule out other reasons for the slowing, although in most cases, the boy simply isfollowing a familial pattern.

Absence of olfaction is a component of Kallmann syndrome, one of the hypothalamichypogonadism disorders. However, this is not a common finding in children who have delayedpuberty. The finding of pubic hair does not permit the identification of gonadal pubertal stagebecause premature adrenal puberty (adrenarche or pubarche) is identified clinically by thepresence of pubic hair without testicular enlargement. Testicular size is a better indicator of truepuberty.

If the boy described in the vignette were small for gestational age at birth, this mightcontribute to his short stature as a young teenager. Children who are born small for gestationalage and do not catch up to the 10th percentile in height by the time they are 2 years of age havea United States Food and Drug Administration-approved indication to receive growth hormone.Even if the children have relatively delayed bone ages and ostensibly good adult heightpredictions, if adult height is a concern, growth hormone often is a reasonable treatmentbecause many such children have pubertal courses that involve limited growth spurts that donot permit them to reach the heights initially predicted. A short boy whose weight is at the 50thpercentile is overweight or obese. Such a finding could be associated with an underlyingendocrine disorder such as hypothyroidism or Cushing disease, but does not influence apotential diagnosis of delayed puberty.

References:

Clark PA. Constitutional growth delay. eMedicine Specialties, Pediatrics: General Medicine,Endocrinology. 2007. Available at: http://www.emedicine.com/ped/topic472.htm

Lazar L, Pollak U, Kalter-Leibovici O, Pertzelan A, Phillip M. Pubertal course of persistently shortchildren born small for gestational age (SGA) compared with idiopathic short children bornappropriate for gestational age (AGA). Eur J Endocrinol. 2003;149:425-432. Available at:http://eje-online.org/cgi/reprint/149/5/425

Pallais JC, Caudill M, Pitteloud N, Seminara S, Crowley WF Jr. Kallmann syndrome. GeneReviews.2007. Available at: http://www.ncbi.nlm.nih.gov/bookshelf/br.fcgi?book=gene&part=kms

Copyright © 2010 by the American Academy of Pediatrics page 226

Page 227: AAP PREP 2010

2010 PREP SA on CD-ROM

Sedlmeyer IL, Palmert MR. Delayed puberty: analysis of a large case series from an academicmedical center. J Clin Endocrinol Metab. 2002;87:1613-1620. Available at:http://jcem.endojournals.org/cgi/content/full/87/4/1613

Wehkalampi K, Widén E, Laine T, Palotie A, Dunkel L. Patterns of inheritance of constitutionaldelay of growth and puberty in families of adolescent girls and boys referred to specialistpediatric care. J Clin Endocrinol Metab. 2008;93:723-728. Abstract available at:http://www.ncbi.nlm.nih.gov/pubmed/18160460

Copyright © 2010 by the American Academy of Pediatrics page 227

Page 228: AAP PREP 2010

2010 PREP SA on CD-ROM

Question: 91

A new patient who is 13 years old comes to your office. You note that he has a long, narrowface and enlarged, protruding ears (Item Q91). On physical examination, he has joint laxity andmacroorchidism. He is very active, has difficulty making eye contact, and engages in some handflapping. His mother has brought his previous medical and school records, and you note that hismost recent testing showed a full-scale intelligence quotient of 45. He is currently in a self-contained special education classroom. Family history reveals that the maternal uncle hasintellectual disability. His mother is pregnant and asks about the potential of having anotheraffected child.

Of the following, the MOST appropriate response is that

A. only male children are affected by this disorder

B. prenatal folate will help prevent having another affected child

C. the disorder is inherited in an autosomal recessive pattern

D. the father is a carrier of the disorder

E. there is a 50% chance of passing this mutation to male offspring

Copyright © 2010 by the American Academy of Pediatrics page 228

Page 229: AAP PREP 2010

2010 PREP SA on CD-ROM

Preferred Response: ECritique: 91

The presentation of the boy described in the vignette is characteristic of fragile X syndrome(FXS). FXS is stated to be the most common genetic cause for developmental disability(DD)/intellectual disability (ID) (also known as mental retardation). However, reviews suggestthat only about 2% of individuals who have ID have a mutation with the gene involved with FXS.About 1 in 4,000 males and about 1 in 6,000 to 8,000 females have FXS. It is an X-linked ID inwhich a CGG repeat expansion within the 5' untranslated regions of the fragile X mentalretardation-1 gene (FMR1) has been identified. Individuals who have 55 to 200 repeats arecarriers of the premutation; those who have FXS have more than 200 repeats and completegene methylation (full mutation). This X-linked dominant disorder with reduced penetrance differsfrom X-linked recessive disorders in that females may be affected. An affected father does notpass the affected gene to his son. A mother who is a carrier, as in this vignette, has a 50%chance of passing the gene to her son. Affected females tend to have more varied clinicalpresentations, ranging from the classic presentation described in males to few symptoms.Females tend to have fewer behavioral symptoms.

A family history is essential when evaluating a child who has ID. Such a history includes athree-generation family history, with attention to other family members who have ID,developmental delays, psychiatric disorders, congenital malformations, miscarriages, stillbirths,and early childhood deaths. The family history may help guide the clinician to a diagnosis,particularly when other family members are affected. Such direction is especially true in thecase of a male who has other male relatives affected by DD/ID related through female relativeswho do not have ID. This may be indicative of a specific form of X-linked inheritance (X-linkedrecessive).

Prenatal folate administration decreases chances of having a child who has spina bifida butdoes not affect the chances of giving birth to a child who has FXS.

References:

Centers for Disease Control and Prevention. Video on genetics of fragile X syndrome. Availableat: http://www.cdc.gov/ncbddd/single_gene/video/Fragile%20X-4.html

Moeschler JB, Shevell M and the Committee on Genetics. Clinical genetic evaluation of the childwith mental retardation or developmental delays. Pediatrics. 2006;117:2304-2316. Available at:http://pediatrics.aappublications.org/cgi/content/full/117/6/2304

Seashore MR. Genetic screening and the pediatrician. Pediatr Ann. 2000;29:272-276

Stroop JB. The family history as a screening tool. Pediatr Ann. 2000;29:279-282

The National Fragile X Foundation web site at: http://www.fragilex.org/html/home.shtml

Copyright © 2010 by the American Academy of Pediatrics page 229

Page 230: AAP PREP 2010

2010 PREP SA on CD-ROM

Question: 92

An 18-year-old exchange student from England who is living in a college dormitory is diagnosedwith acute parotitis due to mumps and admitted to the school infirmary. One of the student’sroommates calls you to discuss his concerns regarding his risk from this exposure. When youreview the calling student’s records, you find that he received a single dose of measles, mumps,rubella vaccine at 15 months of age.

Of the following, the MOST appropriate action at this time is to

A. move him into another dormitory room for 18 days from the exposure

B. prescribe a dose of mumps vaccine

C. prescribe gamma globulin intramuscularly

D. reassure him that he is likely immune

E. screen him for mumps antibodies

Copyright © 2010 by the American Academy of Pediatrics page 230

Page 231: AAP PREP 2010

2010 PREP SA on CD-ROM

Preferred Response: BCritique: 92

Mumps is a member of the paramyxovirus family that spreads by contact with infectedrespiratory secretions. Clinically, mumps is a systemic infection characterized by acutedevelopment of unilateral or bilateral parotid gland swelling (Item C92) lasting at least 2 days.Complications include central nervous system infection (manifested as meningitis or encephalitis)and orchitis. Other rare complications of mumps include monoarticular large joint arthritis,pancreatitis, thyroiditis, myocarditis, and oophoritis. An infected individual is contagious from 1 to2 days before the onset of parotid swelling to at least 5 days after the onset of swelling, and theincubation period ranges from 12 to 25 days after exposure. Isolation of an infected individual isrecommended for 9 days after the onset of parotid swelling.

In the prevaccine era (before 1967), the peak incidence of mumps was in 5 to 14 year olds,and the peak occurrence was between January and May. Subsequent to implementation ofwidespread vaccination in the United States, mumps cases decreased 99% to fewer than 300per year until a multistate outbreak occurred from late 2005 into 2006, with 2,597 reported casesfrom 11 states, primarily involving college students. The highest attack rate occurred in 18 to 24year olds. A large number of cases occurred in individuals who reported having received twoprior doses of mumps vaccine. Theories for this spread on college campuses include thedormitory setting, with frequent and extended close contact for transmission; the failure torecognize early cases; the recognition that two doses of vaccine are not 100% effective; andstrain difference between the infecting strain (mumps [G]) and the vaccine strain (mumps [A]).

Based on these observations, the Advisory Committee on Immunization Practices of theCenters for Disease Control and Prevention updated recommendations for mumps vaccination toinclude documentation of two doses of live mumps vaccine for school-age children(kindergarten to 12th grade) and for high-risk adults (eg, health-care workers, internationaltravelers, and students at post-high school institutions). In addition, a second dose of mumpsvaccine should be considered for children 1 to 4 years old and adults who are in proximity to anoutbreak. Immune globulin is not effective for postexposure prophylaxis.

Although a second dose of vaccine may not be effective for the exposure experienced forthe young man described in the vignette, it would provide future protection. Infection can occurdespite prior vaccination, so reassurance that the young man likely is immune and screening himfor mumps antibodies is not appropriate. Because the young man likely already has beenexposed and the index case is out of the area, moving into another dormitory room is unlikely tobe of value.

References:

American Academy of Pediatrics. Mumps. In: Pickering LK, Baker CJ, Kimberlin DW, Long SS,eds. Red Book: 2009 Report of the Committee on Infectious Diseases. 28th ed. Elk GroveVillage, Ill: American Academy of Pediatrics; 2009:468-472

Centers for Disease Control and Prevention (CDC). Notice to readers: updated recommendationsof the Advisory Committee on Immunization Practices (ACIP) for the control and elimination ofmumps. MMWR Morbid Mortal Wkly Rep. 2006;55:629-630. Available at:

Copyright © 2010 by the American Academy of Pediatrics page 231

Page 232: AAP PREP 2010

2010 PREP SA on CD-ROM

http://www.cdc.gov/mmwr/preview/mmwrhtml/mm5522a4.htm

Centers for Disease Control and Prevention (CDC). Update: multistate outbreak of mumps --United States, January 1-May 2, 2006. MMWR Morbid Mortal Wkly Rep. 2006; 55:559-563.Available at: http://www.cdc.gov/mmwr/preview/mmwrhtml/mm5520a4.htm

Dayan GH, Quinlisk P, Parker AA, et al. Recent resurgence of mumps in the United States. NEngl J Med. 2008;385:1580-1589. Abstract available at:http://www.ncbi.nlm.nih.gov/pubmed/18403766

Copyright © 2010 by the American Academy of Pediatrics page 232

Page 233: AAP PREP 2010

2010 PREP SA on CD-ROM

Question: 93

You are evaluating a 6-year-old girl who has acquired immune deficiency syndrome, failure tothrive, and chronic profuse diarrhea. Her diarrhea has worsened progressively over the past 3months. She has had intermittent temperatures to 38.6°C and anorexia. Her family is very poorlycompliant with her medications. Her viral load is greater than 500,000 copies, and her CD4 count

is 20 cells/mm3. On physical examination, the very thin girl, who weighs 12 kg (<5th percentile),has oral thrush and generalized lymphadenopathy.

Of the following, the MOST likely pathogen causing this patient’s illness is

A. Cryptosporidium

B. cytomegalovirus

C. Mycobacterium avium complex

D. Mycobacterium tuberculosis complex

E. Salmonella

Copyright © 2010 by the American Academy of Pediatrics page 233

Page 234: AAP PREP 2010

2010 PREP SA on CD-ROM

Preferred Response: ACritique: 93

Diagnosing infections in immunocompromised hosts presents a special challenge becausesuch individuals have one or more defects in the body’s natural defense mechanisms thatpredispose them to infections caused by organisms that normally do not cause infection in theimmunocompetent host. Certain organisms are more likely to cause infection, depending on thespecific immunocompromising condition that is present (Item C93).

Cryptosporidium, a protozoan parasite, is most likely to account for the symptoms describedfor the patient in the vignette, who has acquired immune deficiency syndrome (AIDS), chronicdiarrhea, and failure to thrive. For patients who have AIDS, infection with cytomegalovirus mostcommonly manifests as retinitis. Mycobacterium tuberculosis infection manifests withinvolvement of the pulmonary, central nervous system, and other organ systems. Infection withMycobacterium avium complex usually manifests as persistent, constant fevers with drenchingnight sweats, malaise, and anorexia. Gastroenteritis caused by Salmonella has an acute onset;chronic infections with this pathogen usually manifest as recurrent bacteremias.

References:

American Academy of Pediatrics. Cryptosporidiosis. In: Pickering LK, Baker CJ, Kimberlin DW,Long SS, eds. Red Book: 2009 Report of the Committee on Infectious Diseases. 28th ed. ElkGrove Village, Ill: American Academy of Pediatrics; 2009:272-273

Tolan RW Jr. Infections in the immunocompromised host. eMedicine Specialties, Pediatrics:General Medicine, Infectious Disease. 2008. Available at:http://www.emedicine.com/ped/topic3096.htm

Copyright © 2010 by the American Academy of Pediatrics page 234

Page 235: AAP PREP 2010

2010 PREP SA on CD-ROM

Question: 94

A 2½-year-old male presents with a 3-day history of progressive eyelid swelling. He had anupper respiratory tract infection approximately 1 to 2 weeks ago. He has no history of pruritusor bee stings. His mother reports a decreased number of wet diapers per day, absence offever, and absence of gross hematuria. On physical examination, his weight is at the 75thpercentile, height is at the 50th percentile, temperature is normal, pulse rate is 96 beats/min,respiratory rate is 28 breaths/min, and blood pressure is 106/62 mm Hg. He exhibits bilateralperiorbital swelling (Item Q94A), mild scrotal edema (Item Q94B), and mild pitting edema over thepretibial region.

Of the following, the MOST likely laboratory finding expected for this child is

A. depressed complement 1 esterase inhibitor value

B. elevated bradykinin value

C. elevated immunoglobulin E value

D. elevated serum creatinine value

E. large protein on urinalysis

Copyright © 2010 by the American Academy of Pediatrics page 235

Page 236: AAP PREP 2010

2010 PREP SA on CD-ROM

Preferred Response: ECritique: 94

The young boy described in the vignette exhibits generalized edema, with some subjectivedecrease in urine output. The differential diagnosis for this clinical scenario includes nephroticsyndrome (NS) and other causes of hypoalbuminemia, such as protein-losing enteropathy, liverfailure, and severe protein malnutrition (kwashiorkor). Among the other causes of generalizededema are conditions of sodium/fluid retention, such as renal failure, acute glomerulonephritis,congestive heart failure, and response to medications (minoxidil). Causes of generalized edemadue to capillary leak syndrome include sepsis or thermal tissue injury from burns. Based on thechild’s clinical presentation, NS is the most likely diagnosis and, therefore, proteinuria is the mostlikely finding.

Childhood NS is defined as the presence of marked proteinuria, hypoalbuminemia, andedema. The signs and symptoms usually are related to edema or ascites. Most affected childrenpresent with periorbital edema, which may be misdiagnosed as being due to environmentalallergies. Less commonly, a child presents with marked abdominal swelling from ascites.

Other presenting symptoms of NS relate to the complications associated with this disorder,many of which are related to other nonalbumin proteins lost in the urine. For example, childrenwho have NS are at higher risk for infection because of urinary losses of immunoglobulins andcomplement factor B (net effect is decreased bacterial opsonization). These factors, combinedwith the presence of ascites (a potential bacterial culture medium), increase the risk ofspontaneous bacterial peritonitis among children who have NS. Accordingly, children presentingwith acute abdominal pain may be misdiagnosed as having appendicitis until the astute clinicianobtains a urinalysis and appreciates the more than 300 mg/dL of protein present on the dipsticktest.

Another complication of NS may be a hypercoagulable state, the pathogenesis of which ismultifactorial. Although NS may be associated with episodes of thromboembolism (usuallyvenous thrombosis), this complication rarely presents at the time of diagnosis. When present,differential limb swelling from a venous thrombosis or gross hematuria associated with renalvein thrombosis can occur.

Angioedema is a capillary leak at the level of the dermis that results in localized edema. It isdistinguished from generalized edema by its asymmetric involvement and its occurrence innondependent areas. Causes of angioedema include medications such as angiotensin-converting enzyme inhibitors, which can increase bradykinin concentrations. Another cause ofangioedema is hereditary angioedema associated with complement C1 esterase inhibitordeficiency. Elevated immunoglobulin E values can be seen with allergies, but not in the clinicalsetting described. Azotemia or increased serum creatinine is not the most likely finding in thisclinical scenario.

References:

Roth KS, Amaker BH, Chan JCM. Nephrotic syndrome: pathogenesis and management. PediatrRev. 2002;23:237-248. Available at:http://pedsinreview.aappublications.org/cgi/content/full/23/7/237

Copyright © 2010 by the American Academy of Pediatrics page 236

Page 237: AAP PREP 2010

2010 PREP SA on CD-ROM

Valentini RP, Smoyer WE. Nephrotic syndrome. In: Kher KK, Schnaper HW, Makker SP, eds.Clinical Pediatric Nephrology. 2nd ed. London, England: Informa Healthcare; 2007:155-194

Valentini RP. Pathophysiology and etiology of edema in children. UpToDate Online 16.3. 2008.Available for subscription at:http://www.utdol.com/online/content/topic.do?topicKey=pedineph/14964&selectedTitle=1~150&source=search_result630

Copyright © 2010 by the American Academy of Pediatrics page 237

Page 238: AAP PREP 2010

2010 PREP SA on CD-ROM

Question: 95

A 5-year-old boy presents with a 2-year history of chronic nasal congestion. His parentsdescribe him as a "constant mouth breather," with nightly snoring and occasional episodes ofstopping breathing. These symptoms have occurred daily without specific seasonal changesand have not improved after a 3-month course of nasal corticosteroids and second-generationoral antihistamine. The child has no ocular symptoms and has little-to-no sneezing or nasalitching. He is otherwise healthy. On physical examination, the child appears comfortable but hasobvious mouth breathing. On evaluation of his head and neck, you note tonsils that extendbeyond the folds of the pharyngeal wall but do not touch. The rest of his evaluation yieldsnormal results.

Of the following, the MOST likely reason for this boy’s symptoms is

A. adenoidal hypertrophy

B. chronic sinusitis

C. nasal polyposis

D. perennial allergic rhinitis

E. seasonal allergic rhinitis

Copyright © 2010 by the American Academy of Pediatrics page 238

Page 239: AAP PREP 2010

2010 PREP SA on CD-ROM

Preferred Response: ACritique: 95

The clinical history and symptoms described for boy in the vignette are suggestive ofadenoidal hypertrophy, the most common upper airway obstructive lesion in children. Adenoidalhypertrophy presents most commonly with nasal obstruction and persistent mouth breathing.Over time, chronic mouth breathing may lead to changes in facial appearance (so-called"adenoidal facies"), obstructive sleep apnea syndrome, snoring, recurrent sinusitis or otitismedia, and hyponasal speech. Oftentimes, the tonsillar tissue is enlarged but may appearappropriate in the presence of adenoidal hypertrophy. An adenoidectomy/tonsillectomy usually isrecommended in the presence of recurrent sinusitis or otitis media or obstructive sleep apneasyndrome.

Allergic rhinitis due to perennial allergens (eg, house dust mite, pet dander, molds) candevelop within the first postnatal year but typically presents with more classic symptoms (ie,sneezing, rhinorrhea, ocular pruritus) and responds to combinations of oral antihistamines andnasal corticosteroids. Seasonal allergic rhinitis (eg, trees, weeds, grasses) usually does notoccur until the fourth or fifth year after birth and also usually responds well to usual allergymedications.

The absence of purulent rhinorrhea, facial-dental pain, postnasal drainage, headache, andcough make chronic sinus disease less likely for this boy. Nasal polyps in children may causenasal obstruction and anosmia, but they generally do not cause snoring or sleep apneasymptoms. Most polyps originate from the ethmoid sinuses and appear as gelatinous, pale,grapelike structures that do not change in size or appearance after administration of a nasaldecongestant. Children who present with nasal polyposis should be evaluated for cysticfibrosis. Less common causes of chronic nasal symptoms include nonallergic rhinitis, vasomotorrhinitis, cerebrospinal fluid rhinorrhea, nasal sarcoidosis, Wegener granulomatosis, septaldeviation, primary ciliary dyskinesia, rebound congestion from chronic nasal decongestant use,and foreign bodies.

References:

Boat TF, Acton JD. Cystic fibrosis. In: Kliegman RM, Behrman RE, Jenson HB, Stanton BF, eds.Nelson Textbook of Pediatrics. 18th ed. Philadelphia, Pa: Saunders Elsevier; 2007:1803-1816

Gigante J. Tonsillectomy and adenoidectomy. Pediatr Rev. 2005;26:199-203. Available at:http://pedsinreview.aappublications.org/cgi/content/full/26/6/199

Haddad J Jr. Nasal polyps. In: Kliegman RM, Behrman RE, Jenson HB, Stanton BF, eds. NelsonTextbook of Pediatrics. 18th ed. Philadelphia, Pa: Saunders Elsevier; 2007:1746-1747

Pappas DE, Hendley JO. Sinusitis. In: Kliegman RM, Behrman RE, Jenson HB, Stanton BF, eds.Nelson Textbook of Pediatrics. 18th ed. Philadelphia, PAa: Saunders Elsevier; 2007:1749-1751

Taylor A, Adam HM. In brief: sinusitis. Pediatr Rev. 2006;27:395-397. Available at:http://pedsinreview.aappublications.org/cgi/content/full/27/10/395

Copyright © 2010 by the American Academy of Pediatrics page 239

Page 240: AAP PREP 2010

2010 PREP SA on CD-ROM

Wetmore R. Tonsils and adenoids. In: Kliegman RM, Behrman RE, Jenson HB, Stanton BF, eds.Nelson Textbook of Pediatrics. 18th ed. Philadelphia, Pa: Saunders Elsevier; 2007:1756-1757

Copyright © 2010 by the American Academy of Pediatrics page 240

Page 241: AAP PREP 2010

2010 PREP SA on CD-ROM

Question: 96

A 17-year-old soccer player comes to your office with an acute shoulder injury. He reports thathe was running down the field, slipped, and fell backward with his left arm stretched out behindhim. He has been unable to move his arm since the injury. He holds the arm slightly abducted andexternally rotated and resists all attempts at movement. Physical examination reveals asymmetryof his shoulders, with increased shoulder slope on the left. You palpate fullness inferior to theleft mid-clavicle and cannot elicit pinprick sensation over the lateral deltoid.

Of the following, the MOST likely diagnosis is

A. anterior humeral dislocation

B. grade 3 acromioclavicular joint injury

C. posterior sternoclavicular dislocation

D. proximal humerus fracture

E. rotator cuff tear

Copyright © 2010 by the American Academy of Pediatrics page 241

Page 242: AAP PREP 2010

2010 PREP SA on CD-ROM

Preferred Response: ACritique: 96

The clinical presentation of the boy in the vignette points to an anterior humeral dislocation(Item C96A) and (Item C96B). This injury typically results from a fall on a posteriorly rotated,abducted arm, often during sports participation. The age distribution has two peaks, one at 18 to30 years and a second in the elderly. In young adults, this injury occurs most commonly in males(9:1). It is exceedingly uncommon in younger children because such a fall is more likely to causea fracture through an open epiphysis rather than ligamentous failure.

Acromioclavicular (AC) joint injury usually is caused by a fall onto the shoulder (Item C96C).Most commonly seen in adolescent and young adult males after a sports injury, the physicalfindings include tenderness over the acromion and prominence of the distal clavicle. A grade 3injury occurs when the AC ligament is disrupted completely.

Posterior sternoclavicular (SC) dislocation is an uncommon injury resulting from significantdirect force to the posterior shoulder. Physical findings include tenderness over the SC joint andworsening pain with lateral compression. Posterior dislocations also may manifest withrespiratory and vascular compromise if the medial clavicle compresses the trachea or greatvessels.

Proximal humeral fractures present with shoulder and upper arm pain without shoulderasymmetry. These injuries result from blunt trauma to the upper arm.

Injuries to the rotator cuff (supraspinatus, infraspinatus, subscapularis, and teres minormuscles and their ligamentous attachments) in adolescents typically are due to overuse duringsports that require overhead motion (eg, baseball, tennis, weightlifting). Ligamentous tears areuncommon in this age group; more commonly, adolescents develop rotator cuff tendonitis. Onphysical examination, affected patients demonstrate shoulder pain upon raising and lowering thearm without obvious shoulder deformity.

References:

Andrews S, Davies M, Donatelli R, Whiteside JA. Acromioclavicular injuries. In: Griffin LY, ed.Essentials of Musculoskeletal Care. 3rd ed. Rosemont, Ill: American Academy of OrthopaedicSurgeons; 2005:157-160

Andrews S, Davies M, Donatelli R, Whiteside JA. Fracture of the proximal humerus. In: GriffinLY, ed. Essentials of Musculoskeletal Care. 3rd ed. Rosemont, Ill: American Academy ofOrthopaedic Surgeons; 2005:177-179

Kroner K, Lind T, Jensen J. The epidemiology of shoulder dislocations. Arch Orthop TraumaSurg. 1989; 108:288-290. Abstract available at: http://www.ncbi.nlm.nih.gov/pubmed/2789505

Malanga GA. Rotator cuff injuries. eMedicine Specialties, Sports Medicine, Shoulder. 2006.Available at: http://www.emedicine.com/sports/topic115.htm

Rudzinski JP, Kunio N. Sternoclavicular joint injury. eMedicine Specialties, Emergency Medicine,Trauma and Orthopedics. 2007. Available at: http://www.emedicine.com/emerg/TOPIC783.HTM

Copyright © 2010 by the American Academy of Pediatrics page 242

Page 243: AAP PREP 2010

2010 PREP SA on CD-ROM

Wilson SR, Price DD. Dislocation, shoulder. eMedicine Specialties, Emergency Medicine,Trauma and Orthopedics. 2008. Available at: http://www.emedicine.com/emerg/topic148.htm

Copyright © 2010 by the American Academy of Pediatrics page 243

Page 244: AAP PREP 2010

2010 PREP SA on CD-ROM

Question: 97

A 12-year-old girl presents with a 2-week history of anorexia, intermittent low-grade fever, andright-sided, postprandial abdominal discomfort, accompanied by a 5-lb weight loss. She has hadintermittent diarrhea over the past 1½ years, during which time she has not experienced anylinear growth. On physical examination, she appears pale and somewhat ill and has an oraltemperature of 37.4°C. Abdominal examination demonstrates moderate distention and right lowerquadrant (RLQ) tenderness, with bowel sounds present. You palpate a poorly defined mass in

the RLQ. Her development is at Sexual Maturity Rating 2. Initial laboratory data include:

•Hemoglobin, 9.8 g/dL (98 g/L)

•White blood cells, 10.0x103/mcL (10.0x109/L)

•Erythrocyte sedimentation rate, 52 mm/hr

•Albumin, 2.8 g/dL (28 g/L)You obtain abdominal computed tomography scan with contrast (Item Q97).

Of the following, the MOST appropriate approach in this case includes

A. a hydrostatic barium enema study

B. intravenous corticosteroids

C. nasoenteric tube decompression

D. referral for immediate surgical exploration

E. strict bowel rest and parenteral nutrition

Copyright © 2010 by the American Academy of Pediatrics page 244

Page 245: AAP PREP 2010

2010 PREP SA on CD-ROM

Preferred Response: BCritique: 97

The presentation described for the young adolescent in the vignette is typical for partialsmall intestine obstruction related to small intestinal involvement with Crohn disease (CD). Thechronic diarrhea, growth arrest, weight loss, and right lower quadrant (RLQ) abdominal pain areseen in 30% of affected children whose disease activity predominates in the distal smallintestine. For patients who have small bowel CD and limited (generally right-sided) or no largebowel disease, symptoms often persist for 1 to 3 years prior to diagnosis. The physical findingsreported for this patient, including anthropometric measurements, suggestion of a mass in theRLQ (in this case, representing thickened loops of involved bowel), laboratory abnormalities(anemia, hypoalbuminemia, elevated erythrocyte sedimentation rate), and radiographic evidenceof marked bowel wall thickening with reduced intraluminal volume, strongly indicate CD (ItemC97).

The most appropriate treatment course for this patient, directed at relieving the partialintestinal obstruction, is to ameliorate intestinal transmural inflammation by instituting therapy withintravenous corticosteroids. Such anti-inflammatory and immunomodulatory treatment has beenshown to be highly effective in rapidly reducing inflammation in CD, especially for patients whohave ileal involvement, thus relieving signs and symptoms of delayed intestinal transit and partialobstruction and inducing clinical remission. After a rapid clinical response has been achieved,long-term pharmacologic therapy can be initiated to maintain remission and prevent clinicalrelapse. When corticosteroid therapy is employed, efforts should be made to taper the doses asearly as possible to limit the occurrence of steroid-related adverse effects.

The goal of therapy in any patient who presents with signs of partial or complete intestinalobstruction must be to achieve bowel decompression and proceed with a management planaimed at reaching a prompt, definitive diagnosis and relieving the obstructing lesion. In this case,the absence of vomiting, despite the presence of abdominal distention, indicates that theobstruction is partial (which is confirmed radiographically) and, therefore, suggests thatnasogastric decompression is not required. Similarly, surgical exploration would not be indicatedas first-line therapy in a patient whose partial small bowel obstruction likely is the consequenceof CD. The patient’s age, history, mode of presentation, and radiographic findings are notconsistent with obstruction caused by an intussusception. Thus, a hydrostatic barium study isnot indicated.

Corticosteroid therapy likely can relieve the clinical signs and symptoms reported for thispatient, at least initially, but other potential causes for partial small bowel obstruction must beconsidered. Computed tomography scan failed to demonstrate the presence of an abdominalmass that would represent either an inflammatory phlegmon or frank abscess in CD. Broad-spectrum antibiotic therapy is indicated for these conditions and usually is administeredconcurrently with anti-inflammatory medication. First-line medical therapy may not elicit asatisfactory response for such patients, and alternative approaches, including the use ofbiologic agents directed against the proinflammatory cytokine tumor necrosis factor-alpha, maybe required. A recent French population-based study indicated that those whose CD manifestsstricturing behavior or requires corticosteroid therapy have a 2.5- to 3-fold greater overall riskfor surgery for resection of the strictured bowel segment.

Although optimizing nutritional status in CD is recognized as a critical adjunct to drug

Copyright © 2010 by the American Academy of Pediatrics page 245

Page 246: AAP PREP 2010

2010 PREP SA on CD-ROM

treatment, the importance of nutrition alone in inducing remission remains controversial. Availableevidence indicates that up to 80% of patients who have mild-to-moderate disease eventuallyachieve clinical remission by using oral or enteral nutrition, with either monomeric or polymericformulas as the sole therapeutic intervention. However, the data regarding nutrition’s primaryrole as an equally effective alternative to corticosteroids are less clear. Two meta-analysesindicated that corticosteroids were superior to nutrition therapy for induction of remission, but amore recent meta-analysis demonstrated equivalency between these treatments. The long-termconsequences of these, as well as the other therapies for CD, await the outcome of furtherstudy.

References:

Boyle JT. Recurrent abdominal pain: an update. Pediatr Rev. 1997;18:310-321. Available at:http://pedsinreview.aappublications.org/cgi/content/full/18/9/310

Griffiths AM, Hugot J-P. Crohn disease. In: Walker WA, Goulet O, Kleinman RE, Sherman PM,Shneider BL, Sanderson IR, eds. Pediatric Gastrointestinal Diseases: Pathophysiology,Diagnosis, Management. 3rd ed. Hamilton, Ontario, Canada: BC Decker; 2004:789-824

Homan M, Baldassano RN, Mamula P. Managing complicated Crohn's disease in children andadolescents. Nat Clin Pract Gastroenterol Hepatol. 2005;2:572-579. Abstract available at:http://www.ncbi.nlm.nih.gov/pubmed/16327836

Hyams JS. Inflammatory bowel disease. Pediatr Rev. 2005;26:314-320. Available at:http://pedsinreview.aappublications.org/cgi/content/full/26/9/314

Miller G, Boman J, Shrier I, Gordon PH. Etiology of small bowel obstruction. Am J Surg.2000;180:33-36. Abstract available at: http://www.ncbi.nlm.nih.gov/pubmed/11036136

Ryckman FC. Approach to the child with gastrointestinal obstruction. In: Rudolph CD, RudolphAM, Hostetter MK, Lister G, Siegel NJ, eds. Rudolph’s Pediatrics. 21st ed. New York, NY:McGraw-Hill Medical Publishing Division; 2003:1376-1379

Vernier-Massouille G, Balde M, Salleron J, et al. Natural history of pediatric Crohn’s disease: apopulation-based cohort study. Gastroenterology. 2008;135:1106-1113. Abstract available at:http://www.ncbi.nlm.nih.gov/pubmed/18692056

Copyright © 2010 by the American Academy of Pediatrics page 246

Page 247: AAP PREP 2010

2010 PREP SA on CD-ROM

Question: 98

You are evaluating a 3-day-old 1,300-g infant who was born at 30 weeks’ gestation. He did nothave early respiratory distress. He has been receiving intravenous nutrition and slowly startedenteral feedings yesterday. Today he has several episodes of apnea lasting 20 to 30 secondsthat are associated with cyanosis (pulse oximetry reading of 75%) and bradycardia (heart rateof 75 beats/min). He has responded to tactile stimulation and blow-by oxygen. Physicalexamination after one such episode reveals good color, perfusion, tone, and reactivity. Thelungs are clear, and the chest wall is stable. There is no heart murmur. The abdomen is soft andnontender, with normal bowel sounds.

Of the following, the MOST likely cause of the apnea episodes in this infant is

A. anemia of prematurity

B. apnea of prematurity

C. hypoglycemia

D. necrotizing enterocolitis

E. sepsis

Copyright © 2010 by the American Academy of Pediatrics page 247

Page 248: AAP PREP 2010

2010 PREP SA on CD-ROM

Preferred Response: BCritique: 98

Central apnea represents a total cessation of respirations and is mediated throughmechanisms that affect the neural integration of sensory input (stretch receptors,chemoreceptors, and baroreceptors) and the central nervous system’s efferent signaling forinspiration. By definition, such cessations of respiration last 20 seconds or longer or if they areless than 20 seconds in duration, are associated with cyanosis or bradycardia. The newborndescribed in the vignette displays central apnea with cyanosis and bradycardia. Apnea ofprematurity, a form of central apnea, is considered a diagnosis of exclusion, and other potentialcontributing factors must be ruled out. It is treated effectively and safely with the methylxanthinecaffeine.

Central apnea is a common respiratory disorder among very low-birthweight preterminfants, typically appearing in the first 2 weeks of postnatal life. It must be evaluated within thecontext of the individual newborn’s medical history and clinical setting. In addition, it must bedistinguished from obstructive apnea, which is often due to neck flexion with positioning;gastroesophageal reflux with reflex glottic closure and cessation of airflow; and mechanical

obstruction of the trachea or larynx. Numerous conditions may lead to central apnea:

-Sepsis, meningitis, or necrotizing enterocolitis

-Prolonged hypoxemia (lung disease, patent ductus arteriosus, profound anemia)-Central nervous system depressant effects of certain medications (opioids, barbiturates,

and benzodiazepines)-Intracranial hemorrhage (including intraventricular, subdural, or subarachnoid

hemorrhage)

-Hydrocephalus (including posthemorrhagic hydrocephalus)

-Electrolyte abnormalities

-Hypoglycemia

-Temperature instability (hypothermia or hyperthermia)

-Prematurity and immature respiratory drive Anemia of prematurity may contribute to apnea when it is profound (hemoglobin

characteristically <7 g/dL [70 g/L]), but this is a late phenomenon (occurring after 2 to 4 weeksof postnatal age). Because the infant is receiving intravenous alimentation and some enteralfeeding, it is unlikely that he is experiencing hypoglycemia. Necrotizing enterocolitis is veryuncommon in the first 3 days after birth, and its association with apnea characteristically is inthe context of a systemic illness, presenting with shock and lethargy. Sepsis may cause apnea,but the newborn described in the vignette does not exhibit any other clinical signs of sepsis.

References:

Martin RJ, Abu-Shaweesh JM, Baird TM. Pathophysiologic mechanisms underlying apnea ofprematurity. NeoReviews. 2002;3:e59-e65. Available for subscription at:

Copyright © 2010 by the American Academy of Pediatrics page 248

Page 249: AAP PREP 2010

2010 PREP SA on CD-ROM

http://neoreviews.aappublications.org/cgi/content/full/3/4/e59

Schmidt B, Roberts RS, Davis P, et al; Caffeine for Apnea of Prematurity Trial Group. Long-termeffects of caffeine therapy for apnea of prematurity. N Engl J Med. 2007;357:1893-1902.Available at: http://content.nejm.org/cgi/content/full/357/19/1893

Schmidt B, Roberts RS, Davis P, et al; Caffeine for Apnea of Prematurity Trial Group. Caffeinetherapy for apnea of prematurity. N Engl J Med. 2006;354:2112-2121. Available at:http://content.nejm.org/cgi/content/full/354/20/2112

Thilo EH, Rosenberg AA. The newborn infant. In: Hay WW Jr, Levin MJ, Sondheimer JM,Deterding RR, eds. CURRENT Diagnosis & Treatment: Pediatrics. 19th ed. New York, NY: TheMcGraw-Hill Companies; 2009:Chapter 1. Available for subscription at:http://www.accessmedicine.com/content.aspx?aID=3396500

Copyright © 2010 by the American Academy of Pediatrics page 249

Page 250: AAP PREP 2010

2010 PREP SA on CD-ROM

Question: 99

A 7-year-old boy is brought to the clinic because of an itchy rash that has been present for 2weeks. He has been healthy except for intermittent asthma, and his mother reports that hefrequently has very dry patches of skin. She is concerned that these may be psoriasis becausethe boy’s father has psoriasis. You obtain a history for possible exposures and find nothingexcept that he plays outside in the yard a lot. He has numerous linear vesicles and blisters onhis arms, with surrounding erythema and mild edema (Item Q99). He has a few similar lesions onhis anterior legs. He scratches the lesions frequently during your examination.

Of the following, the MOST likely diagnosis is

A. allergic contact dermatitis

B. atopic dermatitis

C. dyshidrotic eczema

D. nummular eczema

E. psoriasis

Copyright © 2010 by the American Academy of Pediatrics page 250

Page 251: AAP PREP 2010

2010 PREP SA on CD-ROM

Preferred Response: ACritique: 99

The erythema and lesions distributed in a linear pattern described for the boy in the vignettesuggest allergic contact dermatitis caused by poison ivy. The skin inflammation is in response tothe exposure. Contact dermatitis may be categorized as irritant contact dermatitis, which iscaused by mechanical or chemical irritation of the skin, or allergic contact dermatitis, whichinvolves antigen introduction and recruitment of previously sensitized lymphocytes to the skin.Common manifestations of the irritant form are diaper dermatitis in infants and perioral dermatitiscaused by frequent licking of the lips. The skin appears reddened, dry, and cracked. Allergiccontact dermatitis may be caused by exposures to certain plants such as poison ivy or othercontact allergens such as metals or fragrances. Potent antigens, such as urushiol in poison ivy,produce an acute dermatitis with erythematous papules (Item C99A), vesicles, or blisters. Incontrast, weaker antigens, such as nickel, usually cause dry-appearing papules, patches, orplaques. The location and distribution of lesions assists in the diagnosis of allergic contactdermatitis. In addition to a linear pattern of lesions, other patterns consistent with the distributionof the exposure (eg, belt buckles [Item C99B] or jewelry [Item C99C]) may be seen.

Atopic dermatitis is common among patients who have other atopic conditions, such asasthma. Clinical features include overall skin dryness, lichenification of some discrete areassuch as the wrists and knees (Item C99D), and possible erythema with vesiculation. Linear skinlesions are not typical. Dyshidrotic eczema is a recurrent or chronic vesicular rash that primarilyaffects the palms, lateral fingers (Item C99E), and soles. Nummular eczema is characterized bycoin-shaped scaling or crusted plaques (Item C99F) that may be hyperpigmented; usually lesionsare located on the arms and legs. Psoriasis is a chronic inherited condition. The lesions typicallyare round, erythematous patches or plaques covered by white or grayish scale (Item C99G).

References:

Mark BJ, Slavin RG. Allergic contact dermatitis. Med Clin North Am. 2006;90:169-185. Abstractavailable at: http://www.ncbi.nlm.nih.gov/pubmed/16310529

Weston WL. Contact dermatitis in children. UpToDate Online 16.8. 2008. Available forsubscription at:http://www.utdol.com/online/content/topic.do?topicKey=ped_derm/2846&selectedTitle=1~150&source=search_result630

Copyright © 2010 by the American Academy of Pediatrics page 251

Page 252: AAP PREP 2010

2010 PREP SA on CD-ROM

Question: 100

A young mother brings in her 7-day-old newborn, accompanied by both grandmothers, for theinfant’s first health supervision visit. The baby has a reddish-orange area of what appears to bemercurochrome applied to the umbilicus, which had been treated with triple dye in the nursery.The infant also has a "belly band," a soft cotton wrap, applied around her abdomen. During yourdiscussions with the family, you sense some conflict about the best care for the umbilical cordstump.

Of the following, the MOST appropriate advice for the family is to

A. alternate applying mercurochrome with alcohol

B. apply moist compresses three times daily to lead to faster separation

C. apply povidone-iodine to the stump

D. dry the stump after washing the baby with mild soap and water

E. remove the "belly band" because it is likely to cause harm

Copyright © 2010 by the American Academy of Pediatrics page 252

Page 253: AAP PREP 2010

2010 PREP SA on CD-ROM

Preferred Response: DCritique: 100

Umbilical cord care is rich in both cultural and medical traditions. The variety of historical andgeographic regimens for cord care is staggering. Goals of cord care include maintaininghygiene, avoiding infection, and promoting cord separation.

Regimens for cord care following hospital delivery in developed countries differ from thosein developing countries where clean water and antimicrobial preparations may not be available.

"Dry cord care," consisting of simple washing of the baby with soap and water andthoroughly drying the cord, has been shown to be as good as application of triple dye, topicalantibiotics, povidone-iodine, or drying agents. Mercurochrome is no longer available in the UnitedStates because of concerns about environmental mercury, but it is available in other countries.Applications of folk remedies, including teas and herbal solutions as well as human milk in somecountries, have been shown in limited studies to cause no harm and possibly to preventinfection. In one study, dry cord care, consisting of bathing the umbilical stump and surroundingarea with soap and water and patting it dry, was not associated with any increase in infectionover use of alcohol. There is a trend toward reduced infection rates with topical antibiotics.Antiseptics such as alcohol may delay separation but allay maternal concerns.

In areas where sterile technique is uncommon, the use of delivery packs and hand washingby caregivers is essential to prevent omphalitis. Chlorhexidine may prove useful in preventinginfection in those same areas as a topical antiseptic for umbilical cord stump care.

Cultural practices such as the "belly band" should be approached as with other folkmedicine issues. If it is doing no harm, it may not be worth questioning the family’s practices, andrespecting its use may foster respect from the family.

As a result of reviewing this information, do you intend to make a change in practiceto provide better patient care?Yes No

References:

Anderson JM, Philip AGS. Management of the umbilical cord: care regimens, colonization,infection, and separation. NeoReviews. 2004;5:e155-e163. Available at:http://neoreviews.aappublications.org/cgi/content/full/5/4/e155

Dore S, Buchan D, Coulas S, et al. Alcohol versus natural drying for newborn cord care. JObstet Gynecol Neonatal Nurs. 1998;27:621—627. Abstract available at:http://www.ncbi.nlm.nih.gov/pubmed/9836156

Gladstone IM, Clapper L, Thorp JW, Wright DI. Randomized study of six umbilical cord careregimens comparing length of attachment, microbial control, and satisfaction. Clin Pediatr.1988;27:127-129. Abstract available at: http://www.ncbi.nlm.nih.gov/pubmed/3277760

Janssen PA, Selwood BL, Dobson SR, Peacock D, Thiessen DN. To dye or not to dye: arandomized, clinical trial of a triple dye/alcohol regime versus dry cord care. Pediatrics.

Copyright © 2010 by the American Academy of Pediatrics page 253

Page 254: AAP PREP 2010

2010 PREP SA on CD-ROM

2003;111:15-20. Available at: http://pediatrics.aappublications.org/cgi/content/full/111/1/15

McConnell TP, Lee CW, Couillard M, Sherrill WW. Trends in umbilical cord care: scientificevidence for practice. Newborn and Infant Nursing Reviews. 2004;4:211-222. Abstract availableat: http://www.sciencedirect.com/science?_ob=ArticleURL&_udi=B758X-4F0G4B7-B&_user=10&_coverDate=12%2F31%2F2004&_rdoc=10&_fmt=high&_orig=browse&_srch=doc-info(%23toc%2312938%232004%23999959995%23551221%23FLA%23display%23Volume)&_cdi=12938&_sort=d&_docanchor=&_ct=15&_acct=C000050221&_version=1&_urlVersion=0&_userid=10&md5=e0adc786153a7f87257a316a89bef153

Mullany LC, Darmstadt GL, Khatry SK, et al. Topical applications of chlorhexidine to the umbilicalcord for prevention of omphalitis and neonatal mortality in southern Nepal: a community-based,cluster-randomised trial. Lancet. 2006;367:910—918. Available at:http://www.pubmedcentral.nih.gov/articlerender.fcgi?tool=pubmed&pubmedid=16546539

Mullany LC, Darmstadt GL, Tielsch JM. Role of antimicrobial applications to the umbilical cord inneonates to prevent bacterial colonization and infection: a review of the evidence. Pediatr InfectDis J. 2003;22:996-1002. Abstract available at: http://www.ncbi.nlm.nih.gov/pubmed/14614373

Zupan J, Garner P, Omari AAA. Topical umbilical cord care at birth. Cochrane Database SystRev. 2004;3:CD001057. Available at:http://www.mrw.interscience.wiley.com/cochrane/clsysrev/articles/CD001057/frame.html

Copyright © 2010 by the American Academy of Pediatrics page 254

Page 255: AAP PREP 2010

2010 PREP SA on CD-ROM

Question: 101

You are evaluating a 12-year-old girl as part of a sports screening program at the local school.She tells you that she has trouble keeping up with her friends during gym class and on thesoccer field. On physical examination, she appears well and is in no distress. Her precordialexamination demonstrates a mild lift. The first heart sound is normal, and the second heart soundis prominently split. There is a 3/6 systolic ejection murmur at the upper left sternal border (ItemQ101). Diastole is clear, and her pulses are normal in all extremities.

Of the following, the MOST likely cause of this patient’s signs and symptoms is

A. aortic stenosis

B. atrial septal defect

C. patent ductus arteriosus

D. pulmonary stenosis

E. ventricular septal defect

Copyright © 2010 by the American Academy of Pediatrics page 255

Page 256: AAP PREP 2010

2010 PREP SA on CD-ROM

Preferred Response: BCritique: 101

Recognition of cardiac anomalies in pediatric patients requires a complete history andphysical examination. The timing and severity of the presentation often depends on the severityof the underlying condition, such as the size of a ventricular septal defect, the degree ofsemilunar valve stenosis, or the extent of obstruction to pulmonary blood flow. Many of thecongenital cardiac anomalies lead to turbulent blood flow within the heart or great vessels,which produces a murmur. The loudness, timing, location, radiation, and pitch of the murmur cansuggest the cause of the anomaly.

The girl described in the vignette has the typical findings of an atrial septal defect. When theleft-to-right shunt at the atrial level is significant, the patient may report a history of decreasedexercise tolerance when compared with peers. Such decreased tolerance likely is the result ofthe dilated right ventricle, which receives the normal blood flow returning to the right atrium fromthe systemic veins as well as the abnormal blood flow that results from the left-to-right shunt ofthe atrial septal defect.

The murmur in patients who have atrial septal defects is not from the blood flow across theatrial septum because this flow usually is not turbulent and at low pressure. Rather, the systolicmurmur results from a relative pulmonary stenosis because the left-to-right atrial shunt and thesubsequent increased right ventricular volume are required to cross the pulmonary valve. Thevalve annulus does not dilate and, therefore, the amount of blood (stroke volume) crossing thevalve is increased. The increased flow across the valve per heart beat necessitates anincrease in the velocity of that blood flow, resulting in turbulence. It is this turbulence thatcreates the murmur that is heard best at the upper left sternal border. Because there is nostructural abnormality of the pulmonary valve, no click is appreciated in patients who have atrialseptal defects. In large atrial septal defects, a murmur also may be noted during diastole due tothe increased amount of blood that must cross the tricuspid valve during ventricular filling.Finally, patients who have atrial septal defects often have a fixed and split second heart soundthat most likely results from the relative prolonged time required for the dilated ventricle to emptyits contents during systole. In contrast, in the healthy heart, the second heart sound splitsvariably with respiration. The lack of variation of the split most likely is due to the freecommunication between the two atria, which allows for equalization of the influence ofrespiration on both the right and left ventricle.

The murmur of pulmonary stenosis often is associated with a systolic click that results fromthe abnormal structure and function of the pulmonary valve itself (Item C101A). The normalsplitting of the second heart sound occurs because the volume of right ventricular blood and itsstroke volume generally are normal.

Similarly, aortic stenosis is associated with an ejection click that does not change withposition, and the accompanying murmur is heard best at the upper right sternal boarder, withradiation into the neck. Affected patients usually have a normal second heart sound.

The patent ductus arteriosus typically produces a continuous murmur that is characterizedas having a "machinery" quality (Item C101B) and usually is loudest at the left infraclaviculararea. The murmur is continuous because the flow between the systemic and pulmonarycirculation is constant due to the higher systemic compared with pulmonary vascular resistancethroughout the cardiac cycle and the lack of a valve to separate the two in the structure of the

Copyright © 2010 by the American Academy of Pediatrics page 256

Page 257: AAP PREP 2010

2010 PREP SA on CD-ROM

ductus. The murmur of a ventricular septal defect typically is holosystolic (Item C101C) because the

left-to-right shunt at the ventricular level begins with the onset of systole, even before the aorticand pulmonary valves open. When the ventricular septal defect is small, it produces a high-pitched murmur, heard along the sternal border, and the second heart sound is normal, with nochange in its normal physiologic splitting (Item C101D).

References:

Allen HD, Phillips JR, Chan DP. History and physical examination. In: Allen HD, Driscoll DJ, ShaddyRE, Feltes TF, eds. Moss and Adams’ Heart Disease in Infants, Children, and Adolescents,Including the Fetus and Young Adult. 7th ed. Philadelphia, Pa: Lippincott Williams & Wilkins;2008:58-65

Moller JH. Clinical history and physical examination. In: Moller JH, Hoffman JIE, eds. PediatricCardiovascular Medicine. Philadelphia, Pa: Churchill Livingstone; 2000:97-110

Copyright © 2010 by the American Academy of Pediatrics page 257

Page 258: AAP PREP 2010

2010 PREP SA on CD-ROM

Question: 102

A 17-year-old boy is brought via ambulance to the emergency department due to a generalizedtonic-clonic seizure that is ongoing. No medications have been administered en route, butintravenous access has been obtained. After administration of 2 mg intravenous lorazepam, theseizure stops. According to his father, the boy has epilepsy that is difficult to control, for whichhe takes valproic acid, oxcarbazepine, and levetiracetam.

Of the following, the MOST appropriate next diagnostic test to obtain is

A. brain magnetic resonance imaging

B. electroencephalography

C. measurement of blood anticonvulsant concentrations

D. measurement of serum calcium, magnesium, and phosphorous

E. measurement of serum electrolytes

Copyright © 2010 by the American Academy of Pediatrics page 258

Page 259: AAP PREP 2010

2010 PREP SA on CD-ROM

Preferred Response: CCritique: 102

The adolescent described in the vignette has been diagnosed with epilepsy, which meansthat he has had two or more unprovoked seizures. More than 50% of children who haveepilepsy become seizure-free on a single antiseizure medication, but others continue to haveseizures. Rational polypharmacy using agents that have multiple mechanisms of action may betried but usually does not render such patients seizure-free. This boy is receiving a very broadspectrum of antiseizure medications, which implies that intractable epilepsy is the most likelycause for his current seizure. This should be the focus of management in the emergencydepartment. Determination of antiseizure drug concentrations, in combination with informationabout the timing and amount of the last doses taken, can aid in treatment decisions, even if thehospital laboratory cannot obtain the results rapidly. Also, low drug concentrations may indicatenoncompliance, a common problem in adolescents.

Serum electrolytes, calcium, magnesium, and phosphorous are frequently orderedmeasurements, but results rarely are helpful in the emergency department for the child who hasknown epilepsy and has suffered a brief or prolonged seizure. Such laboratory tests should beobtained in children younger than age 6 months, in the presence of a concerning history such aschronic diarrhea, and in patients for whom there is no known cause for status epilepticus.Similarly, electroencephalography is unlikely to be helpful for a patient already known to haveepilepsy. The purpose of magnetic resonance imaging in this setting is to identify a structuralcause for seizures, but neuroimaging probably has been obtained previously for a child whohas intractable epilepsy, and repeat imaging has little value in the absence of any new history,such as recent trauma.

References:

Johnston MV. Seizures in childhood. In: Kliegman RM, Behrman RE, Jenson HB, Stanton BF, eds.Nelson Textbook of Pediatrics. 18th ed. Philadelphia, Pa: Saunders Elsevier; 2007:2457-2475

Major P, Thiele EA. Seizures in children: laboratory diagnosis and management. Pediatr Rev.2007;28:405-414. Available at: http://pedsinreview.aappublications.org/cgi/content/full/28/11/405

Riviello JJ Jr, Ashwal S, Hirtz D, et al. Practice parameter: diagnostic assessment of the childwith status epilepticus (an evidence-based review): report of the Quality StandardsSubcommittee of the American Academy of Neurology and the Practice Committee of the ChildNeurology Society. Neurology. 2006;67:1542-1550. Available at:http://www.neurology.org/cgi/content/full/67/9/1542

Walker DM, Teach SJ. Update on the acute management of status epilepticus in children. CurrOpin Pediatr. 2006;18:239-244. Abstract available at:http://www.ncbi.nlm.nih.gov/pubmed/16721142

Copyright © 2010 by the American Academy of Pediatrics page 259

Page 260: AAP PREP 2010

2010 PREP SA on CD-ROM

Question: 103

You are asked to evaluate a 2-day-old baby who was born at term and was admitted to theneonatal intensive care unit due to respiratory distress. Anteroposterior chest radiographyreveals multiple butterfly vertebrae (Item Q103). On physical examination, the baby is normallygrown and formed except for hypoplastic thumbs. You request echocardiography, whichreveals a large ventricular septal defect. The baby is feeding and stooling well.

Of the following, the recommendation that is MOST helpful in guiding further management for thisinfant is

A. fluorescence in situ hybridization

B. hematology consultation

C. renal ultrasonography

D. serum calcium quantitation

E. urine organic acid analysis

Copyright © 2010 by the American Academy of Pediatrics page 260

Page 261: AAP PREP 2010

2010 PREP SA on CD-ROM

Preferred Response: CCritique: 103

The newborn described in the vignette has three malformations that can be associated witha number of diagnoses, including 22q11 deletion syndrome, VATER association, andchromosome abnormalities. Any young infant who has two or more major malformations (thosethat have significant meaning for health) or three or more minor malformations (features thatoccur in 4% or fewer of the normal population, such as epicanthal folds and in-curving of thefifth finger) should undergo evaluations to detect the presence of further organ anomalies. Forthis infant, renal ultrasonography is most important in guiding further medical management.

VATER [Vertebral anomalies (Item C103A), Anal atresia, TracheoEsophageal fistula,Renal/Radial defects] or VACTERL (Vertebral anomalies, Anal atresia, Cardiac defects,TracheoEsophageal fistula, Renal anomalies, Limb [radial] anomalies) association refers to aclustering of defects that occurs more often than expected by chance. VATER is not a"syndrome" because it is has multiple potential causes. Because a variety of renal anomalies(agenesis, dysplasia, and hypoplasia) are observed in VATER association, the kidneys must beevaluated in any infant for whom this is a diagnostic consideration, including the infant in thevignette. It is important to note that VATER association is a diagnosis of exclusion; conditionsthat have overlapping features should be ruled out whenever possible.

Fluorescence in situ hybridization (FISH) uses fluorescently labeled DNA probes to detectspecific regions throughout the genome. This testing can be performed on dividing or interphase(resting) cells and has the advantage of a faster turnaround time than routine chromosomeanalysis (when used for the purpose of detecting aneuploidies, for example) and being able todetect submicroscopic chromosomal deletions and duplications. A specific test must berequested when ordering FISH, such as FISH for 22q11 deletion (Item C103B) or FISH for Prader-Willi syndrome. Because FISH does not detect chromosomal rearrangements and many otherchromosome anomalies, chromosome analysis also is performed routinely in most cases.Although a specific FISH test could help make a diagnosis for the baby in the vignette, it is notthe recommended next step in guiding management.

Infants who have hypoplastic thumbs can have blood dyscrasias, as seen inthrombocytopenia absent radius (TAR) and Rothmund Thomson syndromes. However, prior toconsulting a hematologist, the presence of a blood disorder should be determined.

Serum calcium concentrations may be abnormally low in infants who have 22q11 deletionsyndrome, but they are normal in most syndromes involving multiple congenital anomalies. Urineorganic acid analysis is not helpful in an infant who has multiple congenital anomalies, isotherwise stable, and has normal routine laboratory values.

References:

Hall BD. VATER association. In: Cassidy SB, Allanson JE, eds. Management of GeneticSyndromes. 2nd ed. Hoboken, NJ: Wiley-Liss; 2005:607-613

Jones KL. VATERR association. In: Smith’s Recognizable Patterns of Human Malformation. 6thed. Philadelphia, Pa: Elsevier Saunders; 2006:756-759

Copyright © 2010 by the American Academy of Pediatrics page 261

Page 262: AAP PREP 2010

2010 PREP SA on CD-ROM

Källén K, Mastroiacovo P, Castilla EE, Robert E, Källén B. VATER non-random association ofcongenital malformations: study based on data from four malformation registers. Am J MedGenet. 2001;101:26-32. Abstract available at:http://www.ncbi.nlm.nih.gov/pubmed/11343333?ordinalpos=&itool=EntrezSystem2.PEntrez.Pubmed.Pubmed_ResultsPanel.SmartSearch&log$=citationsensor

Nussbaum, RL, McInnes, RR, Willard HF. Introduction to cytogenetics. In: Thompson andThompson Genetics in Medicine. 7th ed. Philadelphia, Pa: Saunders Elsevier; 2007:59-64

Shprintzen RJ. Velo-cardio-facial syndrome. In: Cassidy SB, Allanson JE, eds. Management ofGenetic Syndromes. 2nd ed. Hoboken, NJ: Wiley-Liss; 2005:615-631

Copyright © 2010 by the American Academy of Pediatrics page 262

Page 263: AAP PREP 2010

2010 PREP SA on CD-ROM

Question: 104

While interviewing a 17-year-old boy, you learn that he is having trouble getting up in the morningon school days, is eating too much, and has gained a lot of weight. Since he broke up with hisgirlfriend a few months ago, he has been less interested in his usual activities, is less focused,and appears fatigued. His parents report that he has been moody and irritable lately, and theyare afraid to talk to him.

Of the following, the MOST important next step is to

A. have his parents present for any further discussion

B. limit further discussion to safe sex education

C. order a urine drug screen

D. reassure parents that this is normal adolescent behavior

E. screen the boy for suicidal ideation

Copyright © 2010 by the American Academy of Pediatrics page 263

Page 264: AAP PREP 2010

2010 PREP SA on CD-ROM

Preferred Response: ECritique: 104

Adolescents who are depressed can present with symptoms similar to those of depressedadults, such as appearing sad and tearful. However, young people may exhibit irritability ratherthan a depressed mood; somatic symptoms such as chest and abdominal pain; or behavioralproblems such as truancy, substance use, or self-destructive actions. The boy described in thevignette has many features of depression, including fatigue, irritability, and overeating withweight gain. Accordingly, he requires evaluation for suicidal ideation.

Suicide is the third leading cause of death for 15- to 19-year-olds, after accidents andhomicide. Many unsuccessful suicide attempts allow possible rescue and often represent adesperate cry for help. It is imperative that, when appropriate, pediatricians screen for suicidalideation and other risk factors. This ensures that any adolescent who has such thoughtsunderstands that he or she is being heard and, therefore, may not feel the need to progress tomaking a lethal attempt. The best approach to screening for suicidal ideation is throughnonjudgmental questions. No data indicate that asking about suicide precipitates such behavior.

The adolescent is more likely to withhold information in the presence of the parents and,therefore, should be interviewed alone. Although intoxication may lower the threshold for asuicide attempt, drug screens should not be ordered without adolescents’ knowledge unlessthey are mentally impaired and cannot provide informed consent. Violating trust increases thedifficulty of engaging the patient in a therapeutic alliance. Sexual orientation issues, sexualabuse, or acquiring a sexually transmitted infection may cause emotional distress, but limiting thediscussion to sexual issues and not asking about suicidal ideation may lead at-risk adolescentsto feel hopeless and unheard, increasing their risk for a suicide attempt. The combination ofsymptoms and their persistence over 3 months for this boy makes it less likely that this is "normalbehavior."

References:

American Academy of Child and Adolescent Psychiatry. Practice parameter for the assessmentand treatment of children and adolescents with suicidal behavior. J Am Acad Child AdolescPsychiatry. 2001;40(7 suppl):24S-51S. Abstract available at:http://www.ncbi.nlm.nih.gov/pubmed/11434483

Shain BN and the Committee on Adolescence. Clinical report: suicide and suicide attempts inadolescents. Pediatrics. 2007;120:669-676. Available at:http://pediatrics.aappublications.org/cgi/content/full/120/3/669

Wintersteen MB, Diamond GS, Fein JA. Screening for suicide risk in the pediatric emergency andacute care setting. Curr Opin Pediatr. 2007;19:398-404. Abstract available at:http://www.ncbi.nlm.nih.gov/pubmed/17630602

Copyright © 2010 by the American Academy of Pediatrics page 264

Page 265: AAP PREP 2010

2010 PREP SA on CD-ROM

Question: 105

You are caring for a 2-year-old girl who has cardiomyopathy and is awaiting cardiactransplantation. She is receiving a continuous infusion of milrinone at 0.5 mcg/kg per minute,intravenous furosemide three times a day, and 2 L/min of oxygen administered via nasalcannula. On physical examination, her temperature is 39.0°C, heart rate is 130 beats/min,respiratory rate is 30 breaths/min, and blood pressure is 80/40 mm Hg. Her oxygen saturation is

92%. An arterial blood gas shows a pH of 7.35, Paco2 of 40 mm Hg, and Pao2 of 50 mm Hg,with a hemoglobin of 8 g/dL (80 g/L).

Of the following, the treatment that can BEST increase her tissue oxygen delivery is

A. administration of 10 mg/kg acetaminophen

B. increased furosemide administration to four times a day

C. increased oxygen flow to achieve an oxygen saturation of 95%

D. reduction of the milrinone infusion to 0.25 mcg/kg per minute

E. transfusion with 15 mL/kg packed red blood cells

Copyright © 2010 by the American Academy of Pediatrics page 265

Page 266: AAP PREP 2010

2010 PREP SA on CD-ROM

Preferred Response: ECritique: 105

Adequate tissue oxygenation requires a complex interaction between the pulmonary,cardiovascular, and hematologic systems and can be disturbed by a variety of diseaseprocesses. Continual supply of oxygen is imperative because cells lack the ability to storeoxygen. Imbalances between tissue demand and supply result in cell injury and death.

The amount of oxygen transport in the blood is dependent on three factors: hemoglobinconcentration, cardiac output, and the amount of hemoglobin that is saturated with oxygen.Although a small amount of oxygen is dissolved in blood, most is carried by hemoglobin, as

expressed in the equation for arterial oxygen content (CaO2):

Cao2 (g O2/mL)= (hemoglobin x 1.34 x Sao2) + (0.003 x Pao2)

The tissue delivery of oxygen (Do2) is calculated using the cardiac output (CO) and the

Cao2 as follows:

Do2 = CO x Cao2Oxygen delivery for the child described in the vignette can be improved most appropriately

by transfusion of packed red blood cells and correction of her anemia. Other interventions maybe beneficial but would not have the impact of transfusion. Reducing her fever withacetaminophen would decrease tissue oxygen demand, increasing the diuretic dose mightincrease cardiac output if her vascular volume is elevated, and increasing oxygen flow wouldincrease the arterial oxygen content. A reduction in the milrinone dose would decrease cardiacoutput and reduce oxygen delivery.

References:

Brierly J, Carcillo JA, Choong J, et al. Clinical practice parameters for hemodynamic support ofpediatric and neonatal septic shock: 2007 update from the American College of Critical CareMedicine. Crit Care Med. 2009;37:666-688

Carcillo JA, Wheeler DS, Kooy NW, Shanley TP. Shock: an overview. In: Wheeler DS, Wong HR,Shanley TP, eds. Pediatric Critical Care Medicine: Basic Science and Clinical Evidence. NewYork, NY: Springer-Verlag London Limited; 2007:274-298

McKiernan MA, Lieberman SA. Circulatory shock in children: an overview. Pediatr Rev.2005;26:451-460. Available at: http://pedsinreview.aappublications.org/cgi/content/full/26/12/451

Copyright © 2010 by the American Academy of Pediatrics page 266

Page 267: AAP PREP 2010

2010 PREP SA on CD-ROM

Question: 106

A mother brings in her 7-year-old daughter because she is worried that the little girl will gothrough puberty too early. The woman tells you that she reached menarche at 9 years of age,and this was a difficult experience. The child’s father, on the other hand, had his growth spurt atthe end of high school.

Of the following, this girl is MOST likely to have early menarche if the physical examinationreveals

A. a body mass index greater than the 85th percentile

B. adult body odor

C. breast tissue

D. facial acne

E. pubic hair

Copyright © 2010 by the American Academy of Pediatrics page 267

Page 268: AAP PREP 2010

2010 PREP SA on CD-ROM

Preferred Response: CCritique: 106

Age at puberty has a heritable component. In some families, the inheritance may beautosomal dominant; in others, it seems polygenic. A 7-year-old girl whose mother reachedmenarche at an early age and whose father was delayed in puberty, as described in thevignette, could have either early or late puberty. However, early menarche at, for example, 9years of age, would be associated with some signs of breast development (thelarche) by 7years of age.

Higher body mass index is associated with early puberty in girls, but not in boys. Adult bodyodor, pubic hair, and acne are all signs of adrenal puberty (adrenarche or pubarche). Thisoccurs more or less independently of gonadal puberty, which, in girls, is identified clinically bythe beginning of breast budding (thelarche).

References:

de Vries L, Kauschansky A, Shohat M, Phillip M. Familial central precocious puberty suggestsautosomal dominant inheritance. J Clin Endocrinol Metab. 2004;89:1794-1800. Available at:http://jcem.endojournals.org/cgi/content/full/89/4/1794

Ibánez L, Jiménez R, de Zegher F. Early puberty-menarche after precocious pubarche: relationto prenatal growth. Pediatrics. 2006;117:117-121. Available at:http://pediatrics.aappublications.org/cgi/content/full/117/1/117

Kaplowitz PB. Link between body fat and the timing of puberty. Pediatrics. 2008;121:S208-S217. Available at: http://pediatrics.aappublications.org/cgi/content/full/121/Supplement_3/S208

Kaplowitz PB. Precocious puberty. eMedicine Specialties, Pediatrics: General Medicine,Endocrinology. 2007. Available at: http://www.emedicine.com/ped/topic1882.htm

Papadimitriou A, Pantsiotou S, Douros K, Papadimitriou DT, Nicolaidou P, Fretzayas A. Timing ofpubertal onset in girls: evidence for non-Gaussian distribution. J Clin Endocrinol Metab.2008;93:4422-4425. Abstract available at: http://www.ncbi.nlm.nih.gov/pubmed/18728173

Copyright © 2010 by the American Academy of Pediatrics page 268

Page 269: AAP PREP 2010

2010 PREP SA on CD-ROM

Question: 107

During the health supervision visit for a 4-year-old girl, her father reports that she has developeda stutter over the past 9 months. He explains that she is a little frustrated by the difficulty inexpressing herself but otherwise seems happy and well-adjusted. In talking with the father, youalso note that he has a mild stutter. He speaks to the child slowly and deliberately andencourages her to take her time when speaking to you.

Of the following, the risk factor that MOST strongly suggests the need for speech therapy forthis girl is the

A. age of onset

B. child’s reaction to stuttering

C. child’s sex

D. father’s stutter

E. time since onset

Copyright © 2010 by the American Academy of Pediatrics page 269

Page 270: AAP PREP 2010

2010 PREP SA on CD-ROM

Preferred Response: DCritique: 107

The onset of stuttering typically occurs during the period of intense speech and languagedevelopment as the child progresses from two-word utterances to the use of complexsentences, generally between the ages of 2 and 5 years but sometimes as early as 18 months.About 5% of all children go through a period of stuttering that lasts 6 months or more. The sexratio for stuttering appears to be equal at the onset of the disorder, but studies indicate thatthree to four times as many boys continue to stutter. Between 75% and 80% of children whostart to stutter stop within 12 to 24 months without speech therapy. Children who beginstuttering before the age of 3.5 years and girls are more likely to outgrow stuttering. Strongevidence shows that almost all children who stutter have a family member who stutters.

Studies have shown that individuals who stutter have a large degree of within-worddysfluencies compared with their typical peers. Examples are: 1) repetition of individual soundsor syllables such as "W-w-w-what is he doing?", 2) prolongation of words such as"Wwwwwwhat is he doing?, and 3) blocks or pauses such as ". . . What is he doing?" Suchdysfluencies may be associated with eyelid closing and physical tension around the lips. Thestandard criteria for diagnosing a child who has stuttering or is at risk to develop stuttering is anaverage of at least three within-word dysfluencies in 100 words of conversation.

Although the girl described in the vignette does not have a strong negative reaction to herspeech impediment, her father stutters, placing her at risk for problematic stuttering andindicating the need for speech therapy. Criteria for referring children who stutter for speechtherapy are presented in Item C107. Referral should be made for children who have severestuttering problems, those who have mild stuttering problems that do not improve markedly within6 to 8 weeks, or those whose parents are very concerned. Referral also is indicated forchildren who have associated behaviors (eg, head, body, or limb movement; audible breathingprior to the disfluencies; and observable muscle tension in the orofacial region). In time, suchchildren may develop anxiety about stuttering that could contribute to the chronicity of thedisorder.

References:

American Speech—Language-Hearing Association Web site. Available at:http://www.asha.org/default.htm

Feldman HM. Evaluation and management of language and speech disorders in preschoolchildren. Pediatr Rev. 2005;26:131-142. Available at:http://pedsinreview.aappublications.org/cgi/content/full/26/4/131

Guitar B, Conture EG. The Child Who Stutters: To the Pediatrician. 4th ed. Memphis, Tenn:Stuttering Foundation of America; 2007. Available at:http://www.stutteringhelp.org/Default.aspx?tabid=12

Zebrowski PM. Developmental stuttering. Pediatr Ann. 2003;32:453-458. Abstract available at:http://www.ncbi.nlm.nih.gov/pubmed/12891762

Copyright © 2010 by the American Academy of Pediatrics page 270

Page 271: AAP PREP 2010

2010 PREP SA on CD-ROM

Question: 108

A 14-year-old girl presents to your emergency department for evaluation of a 3-week history ofprogressive episodes of coughing spasms. She reports several episodes of posttussivevomiting and difficulty sleeping at night. She denies night sweats or weight loss and says shewas previously well. She does not take any medications.

Of the following, the MOST important additional information to obtain is a history of

A. gastroesophageal reflux disease

B. international travel over the past year

C. pet or animal exposures

D. spelunking trips in the last 6 months

E. vaccinations received since 11 years of age

Copyright © 2010 by the American Academy of Pediatrics page 271

Page 272: AAP PREP 2010

2010 PREP SA on CD-ROM

Preferred Response: ECritique: 108

The progressive paroxysmal cough progressing over 3 weeks accompanied by posttussivevomiting reported by the girl in the vignette is typical for adolescent pertussis. The duration ofdisease is 6 to 10 weeks, and complications in adolescents and adults may include sleepdisturbances, secondary pneumonia, and vomiting. Immunity to pertussis wanes 6 or more yearsafter vaccination, which makes adolescents and adults susceptible after completing therecommended childhood pertussis vaccine series at 4 to 6 years of age. In recognition of theincreased occurrence of pertussis in this age range and with the demonstrated safety andefficacy of attenuated acellular pertussis booster vaccines (ie, tetanus toxoid, reduceddiphtheria toxoid, acellular pertussis [Tdap]), the Advisory Committee on Immunization Practicesof the Centers for Disease Control and Prevention recommended in 2005 that a single dose ofTdap be administered at the routine 11- to 12-year-old health supervision visit. A single dose ofTdap also should be administered more than 5 years after the last tetanus toxoid, reduceddiphtheria toxoid (Td) dose for individuals up to 64 years of age. In high-risk situations, the dosecan be administered as soon as 2 years after the last Td vaccination. Accordingly, determiningwhether the girl in the vignette has received a dose of Tdap since 11 years of age would helpconfirm the suspicion of pertussis.

The most common signs and symptoms of gastroesophageal reflux are heartburn,regurgitation, and dysphagia, not primarily coughing spasms. A history of international travelmight help support concerns of exposure to tuberculosis. In addition, such travel might increasethe individual’s exposure to other respiratory agents such as influenza (based on seasonalityand hemisphere) and other vaccine-preventable diseases such as pertussis, but supporting thediagnosis of pertussis would be better aided by knowing the history of Tdap immunization. Anumber of respiratory infections can be transmitted to people from pets or animals (eg, Q fever,Bordetella bronchiseptica), but these infections are much less common than pertussis and arenot as consistent with the clinical illness described. Spelunking (or caving) in areas of theeastern and central United States may increase the risk for exposure to histoplasmosis from birdor bat droppings in the caves. Clinically, histoplasmosis in the healthy host usually isasymptomatic. Acute pulmonary histoplasmosis presents with more of an influenza-like illnessthat resolves over 2 days to 2 weeks.

References:

American Academy of Pediatrics. Histoplasmosis. In: Pickering LK, Baker CJ, Kimberlin DW, LongSS, eds. Red Book: 2009 Report of the Committee on Infectious Diseases. 28th ed. Elk GroveVillage, Ill: American Academy of Pediatrics; 2009:373-375

American Academy of Pediatrics. Pertussis (whooping cough). In: Pickering LK, Baker CJ,Kimberlin DW, Long SS, eds. Red Book: 2009 Report of the Committee on Infectious Diseases.28th ed. Elk Grove Village, Ill: American Academy of Pediatrics; 2009:504-519

Kretsinger K, Broder KR, Cortese MM, et al; Centers for Disease Control and Prevention;Advisory Committee on Immunization Practices; Healthcare Infection Control Practices Advisory

Copyright © 2010 by the American Academy of Pediatrics page 272

Page 273: AAP PREP 2010

2010 PREP SA on CD-ROM

Committee. Preventing tetanus, diphtheria, and pertussis among adults: use of tetanus toxoid,reduced diphtheria toxoid and acellular pertussis vaccine: recommendations of the AdvisoryCommittee on Immunization Practices (ACIP) and recommendation of ACIP, supported by theHealthcare Infection Control Practices Advisory Committee (HICPAC), for use of Tdap amonghealth-care personnel. MMWR Recomm Rep. 2006;55(RR-17):1-37. Available at:http://www.cdc.gov/mmwr/preview/mmwrhtml/rr5517a1.htm

Lee GM, Lett S, Schauer S, et al and the Massachusetts Pertussis Study Group. Societal costsand morbidity of pertussis in adolescents and adults. Clin Infect Dis. 2004;39:1572-1580.Available at: http://www.journals.uchicago.edu/doi/full/10.1086/425006

Copyright © 2010 by the American Academy of Pediatrics page 273

Page 274: AAP PREP 2010

2010 PREP SA on CD-ROM

Question: 109

A 4-year-old boy who recently was diagnosed with acute lymphoblastic leukemia hasexperienced the sudden onset of a temperature to 40.0°C, abdominal pain, vomiting, left flank

pain, and dysuria. Laboratory tests show a peripheral white blood cell count of 3.0x103/mcL

(3.0x109/L), with 60% neutrophils, 5% band forms, and 35% lymphocytes. Microscopicexamination of the urine shows 20 to 40 red blood cells/high-power field and 100 to 200 whiteblood cells/high-power field. A blood culture is positive for Enterococcus gallinarium.

Of the following, the MOST effective antimicrobial agent for the treatment of this infection is

A. ampicillin

B. cefotaxime

C. ciprofloxacin

D. clindamycin

E. vancomycin

Copyright © 2010 by the American Academy of Pediatrics page 274

Page 275: AAP PREP 2010

2010 PREP SA on CD-ROM

Preferred Response: ACritique: 109

Ampicillin is the most active beta-lactam antimicrobial agent against most enterococcalisolates and is the drug of choice for the treatment of infections due to Enterococcus gallinarumand E casseliflavus. These two serotypes of enterococci are unusual in that they aresusceptible to ampicillin but are resistant to vancomycin. For infections caused by E faecalis, Efaecium, and other enterococcal species, vancomycin is an alternative agent to ampicillin,especially for patients who are allergic to penicillin and for organisms that exhibit high-levelpenicillin resistance.

Imipenem, a carbapenem antibiotic; piperacillin/tazobactam, an extended-spectrumpenicillin/inhibitor combination antibiotic; and moxifloxacin, a fluoroquinolone agent, are effectiveagainst both E faecalis and E faecium. Meropenem, another carbapenem antibiotic, has activityagainst E faecalis but less activity against E faecium. Finally, levofloxacin, a fluoroquinolone,has activity against E faecalis but none against E faecium.

Vancomycin resistance among E faecium and E faecalis is a major problem in the hospitalsetting. Five major phenotypes of vancomycin-resistant enterococci (VRE) have been described(VanA, VanB, VanC, VanD, and VanE). Options for the treatment of patients who have life-threatening VRE infections include quinupristin-dalfopristin and linezolid. Quinupristin-dalfopristinis a combination streptogramin antibiotic that is active against most E faecium strains but none ofthe E faecalis isolates. Linezolid is an oxazolidinone antibiotic that is effective against manyisolates of vancomycin-resistant E faecium and E faecalis.

Enterococcal species are considered resistant to the cephalosporins, aminoglycosides,ciprofloxacin, trimethoprim-sulfamethoxazole, and clindamycin.

References:

American Academy of Pediatrics. Non-group A or B streptococcal and enterococcal infections.In: Pickering LK, Baker CJ, Kimberlin DW, Long SS, eds. Red Book: 2009 Report of theCommittee on Infectious Diseases. 28th ed. Elk Grove Village, Ill: American Academy ofPediatrics; 2009:634-636

Moellering RC Jr. Enterococcus species, Streptococcus bovis, and Leuconostoc species. In:Mandell GL, Bennett JE, Dolan R, eds. Mandell, Douglas, and Bennett’s Principles and Practiceof Infectious Diseases. 6th ed. Philadelphia, Pa: Elsevier Churchill Livingstone; 2005:2411-2422

Skae CC, Adam HM. In brief: the misuse of vancomycin. Pediatr Rev. 2003;24:249-250. Availableat: http://pedsinreview.aappublications.org/cgi/content/full/24/7/249

Copyright © 2010 by the American Academy of Pediatrics page 275

Page 276: AAP PREP 2010

2010 PREP SA on CD-ROM

Question: 110

A 3-year-old girl presents with a 2-day history of vomiting and diarrhea. Her mother explains thatalthough the girl has difficulty keeping down fluids, she has managed to take sips of water andeat popsicles. On physical examination, her temperature is 37.0°C, heart rate is 140 beats/min,respiratory rate is 14 breaths/min, blood pressure is 106/60 mm Hg, and weight is 15 kg. She

has dry mucous membranes and a capillary refill of 3 seconds. Laboratory evaluation reveals:

•Sodium, 131 mEq/L (131 mmol/L)

•Potassium, 3.5 mEq/L (3.5 mmol/L)

•Chloride, 94 mEq/L (94 mmol/L)

•Bicarbonate, 16 mEq/L (16 mmol/L)

•Glucose, 70.0 mg/dL (3.9 mmol/L)

•Blood urea nitrogen, 44.0 mg/dL (15.7 mmol/L)

•Creatinine, 1.1 mg/dL (97.2 mcmol/L)Urinalysis reveals a specific gravity of 1.030, pH of 5.5, 3+ ketones, and no blood or protein.

Of the following, the MOST appropriate intravenous fluid order for immediate management is

A. 5% dextrose at 10 mL/hour + urine output replacement

B. 5% dextrose + 0.225% sodium chloride at 50 mL/hr

C. 5% dextrose + 0.33% sodium chloride + 20 mEq/kg potassium chloride at 50 mL/hr

D. 0.225% sodium chloride at a volume of 300 mL over 1 hour

E. 0.9% sodium chloride at a volume of 300 mL over 1 hour

Copyright © 2010 by the American Academy of Pediatrics page 276

Page 277: AAP PREP 2010

2010 PREP SA on CD-ROM

Preferred Response: ECritique: 110

The child described in the vignette has an antecedent gastrointestinal illness and exhibitssigns of volume depletion, including tachycardia, the absence of moist mucous membranes, anddelayed capillary refill. Furthermore, laboratory evaluation demonstrates hyponatremia,increased anion gap metabolic acidosis, and azotemia (elevated blood urea nitrogen andcreatinine values). The patient’s urinalysis demonstrates concentrated urine with ketonuria, butno blood or protein.

In general, results of this patient’s clinical and laboratory evaluation are consistent withhypovolemia and prerenal failure. Prerenal failure is seen in the setting of decreased effectivecirculating blood volume, as can occur with dehydration, congestive heart failure, and sepsis.The essentially negative urinalysis for this patient strongly suggests prerenal failure. The abilityof her kidneys to generate highly concentrated urine is reassuring regarding the viability of renaltubules. In contrast, children who have acute tubular necrosis tend to have isosthenuria (urinespecific gravity = 1.010) and, therefore, are unable to concentrate the urine because of tubularinjury. Further supportive evidence of tubular injury includes hematuria or proteinuria, which canoccur when the urothelium is disrupted.

Fluid management of children who have prerenal failure begins with restoration of theintravascular volume with isotonic saline. Patients who have prerenal failure and depressedeffective circulating blood volume have increased activity of the compensatory mechanisms tonormalize the blood volume, including the renin-angiotensin-aldosterone system, the sympatheticnervous system, and antidiuretic hormone. These compensatory systems prompt avid sodiumand water retention by the nephron, resulting in low urinary sodium excretion (<25 mEq/L) and

low fractional excretion of sodium (FENa <1%). This physiologic commitment to restoreintravascular volume in the setting of hypovolemia necessitates the prescription of intravenousfluids that are isotonic and devoid of free water. Therefore, the only acceptable fluid to increaseintravascular volume is 0.9% sodium chloride, which typically is prescribed at volumes of 20mL/kg for the pediatric patient. Other fluid options that have less than 0.9% sodium chloride(such as 0.225%, 0.33%, or 0.45%) contain free water and increase the risk for hyponatremiadue to high antidiuretic hormone concentrations in the patient.

The fluid prescription of insensible water loss + urine output replacement is ideal for thepatient who has intrinsic renal disease and is euvolemic. However, this fluid prescription isinappropriate for the dehydrated patient who has prerenal failure.

References:

Devarajan P, Goldstein SL. Acute renal failure. In: Kher KK, Schnaper HW, Makker SP, eds.Clinical Pediatric Nephrology. 2nd ed. London, England: Informa Healthcare; 2007:363-376

Whyte DA, Fine RN. Acute renal failure in children. Pediatr Rev. 2008;29:299-307. Available at:http://pedsinreview.aappublications.org/cgi/content/full/29/9/299

Copyright © 2010 by the American Academy of Pediatrics page 277

Page 278: AAP PREP 2010

2010 PREP SA on CD-ROM

Question: 111

A previously healthy 10-year-old girl presents with a 2-week history of bilateral nasalcongestion and yellowish-green rhinorrhea. During the past 3 days, she has had a temperatureof 38.4°C and some increased sinus pain. A review of her chart shows that her immunizationsare up to date, including her pneumococcal conjugate vaccine series and her annual influenzavaccination. She has no history of drug allergies. On physical examination, she has appropriatevital signs for her age, bilateral infraorbital edema, and yellowish mucus in her nares.

Of the following, the MOST appropriate initial antibiotic to consider is

A. amoxicillin

B. amoxicillin/clavulanate

C. cephalexin

D. clindamycin

E. trimethoprim-sulfamethoxazole

Copyright © 2010 by the American Academy of Pediatrics page 278

Page 279: AAP PREP 2010

2010 PREP SA on CD-ROM

Preferred Response: ACritique: 111

Acute bacterial sinusitis is an infection of the paranasal sinuses that lasts more than 7 to 10days. The most common bacteria involved in acute sinusitis are Streptococcus pneumoniae,Haemophilus influenzae, and Moraxella catarrhalis. Other organisms resulting in acute sinusitisinclude viral pathogens causing upper respiratory tract infections, Staphylococcus aureus,anaerobes, and rarely fungal species in immunocompromised individuals.

Prominent symptoms of acute sinusitis include nasal congestion, rhinorrhea, postnasaldrainage, cough, fever, headache, and tooth pain. Physical examination findings do notnecessarily contribute to the diagnosis of acute bacterial sinusitis because they can be similar tothose of an uncomplicated viral infection. Antibiotic treatment of acute rhinosinusitis isrecommended to speed the clinical cure and prevent complications such as periorbital cellulitisand asthma exacerbation. Initial therapy should be with an agent that is active against the likelypathogen and based on local resistance patterns.

Amoxicillin is the drug of choice for the treatment of acute sinusitis. A dose schedule of 45mg/kg per day or 90 mg/kg per day is appropriate, depending on the presence of risk factors.The lower dose can be used for children who are older than 2 years of age, are not in childcare, and have not received antimicrobial therapy in the past month.

Amoxicillin/clavulanate is an appropriate choice if the patient fails to improve after receivingamoxicillin monotherapy and there are concerns regarding beta-lactam-resistant orgranisms.First-generation cephalosporins such as cephalexin are poor choices due to poor coverage forH influenzae and M catarrhalis. For a patient who has severe reactions to penicillin, appropriateoptions include azithromycin, clarithromycin, or clindamycin. Trimethroprim-sulfamethoxazole anderythromycin-sulfisoxazole no longer are recommended due to increased pneumococcalresistance.

References:

Pappas DE, Hendley JO. Sinusitis. In: Kliegman RM, Behrman RE, Jenson HB, Stanton BF, eds.Nelson Textbook of Pediatrics. 18th ed. Philadelphia, Pa: Saunders Elsevier; 2007:1749-1751

Slavin RG, Spector SL, Berstein IL, et al; American Academy of Allergy, Asthma andImmunology; American College of Allergy, Asthma and Immunology; Joint Council of Allergy,Asthma and Immunology. The diagnosis and management of sinusitis: a practice parameterupdate. J Allergy Clin Immunol. 2005;116(6 suppl):S13-S47

Taylor A, Adam HM. In brief: sinusitis. Pediatr Rev. 2006;27:395-397. Available at:http://pedsinreview.aappublications.org/cgi/content/full/27/10/395

Copyright © 2010 by the American Academy of Pediatrics page 279

Page 280: AAP PREP 2010

2010 PREP SA on CD-ROM

Question: 112

A 2-year-old boy presents with bloody drainage from the left ear. His mother reports that he hasbeen crying and holding his left ear since last night, and there was blood on his pillow andaround his left ear this morning when he awoke. On examination of his ears, you noteseropurulent fluid in the external auditory canal and marked edema of the mucosa. Midway in thecanal you can see a portion of a small metallic object. Upon further questioning, the motherreports she had found him in his grandmother’s room yesterday playing with her hearing aids.

Of the following, the MOST appropriate next step is to

A. attempt removal with cerumen spoon

B. irrigate the external auditory canal to remove the foreign body

C. prescribe ciprofloxacin ear drops and schedule a follow-up appointment in 2 days

D. recommend follow-up with an otorhinolaryngologist next week for removal of the foreignbody

E. send the boy to the emergency department for immediate removal of the foreign body

Copyright © 2010 by the American Academy of Pediatrics page 280

Page 281: AAP PREP 2010

2010 PREP SA on CD-ROM

Preferred Response: ECritique: 112

Foreign bodies in the ear are common in children and may have a variety of presentations.Patients may be asymptomatic, but the parent may see something in the ear or the child maydisclose putting something in the ear. Symptomatic patients may complain of ear pain, hearingloss, or otorrhea. Common ear foreign bodies include beads, insects, toys, popcorn, beans, andbutton batteries.

Easily graspable foreign bodies usually can be removed in the office using irrigation,forceps, right angle hook, cerumen spoon, or suction. In some situations, subspecialtyconsultation may be necessary. Hard foreign objects that cannot be grasped or are wedged inthe external auditory canal (EAC), trauma in the ear canal, or lack of success after multipleremoval attempts are common reasons for referral. With the exception of organic foreign bodies(eg, beans, popcorn) and button batteries, most asymptomatic ear foreign bodies do not requireemergent removal.

The patient described in the vignette has a button battery in the EAC. Because of thesignificant risk for tissue damage from leakage of the battery contents, button batteries requireemergent removal. The marked edema and poor visibility of the object in this case necessitatessubspecialty consultation. Attempted removal with a cerumen spoon is not likely to besuccessful and may cause additional trauma, and irrigation never is indicated in this settingbecause water may cause current generation or leakage. Antibiotic drops may be indicatedfollowing the removal procedure.

References:

Bauer CA, Jenkins HA. Otologic symptoms and syndromes. In: Cummings CW, Flint PW, HaugheyBH, et al, eds. Cummings Otolaryngology: Head and Neck Surgery. 4th ed. Philadelphia, Pa:Elsevier Mosby; 2005:2867-2871

Haddad J. The ear: general considerations and evaluation. In: Kleigman RM, Behrman RE, JensonHB, Stanton BF, eds. Nelson Textbook of Pediatrics. 18th ed. Philadelphia, Pa: SaundersElsevier; 2007:2617-2619

Heim SW, Maughan KL. Foreign bodies in the ear, nose, and throat. Am Fam Physician.2007;76:1185-1189. Available at: http://www.aafp.org/afp/20071015/1185.html

Lin VY, Daniel SJ, Papsin BC. Button batteries in the ear, nose and upper aerodigestive tract. IntJ Pediatr Otorhinolaryngol. 2004;68:473-479. Abstract available at:http://www.ncbi.nlm.nih.gov/pubmed/15013616

Copyright © 2010 by the American Academy of Pediatrics page 281

Page 282: AAP PREP 2010

2010 PREP SA on CD-ROM

Question: 113

A 3½-week-old male infant presents to your office with a history of 2 to 3 days of vomiting. Hewas born at term and his birthweight was 3,250 g. He is breastfed. He has been exhibitingnonbilious vomiting after each feeding, and according to his mother, the emesis now appears to"shoot out of his mouth." After vomiting, he seems eager to resume feeding. Over the past 24hours, his mother has noted fewer wet diapers and less stool than usual. The baby hasexperienced no diarrhea or upper respiratory tract symptoms. Physical examinationdemonstrates an alert, afebrile infant who weighs 3,550 g and is sucking vigorously on apacifier. His skin turgor is normal. The remainder of the examination findings are unremarkable,except for slight abdominal distention. You refer the baby to the local emergency department and

order measurement of serum electrolytes. Results include:

•Sodium, 132 mEq/L (132 mmol/L)

•Potassium, 3.2 mEq/L (3.2 mmol/L)

•Chloride, 95 mEq/L (95 mmol/L)•Bicarbonate, 30 mEq/L (30 mmol/L)

Of the following, the MOST appropriate initial treatment for this infant is to

A. administer 70 mL 0.9% sodium chloride intravenously over 1 hour, followed by infusion with5% dextrose and 0.225% sodium chloride at 15 mL/hr

B. begin intravenous fluids with 5% dextrose and 0.3% sodium chloride at 15 mL/hr

C. encourage continued nursing, pending results of further diagnostic tests

D. initiate intravenous fluids containing 5% dextrose and 0.45% sodium chloride at 25 mL/hr

E. institute oral rehydration therapy with a glucose-electrolyte solution

Copyright © 2010 by the American Academy of Pediatrics page 282

Page 283: AAP PREP 2010

2010 PREP SA on CD-ROM

Preferred Response: DCritique: 113

The progressive nonbilious vomiting, weight loss, and hypochloremic metabolic alkalosisdescribed for the infant in the vignette suggest the presence of a gastric outlet obstruction.Infantile hypertrophic pyloric stenosis (HPS) is the most likely cause of such clinical findings. Theinitial goals of therapy are institution of appropriate intravenous rehydration, with gradualcorrection of any electrolyte disturbances. Only when hydration status has normalized andelectrolyte abnormalities have been corrected should surgical pyloromyotomy be undertaken.The presenting weight of 3,550 g for this 3.5-week-old infant, when compared with hisbirthweight of 3,250 g and an estimated 20-g/day weight gain, indicates relatively milddehydration of approximately 5%. Based on the serum electrolyte data, fluid therapy should beinitiated with 5% dextrose and 0.45% sodium chloride at approximately 1.5 times the calculatedmaintenance rate (~400 mL per 24 hours, based on the infant’s hydrated weight) or 25 mL/hr.Oral rehydration therapy, including human milk or glucose electrolyte solutions, is notrecommended in this infant, whose serum electrolye values indicate metabolic derangementsconsistent with HPS, which will require surgical intervention.

Because the infant in the vignette is estimated to be 5% dehydrated, vigorous fluidmanagement with a bolus infusion is not required. In most cases of HPS, appropriate initialintravenous fluid therapy involves a sodium chloride concentration between 0.45% and 0.9%,depending on the magnitude of the electrolyte deficits. Intravenous therapy with concentrationsof less than 0.45% sodium chloride should be avoided, unless the condition is diagnosed early inthe disease course and all electrolyte and acid-base values are normal. Potassium should beadded to the infusate as soon as urinary output is established to prevent progressivehypokalemia. It is important to note that the serum potassium concentration rises as the serum pHnormalizes. Bolus infusions with 10 to 20 mL/kg 0.9% sodium chloride are reserved for infantswho demonstrate clinically moderate-to-severe intravascular fluid depletion (ie, >7%dehydration).

HPS is the most frequent cause of metabolic alkalosis in the first few weeks after birth andthe most common indication for abdominal surgery, with an estimated incidence of 1 in 250 livebirths. The condition occurs more commonly in white males, but greater risk of occurrence infirst-born males has not been proven. There may be a positive family history. Other, lesscommon causes of gastric outlet obstruction include duodenal stenosis, gastric duplication,antral web, and annular pancreas.

Although hypochloremic, hypokalemic metabolic alkalosis represents the classic electrolyteand acid-base imbalances of HPS, many infants have normal serum electrolyte values, thusincreasing the importance of having a "high index of suspicion" for this diagnosis. The magnitudeof any acid-base and electrolyte disturbance is related directly to the duration of vomiting.Persistent emesis results in a progressive loss of gastric fluid and hydrochloric acid.Hypokalemia ensues as the kidneys retain hydrogen ions in favor of potassium. As extracellularfluid volume and hydrogen ions continue to be lost due to continued postprandial emesis,intravascular volume contracts, increasing the relative serum bicarbonate concentration andleading to a contraction alkalosis. Serum potassium values fall further as alkalosis stimulatespotassium-hydrogen exchange across cell membranes, with potassium driven intracellularly andhydrogen ions driven extracellularly.

Copyright © 2010 by the American Academy of Pediatrics page 283

Page 284: AAP PREP 2010

2010 PREP SA on CD-ROM

References:

Dinkevich E, Ozuah PO. In brief: pyloric stenosis. Pediatr Rev. 2000;21:249-250. Available at:http://pedsinreview.aappublications.org/cgi/content/full/21/7/249

Garcia VF, Randolph JG. Pyloric stenosis: diagnosis and management. Pediatr Rev. 1990;11:292-296. Abstract available at:http://pedsinreview.aappublications.org/cgi/content/abstract/11/10/292

Rice HE, Caty MG, Glick PL. Fluid therapy for the pediatric surgical patient. Pediatr Clin NorthAm. 1998;45:719-727. Abstract available at: http://www.ncbi.nlm.nih.gov/pubmed/9728183

Roberts KB. Fluid and electrolytes: parenteral fluid therapy. Pediatr Rev. 2001;22:380-387.Available at: http://pedsinreview.aappublications.org/cgi/content/full/22/11/380

Copyright © 2010 by the American Academy of Pediatrics page 284

Page 285: AAP PREP 2010

2010 PREP SA on CD-ROM

Question: 114

A 3-day-old infant who was born at 29 weeks’ gestation and weighed 1,200 g has experiencedrespiratory distress syndrome and is receiving assisted ventilation. This morning, you note agrade III/VI holosystolic murmur, hyperdynamic precordium, and widened pulse pressures.

Of the following, the MOST appropriate next step is to

A. administer ibuprofen treatment

B. administer indomethacin prophylaxis

C. increase maintenance fluids

D. obtain echocardiography

E. reduce the ventilator settings

Copyright © 2010 by the American Academy of Pediatrics page 285

Page 286: AAP PREP 2010

2010 PREP SA on CD-ROM

Preferred Response: DCritique: 114

The 29-weeks’ gestation very low-birthweight infant described in the vignette, who hasrespiratory distress syndrome, is at risk for a persistent patent ductus arteriosus (PDA). Closureof the ductus arteriosus often is delayed in such infants, but its patency may be of variablehemodynamic or clinical significance. Symptoms or signs resulting from a PDA typically appearbetween the third and seventh postnatal day, although they can appear later.

The significance of the PDA for the infant in the vignette is suggested by the presence ofthe murmur, a widened pulse pressure, and a hyperdynamic precordium. Such findings areindicative of a left-to-right shunt of blood from the descending aorta through the PDA into thepulmonary circulation. This condition may volume overload the right heart, contributing to thehyperdynamic precordium. As the shunt continues, pulse pressures widen and the overalladequacy of cardiac output to distal circulatory beds may be jeopardized, resulting in systemichypotension, metabolic acidemia, oliguria, and potential gastrointestinal compromise. Untreated,the condition may result in pulmonary congestion with reduced pulmonary compliance,congestive heart failure, and hepatomegaly.

Diagnosis of a PDA is made both clinically and echocardiographically. The hemodynamicsignificance of a PDA is determined echocardiographically by assessing the size of the leftatrium, the aortic outlet, the PDA, and the right ventricle as well as the presence of reversed end-diastolic blood flow in the proximal aortic arch. Judicious fluid management (<150 mL/kg per day)in the first week after birth may reduce the likelihood of clinically significant PDA. However, oncethe PDA is diagnosed, fluid restriction is a mainstay of treatment. The use of positive end-expiratory pressure (PEEP) also may help reduce pulmonary overcirculation in the ventilatedpatient. Pharmacologic management consists of intravenous indomethacin or ibuprofen-lysine,usually administered in three successive doses. If the PDA fails to close with thesepharmacologic measures, surgical ligation may be warranted.

Treatment with ibuprofen may follow confirmation of the PDA but should not be initiatedpresumptively due to potential complications and adverse effects. A single dose of intravenousindomethacin may be administered prophylactically in the first 12 to 24 hours after birth, but thewindow for prophylaxis has passed for the infant in the vignette. Increasing fluids might worsencardiopulmonary congestion in the setting of a PDA. Reducing ventilator settings (PEEP,inspiratory time, or pressure) can result in a reduction in mean airway pressure, which allowsfor an increase in left-to-right shunting.

References:

Clyman RI, Chorne N. Patent ductus arteriosus: evidence for and against treatment. J Pediatr.2007;150:216-219

Ohlsson A, Walia R, Shah S. Ibuprofen for the treatment of patent ductus arteriosus in pretermand/or low birth weight infants. Cochrane Database Syst Rev. 2008;1:CD003481. Available at:http://www.mrw.interscience.wiley.com/cochrane/clsysrev/articles/CD003481/frame.html

Sarkar S, Dechert R, Schumacher RE, Donn SM. Is refractory hypotension in preterm infants a

Copyright © 2010 by the American Academy of Pediatrics page 286

Page 287: AAP PREP 2010

2010 PREP SA on CD-ROM

manifestation of early ductal shunting? J Perinatol. 2007;27:353-358. Abstract available at:http://www.ncbi.nlm.nih.gov/pubmed/17443200

Thilo EH, Rosenberg AA. The newborn infant. In: Hay WW Jr, Levin MJ, Sondheimer JM,Deterding RR, eds. CURRENT Diagnosis & Treatment: Pediatrics. 19th ed. New York, NY: TheMcGraw-Hill Companies; 2009:Chapter 1. Available for subscription at:http://www.accessmedicine.com/content.aspx?aID=3396500

Thilo EH, Rosenberg AA. The newborn infant. In: Hay WW Levin MJ, Sondheimer JM, DeterdingRR, eds. CURRENT Diagnosis & Treatment: Pediatrics. 19th ed. New York, NY: The McGraw-Hill Companies; 2009:Chapter 1. Available for subscription at:http://www.accessmedicine.com/content.aspx?aID=3396500

Copyright © 2010 by the American Academy of Pediatrics page 287

Page 288: AAP PREP 2010

2010 PREP SA on CD-ROM

Question: 115

A 16-year-old girl comes to your office for a follow-up visit from the emergency department,where she went for the acute onset of knee pain and swelling. The emergency departmentphysician had ordered an antinuclear antibody test, which was positive at 1:320. Further historyreveals that she has had intermittent joint pains for several weeks and dark-colored urine.Findings on her physical examination are normal except for an effusion in her right knee. Youdecide that further evaluation for systemic lupus erythematosus (SLE) is warranted.

Of the following, the MOST specific test in helping you make the diagnosis of SLE is

A. anticardiolipin antibody measurement

B. anti-double-stranded DNA antibody measurement

C. anti-Ro antibody measurement

D. complement measurement

E. Venereal Disease Research Laboratory (VDRL) test

Copyright © 2010 by the American Academy of Pediatrics page 288

Page 289: AAP PREP 2010

2010 PREP SA on CD-ROM

Preferred Response: BCritique: 115

Systemic lupus erythematosus (SLE) is a chronic autoimmune disorder that involves multipleorgan systems. The diagnostic criteria are well established and include the presence ofcharacteristic laboratory findings as well as clinical manifestations. Four of the following 11

criteria must be met for the diagnosis to made:

•Malar rash

•Discoid rash

•Oral ulcers

•Photosensitivity

•Arthritis

•Serositis

•Hematologic manifestations

•Central nervous system manifestations

•Nephritis•Immunologic manifestations (positive anti-double-stranded DNA [anti-ds DNA] or anti-

Smith antibody, false-positive test for syphilis, elevated antiphospholipid antibodies)

•Elevation of antinuclear antibody (ANA)A good screening test for the presence of SLE is the ANA test because almost all patients

who have SLE have positive results. However, many people who do not have SLE also mayhave a positive ANA test result, including those who have no disease and those who haveother conditions such as juvenile idiopathic arthritis, dermatomyositis, thyroid disease, or recentinfections. When a patient’s laboratory evaluation reveals a positive ANA and there are otherpotential clinical manifestations of SLE, such as those described for the girl in the vignette, morespecific antibodies, such as anti-ds DNA, anti-Smith, anti-Ro, and anti-cardiolipin antibodies,should be obtained. Of these, the anti-ds DNA is the most specific. Anti-Ro is seen mostcommonly in neonatal lupus erythematosus. Complement values may be low in patients whohave SLE, but this finding is not sensitive or specific for SLE. A false-positive Venereal DiseaseResearch Laboratory (VDRL) test occurs commonly among patients who have SLE, making itone of the laboratory criteria for the disease; however, the anti-ds DNA antibody test hasgreater diagnostic specificity.

References:

Benseler SM, Silverman ED. Systemic lupus erythematosus. Pediatr Clin North Am. 2005;52:443-467. Abstract available at: http://www.ncbi.nlm.nih.gov/pubmed/15820375

Gottlieb BS, Ilowite NT. Systemic lupus erythematosus in children and adolescents. Pediatr Rev.2006;27:323-329. Available at: http://pedsinreview.aappublications.org/cgi/content/full/27/9/323

Copyright © 2010 by the American Academy of Pediatrics page 289

Page 290: AAP PREP 2010

2010 PREP SA on CD-ROM

Question: 116

An 18-month-old boy presents to the emergency department with a 2-day history of cough,posttussive emesis, and diminished food intake, although he has been taking liquids. Thecoughing began after eating some popcorn. His mother explains that he has had no fever orrhinorrhea. He had an episode of bronchiolitis at 3 months of age but has no other history ofrespiratory illness, chronic cough, or other health concerns. He is the youngest of four childrencared for at home. On physical examination, his vital signs are normal, his height and weight areat the 50th percentile for age, and the boy is mouth-breathing with mild nasal flaring. Oxygensaturation is 94% on room air. Auscultation of the chest reveals diffuse rhonchi and wheezesthat are markedly louder on the left side of the chest.

Of the following, the MOST likely diagnosis is

A. bronchiolitis

B. community-acquired bacterial pneumonia

C. cystic fibrosis

D. foreign body aspiration

E. reactive airway disease

Copyright © 2010 by the American Academy of Pediatrics page 290

Page 291: AAP PREP 2010

2010 PREP SA on CD-ROM

Preferred Response: DCritique: 116

Cough and wheezing are common presentations in small children and infants. Although themost common causes involve asthma and bronchiolitis, tracheal and esophageal foreign bodiesmay present with the acute or subacute onset of cough and wheezing, especially in the olderinfant and toddler. The age, lack of symptoms since age 3 months, and abrupt onset ofsymptoms after eating a food that could be aspirated, as described for the boy in the vignette,suggest foreign body aspiration rather than the other more indolent causes.

Contrary to popular belief, the aspiration of a small foreign body, including food (such asseeds or nuts) is as common in the left as the right mainstem bronchus in young childrenbecause the bifurcation of the trachea remains symmetric until the aortic knob grows larger inlater childhood. Typically, a chest radiograph may not disclose air trapping for up to 1 day afterthe aspiration, despite the initial coughing. A careful history must be obtained for all children whohave abrupt onset of cough and wheezing, and children whose histories strongly suggestforeign body aspiration should undergo prompt rigid bronchoscopy (Item C116). The child inrespiratory distress should not be sent to the radiology department but should have expiratoryand inspiratory anteroposterior chest radiography performed in the emergency department.Children who have aspirated foreign bodies are at high risk for developing secondary bacterialpneumonia.

An esophageal foreign body, most often a coin, also may cause wheezing, but commonlypatients also have vomiting or an inability to tolerate solid foods. When the foreign body remainsin the upper half of the esophagus, the trachea may be compressed directly in some children,especially infants.

Although bacterial pneumonia occasionally presents with wheezing (especially in the childwho has asthma) and wheezing may be seen with community-acquired pneumonia caused byMycoplasma and Chlamydia pneumoniae, other findings, such as fever, and bilateral signsupon auscultation of the chest, often are present. Cystic fibrosis should be considered inchildren who have persistent pulmonary disease, especially in conjunction with failure to thriveor diarrhea.

References:

Kugelman A, Shaoul R, Goldsher M, Srugo I. Persistent cough and failure to thrive: a presentationof foreign body aspiration in a child with asthma. Pediatrics. 2006;117:e1057-e1060. Availableat: http://pediatrics.aappublications.org/cgi/content/full/117/5/e1057

Mohan P. In brief: aspiration in infants and children. Pediatr Rev. 2002;23:330-331. Available at:http://pedsinreview.aappublications.org/cgi/content/full/23/9/330

Rovin JD, Rodgers BM. Pediatric foreign body aspiration. Pediatr Rev. 2000;21:86-90. Availableat: http://pedsinreview.aappublications.org/cgi/content/full/21/3/86

Skae CC, Adam HM. In brief: esophageal foreign bodies. Pediatr Rev. 2005;26:34-35. Availableat: http://pedsinreview.aappublications.org/cgi/content/full/26/1/34

Copyright © 2010 by the American Academy of Pediatrics page 291

Page 292: AAP PREP 2010

2010 PREP SA on CD-ROM

Weinberger M, Abu-Hasan M. Pseudo-asthma: when cough, wheezing, and dyspnea are notasthma Pediatrics. 2007;120:855-864. Available at:http://pediatrics.aappublications.org/cgi/content/full/120/4/855

Copyright © 2010 by the American Academy of Pediatrics page 292

Page 293: AAP PREP 2010

2010 PREP SA on CD-ROM

Question: 117

You are evaluating a 2-month-old girl as part of a routine health maintenance visit. Her mothertells you that she has no trouble feeding and is gaining weight like her previous children. Herprecordial examination demonstrates a mild lift. The first and second heart sounds are normal.There is a systolic click at the upper left sternal border as well as a 3/6 systolic ejection murmurat the upper left sternal border (Item Q117) with radiation to the axillae. Diastole is clear, and herpulses are normal in all extremities.

Of the following, the MOST likely cause of this patient’s signs and symptoms is

A. aortic stenosis

B. atrial septal defect

C. patent ductus arteriosus

D. pulmonary stenosis

E. ventricular septal defect

Copyright © 2010 by the American Academy of Pediatrics page 293

Page 294: AAP PREP 2010

2010 PREP SA on CD-ROM

Preferred Response: DCritique: 117

The infant described in the vignette has typical findings of pulmonary stenosis, which oftenis associated with a systolic click resulting from the abnormal structure and function of thepulmonary valve. The click is caused by the opening of the thickened valve leaflets duringsystole. In contrast to the normal thin and flexible semilunar valve leaflets, those of the stenoticpulmonary valve have an accentuated sound that is referred to as an opening click. The murmurof pulmonary stenosis results from systolic blood flow from the right ventricle across theabnormally narrowed orifice of the pulmonary valve. The narrowing yields a diminished valvearea through which the stroke volume crosses, creating turbulence. Such turbulence is notedduring auscultation as a systolic ejection murmur and typically is heard best over the pulmonaryvalve and main pulmonary artery. On the chest wall, these structures lie beneath the left sternalborder, with extension cephalad toward the left clavicle. Frequently, the murmur radiates into theback and the axillae as the sound of turbulence follows the course of the branch pulmonaryarteries.

Aortic stenosis also is associated with a systolic ejection click that does not change withposition, but the accompanying murmur is heard best at the upper right sternal border, withradiation into the neck. The murmur associated with an atrial septal defect is not from the bloodflow across the atrial septum, which usually is nonturbulent and at low pressure. Rather, thesystolic murmur created by an atrial septal defect is the result of a relative pulmonary stenosisas the left-to-right atrial shunt and resulting increased right ventricular volume must cross thepulmonary valve. In contrast to pulmonary valve stenosis, there is no structural abnormality ofthe pulmonary valve and, thus, no systolic click. Patent ductus arteriosus typically produces acontinuous murmur characterized as having a "machinery" quality that is usually loudest at theleft infraclavicular area. It is continuous because of the constant flow between the systemic andpulmonary circulation resulting from the higher systemic vascular resistance compared with thepulmonary vascular resistance throughout the cardiac cycle and the lack of a valve to separatethe two circulations. The murmur of a ventricular septal defect typically is holosystolic becausethe left-to-right shunt at the ventricular level begins with the onset of systole, even before theaortic and pulmonary valves open. When the ventricular septal defect is small, it produces a high-pitched murmur, heard along the sternal border, and a normal second heart sound without achange in its normal physiologic splitting.

References:

Allen HD, Phillips JR, Chan DP. History and physical examination. In: Allen HD, Driscoll DJ, ShaddyRE, Feltes TF, eds. Moss and Adams’ Heart Disease in Infants, Children, and Adolescents,Including the Fetus and Young Adult. 7th ed. Philadelphia, Pa: Lippincott Williams & Wilkins;2008:58-65

Moller JH. Clinical history and physical examination. In: Moller JH, Hoffman JIE, eds. PediatricCardiovascular Medicine.Philadelphia, Pa: Churchill Livingstone; 2000:97-110

Copyright © 2010 by the American Academy of Pediatrics page 294

Page 295: AAP PREP 2010

2010 PREP SA on CD-ROM

Question: 118

A 4-year-old boy presents to the emergency department with balance problems. He had beenpreviously healthy, but his walking has worsened progressively for the past 2 days, withstaggering and lurching. On physical examination, the boy is cooperative and alert. His musclesare not tender, and his joints are not red, swollen, or tender. His vision seems functionallynormal, but there is end-gaze nystagmus in all directions. When sitting independently, his headand trunk bob. His strength appears normal, and his reflexes are normal. When asked to standwith his hands outstretched, a symmetric tremor is evident and worsens as he approaches thetarget on finger-to-nose testing. His gait is broad-based. A urine toxicology screen revealsnormal results. Brain magnetic resonance imaging shows no tumors or other gray or whitematter lesions. Lumbar puncture shows 3 white blood cells, 2 red blood cells, protein of 20.0g/dL, and glucose of 50.0 mg/dL (2.8 mmol/L).

Of the following, you are MOST likely to advise the child’s mother that

A. intravenous steroids significantly reduce recurrence risk

B. neuroblastoma is a common cause of these symptoms

C. repeat lumbar puncture is needed in 2 days

D. symptoms may resolve in weeks to months

E. symptoms usually resolve after antibiotic treatment

Copyright © 2010 by the American Academy of Pediatrics page 295

Page 296: AAP PREP 2010

2010 PREP SA on CD-ROM

Preferred Response: DCritique: 118

A child who experiences subacute-onset gait or balance problems should be evaluatedthoroughly in the emergency department based initially on localization obtained through carefulexamination. The differential diagnosis includes structural and immunologically mediateddiseases that can be life-threatening or cause permanent neurologic injury. The bilateralsymptoms with completely normal mental status and no somnolence described for the boy in thevignette makes a cerebral cause less likely. The lateral end-gaze nystagmus, tremor on handactivation and finger-to-nose testing, trunk bobbing (titubation), and broad-based gait localize theproblem to the cerebellum (Item C118). The subacute onset of such symptoms strongly suggestsacute cerebellar ataxia, which usually is acquired after infection or immunization. Because this isa self-limited, monophasic illness, the mother can be advised that the symptoms should resolvein weeks to months.

A repeat lumbar puncture in a few days is not likely to clarify the diagnosis. Otitis mediadoes not cause nystagmus and ataxia, and antibiotics are not indicated (unless there is aconcurrent bacterial infection). The recurrence risk for acute cerebellar ataxia is low, and it isunknown whether steroids reduce the risk further. Although clinical trials are not available toguide management, due in part to the low incidence of this condition, many clinicians empiricallytreat with a short course of high-dose steroids to reduce the duration of illness.

Although the time course makes a structural cerebellar or brainstem lesion unlikely for thispatient, ordering the magnetic resonance imaging is reasonable because children in this agegroup may present with cerebellar and brainstem neoplasms such as astrocytomas, pontinegliomas, primitive neuroectodermal tumors (medulloblastomas), or ependymomas. Rarely, such apresentation may be associated with Guillain-Barré syndrome. Therefore, specialty consultationis advised.

For children presenting with acute ataxia, the clinician should be especially vigilant for thepossibility of neuroblastoma presenting as opsoclonus myoclonus ataxia syndrome. Case serieshave shown that children who have opsoclonus myoclonus often are misdiagnosed initially ashaving acute cerebellar ataxia. Because distinguishing these diagnoses clinically is challenging,specialty consultation is advisable.

References:

Fogel BL, Perlman S. Clinical features and molecular genetics of autosomal recessive cerebellarataxias. Lancet Neurol. 2007;6:245-257. Abstract available at:http://www.ncbi.nlm.nih.gov/pubmed/17303531

Gilbert DL. Ataxia. In: Singer HS, Kossoff EH, Hartman AL, Crawford TO, eds. Treatment ofPediatric Neurologic Disorders. Boca Raton, Fla: Taylor & Francis Group;. 2005:415-422

Johnston MV. Movement disorders. In: Kliegman RM, Behrman RE, Jenson HB, Stanton BF, eds.Nelson Textbook of Pediatrics. 18th ed. Philadelphia, Pa: Saunders Elsevier, 2007:2488-2493

Klockgether T. Ataxias. In: Goetz C, ed. Textbook of Clinical Neurology. 3rd ed. Philadelphia, Pa:

Copyright © 2010 by the American Academy of Pediatrics page 296

Page 297: AAP PREP 2010

2010 PREP SA on CD-ROM

Saunders Elsevier; 2007:741-758

Pranzatelli MR. Opsoclonus-myoclonus-ataxia syndrome. In: Fernandez-Alvarez E, ArzimanoglouA, Tolosa E, eds. Paediatric Movement Disorders: Progress in Understanding. Esher, Surrey,United Kingdom: John Libbey Eurotext; 2005:121-136

Ryan MM, Engle EC. Acute ataxia in childhood. J Child Neurol. 2003;18:309-316. Abstractavailable at: http://www.ncbi.nlm.nih.gov/pubmed/12822814

Copyright © 2010 by the American Academy of Pediatrics page 297

Page 298: AAP PREP 2010

2010 PREP SA on CD-ROM

Question: 119

You are seeing an African American child for evaluation of short stature. While taking the familyhistory, you learn that multiple individuals in three generations on the mother’s side were bornwith postaxial polydactyly (an extra digit on the little finger side) of the hand(s). The mother saysthat the extra digits have been removed, and all affected individuals are doing well and havenormal stature. The medical student working with you asks about the significance of this familyhistory.

Of the following, the MOST likely explanation for postaxial polydactyly in this family is

A. a short stature-polydactyly syndrome

B. an unbalanced chromosome rearrangement

C. autosomal dominant inheritance

D. consanguinity

E. teratogenic exposure

Copyright © 2010 by the American Academy of Pediatrics page 298

Page 299: AAP PREP 2010

2010 PREP SA on CD-ROM

Preferred Response: CCritique: 119

Polydactyly usually is an isolated condition that is inherited as an autosomal dominant trait,but it also is a feature of numerous syndromes. Polydactyly is divided into preaxial (extra digit[s]on the thumb or great toe side of the hand or foot), mesoaxial (extra digit[s] in the middle), andpostaxial (occurring on the fifth finger or little toe side) forms (Item C119). Further subtypes aredescribed based on how well formed the extra digit(s) is(are) and their shape. In recent years,researchers have learned a great deal about genes that control limb development, shedding lighton mechanisms leading to the formation of extra digits.

Postaxial polydactyly occurs in approximately 1 in 1,500 newborns and is particularlycommon as a dominant trait in individuals of African descent. As with any type of birth defect, itis important to obtain a three-generation pedigree to discern whether there are other affectedindividuals, the pattern of inheritance, and the presence or absence of associated anomalies.

Causes for polydactyly are numerous and include chromosome abnormalities, single-genedefects (consanguinity suggests the possibility of increased risk for autosomal recessiveconditions), and teratogens such as thalidomide. Although some syndromes are associated withshort stature and polydactyly, the history of normal stature in family members who havepolydactyly in the vignette makes a short stature-polydactyly syndrome unlikely.

Management of polydactyly varies among types. For the simplest form of postaxialpolydactyly (type B), tying off or transection of a small skin tag may be sufficient. For morecomplicated cases involving bone, soft tissues, and tendons, referral to a hand surgeon isnecessary.

References:

Everman DB. Hands and feet: the polydactylies. In: Stevenson RE, Hall JG, eds. HumanMalformations and Related Anomalies. 2nd ed. New York, NY: Oxford University Press;2006:937-953

Firth HV, Hurst JA. Postaxial polydactyly, preaxial polydactyly. In: Oxford Desk ReferenceClinical Genetics. New York, NY: Oxford University Press; 2005:214-219

Copyright © 2010 by the American Academy of Pediatrics page 299

Page 300: AAP PREP 2010

2010 PREP SA on CD-ROM

Question: 120

You are seeing a 16-year-old girl for complaints of a malodorous vaginal discharge. She has noabdominal pain or urinary or gastrointestinal symptoms. Results of routine screening forgonorrhea and chlamydia were negative 3 months ago, and she has not been sexually activesince that time. She explains that she douches regularly. On pelvic examination, you note ahomogenous gray discharge coating the vaginal walls, normal-appearing cervix, and no uterineor adnexal tenderness on bimanual examination. The pH of her vaginal secretions is 4.8. Youobtain a saline wet mount (Item Q120).

Of the following, the MOST likely diagnosis is

A. bacterial vaginosis

B. chemical vaginitis

C. chlamydial cervicitis

D. physiologic leukorrhea

E. vaginal candidiasis

Copyright © 2010 by the American Academy of Pediatrics page 300

Page 301: AAP PREP 2010

2010 PREP SA on CD-ROM

Preferred Response: ACritique: 120

The patient described in the vignette has bacterial vaginosis, based on the presence ofthree of the four Amstel diagnostic criteria. The four criteria are: a thin, homogenous, gray-whitedischarge uniformly adherent to the vaginal walls; a vaginal pH greater than 4.5; a positive whifftest (fishy or amine odor on addition of 10% potassium hydroxide); and more than 20% clue cellson microscopic examination. Clue cells are epithelial cells that are coated with bacteria, creatinga granular appearance (Item C120). The presence of three or more of the criteria indicates thepresence of bacterial vaginosis.

Bacterial vaginosis is not sexually transmitted, although it is associated with sexual activity.Because it is not inflammatory, no white blood cells are seen on microscopy. It is caused byovergrowth of several anaerobic bacterial species (eg, Mobiluncus and Gardnerella vaginalis)and a decrease in hydrogen peroxide-producing Lactobacillus. Although often asymptomatic,affected patients may complain of a malodorous discharge. Douching may increase the risk ofbacterial vaginosis by causing changes in the vaginal flora and disturbing the vaginal protectivesystems, which are based on hydrogen peroxide-producing lactobacilli. Such disruption of thevaginal microbiology permits overgrowth of the anaerobic and aerobic bacteria responsible forbacterial vaginosis. Because bacterial vaginosis has been linked to the acquisition of humanimmunodeficiency virus, preterm delivery, pelvic inflammatory disease, and other adverseeffects, douching could play an important role in multiple health problems among sexually activewomen.

When examining a patient who complains of a vaginal discharge, it is important to view thecervix. Purulent discharge at the cervical os and easy bleeding of the cervix when swabbed(friability) suggests cervicitis rather than vaginitis. Cervicitis is caused by Chlamydiatrachomatis, Neisseria gonorrhoeae, and herpes simplex virus. Vaginal candidiasis usuallypresents as a pruritic, thick, white or milky discharge. Physiologic discharge, (leucorrhea) isasymptomatic and associated with normal findings on microscopy. Contact of the vaginalmucosa with certain chemicals (eg, soaps or bubble baths) may result in complaints of vaginalburning or swelling. These symptoms also may be related to an allergic reaction (eg, to latex) orto irritation as with tight clothing. On inspection, there may be slight redness of the vaginalmucosa, with an increase in normal discharge.

References:

Burstein GR, Murray PJ. Diagnosis and management of sexually transmitted disease pathogensamong adolescents. Pediatr Rev. 2003;24:75-82. Available at:http://pedsinreview.aappublications.org/cgi/content/full/24/3/75

Burstein GR, Murray PJ. Diagnosis and management of sexually transmitted diseases amongadolescents. Pediatr Rev. 2003;24:119-127. Available at:http://pedsinreview.aappublications.org/cgi/content/full/24/4/119

O'Brien G, Serwint JR. In brief: bacterial vaginosis. Pediatr Rev. 2008;29:209-211. Available at:http://pedsinreview.aappublications.org/cgi/content/full/29/6/209

Copyright © 2010 by the American Academy of Pediatrics page 301

Page 302: AAP PREP 2010

2010 PREP SA on CD-ROM

Owen MK, Clenney TL. Management of vaginitis. Am Fam Physician. 2004;70:2125-2132.Available at: http://www.aafp.org/afp/20041201/2125.html

Rosen DS. Physiologic growth and development during adolescence. Pediatr Rev. 2004;25:194-200. Available at: http://pedsinreview.aappublications.org/cgi/content/full/25/6/194

Copyright © 2010 by the American Academy of Pediatrics page 302

Page 303: AAP PREP 2010

2010 PREP SA on CD-ROM

Question: 121

You are evaluating a 6-month-old girl in the pediatric intensive care unit who is being treated forcongestive heart failure due to a presumed viral myocarditis. On physical examination, her heartrate is 170 beats/min, respiratory rate is 50 breaths/min, and blood pressure is 75/40 mm Hg.She is currently receiving 3 L/min of oxygen via nasal cannula and has an oxygen saturation of92%. The infant appears alert, exhibits diaphoresis, has decreased aeration bilaterally at thelung bases, and has a fourth heart sound. Her liver is palpable 5 cm below her right costalmargin, and her peripheral pulses are diminished. She is receiving milrinone and furosemide

infusions. Results of laboratory studies from 2 hours ago include:

•Serum sodium, 140 mEg/L (140 mmol/L)

•Serum potassium, 4 mEq/L (4 mmol/L)

•Serum chloride, 105 mEq/L (105 mmol/L)

•Serum albumin, 4.0 g/dL (40 g/L)

•Hematocrit, 45.0% (0.45)

•Blood urea nitrogen, 20.0 mg/dL (7.1 mmol/L)•Serum creatinine, 0.6 mg/dL (53.0 mcmol/L)

Of the following, the MOST appropriate treatment to improve this child’s congestive heart failureis

A. administration of 15 mL/kg of 5% albumin

B. discontinuation of the furosemide infusion

C. increase in oxygen administration to 4 L/min

D. initiation of continuous positive airway pressure

E. transfusion with 15 mL/kg packed red blood cells

Copyright © 2010 by the American Academy of Pediatrics page 303

Page 304: AAP PREP 2010

2010 PREP SA on CD-ROM

Preferred Response: DCritique: 121

Pulmonary edema results from a variety of pulmonary and cardiac disease processes thatincrease pulmonary capillary pressure, decrease pulmonary capillary integrity, or decreaseplasma oncotic pressure. Each of these processes results in fluid outflow from the pulmonarycapillaries into the pulmonary interstitium. Increased pulmonary capillary pressure usually resultsfrom cardiac failure (as is the case for the infant in the vignette) or obstruction to flow bystenotic pulmonary veins or tumor compression. Increased capillary permeability typically is seenin sepsis, pneumonia, acute respiratory distress syndrome, near-drowning, or toxin exposure.Decreased oncotic pressure generally results from hypoalbuminemic states, as may occur inrenal or liver disease.

Treatment of pulmonary edema should be directed at addressing the underlying diseaseprocess, but general measures to stabilize the patient’s respiratory status and reduce pulmonaryfluid also are crucial. The administration of positive end-expiratory pressure (PEEP) has beendemonstrated to be very effective in the management of pulmonary edema due to bothpulmonary and cardiac diseases. PEEP decreases the work of breathing and improvespulmonary function by reopening and maintaining the patency of alveolar units. This reducespulmonary resistance rather than forcing fluid from the interstitium. Severe cases of pulmonaryedema likely require positive inspiratory pressures as well.

Supplemental oxygen should be administered, both to increase alveolar oxygen tension andto decrease pulmonary vasoconstriction. However, small increases in supplemental oxygen (eg,from 3 to 4 L/min) increase alveolar oxygen tension only slightly. Diuretics are helpful formanagement of pulmonary edema and, therefore, should not be discontinued. For patients whohave hypoalbuminemia, administration of albumin may restore normal oncotic pressure.Transfusion of packed red blood cells would only be of benefit if the patient was anemic.

References:

Bahk T, Mazor R, Green TP. Pulmonary edema. In: Kliegman RM, Behrman RE, Jenson HB,Stanton BF, eds. Nelson Textbook of Pediatrics. 18th ed. Philadelphia, Pa: Saunders Elsevier;2007:1787-1788

Caples SM, Gay PC. Noninvasive positive pressure ventilation in the intensive care unit: aconcise review. Crit Care Med. 2005;33:2651-2658. Abstract available at:http://www.ncbi.nlm.nih.gov/pubmed/16276193

Chase MA, Wheeler, DS. Disorders of the pediatric chest. In: Wheeler DS, Wong HR, Shanley TP,eds. Pediatric Critical Care Medicine: Basic Science and Clinical Evidence. New York, NY:Springer-Verlag London Limited; 2007:361-375

Copyright © 2010 by the American Academy of Pediatrics page 304

Page 305: AAP PREP 2010

2010 PREP SA on CD-ROM

Question: 122

An anxious mother tells you that following pituitary surgery, her husband has started using atestosterone gel applied to the skin on his abdomen daily. She has read that this medication mightaffect her 2-year-old daughter and 4-year-old son adversely and wonders how best to protectthem.

Of the following, the BEST advice is for the father to

A. allow the gel to dry on the skin, get dressed, and wash his hands with soap and waterbefore touching the children

B. clean his hands and the area of application with an alcohol-containing cleanser beforetouching the children

C. have limited contact with the girl, but the boy should not experience any harmful effects

D. make the room in which he applies the gel off-limits to the children

E. shower immediately after applying the gel so that the preparation does not contaminate hischildren

Copyright © 2010 by the American Academy of Pediatrics page 305

Page 306: AAP PREP 2010

2010 PREP SA on CD-ROM

Preferred Response: ACritique: 122

Testosterone gels applied to the skin to maintain normal testosterone concentrations in menwho have hypogonadism have been reported to cause virilization in family members if usedincorrectly. After applying the gel to an area of skin that will be covered with clothing, the usershould wash the hands with soap and water to remove any residual testosterone. An individualusing the preparation should wait 5 to 6 hours before showering to allow full cutaneousabsorption. If direct skin-to-skin contact with another individual is anticipated, the area ofapplication should be washed thoroughly. Soap and water or alcohol will remove thetestosterone from the skin. Unless the testosterone gel has been applied to room surfaces andanother individual comes into contact with the surfaces, there is no need to limit room access.Both boys and girls can develop masculinization after exposure to testosterone gels, so nochildren should have inappropriate exposure to these potent medications.

References:

Androgel® (testosterone gel 1%) prescribing information. In: Physicians’ Desk Reference. 63rded. Montvale, NJ: Physicians’ Desk Reference Inc; 2008

Brachet C, Vermeulen J, Heinrichs C. Children's virilization and the use of a testosterone gel bytheir fathers. Eur J Pediatr. 2005;164:646-647. Extract available at:http://www.springerlink.com/content/r925428mr2473215/

Kunz GJ, Klein KO, Clemons RD, Gottschalk ME, Jones KL. Virilization of young children aftertopical androgen use by their parents. Pediatrics. 2004;114:282-284. Available at:http://pediatrics.aappublications.org/cgi/content/full/114/1/282

Testim® (teststerone gel 1%) prescribing information. In: Physicians’ Desk Reference. 63rd ed.Montvale, NJ: Physicians’ Desk Reference Inc: 2008

Copyright © 2010 by the American Academy of Pediatrics page 306

Page 307: AAP PREP 2010

2010 PREP SA on CD-ROM

Question: 123

The parents of a newborn in whom a congenital severe-profound hearing loss has beendiagnosed are seeking guidance about how to promote their infant’s language development. Yourecommend enrollment in an early intervention program and obtaining hearing aids when theinfant is young. His parents have never been exposed to an individual who has a hearing loss.They ask your opinion on the best approach for him to learn language.

Of the following, the MOST important factor in language development of an infant or young childwho has hearing loss is the use of

A. a high amount of verbal/nonverbal (gestures) communication between parents and child

B. an oral-aural method emphasizing the teaching of speech and the use of a child’s residualhearing

C. hand-cued speech (using combined speech and hand cues)

D. manually coded English between parents and child

E. the bilingual (ASL and English)-bicultural (hearing and deaf culture) approach

Copyright © 2010 by the American Academy of Pediatrics page 307

Page 308: AAP PREP 2010

2010 PREP SA on CD-ROM

Preferred Response: ACritique: 123

Multiple factors contribute to a family’s decisions regarding their child who has confirmedhearing loss. Language-based early intervention results in superior expressive and receptivecommunication abilities compared with those of children identified at later stages of development.Family involvement, including verbal and nonverbal (gestures) communication, has a moresignificant positive effect on language development than any specific type of intervention.

Hand-cued speech (www.cuedspeech.org) consists of eight different handshapes(represent consonants) and four different hand locations around the speaker’s face (representvowels). Manually coded English employs a visual representation of the spoken Englishlanguage. Signs and finger spelling are used to represent spoken English.

The goal of the oral-aural method of communication is to develop spoken language and gaininclusion in the mainstream both in school and society. People who employ this auditory-oralapproach use their aided residual hearing as well as reading of speech, facial expressions, andnaturally occurring gestures. Such an approach requires consistent use of hearing aids/FMtechnology and provision of auditory training and speech therapy as well as lip-reading.

The bilingual-bicultural (Bi-Bi) philosophy of the National Association of the Deaf(www.nad.org) promotes communication in two languages (visual and a form of spoken) forchildren who are deaf so they can be a part of both deaf and hearing communities. The Bi-Biapproach supports early language learning through American Sign Language (ASL) and a formof spoken English taught as a second language later in elementary school.

Of note, children who have hearing loss of less than 90 dB usually benefit fromconventional amplification systems. For children who have severe-to-profound sensorineuralhearing loss affecting both ears, cochlear implantation may be considered. This procedure leadsto the ability to perceive sound signals. Although results are highly variable, the procedure ismost successful when undertaken early in life.

References:

Feldman HM. Evaluation and management of language and speech disorders in preschoolchildren. Pediatr Rev. 2005;26:131-141. Available at:http://pedsinreview.aappublications.org/cgi/content/full/26/4/131

Gravel JS, O’Gara J Communication options for children with hearing loss. Ment Retard DevDisabil Res Rev. 2003;9:243-251. Abstract available at:http://www.ncbi.nlm.nih.gov/pubmed/14648817

Copyright © 2010 by the American Academy of Pediatrics page 308

Page 309: AAP PREP 2010

2010 PREP SA on CD-ROM

Question: 124

A 14-year-old boy who has cystic fibrosis presents with an acute pulmonary exacerbationinvolving tachypnea, chest wall retractions, and decreased oxygen saturation from baseline.You order a sputum culture.

Of the following, the MOST appropriate choice for empiric antimicrobial therapy pending theculture results is

A. azithromycin and cefuroxime

B. clindamycin and cefotaxime

C. piperacillin/tazobactam and gentamicin

D. trimethoprim-sulfamethoxazole

E. vancomycin

Copyright © 2010 by the American Academy of Pediatrics page 309

Page 310: AAP PREP 2010

2010 PREP SA on CD-ROM

Preferred Response: CCritique: 124

Cystic fibrosis (CF) is an inherited disorder characterized by a defect in the cystic fibrosistransmembrane conductance regulator (CFTR) gene that leads to production of thick, viscousmucus and recurrent severe pulmonary infections. Although initial pulmonary infections havebeen associated with Staphylococcus aureus in patients who have CF, beyond 1 to 2 years ofage, Pseudomonas can be isolated. Over time, the organism persists and transitions to a mucoidphenotype that is associated with progressive deterioration in pulmonary function andintermittent acute deteriorations. Therefore, empiric therapy of a pulmonary exacerbation of CF,as described for the boy in the vignette, must include coverage for Pseudomonas, such as thecombination of piperacillin/tazobactam and gentamicin. The combination of azithromycin andcefuroxime provides excellent coverage for the usual bacterial pathogens (primarilypneumococcus and mycoplasma) seen in pneumonia in otherwise healthy hosts at this age.Clindamycin and cefotaxime might be considered in the severely ill child who has pneumonia thatpotentially is caused by penicillin-resistant pneumococci, staphylococci, and Haemophilus.Trimethoprim-sulfamethoxazole does not have significant activity against Pseudomonas orpneumococcus, making it an inadequate choice for this patient or an otherwise healthy personwho has bacterial pneumonia. Vancomycin only has activity against gram-positive organisms.Ciprofloxacin and possibly levofloxacin are the only quinolones that have significantantipseudomonal activity. Ciprofloxacin has been used as an oral alternative in pulmonaryexacerbations for those who have CF. Carbapenems (imipenem and meropenem) andceftazidime are other antipseudomonal antibiotics that might be considered for treatment in thissetting. Isolates should be processed for antimicrobial sensitivities because resistance canoccur to even the previously mentioned antipseudomonal antibiotics.

P aeruginosa is a gram-negative organism that is common in the environment. It even can becultured from distilled water. Infections in immunocompetent hosts are rare, althoughPseudomonas has been associated with hot tub folliculitis and ocular infections fromcontaminated contact lens solution. Pseudomonas also can be isolated in necrotic cartilage frompatients who have puncture wound osteomyelitis.

In immunocompromised patients, Pseudomonas can cause sepsis or ecthyma gangrenosum.It also has been associated with hospital-acquired infections in debilitated patients, such asventilator-associated pneumonia or infections in burn patients. In these settings, the infection isdue to direct invasion by the organism and related bacterial virulence factors. The host’sinflammatory response to the organism and decreased mucociliary clearance in CF appear to becritical in the pathophysiology of pneumonia in this condition.

References:

American Academy of Pediatrics. Fluoroquinolones. In: Pickering LK, Baker CJ, Kimberlin DW,Long SS, eds. Red Book: 2009 Report of the Committee on Infectious Diseases. 28th ed. ElkGrove Village, Ill: American Academy of Pediatrics; 2009:737-738

Li Z, Kosorok MR, Farrell PM, et al. Longitudinal development of mucoid Pseudomonasaeruginosa infection and lung disease progression in cystic fibrosis. JAMA. 2005;293:581-588.

Copyright © 2010 by the American Academy of Pediatrics page 310

Page 311: AAP PREP 2010

2010 PREP SA on CD-ROM

Available at: http://jama.ama-assn.org/cgi/content/full/293/5/581

Ramphal R. Epidemiology and pathogenesis of Pseudomonas aeruginosa infection. UpToDateOnline 16.3. 2007. Available for subscription at:www.utdol.com/online/content/topic.do?topicKey=gram_rod/7523&view=print

Copyright © 2010 by the American Academy of Pediatrics page 311

Page 312: AAP PREP 2010

2010 PREP SA on CD-ROM

Question: 125

You are evaluating a 2-year-old boy who has a 10-hour history of a temperature to 40.0°C andprogressively worsening diarrhea. Yesterday he attended a birthday party that was held in apetting zoo, but he has no other history of ill contacts or unusual exposures. His mother statesthat he has had eight watery bowel movements with mucus and streaks of blood in the last 10hours. On physical examination, the little boy is irritable and has a temperature of 39.5°C. Hismucous membranes are slightly tacky, and his abdomen is diffusely tender to palpation. The restof the physical examination findings are within normal limits. Laboratory data include a peripheral

white blood cell count of 16.0x103/mcL (16.0x109/L), with 65% neutrophils, 9% band forms, and26% lymphocytes. Microscopic examination of stool shows fecal leukocytes, mucus, and blood.

Of the following, the MOST likely etiologic agent for this patient’s condition is

A. Campylobacter

B. Escherichia coli

C. Salmonella

D. Shigella

E. Yersinia enterocolitica

Copyright © 2010 by the American Academy of Pediatrics page 312

Page 313: AAP PREP 2010

2010 PREP SA on CD-ROM

Preferred Response: CCritique: 125

The child described in the vignette most likely acquired his infection at the birthday party thathe attended at the petting zoo. In this case, infection with Salmonella is the most likely cause ofhis illness. Salmonella organisms are gram-negative bacteria that belong to theEnterobacteriaceae family. The principal reservoirs for nontyphoidal Salmonella organismsinclude poultry, livestock, reptiles, and pets. The major vehicle of transmission is food of animalorigin (eg, poultry, beef, eggs, and dairy products) or other foods (eg, fruits, vegetables, andbakery products) that have been contaminated by contact with an infected animal product orhuman. Other modes of transmission include ingestion of contaminated water; contact withinfected reptiles or amphibians and possibly rodents; and exposure to contaminated medications,dyes, and medical instruments. In contrast to nontyphoidal Salmonella serotypes, S typhi isfound only in humans, and infection occurs through direct contact with an infected person orwith an item contaminated by a carrier.

Age-specific attack rates for Salmonella infection are highest in children 1 to 4 years ofage. Infections with nontyphoidal Salmonella sp can be asymptomatic or patients may havegastroenteritis, bacteremia, or focal infections (eg, osteomyelitis). The most common illnessassociated with nontyphoidal Salmonella infection is gastroenteritis. The abrupt onset ofnausea, vomiting, abdominal cramps, and fever associated with moderate-to-severe waterydiarrhea are the most common manifestations. The incubation period for gastroenteritis rangesfrom 6 to 72 hours. Examination of the stool typically reveals the presence of polymorphonuclearleukocytes, blood, and mucus. Patients commonly have leukocytosis on a complete blood countassociated with a left shift. The symptoms may persist for 2 to 10 days, and treatment issupportive. After resolution of gastroenteritis, the mean duration of carriage of nontyphoidalSalmonella in the stool is 4 to 5 weeks.

Although Campylobacter sp, Escherichia coli, Shigella sp, and Yersinia enterocolitica maycause gastroenteritis with similar symptoms to those of Salmonella infection, the incubationperiod and other factors differ. For example, the incubation period for Campylobacter sp is daysinstead of hours, and the abdominal pain mimics that of appendicitis or intussusception. Theincubation period for E coli gastroenteritis is 2 to 3 days and that for Y enterocolitica is 4 to 6days. Shigella sp infection has an incubation period of 2 to 4 days and is spread from person-to-person because humans are the natural host for the organism.

References:

American Academy of Pediatrics. Salmonella infections. In: Pickering LK, Baker CJ, KimberlinDW, Long SS, eds. Red Book: 2009 Report of the Committee on Infectious Diseases. 28th ed.Elk Grove Village, Ill: American Academy of Pediatrics; 2009:584-589

From the Centers for Disease Control. Outbreak of Salmonella enteritidis infection associatedwith the consumption of raw shell eggs. JAMA. 1992;267:3263-3264

Guerrant RL, Lima AAM. Inflammatory enteritides. In: Mandell GL, Bennett JE, Dolan R, eds.Mandell, Douglas, and Bennett’s Principles and Practice of Infectious Diseases. 6th ed.

Copyright © 2010 by the American Academy of Pediatrics page 313

Page 314: AAP PREP 2010

2010 PREP SA on CD-ROM

Philadelphia, Pa: Elsevier Churchill Livingstone; 2005:1263-1272

Pegues DA, Ohl ME, Miller SI. Salmonella species, including Salmonella typhi. In: Mandell GL,Bennett JE, Dolan R, eds. Mandell, Douglas, and Bennett’s Principles and Practice of InfectiousDiseases. 6th ed. Philadelphia, Pa: Elsevier Churchill Livingstone; 2005:2636-2654

Copyright © 2010 by the American Academy of Pediatrics page 314

Page 315: AAP PREP 2010

2010 PREP SA on CD-ROM

Question: 126

You are seeing for the first time an 8-year-old boy who has end-stage renal disease and hasbeen receiving chronic outpatient dialysis for the past 4 months. His family has just relocated toyour town. According to the boy and his parents, he is doing well except for some decreasedenergy. He has missed taking some of his medications due to the process of moving. Onphysical examination, the boy’s weight and height are at the 25th percentile, his temperature is37.0°C, heart rate is 96 beats/min, respiratory rate is 16 breaths/min, and blood pressure is110/66 mm Hg. He appears alert and slightly pale but in no acute distress. Notable findingsinclude pale conjunctivae, a II/VI systolic flow murmur at the left lower sternal border, aperitoneal catheter in place, and no edema.

Of the following, the MOST likely cause for his symptoms is

A. erythropoietin deficiency

B. fluid overload

C. hyperphosphatemia

D. hypocalcemia

E. pericardial effusion

Copyright © 2010 by the American Academy of Pediatrics page 315

Page 316: AAP PREP 2010

2010 PREP SA on CD-ROM

Preferred Response: ACritique: 126

The child described in the vignette has end-stage renal disease, the most advanced form ofchronic kidney disease (CKD) that, by definition, is dialysis-dependent. The patient’s decreasedenergy level, tachycardia, pale conjunctivae, and grade II/VI flow murmur are due to anemia that,in this setting, is caused by either erythropoietin deficiency, iron deficiency, or both.Hypertension would be expected in dialysis patients who experience fluid overload, but paleconjunctivae or a flow murmur are not usual. Hyperphosphatemia can cause pruritus andoccasionally injected bulbar conjunctivae (from calcium phosphorus precipitation in the vesselwall). However, the other findings would not be seen. Hypocalcemia may result in symptoms ofneurologic irritability with hyperreflexia on examination. A pericardial effusion might have afriction rub, but should not result in pale conjunctivae.

Anemia in children who have CKD is being managed more effectively with the advent ofrecombinant human erythropoietin (rhEPO) therapy in 1989. Since rhEPO therapy has beenintroduced, the need for blood transfusions in stable dialysis patients has nearly vanished,greatly reducing exposure to the infectious risks of blood transfusions. Moreover, rhEPO usereduces the risk of human leukocyte antigen (HLA) sensitization due to blood transfusions. HLAsensitization can lengthen waiting times on the transplant list for children in need of renaltransplantation. For children who have advanced CKD not treated with hemodialysis, rhEPO isadministered one to three times per week via subcutaneous injection. In contrast, childrenundergoing hemodialysis usually receive rhEPO intravenously through their hemodialysis circuit.A newer form of erythropoietin, darbepoetin alfa, has a longer half-life and requires dosing onlytwo to three times monthly. Risks of erythropoietin therapies are polycythemia and its associatedproblems such as hypertension and thrombosis. Children who have CKD have poor enteralabsorption of iron that is exacerbated by concomitant administration of phosphorus binders.Although oral iron therapy generally is prescribed, parenteral iron may be required for patientswhose iron deficiency anemia is refractory. Newer forms of parenteral iron have beendeveloped and include sodium ferric gluconate complex, which was studied in children 6 to 15years of age and found to be safe and effective. The target hemoglobin values in CKD rangefrom 11 to 12 g/dL (110 to 120 g/L). Normalization of hemoglobin concentrations results inimproved energy levels and cognition, reduced cardiac morbidity, and lower hospitalizationrates.

References:

Berns JS. Anemia of chronic kidney disease: target hemoglobin/hematocrit for patients treatedwith erythropoietic agents. UpToDate Online 16.3. 2008. Available for subscription at:http://www.utdol.com/online/content/topic.do?topicKey=dialysis/36521&selectedTitle=5~150&source=search_result630

Warady BA, Seligman PA, Dahl NV. Single-dosage pharmacokinetics of sodium ferric gluconatecomplex in iron-deficient pediatric hemodialysis patients. Clin J Am Soc Nephrol. 2007;2:1140-1146. Available at: http://cjasn.asnjournals.org/cgi/content/full/2/6/1140

Copyright © 2010 by the American Academy of Pediatrics page 316

Page 317: AAP PREP 2010

2010 PREP SA on CD-ROM

Whyte DA, Fine RN. Chronic kidney disease in children. Pediatr Rev. 2008;29:335-341. Availableat: http://pedsinreview.aappublications.org/cgi/content/full/29/10/335

Wong CS, Mak RH. Chronic kidney disease. In: Kher KK, Schnaper HW, Makker SP, eds. ClinicalPediatric Nephrology. 2nd ed. London, England: Informa Healthcare; 2007:339-352

Copyright © 2010 by the American Academy of Pediatrics page 317

Page 318: AAP PREP 2010

2010 PREP SA on CD-ROM

Question: 127

A mother brings in her 2-year-old son because she thinks he has asthma and needs an inhaler.A chart review shows that the boy has had three "wheezing" episodes over the past 12 monthssince attending child care. Each episode seems to be associated with viral upper respiratorytract infections, with one episode resulting in the child being placed on a 5-day course of oralsteroids.

Of the following, the BEST statement regarding the risk for this infant developing persistentasthma is that

A. the risk is decreased if the infant has a diagnosis of atopic dermatitis

B. the risk is decreased if the mother avoided ingestion of hyperallergenic foods duringpregnancy

C. the risk is increased because the wheezing episodes occurred prior to 2 years of age ratherthan after age 6 years

D. the risk is increased if the infant required hospitalization for respiratory syncytial virusbronchiolitis

E. sensitization to aeroallergens within the first postnatal year does not increase the risk forpersistent asthma

Copyright © 2010 by the American Academy of Pediatrics page 318

Page 319: AAP PREP 2010

2010 PREP SA on CD-ROM

Preferred Response: DCritique: 127

During the past 4 decades, there has been a substantial increase in the incidence of atopy(ie, atopic dermatitis, food allergy, asthma, and allergic rhinitis). A history of atopy in a first-degree relative or parent is clearly the greatest risk factor for atopy. However, nongeneticfactors such as infections, child care exposure, antibiotics, food choices, and maternal dietarychoices have been studied intensely to determine why some "at-risk" infants, such as thosewho have parents who have atopy, eventually develop atopic disorders while others do not.

Although the atopy status of the parents of the boy in the vignette is unknown, the multiplewheezing episodes associated with viral upper respiratory tract infections that the boyexperiences suggest lower airway inflammation, a hallmark of asthma. One landmark study, theTucson Children’s Respiratory Group Study, enrolled children at birth and now has followedthem for more than 15 years, assessing the risk factors associated with development of atopicdisorders. Regarding asthma, investigators identified three primary groups of patients: earlytransient wheezers, persistent wheezers, and late wheezers. Almost 80% of the earlywheezers (ie, wheezing associated with a viral illness prior to age 3 years) did not developpersistent asthma. Late wheezers (ie, wheezing episodes occurring after age 6 years) hadincreased risk for persistent asthma. However, if the infant was male and had wheezing duringa respiratory syncytial virus (RSV) infection, he would have a 15-fold increased risk ofdeveloping later asthma. Other risk factors identified for later asthma included positive skin teststo aeroallergens prior to age 6 years, development of atopic dermatitis (the initial step on the so-called "allergic march"), and increased total serum immunoglobulin E concentrations.

The role of maternal diet and atopy risk is still controversial. In the past, mothers wereadvised to avoid highly allergenic foods such as peanuts and tree nuts during pregnancy.Current evidence suggests that maternal dietary restriction of highly allergenic foods does notplay a role in prevention of atopic diseases. Further, in utero exposure to these foods actuallymay provide immunologic tolerance. In 2008, the American Academy of Pediatrics recommendedthat mothers be allowed to eat a regular healthy diet without specific food restriction.

Other general observations from meta-analysis and controlled studies regarding the

development of atopic disorders include:•Breastfeeding for 3 months, compared with ingestion of cow milk formula, afforded a

decreased risk for atopic dermatitis in "at-risk" infants•Exclusive breastfeeding may decrease wheezing episodes for children younger than 4

years of age but may not affect asthma risk in children older than 6 years of age•Delaying the introduction of solid foods past 4 to 6 months, perhaps even highly allergic

ones such as fish, peanut butter, and eggs, does not appear to have a protective effecton development of atopic disease

References:

Greer FR, Sicherer SH, Burks AW and the Committee on Nutrition and Section on Allergy andImmunology. Effects of early nutritional interventions on the development of atopic disease ininfants and children: the role of maternal dietary restriction, breastfeeding, timing of introductionof complementary foods, and hydrolyzed formulas. Pediatrics. 2008;121:183-191. Available at:

Copyright © 2010 by the American Academy of Pediatrics page 319

Page 320: AAP PREP 2010

2010 PREP SA on CD-ROM

http://pediatrics.aappublications.org/cgi/content/full/121/1/183

Guilbert TW, Morgan WJ, Zeiger RS, et al. Atopic characteristics of children with recurrentwheezing at high risk for the development of childhood asthma. J Allergy Clin Immunol.2004;114:1282—1287. Abstract available at: http://www.ncbi.nlm.nih.gov/pubmed/15577824

Guill MF. Asthma update: epidemiology and pathophysiology. Pediatr Rev. 2004;25:299-305.Available at: http://pedsinreview.aappublications.org/cgi/content/full/25/9/299

Leung DYM. Atopic dermatitis (atopic eczema). In: Kliegman RM, Behrman RE, Jenson HB,Stanton BF, eds. Nelson Textbook of Pediatrics. 18th ed. Philadelphia, Pa: Saunders Elsevier;2007:970-974

Liu AH, Covar RA, Spahn JD, Leung DYM. Childhood asthma. In: Kliegman RM, Behrman RE,Jenson HB, Stanton BF, eds. Nelson Textbook of Pediatrics. 18th ed. Philadelphia, Pa: SaundersElsevier; 2007:953-969

Taussig LM, Wright AL, Holberg CJ, Halonen M, Morgan WJ, Martinez FD. Tuscon Children’sRespiratory Study: 1980 to present. J Allergy Clin Immunol. 2003;111:661-676. Abstractavailable at: http://www.ncbi.nlm.nih.gov/pubmed/12704342

Copyright © 2010 by the American Academy of Pediatrics page 320

Page 321: AAP PREP 2010

2010 PREP SA on CD-ROM

Question: 128

A 16-year-old girl comes to your office after sustaining an acute left knee injury 3 days agoduring a soccer game. She reports that as she turned while running down field, an opposingplayer struck her right knee. She felt immediate pain and the sensation of knee instability. Shewas unable to continue playing. On physical examination, you note moderate swelling andtenderness over the medial aspect of her right knee. Valgus stress testing reveals a 5-mmopening at the joint line with associated pain. She has 4/5 strength for resisted straight leg raise.There is no evidence of anterior or posterior cruciate ligament instability. She has a mildlyantalgic gait and feels as if the leg is going to give out when she turns to the right. She wants toknow if she can play next weekend.

Of the following, the MOST appropriate criterion for her return to sports participation is

A. complete resolution of swelling

B. nearly full strength in the knee

C. normal findings on magnetic resonance imaging of the knee

D. range of motion returned to 75% of normal

E. 3-mm opening at the joint line on valgus stress testing

Copyright © 2010 by the American Academy of Pediatrics page 321

Page 322: AAP PREP 2010

2010 PREP SA on CD-ROM

Preferred Response: BCritique: 128

Although no empiric evidence is available to guide the decision of when to allow an injuredpediatric athlete to return to play, expert consensus guidelines have been developed by anumber of professional sports medicine societies, including the American College of SportsMedicine and the American Orthopedic Society for Sports Medicine. Return to play before aninjury is healed and rehabilitated completely places the athlete at risk of additional injury orreinjury, and the guidelines are intended to prevent such occurrences. The guidelines suggestthat the athlete should be free of pain and swelling during and after exercise; have full range ofmotion, flexibility, and stability of the injured extremity; and have return of 95% of normalstrength. In addition, they highly recommend a stepwise return to competition, with gradualincreases in duration and intensity of practice as the injury heals. Finally, it is critical that theathlete regains the ability to perform sport-specific maneuvers (eg, running, cutting and kicking insoccer) before returning to play.

The patient described in the vignette has sustained a medial collateral ligament sprain andshould be advised that she can return to play when the strength in her right knee is nearlyequivalent to that of the left. Swelling usually resolves within 72 hours and precedes return ofrange of motion and flexibility by almost 2 weeks. Therefore, it is not, by itself, a useful criterionfor return to competition. The decision to return an athlete to competition is based on functionalparameters, not solely on the results of radiographic studies. Any ligamentous laxity in the knee(as would be indicated by abnormal valgus stress testing) is an indication that the ligaments arenot completely healed and that full ligamentous strength has not returned.

References:

Landry GL. Management of musculoskeletal injury. In: Kliegman RM, Behrman RE, Jenson HB,Stanton BF, eds. Nelson Textbook of Pediatrics. 18th ed. Philadelphia, Pa: Saunders Elsevier;2007:2851-2861

Magee LM. Return to Play: A Common Sense Guide for Coaches. Indianapolis, Ind: AmericanSociety of Sports Medicine; Available at:http://www.acsm.org/AM/Template.cfm?Section=Brochures2&Template=/CM/ContentDisplay.cfm&ContentID=1532

Pearsall AW IV. Return to Play. Rosemont, Ill: American Orthopaedic Society of Sports Medicine;2006. Available at: http://mdsmc.com/pdf/Return-To-Play.pdf

Copyright © 2010 by the American Academy of Pediatrics page 322

Page 323: AAP PREP 2010

2010 PREP SA on CD-ROM

Question: 129

A 5-month-old infant who has been fed cow milk protein-based formula presents to theemergency department following 2 days of watery diarrhea that she has passed up to 10 timesper day. She weighed 3,100 g when born at term and has been gaining weight and growingnormally. Her weight at the 4-month visit was 5.5 kg. On physical examination today, her weightis 5.8 kg, temperature is 37.7°C, and heart rate is 140 beats/min. She appears alert, her mucousmembranes are dry, and skin recoil is less than 2 seconds. Capillary refill is slightly prolonged.The remainder of the physical examination findings are unremarkable.

Of the following, the MOST appropriate immediate treatment is

A. intravenous hydration with an initial infusion of 120 mL 0.9% sodium chloride over 2 hours

B. intravenous hydration with 5% dextrose and 0.33% sodium chloride at 40 mL/hr

C. oral feedings with half-strength soy-based formula, mixed 1:1 with a glucose-electrolytesolution

D. oral rehydration with a solution containing 20 mEq/L sodium chloride

E. oral rehydration with a solution containing 75 mEq/L sodium chloride

Copyright © 2010 by the American Academy of Pediatrics page 323

Page 324: AAP PREP 2010

2010 PREP SA on CD-ROM

Preferred Response: ECritique: 129

The primary care clinician frequently encounters acute gastroenteritis, particularly during thewinter rotavirus season. The mild tachycardia, dry mucous membranes, prolonged capillary refill,and estimated preillness weight of 6.3 kg (based on a 20-g/day weight gain documented frombirth to 4 months of age) suggest that the infant described in the vignette is moderately (5% to7%) dehydrated. This corresponds to a 50- to 90-mL/kg fluid deficit. Accordingly, rehydrationtherapy should be instituted with an oral glucose-electrolyte solution containing 75 to 90 mEq/L(75 to 90 mmol/L) sodium chloride.

In the United States, acute gastroenteritis is a major cause of morbidity, accounting for morethan 1.5 million outpatient visits, 200,000 hospitalizations, and approximately 300 deaths peryear. Direct medical costs for rotavirus-induced diarrhea, representing one third of allhospitalizations for diarrhea among United States children younger than 5 years of age, areestimated to be $250 million/year. Worldwide, particularly in developing countries whereepidemic diarrheal illness is a major cause of infant and childhood mortality, oral rehydrationtherapy (ORT) has reduced mortality rates significantly. In developed countries and particularlyin the United States, the implementation of ORT has lagged well behind its use in the developingworld, perhaps owing to a habitual and ingrained practice of using intravenous therapy tomanage all levels of dehydration.

Recommendations for ORT content were developed by the World Health Organization,based on the need to provide therapy for moderate-to-severe dehydration caused by bothosmotic and secretory (eg, choleric) diarrhea. All oral rehydration products must contain glucosebecause the primary mechanism responsible for water and electrolyte repletion involves theprocess of coupled sodium-glucose cotransport across the microvillus membrane (MVM) of thesmall intestinal epithelial cell. Facilitated uptake of one glucose molecule across the MVM,accompanied by two sodium molecules for each molecule of glucose, is driven by a downhill

sodium gradient that is maintained by the action of the basolateral membrane Na+ - K+ - ATPasepump. The sodium content of the fluid used for oral rehydration of moderately dehydrated infantsshould be approximately 75 mEq/L. This sodium concentration differs from the widely availableproprietary maintenance solutions, which contain 40 to 50 mEq/L.

ORT should commence by providing 40 to 50 mL/kg over the initial 1 to 4 hours of therapy.Oral rehydration even may be instituted successfully in more than 90% of vomiting infants byusing dropper or spoon feeding. Once adequate perfusion has been restored, indicated byresolution of tachycardia and normalization of capillary refill time, ORT may proceed using astandard maintenance hydration fluid that contains 40 to 50 mEq/L (40 to 50 mmol/L) sodium.Such a standard solution may be used as primary ORT for infants who present with signs of amild illness (ie, normal heart rate and capillary refill), corresponding to a deficit of less than 50mL/kg (ie, <5% dehydration). ORT for dehydrated infants should not include fluids that havesodium contents of significantly less than 40 mEq/L (40 mmol/L). For example, most over-the-counter sports drinks, fruit juices, and carbonated beverages contain too little sodium or toomuch sugar to promote fluid and electrolyte repletion. When treating diarrheal illness, the sodiumcontent (and the sodium-to-glucose ratio) of ORT solutions must be sufficient to drive intestinalglucose-sodium cotransport and its associated water uptake. Oral hydration management usinginfant formula (either full-strength or diluted) should not be attempted for any child who has a

Copyright © 2010 by the American Academy of Pediatrics page 324

Page 325: AAP PREP 2010

2010 PREP SA on CD-ROM

clinically significant fluid deficit, regardless of the formula’s component carbohydrate. Becausethe infant described in the vignette is estimated to have 5% to 7% dehydration, intravenoushydration is not indicated.

References:

Duggan C, Lasche J, McCarty M, et al. Oral rehydration solution for acute diarrhea preventssubsequent unscheduled follow-up visits. Pediatrics. 1999;104:e29. Available at:http://pediatrics.aappublications.org/cgi/content/full/104/3/e29

Finberg L. Dehydration in infancy and childhood. Pediatr Rev. 2002;23:277-282. Available at:http://pedsinreview.aappublications.org/cgi/content/full/23/8/277

King CK, Glass R, Bresee J, Duggan C; Centers for Disease Control and Prevention. Managingacute gastroenteritis among children. MMWR Recomm Rep. 2003;52(RR16):1-16. Available at:http://www.cdc.gov/mmwr/preview/mmwrhtml/rr5216a1.htm

Nazarian LF. A synopsis of the American Academy of Pediatrics’ practice parameter on themanagement of acute gastroenteritis in young children. Pediatr Rev. 1997; 18:221-223. Availableat: http://pedsinreview.aappublications.org/cgi/content/full/18/7/221

Copyright © 2010 by the American Academy of Pediatrics page 325

Page 326: AAP PREP 2010

2010 PREP SA on CD-ROM

Question: 130

You are called to the operative delivery of a 42-weeks’ gestation male following a pregnancycomplicated by oligohydramnios and poor fetal growth. Meconium-stained amniotic fluid wasnoted upon artificial rupture of membranes. Fetal bradycardia resulted in a decision for cesareandelivery. Resuscitation of the infant requires intubation, tracheal suctioning, assisted ventilation,chest compressions, and intravenous epinephrine. Apgar scores are 1, 3, and 5 at 1, 5, and 10minutes, respectively. You transfer the newborn to the neonatal intensive care unit, where heappears cyanotic, in respiratory distress, and agitated. Systemic blood pressure is 35/17 mm Hg.On 100% oxygen by assisted ventilation, pulse oximetry in the right upper and left lowerextremities reveals saturations of 94% and 80%, respectively. You obtain chest radiography(Item Q130).

Of the following, the MOST likely diagnosis is

A. congenital diaphragmatic hernia

B. congenital pneumonia

C. cyanotic congenital heart disease

D. persistent pulmonary hypertension

E. respiratory distress syndrome

Copyright © 2010 by the American Academy of Pediatrics page 326

Page 327: AAP PREP 2010

2010 PREP SA on CD-ROM

Preferred Response: DCritique: 130

The growth restriction, 42-week gestation, oligohydramnios, and depressed conditionrequiring vigorous resuscitation at birth reported for the newborn in the vignette strongly indicatefetal compromise due to chronic hypoxemia. Although meconium expression does not alwaysequate with fetal stress and may be a normal finding in many term pregnancies, the risk formeconium aspiration must be acknowledged when fetal stress is accompanied by abnormal fetalheart rate findings (bradycardia) and perinatal depression requiring resuscitation. Viewing andsuctioning the trachea is an essential step in this newborn’s resuscitation.

Tracheal suctioning may reveal particulate meconium, meconium-stained mucous secretions,or no meconium. The degree of (any) meconium aspiration cannot be determined by deliveryroom suctioning alone. Clinical, biochemical, and radiographic evaluation must follow for thenewborn who has perinatal depression because of the risk for parenchymal lung injury due toaspiration of blood, amniotic fluid, or meconium (meconium aspiration syndrome); meconiumobstruction of the small and large airways; air-leak syndromes (pneumothorax,pneumomediastinum); and pulmonary vascular reactivity. The latter is due to hypoxemia andreactive pulmonary vascular constriction, leading to a condition of pulmonary hypertension.

The difference in pre- and postductal arterial oxygen saturations indicates that the newbornin the vignette has a right-to-left shunt and persistent pulmonary hypertension of the newborn(PPHN). The radiograph in the newborn who has PPHN associated with meconium aspirationsyndrome reveals generally hyperinflated lung fields, patchy infiltrates, and varying areas ofatelectasis and hyperaeration (Item C130A).

Congenital diaphragmatic hernia is a defect in the embryologic closure of the diaphragm inwhich abdominal contents occupy the thorax and compress lung development (Item C130B).Congenital pneumonia typically is seen in the presence of prolonged rupture of fetal membranesor chorioamnionitis with subsequent neonatal respiratory distress and a lobar or diffuseconsolidation. Pneumonia due to group B streptococci may cause respiratory failure in the termnewborn and presents a radiographic picture indistinguishable from respiratory distresssyndrome due to surfactant deficiency in the preterm infant, in which there are low lungvolumes, diffuse ground-glass densities, and air bronchograms (Item C130C). The radiographicappearance of cyanotic congenital heart disease may show cardiomegaly, pulmonary vascularengorgement, or relative pulmonary oligemia.

References:

Aly H. Respiratory disorders in the newborn: identification and diagnosis. Pediatr Rev.2004;25:201-208. Available at: http://pedsinreview.aappublications.org/cgi/content/full/25/6/201

Dargaville PA, Copnell B, for the Australian and New Zealand Neonatal Network. Theepidemiology of meconium aspiration syndrome: incidence, risk factors, therapies, and outcome.Pediatrics. 2006;117:1712-1721. Available at:http://pediatrics.aappublications.org/cgi/content/full/117/5/1712

Hernández-Díaz S, Van Marter LJ, Werler MM, Louik C, Mitchell AA. Risk factors for persistent

Copyright © 2010 by the American Academy of Pediatrics page 327

Page 328: AAP PREP 2010

2010 PREP SA on CD-ROM

pulmonary hypertension of the newborn. Pediatrics. 2007;120:e272-e282. Available at:http://pediatrics.aappublications.org/cgi/content/full/120/2/e272

Steinhorn RH, Farrow KN. Pulmonary hypertension in the neonate. NeoReviews. 2007;8:e14-e21. Available for subscription at: http://neoreviews.aappublications.org/cgi/content/full/8/1/e14

Thilo EH, Rosenberg AA. The newborn infant. In: Hay WW Jr, Levin MJ, Sondheimer JM,Deterding RR, eds. CURRENT Diagnosis & Treatment: Pediatrics. 19th ed. New York, NY: TheMcGraw-Hill Companies; 2009:Chapter 1. Available for subscription at:http://www.accessmedicine.com/content.aspx?aID=3396500

Copyright © 2010 by the American Academy of Pediatrics page 328

Page 329: AAP PREP 2010

2010 PREP SA on CD-ROM

Question: 131

As the sports physician for the local high school football team, you are asked to give a lecture tocoaches about medical conditions and safe sports participation. You tell them that someconditions are relative or absolute contraindications to playing football because of increased riskto the athlete’s health.

Of the following, the condition that is a CONTRAINDICATION to playing football is

A. diabetes mellitus

B. febrile illness

C. human immunodeficiency virus infection

D. seizure disorder

E. sickle cell disease

Copyright © 2010 by the American Academy of Pediatrics page 329

Page 330: AAP PREP 2010

2010 PREP SA on CD-ROM

Preferred Response: BCritique: 131

Sports participation can help children and adolescents learn physical fitness and team-building skills. However, some medical conditions warrant special consideration with regard toparticipation, and both the type of sport and nature of the medical condition should beconsidered when making decisions. Sports are classified according to contact and intensity, andspecific guidelines for participation in each sport have been outlined in the 36th Bethesdaconference.

Absolute contraindications to participation in any sport include carditis, which may result insudden death with exertion, and febrile illness, which can result in decreased heat tolerance,increased risk of heat illness, and dehydration. In addition, fever may be the heralding sign of anunderlying condition that may put the athlete at additional risk during exercise, such asmyocarditis, pulmonary infection, or infectious mononucleosis with splenomegaly. Athletesshould be advised to avoid sports until the febrile illness has resolved.

Children and adolescents who have diabetes mellitus and sickle cell disease may participatein sports, but special care should be taken to avoid dehydration and overheating, and bloodglucose should be monitored frequently for those who have diabetes mellitus. Because the riskof transmitting human immunodeficiency virus to others during sports is very low, those whohave the infection should be encouraged to participate in sports. Skin lesions should be coveredfully, and sports requiring high skin contact, including wrestling and boxing, should be avoided ifthe viral load is elevated. Children who have seizure disorders may participate in sports, but ifthe seizures are not well controlled, they should avoid sports such as swimming, weightlifting,riflery, and archery because the risk of harm to themselves or others is high should a seizureoccur during the activity.

As a result of reviewing this information, do you intend to make a change in practiceto provide better patient care?Yes No

References:

Rice SG and The Council on Sports Medicine and Fitness. Medical conditions affecting sportsparticipation. Pediatrics. 2008;121:841-848. Available at:http://pediatrics.aappublications.org/cgi/content/full/121/4/841

Tucker A, Grady M. Role of the adolescent preparticipation physical examination. Phys MedRehabil Clin N Am. 2008;19:217-234

36th Bethesda conference: eligibility recommendations for competitive athletes withcardiovascular abnormalities. J Am Coll Cardiol. 2005;45:1312-1375

Copyright © 2010 by the American Academy of Pediatrics page 330

Page 331: AAP PREP 2010

2010 PREP SA on CD-ROM

Question: 132

A nursing student is preparing a family for the discharge of a 4-week-old infant who wasadmitted to the hospital for evaluation of an apparent life-threatening event (ALTE). During the 3-day hospitalization, results of all studies were negative. The infant, who was born at 39 weeks’gestation, has had stable vital signs and no apnea or cyanosis during this hospitalization.Therapy for reflux has been initiated. The nurse asks you, in the presence of the parents, if ahome apnea and bradycardia monitor should be prescribed.

Of the following, the BEST evidence-based information regarding the use of home apneamonitoring is that it

A. has been proven to decrease the risk of sudden infant death syndrome in infants who havehad prior ALTEs

B. has been proven to prevent death in term newborns who have been diagnosed with ALTE ofunknown cause

C. is highly effective in preventing recurrent ALTE attributed to gastroesophageal reflux

D. may be efficacious in monitoring preterm infants who have hypoxia due to apnea ofprematurity

E. usually is prescribed for the first 9 months in high-risk preterm infants

Copyright © 2010 by the American Academy of Pediatrics page 331

Page 332: AAP PREP 2010

2010 PREP SA on CD-ROM

Preferred Response: DCritique: 132

Although many parents believe that the use of an infant cardiorespiratory monitor in thehome after an acute life-threatening event (ALTE) may be lifesaving, there is no evidence tosupport this belief. However, there is some evidence supporting the use of home apneamonitoring for an infant who has apnea of prematurity accompanied by hypoxia that responds tostimulation when the parents are trained to respond appropriately to emergencies. Monitoring iscontinued until 2 to 3 months have passed without an apneic episode.

Skills of communication and professionalism are very important in assisting parents andother health-care professionals in understanding the use of evidence-based medicine to assignbenefit and risk determinations to specific diagnostic and management strategies. In particular,the use of a home apnea monitor for an infant not proven to have a risk of recurrent paroxysmalcardiorespiratory event may increase the possibility of harms such as vulnerable childsyndrome, parental anxiety and depression, and unnecessary administration of cardiopulmonaryor other resuscitative efforts because false alarms and instrument malfunction are far morecommon than reversible infant cardiorespiratory events.

There is no evidence that using a home apnea monitor reduces the risk of sudden infantdeath syndrome (SIDS) for a child who has apnea of prematurity, prior ALTE, or a sibling whosuffered SIDS. Home monitoring will not prevent future episodes of ALTE in the child who hasgastroesophageal reflux. Available evidence has failed to demonstrate a conclusive associationbetween gastroesophageal reflux and ALTE.

As a result of reviewing this information, do you intend to make a change in practiceto provide better patient care?Yes No

References:

American Academy of Pediatrics Committee on Fetus and Newborn. Apnea, sudden infant deathsyndrome, and home monitoring. Pediatrics. 2003;111:914-917. Available at:http://pediatrics.aappublications.org/cgi/content/full/111/4/914

Esani N, Hodgman JE, Ehsani N, Hoppenbrouwers T. Apparent life-threatening events andsudden infant death syndrome: comparison of risk factors. J Pediatr. 2008;152:365-370.Abstract available at: http://www.ncbi.nlm.nih.gov/pubmed/18280841

Farrell PA, Weiner GM, Lemons JA. SIDS, ALTE, apnea and the use of home monitors. PediatrRev. 2002;23:3-9. Available at: http://pedsinreview.aappublications.org/cgi/content/full/23/1/3

Moon RY, Fu LY. Sudden infant death syndrome. Pediatr Rev. 2007;28:209-214. Available at:http://pedsinreview.aappublications.org/cgi/content/full/28/6/209

Copyright © 2010 by the American Academy of Pediatrics page 332

Page 333: AAP PREP 2010

2010 PREP SA on CD-ROM

Question: 133

You are evaluating a 15-year-old boy who will be attending sports camp in the summer. He tellsyou that he is very athletic, has no trouble keeping up with his peers during physical activities,and, in fact, has less fatigue with activities than most of his friends. On physical examination, heis well-developed and comfortable. The first and second heart sounds are normal. There is asystolic click at the upper right sternal border as well as a 3/6 systolic ejection murmur at theupper right sternal border (Item Q133). There is a thrill in his suprasternal notch. Diastole is clear,and his pulses are normal in all extremities.

Of the following, the MOST likely cause of this patient’s signs and symptoms is

A. aortic stenosis

B. atrial septal defect

C. patent ductus arteriosus

D. pulmonary stenosis

E. ventricular septal defect

Copyright © 2010 by the American Academy of Pediatrics page 333

Page 334: AAP PREP 2010

2010 PREP SA on CD-ROM

Preferred Response: ACritique: 133

The patient described in the vignette has the typical findings of aortic stenosis, which oftenis associated with a systolic click that results from the abnormal structure and function of thevalve. The click occurs with opening of the thickened semilunar valve leaflets during systole. Incontrast to the normal thin and flexible valve leaflets, those of the stenotic aortic valve have anaccentuated sound that is referred to as an opening click. The murmur of aortic stenosis resultsfrom systolic blood flow from the left ventricle across the abnormally narrowed orifice of theaortic valve. The narrowing yields a diminished valve area through which the stroke volumecrosses, creating turbulence that is noted during auscultation as a systolic ejection murmur andtypically is heard best over the aortic valve and ascending aorta. On the chest wall, thesestructures lie beneath the right sternal border, with extension up toward the right clavicle. Themurmur often radiates into the neck. A thrill may be appreciated in the suprasternal notch, withthe turbulent blood flow in the transverse aortic arch being palpable in some patients.

Pulmonary stenosis is associated with a systolic ejection click that does not change withposition, but the accompanying murmur is heard best at the upper left sternal border, withradiation into the back and axillae. The murmur associated with an atrial septal defect is not fromthe blood flow across the atrial septum, which usually is not turbulent and at low pressure.Rather, the systolic murmur created by an atrial septal defect is caused by a relative pulmonarystenosis because the left-to-right atrial shunt and resulting increased right ventricular volumemust cross the pulmonary valve. In contrast to pulmonary valve stenosis, there is no structuralabnormality of the pulmonary valve and, thus, no systolic click.

A patent ductus arteriosus typically produces a continuous murmur that is characterized ashaving a "machinery" quality and usually is loudest at the left infraclavicular area. It is continuousbecause of the constant flow between the systemic and pulmonary circulation, with the highersystemic than pulmonary vascular resistance throughout the cardiac cycle and no valve toseparate the two in the structure of the ductus. The murmur of a ventricular septal defect istypically holosystolic because the left-to-right shunt at the ventricular level begins with the onsetof systole, even before the aortic and pulmonary valves open. When the ventricular septaldefect is small, it produces a high-pitched murmur, heard along the sternal border, and thesecond heart sound is normal, with no change in its normal physiologic splitting.

References:

Allen HD, Phillips JR, Chan DP. History and physical examination. In: Allen HD, Driscoll DJ, ShaddyRE, Feltes TF, eds. Moss and Adams’ Heart Disease in Infants, Children, and Adolescents,Including the Fetus and Young Adult. 7th ed. Philadelphia, Pa: Lippincott Williams & Wilkins;2008:58-65

Moller JH. Clinical history and physical examination. In: Moller JH, Hoffman JIE, eds. PediatricCardiovascular Medicine. Philadelphia, Pa: Churchill Livingstone; 2000:97-110

Copyright © 2010 by the American Academy of Pediatrics page 334

Page 335: AAP PREP 2010

2010 PREP SA on CD-ROM

Question: 134

A 10-year-old boy presents to the emergency department with a 2-day history of progressivedifficulty with speech and coordination. On physical examination, the restless but otherwisequiet child has a normal mental status and eye movements. His speech is slurred, and he cannotmaintain tongue protrusion without an in-and-out darting movement. Continuous flowing andjerky movements occur when he holds his hands outstretched or overhead (Item Q134).Although his grip is strong, he cannot maintain it well because of irregular hand and armmovements. He had a sore throat and fever 2 months ago. You diagnose chorea.

Of the following, the MOST effective treatment for suppressing the chorea for this boy is

A. carbamazepine

B. clonazepam

C. haloperidol

D. penicillin

E. trihexyphenidyl

Copyright © 2010 by the American Academy of Pediatrics page 335

Page 336: AAP PREP 2010

2010 PREP SA on CD-ROM

Preferred Response: CCritique: 134

The restless, continuous, involuntary movements that are irregular in direction and amplitudedescribed for the boy in the vignette represent classic chorea. He also has the classic "milk-maid's grip" and "darting tongue" signs, in which a simple motor command cannot be maintaineddue to choreic intrusions or involuntary relaxations (Item C134). The most common form ispoststreptococcal, immune-mediated chorea, that is, Sydenham chorea. The prior sore throatand fever reported for the boy may have been an undiagnosed group A beta-hemolyticstreptococcal infection. This possibility should be evaluated with blood testing for twoantistreptococcal antibodies: antistreptolysin O and anti-DNAse B. Other diagnostic possibilitiesinclude chorea associated with systemic lupus erythematosus, antiphospholipid antibodysyndrome, or hyperthyroidism.

Haloperidol is a dopamine receptor-blocking agent that also is referred to as a neuroleptic.This high-potency antipsychotic can suppress chorea effectively, but the incidence of adverseeffects is high. Fortunately, Sydenham chorea usually is self-limited and a low dose is generallyhelpful. Other antipsychotics (fluphenazine or risperidone) or dopamine-depleting agents(tetrabenazine) also may be used. Because of the diagnostic complexity and therapeuticconsiderations, the boy should receive prompt specialty consultation with a neurologist or amovement disorder specialist.

Carbamazepine is an anticonvulsant. It does not treat chorea and occasionally may induce it.Valproic acid, another anticonvulsant, sometimes is used to suppress chorea but probably isless effective than haloperidol. Penicillin is indicated for secondary prevention among patientswho have rheumatic fever, even with chorea as the sole manifestation. However, penicillin doesnot treat the chorea. Clonazepam is a benzodiazepine that may be helpful, but sedating dosesoften are required for efficacy. Trihexyphenidyl is an anticholinergic medication that may reducedystonia, but it tends to worsen the symptoms of Sydenham chorea.

References:

American Academy of Pediatrics. Group A streptococcal infections. In: Pickering LK, Baker CJ,Kimberlin DW, Long SS, eds. Red Book: 2009 Report of the Committee on Infectious Diseases.28th ed. Elk Grove Village, Ill: American Academy of Pediatrics; 2009:616-628

Johnston MV. Movement disorders. In: Kliegman RM, Behrman RE, Jenson HB, Stanton BF, eds.Nelson Textbook of Pediatrics. 18th ed. Philadelphia, Pa: Saunders Elsevier; 2007:2488-2493

Kiechl-Kohlendorfer U, Ellemunter H, Kiechl S. Chorea as the presenting clinical feature ofprimary antiphospholipid syndrome in childhood. Neuropediatrics. 1999;30:96-98. Abstractavailable at: http://www.ncbi.nlm.nih.gov/pubmed/10401693

Paz JA, Silva CA, Marques-Dias MJ. Randomized double-blind study with prednisone inSydenham's chorea. Pediatr Neurol. 2006;34:264-269. Available at:http://www.ncbi.nlm.nih.gov/pubmed/16638499

Copyright © 2010 by the American Academy of Pediatrics page 336

Page 337: AAP PREP 2010

2010 PREP SA on CD-ROM

Question: 135

You are called to the newborn nursery to evaluate a baby who is having difficulty withfeedings. He seems ravenously hungry when given the nipple but becomes fretful and criesafter sucking for more than a few seconds. Nurses have noticed that he becomes dusky whensleeping, but when he wakes and cries, his color improves. Echocardiography reveals astructurally normal heart. You suspect that the baby has choanal atresia.

Of the following, the MOST likely associated condition in this child is

A. achondroplasia

B. Crouzon syndrome

C. Down syndrome

D. fetal alcohol syndrome

E. Prader-Willi syndrome

Copyright © 2010 by the American Academy of Pediatrics page 337

Page 338: AAP PREP 2010

2010 PREP SA on CD-ROM

Preferred Response: BCritique: 135

Choanal atresia is the congenital obstruction of the posterior choana or choanae (theposterior aperture[s] of the nose that open(s) into the nasopharynx) (Item C135). It may beunilateral or bilateral, and it may be membranous, membranous with a bony rim, or bony. Bilateralatresia is associated with neonatal symptoms as described for the infant in the vignette.Unilateral atresia may be associated only with a mucous discharge on the affected side.Diagnosis is confirmed by computed tomography scan. The male-to-female ratio is 1:2, and theunilateral-to-bilateral ratio is 2:1. Approximately 50% of affected individuals have isolatedchoanal atresia; the other 50% have associated anomalies.

One of the most common disorders associated with choanal atresia is CHARGE (Coloboma,Heart defect, Atresia choanae, Retardation of growth/development, Genital anomalies, Earanomalies) syndrome. For years, this was believed to be an association, but the CHD7 genethat causes this condition was discovered in 2004, and it is now a "syndrome." CHARGEsyndrome is inherited as an autosomal dominant trait; most cases represent new mutations.

Any condition that causes significant depression of the nasal bridge or midface retractioncan be associated with choanal atresia. Examples include the craniosynostosis syndromessuch as Crouzon, Pfeiffer, and Antley-Bixler.

Although achondroplasia, Down syndrome, and fetal alcohol syndrome all include midfacehypoplasia, they are not associated with retraction and choanal atresia. Prader-Willi syndrome isnot associated with midface hypoplasia or retraction.

References:

Cohen MM Jr. Nose: nostril atresia, choanal atresia. In: Stevenson RE, Hall JG, eds. HumanMalformations and Related Anomalies. 2nd ed. New York, NY: Oxford University Press;2006:377-378

Firth HV, Hurst JA. Nasal anomalies. In: Oxford Desk Reference Clinical Genetics. New York,NY: Oxford University Press; 2005:182-185

Copyright © 2010 by the American Academy of Pediatrics page 338

Page 339: AAP PREP 2010

2010 PREP SA on CD-ROM

Question: 136

An adolescent girl from your practice has been hospitalized after taking an overdose ofmedications. She has recovered from her overdose and says she regrets having made thesuicide attempt and would like to go home. You undertake an extensive psychosocial history todetermine if she is at risk for another suicide attempt.

Of the following, the psychosocial risk factor that would be MOST concerning for this girl is ahistory of

A. experimentation with marijuana

B. high parental expectations for school achievement

C. lack of involvement with her school activities

D. many people living in the house

E. previous suicide attempt

Copyright © 2010 by the American Academy of Pediatrics page 339

Page 340: AAP PREP 2010

2010 PREP SA on CD-ROM

Preferred Response: ECritique: 136

A previous suicide attempt and a family history of suicide are strong predicators forcompleted suicide. Demographic risk factors also have been identified, with females being threetimes less likely than males to complete suicide but more likely to have persistent suicidalideation. Females also are nearly twice as likely as males to engage in self-harm behaviorwithout an expressed intent to die. Native American male adolescents have the highest risk ofsuicide, and African American females have the lowest risk. Almost 90% of youth who attemptsuicide have had at least one prior psychiatric diagnosis, such as mood disorders or substanceabuse. A history of physical or sexual abuse has been associated with a heightened suicidalrisk. Some evidence suggests that gay, lesbian, and bisexual youth are at higher risk for suicide.Affective factors such as impulsivity and aggression, especially if combined with adverse lifeevents, have been linked to suicide risk. Youth are also at greater risk of imitating suicidalbehavior from media exposure. The lack of risk factors, however, does not make the adolescentsafe from suicide.

A reported 2% increased risk of suicidal thinking and behavior with the use of selectiveserotonin reuptake inhibitors versus placebo prompted the United States Food and DrugAdministration to issue a warning related to these agents. Re-evaluation of the data indicated alower risk than previously stated. In addition, no suicides occurred during any of these studies.Nevertheless, careful monitoring of all adolescents’ mental health and behavioral status isimportant when starting or changing pharmacologic treatment for depression.

High parental expectations for their child’s school achievement, high grade point average,more people living in the household, and religiosity have been shown to be protective for someboys, but not for girls. Lack of involvement with school activities or experimentation withmarijuana without other symptoms of a mood disorder or other areas of decreased functioningare not high-risk factors for suicide.

Pediatricians should seek to identify adolescents who have risk factors as well as warningsigns of suicide, such as having a current plan or exhibiting a sense of hopelessness,purposelessness, social withdrawal, anxiety, anger, recklessness, increased substance use,and mood swings. The presence of these signs requires immediate further evaluation.

References:

Bridge JA, Goldstein TR, Brent DA. Adolescent suicide and suicidal behavior. J Child PsycholPsychiatry. 2006; 47:372-394. Abstract available at:http://www.ncbi.nlm.nih.gov/pubmed/16492264

Shain BN and the Committee on Adolescence. Clinical report: suicide and suicide attempts inadolescents. Pediatrics. 2007;120:669-676. Available at:http://pediatrics.aappublications.org/cgi/content/full/120/3/669

Wagman Borowsky I, Ireland M, Resnick MD. Adolescent suicide attempts: risks and protectors.Pediatrics. 2001;107:485-493. Available at:http://pediatrics.aappublications.org/cgi/content/full/107/3/485

Copyright © 2010 by the American Academy of Pediatrics page 340

Page 341: AAP PREP 2010

2010 PREP SA on CD-ROM

Wintersteen MB, Diamond GS, Fein JA. Screening for suicide risk in the pediatric emergency andacute care setting. Curr Opin Pediatr. 2007;19:398-404. Abstract available at:http://www.ncbi.nlm.nih.gov/pubmed/17630602

Copyright © 2010 by the American Academy of Pediatrics page 341

Page 342: AAP PREP 2010

2010 PREP SA on CD-ROM

Question: 137

You are caring for an 8-month-old infant in the pediatric intensive care unit. She has beenhospitalized for 1 week with respiratory syncytial virus bronchiolitis. In discussions with themother, she reports that the child has very frequent, large, foul-smelling stools. Physicalexamination of the child reveals a temperature of 37.0°C, heart rate of 140 beats/min, respiratoryrate of 35 breaths/min on the ventilator, and blood pressure of 80/40 mm Hg. Her oxygen

saturation is 90%, and her most recent arterial blood gas shows a pH of 7.25, Paco2 of 70 mm

Hg, and Pao2 of 70 mm Hg. Her weight is 7 kg, and she appears malnourished, with decreasedmuscle development. No obvious congenital abnormalities are apparent. Her chest appearshyperinflated, she has no heart murmur, her pulses are equal and strong, and her abdomen isprotuberant with normal bowel sounds. Computed tomography scan of her chest was obtainedearlier today to evaluate enlarging cystic-appearing lesions on her chest radiograph (Item Q137).

Of the following, the MOST likely underlying diagnosis in addition to bronchiolitis is

A. Clostridium difficile infection

B. congenital lobar emphysema

C. cystic adenomatoid malformation

D. cystic fibrosis

E. pulmonary sequestration

Copyright © 2010 by the American Academy of Pediatrics page 342

Page 343: AAP PREP 2010

2010 PREP SA on CD-ROM

Preferred Response: DCritique: 137

The child described in the vignette was admitted to the pediatric intensive care unit forbronchiolitis, but her history of failure to thrive and malabsorption as well as cystic lesions onradiologic imaging are consistent with cystic fibrosis. Cystic fibrosis can have a variety ofpresentations, including failure to thrive, delayed passage of stool, abnormal stools, rectalprolapse, and electrolyte abnormalities, but acute or chronic respiratory symptoms account formore than 50% of all presentations. Cough frequently is the first respiratory symptom, butprogression to wheezing, shortness of breath, and recurrent pneumonia is common. Infantsyounger than 1 year of age can have bronchiolitis with wheezing, as described for the child inthe vignette. Radiographic findings include hyperinflation, bronchial thickening, and patchyatelectasis. Progressive disease is evidenced by cyst formation, bronchiectasis, and lobaratelectasis. Severe complications of pulmonary disease that may be life-threatening can includedevelopment of pneumothoraces, cor pulmonale, and hemoptysis.

Congenital disorders of the lung such as congenital cystic adenomatoid malformation,congenital lobar emphysema (Item C137), or pulmonary sequestration may present asrespiratory distress during infancy, but they generally are not associated with malabsorption orfailure to thrive. Clostridium difficile infection, which usually develops following antibiotic usage,can cause foul-smelling diarrhea, but failure to thrive is not a characteristic finding.

References:

Boat TF, Acton JD. Cystic fibrosis. In: Kliegman RM, Behrman RE, Jenson HB, Stanton BF, eds.Nelson Textbook of Pediatrics. 18th ed. Philadelphia, Pa: Saunders Elsevier; 2007:1803-1816

Davis PB. Cystic fibrosis. Pediatr Rev. 2001;22:257-264. Available at:http://pedsinreview.aappublications.org/cgi/content/full/22/8/257

Copyright © 2010 by the American Academy of Pediatrics page 343

Page 344: AAP PREP 2010

2010 PREP SA on CD-ROM

Question: 138

A 12-year-old boy who has chronic lymphocytic thyroiditis presents to the emergencydepartment with a 1-week history of nausea, vomiting, and muscle pains. On physicalexamination, the child is dehydrated, has a blood pressure of 80/40 mm Hg and a heart rate of110 beats/min, and appears tanned even though it is November and he lives in Minnesota. Yoususpect adrenal insufficiency (Addison disease) and order laboratory tests for serum cortisoland adrenocorticotropic hormone as well as serum and urine electrolytes (Item Q138).

Of the following, the MOST typical electrolyte pattern for primary adrenal insufficiency is

A. Row A of Item Q138

B. Row B of Item Q138

C. Row C of Item Q138

D. Row D of Item Q138

E. Row E of Item Q138

Copyright © 2010 by the American Academy of Pediatrics page 344

Page 345: AAP PREP 2010

2010 PREP SA on CD-ROM

Preferred Response: ACritique: 138

Children who have primary adrenal insufficiency (Addison disease) are unable to retainsodium and excrete potassium because of aldosterone deficiency. They have lowconcentrations of cortisol and high concentrations of circulating adrenocorticotrophic hormone(ACTH). They become dehydrated and break down muscle tissue, developing hyponatremia,hyperkalemia, an elevated blood urea nitrogen, and acidosis. Their urine electrolytes (increasedsodium and decreased potassium) reflect the aldosterone deficiency.

Children who have ACTH deficiency (ie, secondary adrenal insufficiency) also manifest theeffects of cortisol deficiency: weight loss, nausea, and inability to maintain blood pressure. Theyoften have hyponatremia because the low intravascular volume resulting from cortisoldeficiency leads to release of vasopressin. Because they can release aldosterone, they do notdevelop hyperkalemia. They also do not develop hyperpigmentation.

References:

Liotta EA, Elston DM, Brough A, Erickson QL. Addison disease. eMedicine Specialties,Dermatology, Internal Medicine. 2007. Available at:http://www.emedicine.com/derm/topic761.htm

Ten S, New M, Maclaren N. Clinical review 130: Addison's disease 2001. J Clin EndocrinolMetab. 2001;86:2909-2922. Available at: http://jcem.endojournals.org/cgi/content/full/86/7/2909

Wilson TA, Speiser PW. Adrenal insufficiency. eMedicine Specialties, Pediatrics: GeneralMedicine, Endocrinology. 2007. Available at: http://www.emedicine.com/PED/topic47.htm

Copyright © 2010 by the American Academy of Pediatrics page 345

Page 346: AAP PREP 2010

2010 PREP SA on CD-ROM

Question: 139

A well-nourished, healthy child comes to your office for his 2-year health supervision visit. Youfind that he has failed an autism-specific screening tool (the Modified Checklist for Autism inToddlers) completed by his parents while in the waiting room. Results of a previous audiologyevaluation are normal. His head circumference and growth parameters are normal. Findings on ageneral physical examination are unremarkable, and the neurologic examination producesnonfocal findings. Muscle mass, strength, and tone are within normal limits. His deep tendonreflexes are physiologic and symmetric.

Of the following, the MOST appropriate next step is

A. early intervention referral

B. genetics evaluation

C. head magnetic resonance imaging

D. metabolic evaluation

E. sleep-deprived electroencephalography

Copyright © 2010 by the American Academy of Pediatrics page 346

Page 347: AAP PREP 2010

2010 PREP SA on CD-ROM

Preferred Response: ACritique: 139

The boy described in the vignette has failed The Modified Checklist for Autism in Toddlers (M-CHAT). The M-CHAT is an autism-specific screening tool that has a sensitivity of 0.85 and aspecificity of 0.93. The sensitivity measures the percentage of individuals correctly identified ashaving autism (85%). The specificity measures the percentage of individuals who are correctlyidentified as not having autism (93%). The results described for the boy in the vignette suggestthat he has autism, and the clinician should not wait to confirm the diagnosis before referring thechild for early intervention services. Such services are beneficial in addressing the child’sdeficits. If the diagnosis is confirmed, the interventions may be altered to a more specificintervention for autism spectrum disorder (ASD).

ASDs are biologically based neurodevelopmental disorders. Although genetic evaluation maybe indicated, especially when there is a family history of autism, referral for early intervention ismore likely to improve the child’s prognosis than genetic evaluation. Some of the conditions thatare associated with autism include fragile X syndrome, tuberous sclerosis, fetal alcoholsyndrome, Angelman syndrome, and Rett syndrome. There are only a few reports ofmitochondrial or metabolic abnormalities being associated with ASD, and most affected childrenhave normal immune function. No relationship has been demonstrated between autism and themeasles, mumps, rubella vaccine or thimerosal.

There is no evidence that sleep-deprived electroencephalography and magnetic resonanceimaging for children who have ASDs is needed in the absence of specific clinical findings (eg,seizures, hypopigmented macules). Because metabolic abnormalities rarely are associated withautism, metabolic evaluation also is not warranted.

As a result of reviewing this information, do you intend to make a change in practiceto provide better patient care?Yes No

References:

American Academy of Pediatrics. Caring for Children With Autism Spectrum Disorders: AResource Toolkit for Clinicians. Elk Grove Village, Ill: American Academy of Pediatrics; 2007

Johnson CP Myers SM. Overview of the AAP autism spectrum disorders toolkit and guidelines: apediatrician’s roadmap to the latest ASD guidelines. Contemp Pediatr. 2008. October:43-67

Johnson CP, Myers SM, and the Council on Children With Disabilities. Identification and evaluationof children with autism spectrum disorders. Pediatrics. 2007;120:1183-1251. Available at:http://pediatrics.aappublications.org/cgi/content/full/120/5/1183

Pickler L, Elias E. Genetic evaluation of the child with an autism spectrum disorder. Pediatr Ann.2009;38:26-29

Copyright © 2010 by the American Academy of Pediatrics page 347

Page 348: AAP PREP 2010

2010 PREP SA on CD-ROM

Question: 140

A previously healthy 3-year-old boy presents to the emergency department (ED) with a 5-dayhistory of a "barky" cough and low-grade fever (temperature no greater than 38.4°C. Yesterdayhis temperature increased to 39.5°C, he developed increased hoarseness, and he refused toeat. In the ED, his temperature is 39.4°C, he appears toxic, he exhibits stridor and intercostalretractions, and his oxygen saturation is 91% on room air.

Of the following, the MOST appropriate next step is to

A. administer dexamethasone intramuscularly

B. endotracheally intubate the child

C. obtain a lateral neck radiograph

D. perform an emergency tracheostomy

E. perform pulmonary function testing

Copyright © 2010 by the American Academy of Pediatrics page 348

Page 349: AAP PREP 2010

2010 PREP SA on CD-ROM

Preferred Response: BCritique: 140

The sudden deterioration and development of toxicity 5 days after the onset of an apparentcrouplike illness described for the boy in the vignette is most consistent with development ofbacterial tracheitis. The stridor, retractions, and decreased oxygen saturation all are consistentwith this diagnosis. The rapidity of symptom progression and potential for increased purulentsecretions to obstruct the airway mandate early endotracheal intubation to maintain and clearthe airway. Parenteral antibiotic therapy, including coverage for Staphylococcus aureus, isanother appropriate component of treatment. Of note, bacterial tracheitis is a potentialcomplication of measles infection. A prolonged period of intubation may be necessary until thesecretions clear with antimicrobial therapy.

Although dexamethasone may be indicated for treatment of viral croup, the duration ofillness and acute deterioration described for this boy argue for an alternative diagnosis. A lateralneck radiograph and pulmonary function testing are not helpful in the diagnosis of bacterialtracheitis and might be dangerous in a potentially critically ill patient by delaying antibiotic therapyand airway control. An emergency tracheostomy is not indicated if endotracheal intubation canbe accomplished successfully. However, a tracheostomy may be necessary and possiblylifesaving in the presence of a fixed obstruction in the upper airway, such as epiglottitis, tobypass the lesion. Epiglottitis typically presents acutely, with rapid progression over hours fromonset of symptoms to toxicity and impending airway obstruction. Fortunately, the advent of theconjugate Haemophilus influenzae type b vaccine has made acute epiglottis an extremely rareentity.

References:

Cherry JD. Clinical practice: croup. N Engl J Med. 2008;358:384-391. Extract available at:http://content.nejm.org/cgi/content/extract/358/4/384

Hopkins A, Lahiri T, Slaerno R, Heath B. Changing epidemiology of life-threatening upper airwayinfections: the reemergence of bacterial tracheitis. Pediatrics. 2006;118:1418-1421. Available at:http://pediatrics.aappublications.org/cgi/content/full/118/4/1418

Copyright © 2010 by the American Academy of Pediatrics page 349

Page 350: AAP PREP 2010

2010 PREP SA on CD-ROM

Question: 141

You are evaluating an 11-year-old boy for a puncture wound to the sole of his foot thatoccurred 3 days ago. The patient states that he was climbing over a fence in a nearby forestpreserve when he slipped and landed on his feet in a pile of trash. He noticed that a nail hadpunctured the sole of his tennis shoe, and he pulled it out. Physical examination shows edema,erythema, induration, and point tenderness at the puncture site, with some dried blood andminimal purulent discharge.

Of the following, the MOST likely organism causing this patient’s infection is

A. atypical mycobacteria

B. Klebsiella

C. Pseudomonas aeruginosa

D. Serratia marcescens

E. Staphylococcus aureus

Copyright © 2010 by the American Academy of Pediatrics page 350

Page 351: AAP PREP 2010

2010 PREP SA on CD-ROM

Preferred Response: CCritique: 141

A host of organisms may live on the skin, but the most common organisms comprising theindigenous flora are Staphylococcus aureus, Streptococcus pyogenes (group A beta-hemolyticStreptococcus), S epidermidis, other coagulase-negative staphylococci (CoNS),Propionibacterium acnes, and Candida albicans. The ability of the organism to cause aninfection is influenced, in part, by the patient’s age and immunologic status and thecircumstances surrounding the injury.

By far, the majority of skin and soft-tissue infections in all age groups are caused by Saureus (methicillin-susceptible and methicillin-resistant) and S pyogenes because these are thepredominant organisms found on the skin. Infants and children in intensive care units (bothpediatric and neonatal) have the highest incidence of CoNS skin and bloodstream infections.CoNS colonizing the skin can be introduced at the time of medical device placement, throughmucous membrane or skin breaks, or during catheter manipulation. Patients who have pre-existing breaks in the integrity of the skin (eg, burns, decubitus ulcers) may be colonized andsecondarily infected by a variety of organisms, including Proteus, Pseudomonas aeruginosa,enterococci, gram-negative enteric organisms, and anaerobes. Immunocompromised patientsmay have a variety of unusual pathogens causing skin infections after local minor trauma. Theseinclude various fungi (Fusarium, Paecilomyces, Penicillium, Trichosporon, Alternaria),mycobacteria (M marinum), and other bacteria (Bacillus cereus). Patients who have acquiredimmune deficiency syndrome may have skin lesions caused by Bartonella quintana (bacillaryangiomatosis). P aeruginosa often inhabits the external auditory canal in association with injury,maceration, inflammation, or wet or humid conditions and is the predominant bacterial pathogenof otitis externa. Candida albicans is a common cause of diaper dermatitis in infants andvulvovaginitis in the setting of diabetes mellitus, antibiotic therapy, pregnancy, and moistenvironmental conditions.

Occupationally related or contaminated traumatic wounds of the skin have differentinfectious pathogens. Infections of puncture wounds through the sole of a tennis shoe, asdescribed for the boy in the vignette, usually are caused by P aeruginosa; skin woundssustained in factories or on farms are caused by Enterobacter, P maltophilia, and other gram-negative bacilli; and wounds sustained during sport fishing or other water-related recreationalactivities may be infected with mycobacteria (M marinum) or gram-negative bacilli found inponds and lakes (eg, Aeromonas hydrophilia, Klebsiella, Escherichia coli).

Although atypical mycobacteria, Klebsiella, Serratia marcescens, and S aureus may befound in the environment, they are less likely to cause the infection described for the boy in thevignette.

As a result of reviewing this information, do you intend to make a change in practiceto provide better patient care?Yes No

References:

American Academy of Pediatrics. Candidiasis (moniliasis, thrush). In: Pickering LK, Baker CJ,

Copyright © 2010 by the American Academy of Pediatrics page 351

Page 352: AAP PREP 2010

2010 PREP SA on CD-ROM

Kimberlin DW, Long SS, eds. Red Book: 2009 Report of the Committee on Infectious Diseases.28th ed. Elk Grove Village, Ill: American Academy of Pediatrics; 2009:245-249

American Academy of Pediatrics. Group A streptococcal infections. In: Pickering LK, Baker CJ,Kimberlin DW, Long SS, eds. Red Book: 2009 Report of the Committee on Infectious Diseases.28th ed. Elk Grove Village, Ill: American Academy of Pediatrics; 2009:616-628

American Academy of Pediatrics. Staphylococcal infections. In: Pickering LK, Baker CJ, KimberlinDW, Long SS, eds. Red Book: 2009 Report of the Committee on Infectious Diseases. 28th ed.Elk Grove Village, Ill: American Academy of Pediatrics; 2009:601-615

Pier GB, Ramphal R. Pseudomonas aeruginosa. In: Mandell GL, Bennett JE, Dolan R, eds.Mandell, Douglas, and Bennett’s Principles and Practice of Infectious Diseases. 6th ed.Philadelphia, Pa: Elsevier Churchill Livingstone; 2005:2587-2614

Swartz MN, Pasternack MS. Cellulitis and subcutaneous tissue infections. In: Mandell GL,Bennett JE, Dolan R, eds. Mandell, Douglas, and Bennett’s Principles and Practice of InfectiousDiseases. 6th ed. Philadelphia, Pa: Elsevier Churchill Livingstone; 2005:1172-1194

Copyright © 2010 by the American Academy of Pediatrics page 352

Page 353: AAP PREP 2010

2010 PREP SA on CD-ROM

Question: 142

An 8-year-old girl who has previously diagnosed Takayasu arteritis presents to the clinic for aroutine evaluation. Her vasculitis has been in clinical remission, and she presently is taking nomedications. She complains of intermittent headaches. On physical examination, her heart rate is88 beats/min, and her blood pressure is 160/110 mm Hg. On abdominal examination, you note

bruits to the right and left of the midline. Laboratory results include:

•Sodium, 140 mEq/L (140 mmol/L)

•Potassium, 3.2 mEq/L (3.2 mmol/L)

•Chloride, 100 mEq/L (100 mmol/L)

•Bicarbonate, 32 mEq/L (32 mmol/L)

•Blood urea nitrogen, 16 mg/dL (5.7 mmol/L)

•Creatinine, 0.6 mg/dL (53.0 mcmol/L)Renal ultrasonography reveals kidneys that are slightly small for age but have normalechotexture.

Of the following, the MOST appropriate treatment for this patient is

A. atenolol

B. enalapril

C. furosemide

D. hydrochlorothiazide

E. losartan

Copyright © 2010 by the American Academy of Pediatrics page 353

Page 354: AAP PREP 2010

2010 PREP SA on CD-ROM

Preferred Response: ACritique: 142

The child described in the vignette, who has an underlying large-vessel vasculitis (Takayasuarteritis), presents with marked hypertension associated with bilateral abdominal bruits and ahypokalemic, metabolic alkalosis. This clinical scenario is consistent with bilateral renal arterystenosis.

The usual approach to hypertension in the pediatric patient is to prescribe an agent that willalter at least one of the components of the blood pressure equation: systemic vascularresistance (SVR) or cardiac output (comprised of heart rate and stroke volume). Whileaddressing the mechanism of hypertension, the antihypertensive agent also must avoidundesirable adverse effects. Medications that reduce the SVR include: dihydropyridine calciumchannel blockers (nifedipine or amlodipine), alpha blockers (prazosin), vasodilators (hydralazineor minoxodil), angiotensin-converting enzyme inhibitors (ACEIs) (enalapril or lisinopril), andangiotensin receptor blockers (ARBs) (losartan). Agents that reduce heart rate include: betablockers (propranolol or atenolol) and alpha-2 agonists (clonidine). Drugs that reduce the strokevolume include diuretics (thiazides, loop diuretics [furosemide], and spironolactone). ACEIs andARBs also lower stroke volume through their diuretic effects via reduction of aldosteroneconcentrations by reducing angiotensin II effects. Beta blockers also have effects on the renin-angiotensin-aldosterone system (RAAS) due to the presence of beta-1 receptors on thejuxtaglomerular apparatus (site of renin production) in the kidney. As such, beta blockers canhave some effects on the RAAS, but are generally milder in that regard compared with ACEIsand ARBs.

The treatment of the patient in the vignette (who has bilateral renal artery stenosis) must beapproached with caution because of the unique situational need for angiotensin II to provideefferent arteriolar vasoconstriction to maintain glomerular filtration. Therefore, the use of ACEIsand ARBs generally is contraindicated due the possible precipitous drop in renal function and thedevelopment of acute renal failure. Accordingly, neither enalapril nor losartan should be used inthis patient. Because the mechanism of hypertension is primarily due to the effects ofangiotensin II, which is a potent vasoconstrictor, the use of a beta blocker with its milder effectson the RAAS is ideal. Diuretics (either hydrochlorothiazide or furosemide) may have anadjunctive role, but as first-line therapy, they are only marginally effective. A dihydropyridinecalcium channel blocker also may be effective, but probably less so than a beta blocker.

References:

Flynn JT. Management of hypertension in children and adolescents. In: Kher KK, Schnaper HW,Makker SP, eds. Clinical Pediatric Nephrology. 2nd ed. London, England: Informa Healthcare;2007:481-492

National High Blood Pressure Education Program Working Group on High Blood Pressure inChildren and Adolescents. The fourth report on the diagnosis, evaluation, and treatment of highblood pressure in children and adolescents. Pediatrics. 2004;114:555—576. Available at:http://pediatrics.aappublications.org/cgi/content/full/114/2/S2/555

Copyright © 2010 by the American Academy of Pediatrics page 354

Page 355: AAP PREP 2010

2010 PREP SA on CD-ROM

Question: 143

A 16-year-old boy who runs on his high school cross-country team has been having troublewith coughing and wheezing during exercise. He has a history of intermittent asthma that is wellcontrolled when he is not running. His coach is worried that he may have exercise-inducedasthma (EIA) and would like him to use an inhaler when he exercises.

Of the following, the MOST appropriate response regarding EIA is that

A. a decrease of 5% in forced expiratory volume in 1 second during an exercise challenge isconsistent with EIA

B. a slow warm-up prior to exercise has not been shown to improve EIA

C. most children who have a diagnosis of asthma do not have EIA symptoms

D. sports that require short, quick bursts of activity are more likely to cause symptomscompared with continuous activities

E. symptoms can start within 5 minutes of exercise, but peak symptoms usually occur after theexercise is stopped

Copyright © 2010 by the American Academy of Pediatrics page 355

Page 356: AAP PREP 2010

2010 PREP SA on CD-ROM

Preferred Response: ECritique: 143

Exercised-induced asthma (EIA) results in a transient narrowing of the airway followingvigorous exercise and is associated with a 10% to 15% decrease in the forced expiratory

volume in 1 second (FEV1). A number of mechanisms are postulated for EIA, but the mostaccepted one is based on the decreased warming and humidification of inhaled air duringexercise. Upon completion of exercise, there is a marked influx of fluid due to rapid rewarming,which results in mucus production and airway narrowing. Because temperature change is a keyaspect of EIA, exercise or sports in cold weather provokes EIA more readily than in warmerweather. Also, a slow warm-up, particularly in cold weather, results in a more gradual changein airway temperature and can lessen EIA symptoms.

EIA symptoms such as cough, shortness of breath, and wheezing may begin duringexercise but classically peak 5 to 10 minutes after completion of exercise. The more strenuousthe exercise, the more intense the asthma attack. For example, running produces more severeairflow limitation than jogging, which, in turn, produces more limitation that walking. Short, intensebouts of work lasting fewer than 2 minutes produce fewer problems than longer periods ofeffort because of the shortened time of airway cooling. EIA symptoms usually resolvespontaneously after 30 minutes of rest.

Exercise is a very common asthma trigger for children, with 70% to 90% of children who

have asthma demonstrating EIA symptoms. Pretreatment with a short-acting beta2 agonist isappropriate for isolated EIA. Because most patients who have EIA will be identified withpersistent asthma, the latest Global Initiative for Asthma guidelines, published in late 2008,continue to recommend treatment with inhaled corticosteroids to control any underlyingpersistent asthma. Finally, acute episodes can be attenuated by having the athlete warm upbefore strenuous exercise.

Clinically, exercise challenges usually are performed if the diagnosis of EIA is in doubt or if

the patient is not responding appropriately to usual EIA therapies such as beta2 agonists, inhaledcorticosteroids, or leukotriene antagonists. Prior to exercise, two baseline spirometry

measurements are performed. Ideally, the FEV1 measurements should not differ by more than3%. The patient then exercises, preferably for the same duration and in the same conditions (eg,running outside on a cold day) that have provoked symptoms previously. Postexercisespirometry is performed at 5, 10, 15, and 30 minutes after exercise. During this challenge, a

decrease of 15% in FEV1 is considered consistent with EIA. Other protocols available in amonitored setting include eucapnic voluntary hyperpnea, hypertonic saline challenge, and

inhaled powder mannitol challenge. These require a 10% drop in FEV1 to be consistent with EIA.Other conditions that may result in poor exercise performance include vocal cord

dysfunction, poor physical conditioning (ie, overweight, out of shape), gastrointestinal reflux,poor cardiac function, and restrictive lung disease.

References:

Global Initiative for Asthma. Global Strategy for Asthma Management and Prevention. UpdatedDecember 2008. Available at: http://www.ginasthma.com/Guidelineitem.asp??l1=2&l2=1&intId=60

Copyright © 2010 by the American Academy of Pediatrics page 356

Page 357: AAP PREP 2010

2010 PREP SA on CD-ROM

McFadden ER Jr. Exercise-induced airway narrowing. In: Adkinson NF Jr, Yunginger JW, BusseWW, Bochner BS, Holgate ST, Simons FER, eds. Middleton’s Allergy Principles and Practice.6th ed. Philadelphia, Pa: Mosby Elsevier; 2003:1323-1332

National Heart, Lung, and Blood Institute and the National Asthma Education and PreventionProgram. Expert Panel Report 3 (EPR3): Guidelines for the Diagnosis and Management ofAsthma. Bethesda, Md: U.S. Department of Health and Human Services; 2007. Available at:http://www.nhlbi.nih.gov/guidelines/asthma/asthgdln.htm

Stempel DA. Asthma and the athlete. In: Leung DYM, Sampson HA, Geha RS, Szefler SJ, eds.Pediatric Allergy Principles and Practice. St. Louis, Mo: Mosby Elsevier; 2003:435-443

Copyright © 2010 by the American Academy of Pediatrics page 357

Page 358: AAP PREP 2010

2010 PREP SA on CD-ROM

Question: 144

A 14-year-old boy is brought to the emergency department from a local skateboard park aftersustaining a left elbow injury while skateboarding. He reports that he was skating down anincline at high speed when he lost his balance and fell, breaking his fall with his left arm. Hesays that he had immediate pain in his left elbow and noticed that it "looked out of place." Hismother placed his arm in a sling and brought him to the emergency department. Physicalexamination reveals moderate swelling of the elbow with prominence of the olecranon.Radiographs of his elbow reveal posterior elbow dislocation (Item Q144).

Of the following, the MOST likely additional finding on physical examination for this boy is loss of

A. sensation over the palmar aspect of the thumb and index finger

B. sensation over the fifth finger

C. thumb flexion

D. thumb extension

E. wrist extension

Copyright © 2010 by the American Academy of Pediatrics page 358

Page 359: AAP PREP 2010

2010 PREP SA on CD-ROM

Preferred Response: BCritique: 144

Posterior elbow dislocation is the most common joint dislocation seen in children. Mostfrequently caused by a fall onto an outstretched hand with a supinated forearm, the patienttypically presents with elbow pain and olecranon prominence, as described for the boy in thevignette. In addition, if there is concomitant nerve injury, neurologic deficits may be observed.Ulnar nerve injury, which results in decreased sensation over the fifth finger as well as loss ofwrist flexion and finger abduction, is present in 10% of posterior elbow dislocations. Mediannerve injury, leading to decreased sensation over the palmar aspect of the thumb and first fingeras well as decreased thumb flexion, occurs less frequently. Radial nerve injury, which causesloss of wrist and thumb extension, is not a common complication of this injury. Brachial arteryinjuries may occur and should be suspected in patients who exhibit decreased radial pulse,pallor, and forearm paresthesias.

Identification of neurovascular injury in a patient who has a posterior elbow dislocation iscritical because this finding increases the urgency of reduction. In addition, brachial arteryinjuries require early repair to decrease long-term complications, including ulnar and mediannerve palsies and hand contracture. Acutely, the arm should be splinted, analgesia be providedif possible, and the patient be referred emergently to an orthopedist who can perform thereduction and any necessary neurovascular repairs.

References:

Chorley J. Elbow injuries in the young athlete. UpToDate Online 16.3. 2008. Available at:http://www.utdol.com/online/content/topic.do?topicKey=ped_trau/13894&selectedTitle=1~7&source=search_result

Kuhn MA, Ross G. Acute elbow dislocations. Orthop Clin North Am. 2008;39:155-161. Abstractavailable at: http://www.ncbi.nlm.nih.gov/pubmed/18374806

Wheeless CR III. Elbow dislocations in children. Wheeless’ Textbook of Orthopaedics. 2008.Available at: http://www.wheelessonline.com/ortho/elbow_dislocations_in_children

Copyright © 2010 by the American Academy of Pediatrics page 359

Page 360: AAP PREP 2010

2010 PREP SA on CD-ROM

Question: 145

A 5-year-old girl has a 2-year history of intermittent, poorly localized abdominal pain. She nowpresents with a recurrence of crampy pain, and she has vomited after each meal for the past 24hours. She has no history of fever or diarrhea. The child woke frequently last night because ofpain, and she seemed more comfortable lying on her side in a knee-chest position. The familyhistory is negative for gastrointestinal disease. Both parents are 42 years of age, and the child’sfather underwent a coronary artery bypass procedure last year. The girl is difficult to examine,

complaining of pain wherever her abdomen is palpated. Initial laboratory data include:

•White blood cell count, 10.4x103/mcL (10.4x109/L)

•Hemoglobin, 12.5 g/dL (125 g/L)

•Sodium, 135 mEq/L (135 mmol/L)

•Chloride, 100 mEq/L (100 mmol/L)

•Potassium, 4.5 mEq/L (4.5 mmol/L)

•Amylase, 240 units/L

•Lipase, 700 units/L

•Aspartate aminotransferase, 60 units/L•Alanine aminotransferase, 70 units/L

Of the following, the test that is MOST likely to demonstrate the underlying cause of this girl’sillness is

A. abdominal ultrasonography

B. magnetic resonance cholangiopancreatography

C. serum calcium measurement

D. serum lipid measurement

E. sweat chloride test

Copyright © 2010 by the American Academy of Pediatrics page 360

Page 361: AAP PREP 2010

2010 PREP SA on CD-ROM

Preferred Response: DCritique: 145

The severe abdominal pain, vomiting, and antalgic knee-chest positioning (assumed torelieve pressure on the pancreas, a retroperitoneal structure lying between the duodenum andthe spine), coupled with marked pancreatic enzyme elevations described for the girl in thevignette suggests pancreatitis. The 2-year history of apparently similar, yet less severeepisodes indicates the recurrent nature of the problem. Recurrent pancreatitis is the secondmost common pancreatic disorder, after cystic fibrosis, in the pediatric age group. The earlypresentation of the condition in this child, coupled with her father’s history of prematureatherosclerotic coronary artery disease, indicate that a familial dyslipidemia is an importantpotential cause for her illness. Accordingly, the clinician should obtain a serum lipid profile.Abdominal ultrasonography is not required to establish a diagnosis of pancreatitis. Magneticresonance cholangiopancreatography or the currently preferred technique of endoscopicultrasonography is useful in the diagnosis of gallstone pancreatitis, but the absence of othersigns of cholestasis (eg, jaundice) makes this diagnosis unlikely. Hypercalcemia, particularly inthe setting of hyperparathyroidism, is a known cause of acute and recurrent pancreatitis, butthe family history and presentation strongly suggest a lipid metabolic disorder.

Acute, sporadic pancreatitis is more common than recurrent, chronic pancreatitis duringchildhood and adolescence. Most cases of acute, sporadic pancreatitis are due to bluntabdominal trauma or are classified as "idiopathic." Causes of chronic, relapsing pancreatitisinclude infections, autoimmune disorders, genetic conditions, and drugs.

Pancreatitis is a common complication of a lipoprotein metabolic disorders that areassociated with hypertriglyceridemia. Of the heritable hyperlipidemias, familial lipoprotein lipase(LPL) deficiency and type V hyperlipidemia are the disorders most frequently associated withchronic, relapsing pancreatitis. The LPL enzyme is responsible for intravascular hydrolysis ofdietary triglycerides carried in chylomicrons. Affected patients have functionally absent LPL andmanifest grossly lipemic plasma in the fasting state, with plasma triglyceride measurements inexcess of 1,500 mg/dL (17.0 mmol/L) (and as high as 25,000 mg/dL [282.5 mmol/L]).Hypercholesterolemia is another characteristic finding, with total plasma cholesterolconcentrations approximating 10% to 20% of the triglyceride value. Familial type V hyperlipidemiaalso is characterized by extremely high triglyceride concentrations, typically ranging from 500 to10,000 mg/dL (5.7 to 113.0 mmol/L), and presents with elevations in both chylomicron and low-density lipoprotein concentrations.

The most common cause of recurrent pancreatitis in the United States is hereditarypancreatitis, of which two types have been described. The molecular defects for both types Iand II involve mutations in the gene on chromosome 7q35 that codes for cationic trypsinogen,and both types are transmitted as autosomal dominant disorders with 80% penetrance. Genemutations allow for autoactivation of trypsinogen in the pancreas instead of in the duodenum,leading to intrapancreatic activation of other enzymes and subsequent pancreatic autodigestionand inflammation.

A sweat chloride test may be helpful in the diagnosis of cystic fibrosis (CF). In CF, mutationin the cystic fibrosis transmembrane conductance regulator (CFTR), which serves as a chloridechannel, results in reduced chloride and water secretion. The consequence is production ofabnormally thick, viscid mucus by all exocrine organs, including the pancreas, where the CFTR

Copyright © 2010 by the American Academy of Pediatrics page 361

Page 362: AAP PREP 2010

2010 PREP SA on CD-ROM

is located on the apical membrane of pancreatic ductular epithelium. In addition to causingexocrine pancreatic fibrosis and insufficiency, ductular plugging leads to one or more episodesof pancreatitis in a small percentage (<5%) of those who have CF.

References:

Belamarich PF. In brief: lipoprotein disorders. Pediatr Rev. 1996;17:144. Abstract available at:http://pedsinreview.aappublications.org/cgi/content/abstract/17/4/144

De Boeck K, Weren M, Proesmans M, Kerem E. Pancreatitis among patients with cystic fibrosis:correlation with pancreatic status and genotype. Pediatrics. 2005;115:e463-e469. Available at:http://pediatrics.aappublications.org/cgi/content/full/115/4/e463

Kandula L, Lowe ME. Etiology and outcome of acute pancreatitis in infants and toddlers. JPediatr. 2008;152:106-110. Abstract available at:http://www.ncbi.nlm.nih.gov/pubmed/18154910

Pietzak MM, Thomas DW. Pancreatitis in childhood. Pediatr Rev. 2000;21:406-412. Available at:http://pedsinreview.aappublications.org/cgi/content/full/21/12/406

Whitcomb DC, Lowe ME. Acute and chronic pancreatitis. In: Walker WA, Goulet O, Kleinman RE,Sherman PM, Shneider BL, Sanderson IR, eds. Pediatric Gastrointestinal Disease:Pathophysiology, Diagnosis, Management. 3rd ed. Hamilton, Ontario, Canada: BC Decker Inc;2004:1584-1597

Wittrup HH, Tybjaerg-Hansen A, Abildgaard S, Steffensen R, Schnohr P, Nordestgaard BG. Acommon substitution (Asn291Ser) in lipoprotein lipase is associated with increased risk ofischemic heart disease. J Clin Invest. 1997;99:1606—1613. Available at:http://www.jci.org/articles/view/119323/pdf

Copyright © 2010 by the American Academy of Pediatrics page 362

Page 363: AAP PREP 2010

2010 PREP SA on CD-ROM

Question: 146

You are discussing infants who have low birthweights on rounds with your residents. Youremind them that some low-birthweight (LBW) infants (<2,500 g) are born at term, aftercompleting 37 weeks’ gestation, and are considered small for gestational age (SGA). Other LBWinfants are born preterm, and their LBW status simply reflects this.

Of the following, the MOST likely outcome for term SGA infants is

A. head growth at 6 months postnatal age that lags behind weight and linear growth

B. linear growth that reaches the 50th percentile at 6 months postnatal age

C. more common neurodevelopmental impairment at 2 years postnatal age than seen in pretermintrauterine growth restriction infants

D. neurodevelopmental outcomes at 2 years postnatal age that compare favorably with otherterm infants

E. prediction of school performance is better at age 5 years than 2 years

Copyright © 2010 by the American Academy of Pediatrics page 363

Page 364: AAP PREP 2010

2010 PREP SA on CD-ROM

Preferred Response: ECritique: 146

Small for gestational age (SGA) infants are defined as weighing less than 2,500 g at birth bythe World Health Organization. When this condition exists in the preterm infant, it often is termedintrauterine growth restriction (IUGR). The preterm birth rate in the United States is 12.8%, andas many as 15% to 20% of such infants may be SGA when their gestational ages are evaluatedcarefully. Many SGA infants are born at term. In the United States, the rate of SGA infants hasbeen rising slowly over the past 15 years from 7% to 8.3% of all live births. These findings areindicative of mounting health-care, nutritional, social, and political realities that affect generalhealth and in particular, prenatal and interpregnancy health in women of reproductive age(between 18 and 45 years) (Item C146). Teen mothers have a greater risk for delivering SGAinfants.

IUGR in a term or preterm infant confers both short- and long-term outcome risks, includingincreased morbidity and mortality in the first postnatal year. Among IUGR preterm infants whoare of very low birthweight (<1,500 g), there is increased severity of retinopathy of prematurity,chronic lung disease, and necrotizing enterocolitis as well as an increased mortality rate. Therisk for neurodevelopmental disability in such preterm infants exceeds that seen in preterminfants who are appropriate for gestational age and term SGA infants.

Term SGA infants have a greater risk for neurodevelopmental disability in preschool yearsand beyond compared with term appropriately grown infant peers. The risk is predicted best byserial neurodevelopmental testing and results obtained at preschool age (3 to 5 years) ratherthan earlier (age 2 years or younger). Relative head sparing occurs among SGA infants, suchthat barring overt central nervous system injury, head growth is preserved even when weightor linear growth are poor. Linear growth in SGA infants, especially in very low-birthweightinfants who have IUGR, does not typically show "catch up" until growth accelerations at ages 2years and again in puberty.

References:

Casey PH, Whiteside-Mansell L, Barrett K, Bradley RH, Gargus R. Impact of prenatal and/orpostnatal growth problems in low birth weight preterm infants on school-age outcomes: an 8-year longitudinal evaluation. Pediatrics. 2006;118:1078-1086. Available at:http://pediatrics.aappublications.org/cgi/content/full/118/3/1078

Feldman R, Eidelman AI. Neonatal state organization, neuromaturation, mother-infant interaction,and cognitive development in small-for-gestational-age premature infants. Pediatrics.2006;118:e869-e878. Available at:http://pediatrics.aappublications.org/cgi/content/full/118/3/e869

Martin JA, Kung H-C, Mathews TJ, Hoyert DL, Strobino DM, Guyer B, Sutton SR. Annualsummary of vital statistics: 2006. Pediatrics. 2008; 121:788-801. Available at:http://pediatrics.aappublications.org/cgi/content/full/121/4/788

Regev RH, Lusky A, Dolfin T, Litmanovitz I, Arnon S, Reichman B, in collaboration with the Israel

Copyright © 2010 by the American Academy of Pediatrics page 364

Page 365: AAP PREP 2010

2010 PREP SA on CD-ROM

Neonatal Network. Excess mortality and morbidity among small-for-gestational-age prematureinfants: a population-based study. J Pediatr. 2003;143:186-191. Abstract available at:http://www.ncbi.nlm.nih.gov/pubmed/12970630

Thilo EH, Rosenberg AA. The newborn infant. In: Hay WW Jr, Levin MJ, Sondheimer JM,Deterding RR, eds. CURRENT Diagnosis & Treatment: Pediatrics. 19th ed. New York, NY: TheMcGraw-Hill Companies; 2009:Chapter 1. Available for subscription at:http://www.accessmedicine.com/content.aspx?aID=3396500

Thureen PJ, Anderson MS, Hay WW Jr. The small-for-gestational age infant. NeoReviews.2001;2:e139-e149. Available for subscription at:http://neoreviews.aappublications.org/cgi/content/full/2/6/e139

Zaw W, Gagnon R, da Silva O. The risks of adverse neonatal outcome among preterm small forgestational age infants according to neonatal versus fetal growth standards. Pediatrics.2003;111:1273-1277. Available at:http://pediatrics.aappublications.org/cgi/content/full/111/6/1273

Copyright © 2010 by the American Academy of Pediatrics page 365

Page 366: AAP PREP 2010

2010 PREP SA on CD-ROM

Question: 147

You are evaluating 5- and 7-year-old siblings who were brought to your office because of scalpitching for 3 days. Their mother denies similar symptoms in other family members and is unawareof similar symptoms in schoolmates. Both boys frequently scratch their heads during theexamination, and excoriations are visible behind their ears. One of the boys has small whiteflecks in his hair (Item Q147).

Of the following, the MOST appropriate treatment for this condition is

A. oral ivermectin

B. oral mebendazole

C. topical crotamiton

D. topical lindane

E. topical permethrin

Copyright © 2010 by the American Academy of Pediatrics page 366

Page 367: AAP PREP 2010

2010 PREP SA on CD-ROM

Preferred Response: ECritique: 147

The boys described in the vignette have signs of head lice infestation (pediculosis), which iscaused by the head louse Pediculus humanus capitis. This condition affects millions of children,primarily those of school age, each year. Itching of the scalp is the most common symptom,although it is not always present. Secondary excoriations are common, and bacterialsuperinfection may occur. The diagnosis can be made by direct viewing of a live louse on thehair shaft. Another sign of potential active infestation is the presence of white structures (nits)on the hair shaft, which may represent either the eggs themselves or the empty egg cases thatare left behind as the eggs hatch. Empty egg cases are whiter than eggs and, therefore, areseen more easily. The location of the nits on the hair shaft also may be helpful in determining ifthe infection is active. Those that are closer to the scalp are more likely to be freshly laid eggs;those further away from the scalp likely represent egg cases (they are further away becauseof hair growth and have likely been there longer).

Head lice do not carry a risk of disease. The primary morbidity is school absence becausemany schools are reluctant to readmit children if nits are still visible. However, because nits,particularly when white and located far from the scalp, likely are not viable, exclusion fromschool is not warranted. Therefore, children may return to school after the first treatment with atopical pediculicide.

The treatment for head lice should be both ovicidal and insecticidal. Topical permethrin is themost widely available therapy. The 1% cream rinse should be applied for 10 minutes to washedand towel-dried hair. Permethrin has excellent ovicidal and insecticidal activity and is effectivefor 2 weeks, but most experts recommend a second dose in 7 to 10 days. Topical lindane nolonger is a first-line treatment for head lice because of concerns about neurotoxicity. It may beused in shampoo form for children who have failed pediculosis treatment with other agents. Oralivermectin, administered as two doses 7 to 10 days apart, may be effective treatment, but it isnot approved for this indication by the United States Food and Drug Administration (FDA). Topicalcrotamiton has been used effectively for the treatment of scabies and also may be effective forthe treatment of pediculosis, but it is not approved by the FDA for head lice. Oral mebendazole isnot indicated for the treatment of head lice. Another potential treatment option for head lice istopical malathion, but it is more expensive than permethrin. Application of occlusive agents suchas petrolatum has been proposed as a treatment, but its efficacy has not been proven.

In addition to adequate pharmacologic treatment, parents should take care to disinfestheadgear, pillow cases, and towels by washing in hot water and drying in a hot dryer. Drycleaning is another effective method for removing lice from fabrics. Hair care instruments maybe washed in pediculicide shampoo. Bedmates and close household contacts should be treatedprophylactically, but this is not necessary for classroom contacts.

As a result of reviewing this information, do you intend to make a change in practiceto provide better patient care?Yes No

References:

Copyright © 2010 by the American Academy of Pediatrics page 367

Page 368: AAP PREP 2010

2010 PREP SA on CD-ROM

American Academy of Pediatrics. Pediculosis capitis (head lice). In: Pickering LK, Baker CJ,Kimberlin DW, Long SS, eds. Red Book: 2009 Report of the Committee on Infectious Diseases.28th ed. Elk Grove Village, Ill: American Academy of Pediatrics; 2009:495-497

Broomfield D. In brief: head lice. Pediatr Rev. 2002;23:34-35. Available at:http://pedsinreview.aappublications.org/cgi/content/full/23/1/34

Copyright © 2010 by the American Academy of Pediatrics page 368

Page 369: AAP PREP 2010

2010 PREP SA on CD-ROM

Question: 148

The parents of a 1-week-old female infant page you on a Sunday afternoon with concernsabout a dime-sized spot of blood in her diaper.

Of the following, the MOST accurate explanation of the cause of this bleeding is that it resultsfrom withdrawal of maternal serum

A. beta human chorionic gonadotropin

B. estrogen

C. oxytocin

D. progesterone

E. relaxin

Copyright © 2010 by the American Academy of Pediatrics page 369

Page 370: AAP PREP 2010

2010 PREP SA on CD-ROM

Preferred Response: BCritique: 148

The female infant described in the vignette has blood in her diaper that is likely due tomaternal estrogen withdrawal. Maternal estrogen causes the development of endometrium in thefemale fetus. Endometrial shedding occurs when the estrogen concentrations decrease afterbirth. This small quantity of vaginal bleeding often is termed "pseudomenses." Withdrawal ofmaternal progesterone, beta human chorionic gonadotropin, oxytocin, and relaxin do not result invaginal bleeding.

Vaginal bleeding may occur as early as 2 to 3 days after delivery but often is seen duringthe second week. If bleeding occurs after 3 to 4 weeks of age, it is unlikely to be due toestrogen withdrawal and necessitates a search for underlying disease such as congenitaladrenal hyperplasia, trauma, hemangioma of the genital tract, or other lesion.

References:

Sanfilippo JS. Gynecologic problems of childhood: history and physical examination. In: KliegmanRM, Behrman RE, Jenson HB, Stanton BF, eds. Nelson Textbook of Pediatrics. 18th ed.Philadelphia, Pa: Saunders Elsevier; 2007:2273

Uli N, Chin D, David R, et al. Menstrual bleeding in a female infant with congenital adrenalhyperplasia: altered maturation of the hypothalamic-pituitary-ovarian axis. J Clin EndocrinolMetab. 1997;82:3298-3302. Available at: http://jcem.endojournals.org/cgi/content/full/82/10/3298

Copyright © 2010 by the American Academy of Pediatrics page 370

Page 371: AAP PREP 2010

2010 PREP SA on CD-ROM

Question: 149

You are supervising pediatric residents in an urgent care clinic. One of the residents presents apatient who has a history of systemic lupus erythematosus and is well known to the clinic. Shepresents today with a decreased appetite and emesis. The resident is concerned about thepatient’s appearance. When you join her in the patient’s room, you find a pale, diaphoretic 15-year-old girl who has a heart rate of 130 beats/min and cool, mottled extremities. Hercardiovascular examination reveals distant heart sounds, no murmurs, and jugular venousdistention.

Of the following, the BEST explanation for this patient’s current findings is

A. arrhythmia

B. dehydration

C. pericardial tamponade

D. pleural effusion

E. sepsis

Copyright © 2010 by the American Academy of Pediatrics page 371

Page 372: AAP PREP 2010

2010 PREP SA on CD-ROM

Preferred Response: CCritique: 149

The patient described in the vignette is in shock with decreased perfusion. Jugular venousdistention results from increased filling pressure in the right atrium, a direct result of fluid and,thus, pressure in the pericardial space (pericardial tamponade). The pressure is transmitted tothe thin-walled right atrium and subsequently the venous return of the superior and inferior venacavae. As these veins drain by passive (nonpulsatile) flow, any increase in the filling pressurerequires that they overcome a greater pressure. As the filling of the right atrium decreases, sotoo does its stroke volume. With progression, tachycardia develops in an effort to maintaincardiac output: cardiac output = heart rate x stroke volume. The diminished cardiac output leadsto cardiogenic shock and the resulting signs and symptoms of pallor; diaphoresis; and poorlyperfused, cool, mottled extremities noted in this patient. The distant heart sounds are the resultof excessive fluid in the pericardium creating even further space between the stethoscope andthe heart.

Although the blood pressure was not reported for this patient, it is not uncommon for thosewho have large pericardial effusion and tamponade to demonstrate pulsus paradoxus. Pulsusparadoxus is defined as a decrease in the blood pressure of more than 10 mm Hg duringinspiration. Normally, a mild (4— to 6-mm Hg) decrease in blood pressure occurs as the negativeintrathoracic pressure in combination with increased pulmonary capacitance leads to adecrease in pulmonary venous return to the left atrium and a resulting decrease in leftventricular output. When this normal physiologic phenomenon is exaggerated, such as in casesof significant pericardial effusion, severe asthma, and other respiratory diseases, it is referredto as pulsus paradoxus.

Arrhythmia, dehydration, pleural effusion, or sepsis may lead to critical illness, but thealarming constellation of poor peripheral perfusion, jugular venous distention, distant heartsounds, diaphoresis, and an underlying condition that is associated with pericardial diseasedescribed for the patient in the vignette point specifically to pericardial tamponade.

References:

Issenberg HJ. Miscellaneous topics. In: Chang AC, Hanley FL, Wernovsky G. Wessel DL, eds.Pediatric Cardiac Intensive Care. Baltimore, Md: Williams & Wilkins; 1998:507-518

Kühn B, Peters J, Marx GR, Breitbart RE Etiology, management, and outcome of pediatricpericardial effusions. Pediatr Cardiol. 2008;29:90-94. Abstract available at:http://www.ncbi.nlm.nih.gov/pubmed/17674083

Nowlen TT, Bricker JT. Pericardial disease. In: Moller JH, Hoffman JIE, eds. PediatricCardiovascular Medicine. Philadelphia, Pa: Churchill Livingstone; 2000:780-792

Copyright © 2010 by the American Academy of Pediatrics page 372

Page 373: AAP PREP 2010

2010 PREP SA on CD-ROM

Question: 150

During the health supervision visit for a 6-week-old boy, his father expresses concern that hisson "doesn't look like" his other children. Growth parameters are normal except for a headcircumference of 35.5 cm (less than the 5th percentile). On physical examination, you note thatthe infant does not appear to fixate or track your face visually. There is "slip through" on verticalsuspension and "draping over" on horizontal suspension (Item Q150). Deep tendon reflexes arebrisk. Moro reflex is present and brisk.

Of the following, the MOST likely cause of this infant's hypotonia is

A. anterior horn cell disease

B. congenital brain malformation

C. congenital myasthenic syndrome

D. congenital myopathy

E. spinal cord disease

Copyright © 2010 by the American Academy of Pediatrics page 373

Page 374: AAP PREP 2010

2010 PREP SA on CD-ROM

Preferred Response: BCritique: 150

The diagnostic process for a "floppy" infant involves the same thinking process as for acuteweakness, with the primary difference being the urgency. Chronic generalized hypotonia at ahealth supervision visit does not require emergency evaluation unless there is difficultybreathing. The first step is to try to determine the responsible neurologic level, based on historyand physical examination. The brain, brainstem, spinal cord, anterior horn cell, root, nerve,junction, and muscle should be considered systematically in each case.

Three findings for the child described in the vignette localize his problem, at least in part, tothe cerebrum. First, and most importantly, the head circumference of this 6-week-old isessentially that of a newborn. It is important to determine by reviewing prior measurementswhether head growth has arrested or the microcephaly was present at birth. Second, the infantis not tracking visually. Because this could be an eyeball or brainstem problem, it is important toassess the red reflex, pupil reactions, and vestibular ocular reflex (manually move the headbriskly but gently to each side and determine if the eyes move the expected, opposite direction).In the presence of microcephaly, cortical visual processing problems are more likely than eye orbrainstem problems. Finally, the infant’s reflexes are brisk, which is usually indicative of brainpathology. Absent reflexes point to a peripheral nervous system problem. The compilation ofthese findings suggests that the most likely cause is a congenital brain malformation.

Spinal cord lesions can cause weakness and hypotonia caudal to the level of the lesion, butthey do not result in a small head or poor visual tracking. Similarly, spinal muscularatrophy/anterior horn cell disease, congenital myasthenia (neuromuscular junction dysfunction),or congenital myopathies would not account for this symptom complex.

References:

Sarnat HB. Neuromuscular disorders: evaluation and investigation. In: Kliegman RM, Behrman RE,Jenson HB, Stanton BF, eds. Nelson Textbook of Pediatrics. 18th ed. Philadelphia, Pa: SaundersElsevier; 2007:2531-2533

Sarnat HB. Disorders of neuromuscular transmission and of motor neurons. In: Kliegman RM,Behrman RE, Jenson HB, Stanton BF, eds. Nelson Textbook of Pediatrics. 18th ed. Philadelphia,Pa: Saunders Elsevier; 2007:2554-2558

Copyright © 2010 by the American Academy of Pediatrics page 374

Page 375: AAP PREP 2010

2010 PREP SA on CD-ROM

Question: 151

A white couple has come to you for preconceptional genetic counseling. A review of their familyhistories reveals that the man has two full siblings who have cystic fibrosis (CF) and one fullsibling who is unaffected. He is healthy. The woman does not know of anyone in her family whohas CF, and the remainder of their family histories is noncontributory.

Of the following, the likelihood that the man is a CF carrier is CLOSEST to

A. <1/100

B. 1/20

C. 1/4

D. 1/2

E. 2/3

Copyright © 2010 by the American Academy of Pediatrics page 375

Page 376: AAP PREP 2010

2010 PREP SA on CD-ROM

Preferred Response: ECritique: 151

Cystic fibrosis (CF) is an autosomal recessive condition. Mutations in the cystic fibrosistransmembrane conductance regulator (CFTR) gene on chromosome 7 cause CF. The conditionis "recessive" because both copies of the CFTR gene must be altered for the condition to occur.Individuals who have a mutation in only one copy of the gene are "carriers," and they arehealthy.

Each of the parents of the man in the vignette carries a CFTR mutation because they havetwo affected children. For such a couple, each time they have a child, there are four possible

outcomes related to the CFTR gene mutation:1)They could each pass on the normal copy of their gene, and their child would neither

have CF nor be a carrier.2)The mother could pass on her mutated gene, the father could pass on his normal copy,

and their child would be a carrier and healthy.3)The father could pass on his mutated gene, the mother could pass on her normal copy,

and their child would be a carrier and healthy.

4)They could each pass on their mutated gene, and their child would be affected.Three of these outcomes result in a child who does NOT have CF. Of these three outcomes,

two individuals are carriers for CF. Therefore, the likelihood that this healthy man is a CF carrieris 2/3.

To calculate the couple’s risk of having a child affected by CF, the man’s risk to be a carrier(2/3) is multiplied by the woman’s risk (1/29 in the white population), and that result is multipliedby the likelihood that two carriers would have an affected child (1/4). Therefore, the risk to thiscouple is 1 in 174. When determining recurrence risk for autosomal recessive conditions, it isimportant to recognize that carrier frequency may vary with ethnicity.

References:

Moskowitz SM, Chmiel JF, Sternen DL, Cheng E, Cutting GR. CFTR-related disorders.GeneReviews. 2008. Available at:http://www.ncbi.nlm.nih.gov/bookshelf/br.fcgi?book=gene&part=cf

Nussbaum RL, McInnes RR, Willard HF. Clinical case studies illustrating genetic principles. In:Thompson and Thompson Genetics in Medicine. 7th ed. Philadelphia, Pa: Saunders Elsevier;2007:231-322

Read A, Donnai D. When is screening useful? Cases 2, 21. New Clinical Genetics. Bloxham,Oxfordshire, United Kingdom: Scion Publishing Ltd; 2007:279-281

Copyright © 2010 by the American Academy of Pediatrics page 376

Page 377: AAP PREP 2010

2010 PREP SA on CD-ROM

Question: 152

A 15-year-old girl complains of significant pain with her monthly menstruation that results in hermissing school for 1 day each month. The pain is worse on the first day and subsidesspontaneously over the next 2 days. She has tried ibuprofen and naproxen sodium with norelief. On physical examination, you note pustular acne diffusely over her face and trunk. Otherfindings are normal.

Of the following, the medication that is MOST likely to be of benefit for both of this girl’s problemsis

A. acetaminophen

B. diuretic with menses

C. isotretinoin

D. omega-3 fatty acids

E. oral hormonal contraception

Copyright © 2010 by the American Academy of Pediatrics page 377

Page 378: AAP PREP 2010

2010 PREP SA on CD-ROM

Preferred Response: ECritique: 152

Hormonal methods of contraception are associated with health benefits beyond pregnancyprevention. An understanding of the noncontraceptive benefits of these birth control methodscan increase compliance and continuation of use, thus decreasing unplanned pregnancy. Thenoncontraceptive benefits include a decrease in ovarian cyst formation as well as ovarian,endometrial, and colorectal cancer. They offer some protection against salpingitis and ectopicpregnancies and decrease benign breast disease and acne. They also decrease dysmenorrheaand menorrhagia, reduce iron deficiency anemia and osteoporosis, and decrease the symptomsof polycystic ovarian syndrome. Accordingly, the girl described in the vignette may experiencerelief of her dysmenorrhea and some improvement in her acne with oral hormonal contraception.

Diuretics have been used for the treatment of bloating and weight gain associated withmenstruation. Spironolactone, a potassium-sparing diuretic and androgen receptor blocker, hasachieved the greatest popularity, largely as a result of specific properties that make this agentuniquely suited to hormonally based disorders. Other diuretics have been tried for treatment ofpremenstrual syndrome, with variable success in alleviating water retention symptoms.However, diuretics are not effective in the management of dysmenorrhea.

Acetaminophen does not target the prostaglandins causing menstrual cramps. Earlyevidence suggested possible benefits of fish oil/omega-3 fatty acids for women who havedysmenorrhea, but further studies are required to confirm these findings. Isotretinoin is indicatedfor nodular acne or severe acne that is unresponsive to conventional therapies but would notimprove dysmenorrhea.

As a result of reviewing this information, do you intend to make a change in practiceto provide better patient care?Yes No

References:

Rager KM, Omar HA. Hormonal contraception: noncontraceptive benefits and medicalcontraindications. Adolesc Med Clin. 2005;16:539-551. Abstract available at:http://www.ncbi.nlm.nih.gov/pubmed/16183538

Rimsza ME. Counseling the adolescent about contraception. Pediatr Rev. 2003;24:162-170.Available at: http://pedsinreview.aappublications.org/cgi/content/full/24/5/162

Trussell J. Choosing a contraceptive: efficacy, safety, and personal considerations. In: HatcherRA, Trussell J, Nelson AL, Cates W Jr, Stewart FH, Kowal D. Contraceptive Technology. 19thed. New York, NY: Ardent Media, Inc; 2007:19-48

Copyright © 2010 by the American Academy of Pediatrics page 378

Page 379: AAP PREP 2010

2010 PREP SA on CD-ROM

Question: 153

A mother brings her 6-month-old boy to the emergency department because of decreasedactivity and poor feeding for the past 2 days. She says that he has not had a fever. In theemergency department, his temperature is 37.0°C, heart rate is 150 beats/min, respiratory rate is45 breaths/min, and blood pressure is 65/40 mm Hg. His capillary refill time is greater than 4seconds, and peripheral pulses are difficult to palpate. An initial chest radiograph is obtained(Item Q153A). The emergency department staff obtain laboratory studies, order blood cultures,and administer a total of 60 mL/kg of isotonic fluid as well as antibiotics. The infant becomes lessresponsive, with a heart rate of 170 beats/min, respiratory rate of 60 breaths/min, bloodpressure of 50/28 mm Hg, and capillary refill time of greater than 6 seconds. A repeat chestradiograph is obtained (Item Q153B).

Of the following, the MOST appropriate next step is administration of

A. amphotericin B

B. dopamine infusion

C. isotonic fluid (20 mL/kg)

D. sodium bicarbonate

E. vasopressin infusion

Copyright © 2010 by the American Academy of Pediatrics page 379

Page 380: AAP PREP 2010

2010 PREP SA on CD-ROM

Preferred Response: BCritique: 153

The child described in the vignette is in shock, as evidenced by tachycardia and poorperfusion. Following aggressive resuscitation with fluid, he deteriorates clinically, andradiography documents cardiomegaly and pulmonary edema, which is highly suggestive ofcardiac disease (Item C153A). The appropriate treatment is respiratory stabilization, diureticadministration, and initiation of inotropic medications to improve cardiac function. The choice ofinotropic agent depends on the type of shock seen (Item C153B). Dopamine is a good choice inthis scenario due to its positive effect on contractility and systemic vascular resistance.

Shock, the inadequate delivery of oxygen to meet the metabolic demand of tissues, is animportant cause of morbidity and mortality in pediatrics. It is critical to recognize and treat shockpromptly because mortality rates of 20% to 50% have been reported. Shock is the culmination ofdisturbances in cardiac output and systemic vascular resistance and generally is divided intoseveral types (Item C153C).

Vasopressin is a powerful vasoconstrictor that has a role in severe shock, but because itoften causes tachycardia and can reduce splanchnic blood flow, it usually is reserved forshock resistant to standard inotropes. The chest radiograph for the child in the vignettedemonstrates significant pulmonary edema, which likely only would be worsened by additionalfluid boluses. Because there is no evidence of systemic fungal infection, amphotericin B therapyis unnecessary. Administration of sodium bicarbonate is not indicated because the patient’smetabolic status is not known and there is little documented evidence of its benefit in acuteresuscitation, especially in neonates and infants.

References:

Aschner JL, Poland RL. Sodium bicarbonate: basically useless therapy. Pediatrics.2008;122:831-835. Available at: http://pediatrics.aappublications.org/cgi/content/full/122/4/831

Brierly J, Carcillo JA, Choong J, et al. Clinical practice parameters for hemodynamic support ofpediatric and neonatal septic shock: 2007 update from the American College of Critical CareMedicine. Crit Care Med. 2009;37:666-688

Frankel LR, Kache S. Shock. In: Kliegman RM, Behrman RE, Jenson HB, Stanton BF, eds. NelsonTextbook of Pediatrics. 18th ed. Philadelphia, Pa: Saunders Elsevier; 2007:413-420

McKiernan CA, Lieberman SA. Circulatory shock in children: an overview. Pediatr Rev.2005;26:451-460. Available at: http://pedsinreview.aappublications.org/cgi/content/full/26/12/451

Copyright © 2010 by the American Academy of Pediatrics page 380

Page 381: AAP PREP 2010

2010 PREP SA on CD-ROM

Question: 154

You are covering your group’s pediatric practice over the weekend. A mother calls you at 3 pmSaturday afternoon to tell you that her 9-year-old daughter wet the bed the night before,although she has not been enuretic since she was a toddler. She is tired and has been nappingon and off all day. She also has been very thirsty for about a week, with increased thirst in thepast day. The mother says she looked these symptoms up on the Internet and is worried that herdaughter could have diabetes. On questioning, she reports that she has not noticed weight loss,and the girl’s appetite has been normal. A maternal great grandmother developed diabetes whenshe was 75 years old.

Of the following, the MOST appropriate action is to

A. arrange for blood tests and a urine culture at a local laboratory on Monday morning

B. arrange for her daughter to be seen as an outpatient on Sunday

C. reassure her and ask her to bring her daughter to the office on Monday

D. reassure her and ask her to come to the office if the symptoms persist for several days

E. tell her to bring her daughter to the local hospital emergency department immediately

Copyright © 2010 by the American Academy of Pediatrics page 381

Page 382: AAP PREP 2010

2010 PREP SA on CD-ROM

Preferred Response: ECritique: 154

More than 50% of children who have type 1 diabetes mellitus diagnosed in the United Statesare identified because of early symptoms and do not present initially in diabetic ketoacidosis.Early diagnosis is essential to preventing the serious consequences of uncompensated type 1diabetes. Increased public awareness of the symptoms of diabetes can decrease further thenumber of children who present with diabetic ketoacidosis.

Most children who have type 1 diabetes do not have an affected family member. Initialsymptoms of the disease are reflective of insulin deficiency, hyperglycemia, and glycosuria andinclude weight loss with increased appetite and thirst. Polyuria as a result of glycosuria maymanifest as frequent nocturnal urination or as secondary enuresis. Anorexia, continuedinsatiable thirst, nausea, and vomiting are late manifestations of uncompensated diabetesassociated with developing ketoacidosis. Coma and eventual death is the outcome of untreatedsevere hyperosmolar dehydration and acidosis.

The early diagnosis of diabetes can be particularly difficult in young children who still are indiapers and receive much of their nutrition in liquid form. Frequency of urination cannot be usedas a marker of hydration in vomiting children who have diabetes because urination reflects themarked glycosuria and osmotic diuresis, not hydration status. Symptoms of type 1 diabetes canworsen rapidly, and diabetic ketoacidosis can develop within hours. Therefore, if diabetes issuspected, as suggested by the symptoms described for the girl in the vignette, the child’s bloodand urine should be checked for glucose and ketones without delay. If glucose values areelevated, appropriate laboratory studies to define severity should be obtained, and the childshould be treated promptly.

As a result of reviewing this information, do you intend to make a change in practiceto provide better patient care?Yes No

References:

Cooke DW, Plotnick L. Type 1 diabetes mellitus in pediatrics. Pediatr Rev. 2008;29:374-385.Available at: http://pedsinreview.aappublications.org/cgi/content/full/29/11/374

Levitsky LL, Misra M. Epidemiology, presentation, and diagnosis of type 1 diabetes mellitus inchildren and adolescents. UpToDate Online 16.3. 2008. Available for subscription at:http://www.utdol.com/online/content/topic.do?topicKey=pediendo/16769

Levitsky LL, Misra M. Management of type 1 diabetes mellitus in children and adolescents.UpToDate Online 16.3. 2008. Available at:http://www.utdol.com/online/content/topic.do?topicKey=pediendo/16910

Copyright © 2010 by the American Academy of Pediatrics page 382

Page 383: AAP PREP 2010

2010 PREP SA on CD-ROM

Question: 155

The parents of a 30-month-old girl are concerned because she has sucked her thumb constantlysince she was an infant. They ask you if they should intervene to decrease this behavior.

Of the following, the MOST appropriate response is to

A. have them discipline her every time she sucks her thumb

B. have them put a bitter-tasting substance on her thumb

C. have them put gloves on her hands when she sleeps

D. reassure them that most children stop thumb sucking by 4 years of age

E. refer her to the dentist for an appliance to decrease thumb sucking

Copyright © 2010 by the American Academy of Pediatrics page 383

Page 384: AAP PREP 2010

2010 PREP SA on CD-ROM

Preferred Response: DCritique: 155

Thumb sucking, which tends to peak at 18 to 21 months of age, has onset in utero as earlyas 18 weeks’ gestation. Eighty percent of infants may suck their finger or toes. Thumb sucking isslightly more prevalent in girls than boys. Thumb sucking is viewed as a means of self-comforting and tends to occur most often when the child is falling asleep, tired, hungry, bored,or anxious. When thumb sucking occurs, the family should try to distract but not discipline thechild to avoid secondary gains.

Thumb sucking is normal in a child younger than 4 years of age and generally should beignored. For the child older than 4 years of age in whom the thumb sucking is frequent andproblematic, behavioral techniques may be used, such as gentle reminders and praise.Application of a bitter liquid to the thumb may serve as a reminder to help the child stop. Glovesmay be tried for the child who sucks his or her thumb at night. Referral to a pediatric dentist toplace an intraoral device may be useful for an older child to prevent malocclusion.

References:

Christophersen ER. Behavioral management: theory and practice. In: Parker S, Zukerman B,Augustyn M, eds. Developmental and Behavioral Pediatrics: A Handbook for Primary Care. 2nded. Philadelphia, Pa: Lippincott Williams & Wilkins; 2005:55-60

Christophersen ER. Management of habit disorders. In: Pediatric Compliance: A Guide for thePrimary Care Physician (Critical Issues in Developmental and Behavioral Pediatrics). NewYork, NY: Plenum Medical Book Company; 1994:261-278

Copyright © 2010 by the American Academy of Pediatrics page 384

Page 385: AAP PREP 2010

2010 PREP SA on CD-ROM

Question: 156

A 16-year-old girl comes to your office in August for evaluation of fever, headache with retro-orbital pain, and marked achiness of her joints and muscles. She returned 1 week ago from achurch mission trip to El Salvador. A number of other people on the trip were diagnosed with a

viral syndrome. Findings on physical examination are normal except for:

•Temperature of 39.1°C

•Heart rate of 104 beats/min

•Conjunctival injection without discharge

•Erythema of the pharynx without exudate

•Mild, tender hepatomegaly•Maculopapular rash on the trunk and scattered petechiae

Laboratory results include:

•White blood cell count, 3.2x103/mcL (3.2x109/L)

•Hemoglobin, 14.0 mg/dL (140 g/L)

•Hematocrit, 41.8% (0.418)

•Platelet count, 78x103/mcL (78x109/L)

•Aspartate aminotransferase, 212 units/L

•Alanine aminotransferase, 187 units/L

•Total bilirubin, 0.8 mg/dL (13.7 mcmol/L)

•Blood urea nitrogen, 18.0 mg/dL (6.2 mmol/L) •Creatinine, 0.8 mg/dL (70.7 mcmol/L)

Of the following, the MOST likely diagnosis is

A. dengue fever

B. Epstein-Barr virus infection

C. hepatitis A infection

D. malaria

E. typhoid fever

Copyright © 2010 by the American Academy of Pediatrics page 385

Page 386: AAP PREP 2010

2010 PREP SA on CD-ROM

Preferred Response: ACritique: 156

The fever, headache with retro-orbital pain, severe myalgias and arthralgias, conjunctivalinjection, pharyngeal erythema, hepatomegaly with mild elevation of liver enzyme values, rash,thrombocytopenia, and recent travel to El Salvador reported for the girl in the vignette all point tothe diagnosis of dengue fever. The extent of musculoskeletal symptoms has led to the illnessbeing called "break-bone fever." The diagnosis is made by serologic testing. Dengue virus is themost common arthropod-borne (arbo) virus disease in the world, with an estimated 100 millioncases occurring each year, an incidence that has been increasing in recent years. Denguefever has been reported in Southeast Asia, the South Pacific Islands, Latin America, and theCaribbean.

Infectious mononucleosis or acute Epstein-Barr virus infection can cause many of thesymptoms and laboratory abnormalities (thrombocytopenia, elevated liver enzymes) described inthe vignette, but the absence of splenomegaly and exudative pharyngitis and the extent of themyalgias and arthralgias are less typical of this condition. Similarly, the degree of systemiccomplaints is not consistent with hepatitis A infection. Malaria can present with many of thefeatures described for this girl, but the degree of myalgias and arthralgias is somewhat severefor non-falciparum malaria, which are the types seen in Central America. In addition, anemia,hypersplenism, and pallor would be present in malaria. Furthermore, the viral illnesses reportedin other members of the group are more consistent with a viral diagnosis than malaria. Typhoidfever also may manifest many of the complaints described for this girl, but the rash of typhoid(rose spots) is much more subtle, and abdominal pain generally is more prominent.

Arboviruses refer to diverse groups of viruses that are carried by mosquitoes or ticks intheir salivary glands and transmitted to people when the insect injects infective salivary fluidduring a blood meal. The primary hosts for the viruses are birds or rodents. After the tick ormosquito feeds on the infected animal, the virus replicates in the insect’s salivary glands, and aninfected arthropod feeding on a human may transmit the virus through the saliva into theperson’s bloodstream. Thus, these infections are prevented best through measures to controlmosquito and tick exposure. Humans are accidental hosts and do not participate in the ongoinglife cycle of these viruses.

Because of the critical role of mosquitos and ticks in the transmission of arboviruses, theseillnesses occur during peak vector activity. Thus, in the United States, arboviral illnesses usuallyoccur during summer and autumn, although in the southern states, cases may occur throughoutthe year. Each arbovirus has its own geographic distribution, based on the primary animal hostand particular mosquito and tick species involved in their life cycle, but such infections are seenworldwide.

Many of the arboviral infections involve the central nervous system, leading to encephalitis.Examples of causative pathogens are West Nile virus, eastern and western equine encephalitisvirus, St Louis encephalitis virus, LaCrosse encephalitis virus, and Japanese encephalitis virus.Each of these agents has its own unique geographic distribution.

Other arthropod-borne illnesses include hemorrhagic fevers and febrile illnesses, often withrashes or hepatitis. Dengue fever is the most prominent of such infections. Others includeColorado tick fever and yellow fever.

Copyright © 2010 by the American Academy of Pediatrics page 386

Page 387: AAP PREP 2010

2010 PREP SA on CD-ROM

References:

American Academy of Pediatrics. Arboviruses. In: Pickering LK, Baker CJ, Kimberlin DW, LongSS, eds. Red Book: 2009 Report of the Committee on Infectious Diseases. 28th ed. Elk GroveVillage, Ill: American Academy of Pediatrics; 2009:214-220

Centers for Disease Control and Prevention. Chapter 2. Pre- and post-travel general healthrecommendations: protection against mosquitoes, ticks, fleas and other insects and arthropods.In: Arguin PM, Kozarsky PE, Reed C, eds. Health Information for International Travel 2008.Atlanta, Ga: US Department of Health and Human Services, Public Health Service; 2007.Available at: http://wwwn.cdc.gov/travel/yellowBookCh2-InsectsArthropods.aspx

Wilder-Smith A, Schwartz E. Dengue in travelers. N Engl J Med. 2005;353:924-932. Extractavailable at: http://content.nejm.org/cgi/content/extract/353/9/924

Copyright © 2010 by the American Academy of Pediatrics page 387

Page 388: AAP PREP 2010

2010 PREP SA on CD-ROM

Question: 157

A 3-year-old previously healthy girl presents for evaluation of a rapidly worsening cellulitis onher back. Four days ago, she developed a temperature of 38.9°C and broke out with varicellalesions involving her face, trunk, back, and abdomen. Over the past 8 hours, her mother notedthat one of the varicella lesions on her back became very erythematous, with some mildsurrounding swelling and tenderness. On physical examination, the girl appears tired and quietand has a temperature of 38.6°C. She has a 4x4-cm area of induration underlying a 0.5x0.5-cmvaricella lesion, with a 6x6-cm area of surrounding erythema and duskiness. The area isexquisitely tender to palpation, and a 1x1-cm blister has formed adjacent to the varicella lesion.

A complete blood count shows a peripheral white blood cell count of 27.0x103/mcL

(27.0x109/L), with a differential count of 85% neutrophils, 5% band forms, 8% lymphocytes, and2% monocytes.

Of the following, in addition to parenteral antibiotic therapy, the MOST appropriate next step inthe management of this patient’s condition is

A. bedside incision and drainage

B. computed tomography scan

C. magnetic resonance imaging

D. operating room drainage

E. ultrasonography

Copyright © 2010 by the American Academy of Pediatrics page 388

Page 389: AAP PREP 2010

2010 PREP SA on CD-ROM

Preferred Response: DCritique: 157

Streptococcus pyogenes or group A beta-hemolytic Streptococcus (GABHS) is a part of thenormal flora of the skin, but if there is a break in the skin integrity, GABHS may cause infection ofthe skin and soft tissues (pyoderma, erysipelas, cellulitis) (Item C157A). The most serioussequelae of streptococcal skin infections are necrotizing fasciitis, toxic shock syndrome, andpoststreptococcal acute glomerulonephritis (PSAGN).

Necrotizing fasciitis is an infection of the deeper subcutaneous tissues and fascia that ischaracterized by extensive and rapidly spreading necrosis and gangrene of the skin andunderlying structures. The skin becomes dusky and subsequently purplish, with the appearanceof bullae containing yellow or hemorrhagic fluid (Item C157B). Management of necrotizingfasciitis requires a combination of parenteral antibiotic therapy and aggressive and extensivesurgical debridement in the operating room.

Streptococcal toxic shock syndrome may occur in patients who have GABHS invasive soft-tissue infections (eg, necrotizing fasciitis) and has clinical features that overlap with necrotizingfasciitis. The clinical features include the development of intense local pain, high fever, localizedswelling, and erythema at the site of local skin trauma. The infection progresses to thedevelopment of vesicles or bullae and the appearance of a violaceous or bluish hue. The patientrapidly develops hypotension and multiorgan failure. Mortality may be as high as 40%.

PSAGN is an acute inflammatory disorder of the renal glomerulus that is characterizedpathologically by diffuse proliferative glomerular lesions and clinically by edema, hypertension,hematuria, and proteinuria. It is a delayed nonsuppurative sequela of pharyngeal or cutaneousinfection with certain "nephritogenic" group A streptococcal strains.

The patient described in the vignette has a GABHS bacterial superinfection of her varicellalesions that has progressed to the development of a necrotizing fasciitis. Prompt and aggressivesurgical intervention in the operating room for drainage is critical in the management of thisinfection, and delays for any reason (eg, radiographic studies) should be avoided.

References:

American Academy of Pediatrics. Group A streptococcal infections. In: Pickering LK, Baker CJ,Kimberlin DW, Long SS, eds. Red Book: 2009 Report of the Committee on Infectious Diseases.28th ed. Elk Grove Village, Ill: American Academy of Pediatrics; 2009:616-628

Bisno AL, Stevens DL. Streptococcus pyogenes. In: Mandell GL, Bennett JE, Dolan R, eds.Mandell, Douglas, and Bennett’s Principles and Practice of Infectious Diseases. 6th ed.Philadelphia, Pa: Elsevier Churchill Livingstone; 2005:2362-2379

Jaggi P, Shulman ST. Group A streptococcal infections. Pediatr Rev. 2006;27:99-105. Availableat: http://pedsinreview.aappublications.org/cgi/content/full/27/3/99

Copyright © 2010 by the American Academy of Pediatrics page 389

Page 390: AAP PREP 2010

2010 PREP SA on CD-ROM

Question: 158

A 1-year-old girl presents for a health supervision visit. Her father underwent kidneytransplantation for end-stage renal disease due to autosomal dominant polycystic kidneydisease. The parents request assessment of the child’s kidney function. Her serum creatininemeasurement is normal for age.

Of the following, the MOST correct statement about serum creatinine measurements in thepediatric patient is that

A. a serum creatinine value of 0.6 mg/dL (53.1 mcmol/L) is considered normal for a 1-year-oldchild

B. healthy preterm infants (<36 weeks’ gestation) typically have lower serum creatinine valuesthan term infants

C. males and females have similar serum creatinine values up to 17 years of age

D. newborns in the first few days after birth have lower serum creatinine concentrations thanthose seen in 2-month-old infants

E. serum creatinine values are related more closely to patient height than to patient age inchildren

Copyright © 2010 by the American Academy of Pediatrics page 390

Page 391: AAP PREP 2010

2010 PREP SA on CD-ROM

Preferred Response: ECritique: 158

Kidney function is assessed best by measurement of the glomerular filtration rate (GFR).GFR typically is quantitated by measuring the plasma clearance of a particular marker (eg,creatinine) that is eliminated by the kidneys. An ideal marker is a substance that is filtered freelyat the glomerulus, not reabsorbed or secreted, and biologically inert (ie, neither metabolized norsynthesized by the kidney). The serum creatinine is the most frequently used agent to assessGFR clinically. Creatinine is derived from metabolism of the skeletal muscle protein creatine. Itsgeneration is essentially constant, but because creatinine concentrations correlate with musclemass, the pediatric patient, who has ever-increasing muscle mass due to growth, hasincreasing steady-state serum creatinine values with advancing pediatric age.

In general, males and females have equivalent creatinine values until early adolescence,when the increasing muscle mass of the male adolescent alters the "normal" creatinine values

seen in this age group. Normal GFR is approximately 100 mL/min per 1.73 m2 in females and 125

mL/min per 1.73 m2 in males. The GFR in the pediatric patient increases throughout the firstpostnatal year before reaching adult values by approximately 18 months of age.

Creatinine clearance is calculated with serum creatinine, urine creatinine, and 24-hour urinevolume. Due to difficulties with 24-hour urine collection in children, the Schwartz formula wascreated to allow the clinician to calculate an estimated GFR (eGFR) using the length or height of

the child corrected to standard units of mL/min per 1.73 m2. This formula (eGFR=kL/SCr) allowsthe pediatrician to estimate the GFR in infants, children, and adolescents with only the child’slength or height (L) and the serum creatinine. The k is a constant that varies, based on age andsex. The value for k up to 1 year of age is 0.45; the k value for children from 1 to 13 years ofage is 0.55. Due to considerations of increased muscle mass in adolescent males, the k valuefor males 13 to 21 years of age is 0.7, but this value remains 0.55 for adolescent females.

The 1-year-old child in the vignette has a serum creatinine that is stated to be normal forage. Because a 1-year-old is approximately 75 cm in length, for the eGFR to be normal at 100

mL/min per 1.73 m2, the serum creatinine would be SCr=kL/eGFR or (0.45 x 75)/100 = 0.3 mg/dL(26.5 mcmol/L). It is well known that serum creatinine is elevated in the newborn period andrepresents the maternal value. The values can remain elevated until approximately 10 days ofage, after which time the creatinine returns to infant values in the 0.2 to 0.4 mg/dL (17.7 to 35.4mcmol/L) range. Finney and associates demonstrated higher creatinine values in preterm infants(both 24 to 28 weeks’ and 29 to 36 weeks’ gestation) compared with term infants.

References:

Finney H, Newman DJ, Thakkar H, Fell JM, Price CP. Reference ranges for plasma cystatin C andcreatinine measurements in premature infants, neonates, and older children. Arch Dis Child.2000;82:71-75. Abstract available at: http://www.ncbi.nlm.nih.gov/pubmed/10630919

Rose BD, Post TW. Renal circulation and glomerular filtration rate. In: Clinical Physiology of Acid-base and Electrolyte Disorders. 5th ed. New York, NY: McGraw-Hill Medical Publishing Division;2001:21-70

Copyright © 2010 by the American Academy of Pediatrics page 391

Page 392: AAP PREP 2010

2010 PREP SA on CD-ROM

Schwartz GJ, Feld LG, Langford DJ. A simple estimate of glomerular filtration rate in full-terminfants during the first year of life. J Pediatr. 1984;104:849-854. Abstract available at:http://www.ncbi.nlm.nih.gov/pubmed/6726515

Schwartz GJ, Gauthier B. A simple estimate of glomerular filtration rate in adolescent boys. JPediatr. 1985;106:522-526. Abstract available at: http://www.ncbi.nlm.nih.gov/pubmed/3973793

Schwartz GJ. Clinical assessment of renal function. In: Kher KK, Schnaper HW, Makker SP, eds.Clinical Pediatric Nephrology. 2nd ed. London, England: Informa Healthcare; 2007:71-93

Copyright © 2010 by the American Academy of Pediatrics page 392

Page 393: AAP PREP 2010

2010 PREP SA on CD-ROM

Question: 159

A 5-year-old girl who has mild persistent asthma presents to the emergency department with a 4-day history of cough and shortness of breath that started after a viral upper respiratory tract

infection. She has used her beta2 agonist metered dose inhaler (MDI) and inhaled corticosteroidstwice each day without significant improvement. On physical examination, she coughsfrequently but is sitting comfortably. Her vital signs include a temperature of 36.9°C, respiratoryrate of 28 breaths/min, heart rate of 120 beats/min, and a pulse oximetry reading of 95% onroom air. She has scattered expiratory wheezing on auscultation but no crackles, retractions, orstridor. A chest radiograph shows some mild hyperinflation but also opacification of the rightmiddle lobe.

Of the following, the MOST appropriate treatment for this girl is to

A. admit her to the hospital for intravenous corticosteroids

B. continue her current medication

C. double her inhaled steroid dose

D. start a 3- to 5-day course of oral corticosteroids

E. start a 7- to 10-day course of antibiotics

Copyright © 2010 by the American Academy of Pediatrics page 393

Page 394: AAP PREP 2010

2010 PREP SA on CD-ROM

Preferred Response: DCritique: 159

The girl described in the vignette is suffering an exacerbation of her asthma. Signs andsymptoms of asthma exacerbations include cough, wheezing, shortness of breath, hypoxia,tachypnea, and atelectasis. Treatment depends on the age of the patient, current medicationuse, and severity of the episode. Mild exacerbations may be treated at home with the assistance

of a physician-directed action plan. Initial therapy involves the use of a rapid-acting beta2agonist.

For mild exacerbations treated at home, high doses of inhaled corticosteroids may avert theprogression of symptoms. For moderate-to-severe exacerbations, as experienced by thispatient, oral or intravenous corticosteroids are required. A 3- to 5-day course of oral steroids isadequate for most children, providing a similar outcome compared with intravenous steroids.

The need for hospital admission typically is based on the response to therapy in the clinic oremergency department. Hospital admission would be considered if a patient presents withcyanosis, apnea, history of previous intensive care unit admission or respiratory failure, peak

expiratory flow less than 60% of predicted after beta2 agonist therapy, increased Pco2 on bloodgas analysis, or requirement for persistent nebulization therapy or supplemental oxygen.

A common practice for many clinicians is to increase the dose of inhaled corticosteroidsduring an asthma exacerbation but not start oral steroids. One study supported quadrupling thedose during an asthma exacerbation, but other studies, as well as current asthma guidelines,recommend oral steroids as the mainstay therapy for an exacerbation that does not respond to

beta2 agonists.National Asthma Education and Prevention Program guidelines address the issue of

antibiotics for asthma exacerbations. Clear manifestations of a bacterial infection should bepresent before beginning antibiotic therapy during an asthma exacerbation. Because viralinfections are the most common trigger in younger children, as described in the vignette,antibiotics are not indicated.

Chest radiography for the girl in the vignette documents right middle lobe atelectasis,characterized by a wedge-shaped density that extends anteriorly and inferiorly from the hilum ofthe lung. This finding often is diagnosed inappropriately as pneumonia and treated withantibiotics. Atelectasis due to mucus plugging is not an uncommon finding in children who haveasthma exacerbations. Right middle lobe atelectasis and other scattered areas of atelectasisusually respond to standard asthma therapy (eg, oral corticosteroids, inhaled asthmamedications) and do not require bronchoscopy or chest physiotherapy.

As a result of reviewing this information, do you intend to make a change in practiceto provide better patient care?Yes No

References:

Guill MF. Asthma update: clinical aspects and management. Pediatr Rev. 2004;25:335-344.Available at: http://pedsinreview.aappublications.org/cgi/content/full/25/10/335

Copyright © 2010 by the American Academy of Pediatrics page 394

Page 395: AAP PREP 2010

2010 PREP SA on CD-ROM

Lemanske RF Jr, Busse WW. 6. Asthma. J Allergy Clin Immunol. 2003;111(2 suppl):S502-S519.Abstract available at: http://www.ncbi.nlm.nih.gov/pubmed/12592297

Liu AH, Covar RA, Spahn JD, Leung DYM. Childhood asthma. In: Kliegman RM, Behrman RE,Jenson HB, Stanton BF, eds. Nelson Textbook of Pediatrics. 18th ed. Philadelphia, Pa: SaundersElsevier; 2007:953-969

National Heart, Lung, and Blood Institute and the National Asthma Education and PreventionProgram. Expert Panel Report 3 (EPR3): Guidelines for the Diagnosis and Management ofAsthma. Bethesda, Md: U.S. Department of Health and Human Services; 2007. Available athttp://www.nhlbi.nih.gov/guidelines/asthma/asthgdln.htm

Schuh S, Dick PT, Stephens D, et al. High-dose inhaled fluticasone does not replace oralprednisolone in children with mild to moderate acute asthma. Pediatrics. 2006;118:644—650.Available at: http://pediatrics.aappublications.org/cgi/content/full/118/2/644

Copyright © 2010 by the American Academy of Pediatrics page 395

Page 396: AAP PREP 2010

2010 PREP SA on CD-ROM

Question: 160

A 10-year-old football player is brought to the emergency department on a hot and humid day 30minutes after being found wandering the sidelines with altered mental status. According to thecoach, 1 hour into the second of two-a-day football practices, the boy became disoriented afterrunning sprints. In the emergency department, the patient is moaning incomprehensibly, histemperature is 39.2°C, heart rate is 140 beats/min, respiratory rate is 24 breaths/min, and bloodpressure 100/30 mm Hg. His pupils are 4 mm and sluggishly reactive. There is no history of headtrauma.

Of the following, the MOST likely electrolyte abnormality in this patient is

A. hypercalcemia

B. hyperglycemia

C. hyperkalemia

D. hypermagnesemia

E. hypernatremia

Copyright © 2010 by the American Academy of Pediatrics page 396

Page 397: AAP PREP 2010

2010 PREP SA on CD-ROM

Preferred Response: ECritique: 160

The altered mental status, tachycardia, hypotension, widened pulse pressure, and elevatedbody temperature after vigorous physical activity in a hot environment described for the boy inthe vignette are characteristic of exertional heat stroke (EHS). Although the strict definition ofEHS includes a body temperature of greater than 40.0°C and anhidrosis, younger individualswho have EHS often retain their ability to sweat and may cool down en route to medicalattention. The diagnosis should not be discounted in patients who develop mental statuschanges during exertion, even if they are sweating and have temperatures less than 40.0°C.

Because sweat is hypotonic, affected patients have had hypotonic fluid losses fromexcessive sweating. Thus, without adequate replacement of intravascular fluid deficits,hypernatremia is the most common electrolyte abnormality seen early in the course of EHS.Hypoglycemia and hypokalemia also may be seen. If rhabdomyolysis develops, hyperkalemia,hyperphosphatemia, hypocalcemia, and hypomagnesemia may occur later.

Treatment of EHS begins with aggressive active external cooling to less than 39.0°C inconjunction with airway management, oxygen delivery, and treatment of hypovolemic shockwith fluid resuscitation. The patient should be monitored for pulmonary edema, rhabdomyolysis,disseminated intravascular coagulation, and hepatic failure.

Risk factors for the development of EHS include dehydration, fatigue, and inadequateconditioning or acclimatization. EHS is seen frequently when sport practices begin for theseason in midsummer. Prepubertal children are at particular risk because of relatively large bodysurface area-to-mass, decreased sweat gland efficiency, and decreased ability to acclimatize.

Prevention of EHS includes aggressive monitoring of hydration before, during, and afterathletic events; providing adequate water supplies; ensuring adequate rest prior to practice andevents; and adapting athletes to environmental conditions over 10 to 14 days.

References:

Binkley HM, Beckett J, Casa DJ, Kleiner DM, Plummer PE. National Athletic Trainers' Associationposition statement: exertional heat illnesses. J Athl Train. 2002;37:329—343. Available at:http://www.pubmedcentral.nih.gov/articlerender.fcgi?tool=pubmed&pubmedid=12937591

Casa DJ, Armstrong LE, Hillman SK, et al. National Athletic Trainers' Association positionstatement: fluid replacement for athletes. J Athl Train. 2000;35:212—224. Available at:http://www.pubmedcentral.nih.gov/articlerender.fcgi?tool=pubmed&pubmedid=16558633

Sawka MN, Montain SJ. Fluid and electrolyte supplementation for exercise heat stress. Am JClin Nutr. 2000;72(2 suppl):564S-572S. Available at:http://www.ajcn.org/cgi/content/full/72/2/564S

Copyright © 2010 by the American Academy of Pediatrics page 397

Page 398: AAP PREP 2010

2010 PREP SA on CD-ROM

Question: 161

A 9 year-old boy presents with right and left lower quadrant abdominal pain of several months’duration, which now is increasing in frequency and severity. The pain occurs at any time of dayor night and has awakened him from sleep on several occasions. Past history includes anappendectomy at age 6 years and a tonsilloadenoidectomy at age 8 years. On physicalexamination, his abdomen appears moderately distended and, while being examined, he againbegins to feel discomfort and vomits a moderate amount of yellow-green fluid. You order anabdominal plain radiograph (Item Q161).

Of the following, the MOST appropriate next step is to:

A. begin intravenous hydration and bowel rest

B. insert a nasogastric tube for decompression

C. obtain a barium enema

D. obtain abdominal computed tomography scan

E. refer the boy for surgical exploration

Copyright © 2010 by the American Academy of Pediatrics page 398

Page 399: AAP PREP 2010

2010 PREP SA on CD-ROM

Preferred Response: BCritique: 161

The concomitant findings of abdominal distention and an obstructive gas pattern onabdominal radiography (Item C161A) reported for the boy in the vignette strongly suggest eithera functional or mechanical small bowel obstruction. In adults, postoperative adhesions are theleading cause of small bowel obstruction, comprising nearly 75% of cases in one study.However, small bowel obstruction due to postoperative adhesions is much less common inchildren. In one pediatric study, the overall incidence of bowel obstruction was 1.1% during the5 years after primary surgery. The highest risk occurred after ileal surgery (9.2%), followed byexploratory laparotomy (6.5%) and duodenal surgery (4.7%). For children undergoingappendectomy, readmission for obstruction occurred in only 0.3% of cases.

Regardless of the specific cause, the initial goal of therapy for the child who has eitherfunctional or mechanical bowel obstruction is to relieve the associated signs and symptoms,manifested in the boy in the vignette by abdominal distention and bilious emesis, with the lattersign suggesting complete bowel obstruction. Relief can be achieved for this boy by immediateinstitution of nasogastric decompression. Although intravenous fluids and bowel rest eventuallywill be instituted, the first goal of therapy is intestinal decompression. Computed tomographyscan plays little primary role in management but may be useful in identifying suspectedobstructing lesions. Barium enema may be considered in children whose age and mode ofpresentation suggest a diagnosis of intussusception. Finally, emergency surgical interventionshould be undertaken only after achieving decompression and identifying the site of obstructionwith an upper gastrointestinal tract contrast radiographic series, at least to the ligament of Treitz(to detect intestinal malrotation) and with small intestinal follow-through if needed.

The overall incidence of postappendectomy adhesive small bowel obstruction is low, butadhesions develop with a much higher frequency when the surgical presentation is complicatedby appendiceal perforation. Lesions causing partial or complete luminal obstruction distal to theligament of Treitz often present with bilious vomiting. In the newborn period, this is an ominoussign that suggests a surgical diagnosis, including intestinal stenosis, atresias, and malrotationwith or without volvulus. Beyond infancy, the significance of bile-stained vomiting is less clear.In one report of 230 children presenting to an emergency department because of yellow orgreen emesis, only 20 were determined to have a surgical disease. In another pediatric study of44 patients who required a Ladd procedure for malrotation and midgut volvulus, 80% presentedwith green bilious emesis.

The most common causes of bilious vomiting are shown in Item C161B. Considering theclinical history and age at presentation of the boy in the vignette, postoperative adhesive smallintestinal obstruction represents the most likely diagnosis. Once nasogastric decompression isachieved, further management must include additional diagnostic studies and consideration ofsurgical intervention. Prior data have shown that plain roentgenograms alone often are sufficientfor identifying intestinal malrotation by demonstrating a characteristic gas pattern. In thenewborn period, bilious vomiting should be considered a consequence of mechanical bowelobstruction until proven otherwise. Once initial bowel decompression has been achieved, asmall intestinal contrast study can identify the nature and location of the obstructing lesion priorto surgery. In older patients, for whom malrotation with volvulus is much less prevalent, thetiming of additional diagnostic studies depends on the response to nonoperative management.

Copyright © 2010 by the American Academy of Pediatrics page 399

Page 400: AAP PREP 2010

2010 PREP SA on CD-ROM

Prior investigations in patients who had adhesive small intestine obstructions have demonstratedthat a significant percentage may respond to nasogastric decompression alone, obviating theneed for emergency surgery. Under these conditions, intestinal contrast studies using water-soluble contrast material may be used to confirm luminal integrity and guide further therapy.

References:

Abbas S, Bissett IP, Parry BR. Oral water soluble contrast for the management of adhesive smallbowel obstruction. Cochrane Database Syst Rev. 2007;3:CD004651. Available athttp://www.mrw.interscience.wiley.com/cochrane/clsysrev/articles/CD004651/frame.html

Fleshner PR, Siegman MG, Slater GI, Brolin RE, Chandler JC, Aufses AH Jr. A prospective,randomized trial of short versus long tubes in adhesive small-bowel obstruction. Am J Surg.1995;170:366-370. Abstract available at: http://www.ncbi.nlm.nih.gov/pubmed/7573730

Grant HW, Parker MC, Wilson MS, et al. Adhesions after abdominal surgery in children. J PediatrSurg. 2008;43:152-156. Abstract available at: http://www.ncbi.nlm.nih.gov/pubmed/18206474

Lloyd DA, Kenny SE. The surgical abdomen. In: Walker WA, Goulet O, Kleinman RE, Sherman PM,Shneider BL, Sanderson IR, eds. Pediatric Gastrointestinal Disease. 4th ed. Hamilton, Ontario,Canada: BC Decker; 2004:604-614

Miller G, Boman J, Shrier I, Gordon PH. Etiology of small bowel obstruction. Am J Surg.2000;180:33-36. Abstract available at: http://www.ncbi.nlm.nih.gov/pubmed/11036136

Murray KF, Christie DL. Vomiting. Pediatr Rev. 1998;19:337-341. Available at:http://pedsinreview.aappublications.org/cgi/content/full/19/10/337

Ryckman FC. Approach to the child with gastrointestinal obstruction. In Rudolph CD, Rudolph AM,Hostetter MK, Lister G, Siegel NJ, eds. Rudolph’s Pediatrics. 21st ed. New York, NY: McGraw-Hill Medical Publishing Division; 2003:1376-1379

Sadow KB, Atabaki SM, Johns CM, Chamberlain JM, Teach SJ. Bilious emesis in the pediatricemergency department: etiology and outcome. Clin Pediatr (Phila). 2002;41:475-479. Abstractavailable at: http://www.ncbi.nlm.nih.gov/pubmed/12365309

Copyright © 2010 by the American Academy of Pediatrics page 400

Page 401: AAP PREP 2010

2010 PREP SA on CD-ROM

Question: 162

You are called to examine a newborn male who weighs 2,480 g after a gestation of 42-5/7weeks. The physical examination reveals a normocephalic head, a positive red reflex bilaterally,clear lung fields upon auscultation, no heart murmur, and a soft abdomen without palpablevisceromegaly.

Of the following, the MOST likely additional physical finding expected for this postterm infant is

A. absence of scrotal rugae

B. acrocyanosis

C. dry, peeling skin

D. hypoplastic fingernails

E. poor elastic recoil of ear cartilage

Copyright © 2010 by the American Academy of Pediatrics page 401

Page 402: AAP PREP 2010

2010 PREP SA on CD-ROM

Preferred Response: CCritique: 162

Infants delivered 2 weeks or more after the normal 280-day gestational period (calculatedfrom the mother’s last menstrual period) are defined as postterm. Postterm (often termedprolonged) pregnancies complicate 10% of all live births in the United States and are associatedwith stillbirth, excess neonatal morbidity, and infant mortality. The postterm condition alsorenders the fetus as postmature, a term that reflects the senescence of the placenta andinability to contribute to further fetal growth.

Postterm infants generally are either appropriate-for-gestational age or large-for-gestationalage. Among the other findings are cracked, dry, peeling skin (Item C162); creases covering theentire sole of the foot; mature and long fingernails; absence of lanugo over the back; palpablebreast buds; a stiff, cartilaginous ear with ready recoil; a well-pigmented and rugated scrotum inboys; and labia majora that entirely cover the labia minora and clitoris in girls. Acrocyanosis is nomore common in postterm infants than in term infants, although the risk for meconium aspirationand pulmonary hypertension is increased in postterm infants.

References:

Bruckner TA, Cheng YW, Caughey AB. Increased neonatal mortality among normal-weight birthsbeyond 41 weeks of gestation in California. Am J Obstet Gynecol. 2008;199:421.e1-7. Abstractavailable at: http://www.ncbi.nlm.nih.gov/pubmed/18639211

Cunningham FG, Leveno KL, Bloom SL, Gilstrap LC III, Wenstrom KD. Postterm pregnancy. In:Cunningham FG, Leveno KL, Bloom SL, Hauth JC, Gilstrap LC III, Wenstrom KD, eds. WilliamsObstetrics. 22nd ed. New York, NY: The McGraw-Hill Companies, Inc; 2005: Chapter 37.Available for subscription at: http://www.accessmedicine.com/content.aspx?aID=732448

Dargaville PA, Copnell B for the Australian and New Zealand Neonatal Network. Theepidemiology of meconium aspiration syndrome: incidence, risk factors, therapies, and outcome.Pediatrics. 2006;117:1712-1721. Available at:http://pediatrics.aappublications.org/cgi/content/full/117/5/1712

Hernández-Díaz S, Van Marter LJ, Werler MM, Louik C, Mitchell AA. Risk factors for persistentpulmonary hypertension of the newborn. Pediatrics. 2007;120:e272-e282. Available at:http://pediatrics.aappublications.org/cgi/content/full/120/2/e272

Lissauer T. Physical examination of the newborn. In: Martin RJ, Fanaroff AA, Walsh MC, eds.Fanaroff and Martin's Neonatal-Perinatal Medicine: Diseases of the Fetus and Infant. 8th ed.Philadelphia, Pa: Mosby Elsevier; 2006:513-528

McLean FH, Boyd ME, Usher RH, Kramer MS. Postterm infants: too big or too small? Am J ObstetGynecol. 1991;164:619-624. Abstract available at:http://www.ncbi.nlm.nih.gov/pubmed/1992713

Copyright © 2010 by the American Academy of Pediatrics page 402

Page 403: AAP PREP 2010

2010 PREP SA on CD-ROM

Norwitz ER. Postterm pregnancy. UpToDate Online 16.3. 2008. Available for subscription at:http://www.utdol.com/online/content/topic.do?topicKey=pregcomp/14258&selectedTitle=1~24&source=search_result630

Copyright © 2010 by the American Academy of Pediatrics page 403

Page 404: AAP PREP 2010

2010 PREP SA on CD-ROM

Question: 163

A 12-year-old girl is brought to your office with the complaint of right earache for the past week.She denies fever and drainage from the ear and otherwise feels well. You examine both earsand discover normal findings, so you ask further questions to investigate causes for referredear pain. The mother reports that the girl often grinds her teeth at night, which makes yoususpect temporomandibular joint (TMJ) dysfunction as the cause for her ear pain.

Of the following, the additional historical feature that is MOST suspicious for TMJ dysfunction is

A. gingival tenderness

B. headache

C. neck stiffness

D. sore throat

E. tinnitus

Copyright © 2010 by the American Academy of Pediatrics page 404

Page 405: AAP PREP 2010

2010 PREP SA on CD-ROM

Preferred Response: BCritique: 163

Earache (otalgia) is a common complaint in pediatrics. Primary otalgia is caused by anabnormality of the ear itself, such as otitis media or otitis externa. When the ear appears normalon physical examination, as for the girl in the vignette, causes of secondary otalgia should beinvestigated. These include temporomandibular joint (TMJ) dysfunction, dental disease,pharyngitis, and cervical spine abnormalities. Careful evaluation of the oropharynx and theremainder of the head and neck is indicated to look for evidence of these problems.

TMJ dysfunction refers to conditions in the TMJ that cause pain, limited movement, or clickingon movement. The exact cause is unknown, but abnormal dental occlusion, teeth grinding(bruxism), stress, or abnormalities in the joint itself may be contributing factors. Besides ear andfacial pain with mastication, patients commonly complain of nonspecific headache. Findings onphysical examination may include clicking of the joint, which also is present in many people whodo not have TMJ dysfunction, and pain or swelling over the joint.

Gingival tenderness is most suggestive of a dental cause for ear pain. Neck stiffness islikely in a patient who has cervical spine arthritis or another cervical spine abnormality. Sorethroat probably indicates pharyngitis. Tinnitus, although occasionally reported with TMJdysfunction, is not as common as headache in affected patients.

References:

Auvenshine RC. Temporomandibular disorders: associated features. Dent Clin North Am.2007;51:105-127. Abstract available at: http://www.ncbi.nlm.nih.gov/pubmed/17185062

Bonjardim LR, Gaviao MB, Carmagnani FG, Pereira LJ, Castelo PM. Signs and symptoms oftemporomandibular joint dysfunction in children with primary dentition. J Clin Pediatr Dent.2003;28:53-58. Abstract available at: http://www.ncbi.nlm.nih.gov/pubmed/14604143

Ely JW, Hansen MR, Clark EC. Diagnosis of ear pain. Am Fam Physician. 2008;77:621-628.Abstract available at: http://www.ncbi.nlm.nih.gov/pubmed/18350760

Copyright © 2010 by the American Academy of Pediatrics page 405

Page 406: AAP PREP 2010

2010 PREP SA on CD-ROM

Question: 164

You are meeting with the parents of an otherwise healthy newborn who has glanularhypospadias and bilaterally normal descended testes. They ask you what this finding means fortheir infant.

Of the following, presence of hypospadias is MOST likely to

A. increase his risk of urinary tract infection

B. necessitate renal ultrasonography

C. require immediate surgery

D. require postponement of circumcision

E. resolve spontaneously

Copyright © 2010 by the American Academy of Pediatrics page 406

Page 407: AAP PREP 2010

2010 PREP SA on CD-ROM

Preferred Response: DCritique: 164

Hypospadias is the ventral displacement of the urethral meatal opening on the penis thatoften occurs as an isolated urinary malformation, although it also may accompany chordee,ventral curvature of the penis, and other anomalies. Because it is an anatomic anomaly, it doesnot resolve spontaneously. Depending on the degree of hypospadias (which ranges fromglanular, in which the urethral opening appears on the glans, to proximal, in which the urethraopens on the ventral surface of the penis at the junction of the scrotum) (Item C164), surgicalrepair may be required for both cosmetic and functional purposes related to voiding and sexualfunction. However, surgery is not emergent and generally is performed at about 6 months ofage. If the hypospadias is mild and discovered only at circumcision, surgical repair still ispossible and cosmetically feasible.

The primary concern when counseling parents of a boy affected by hypospadias is toensure that conditions for surgical repair are optimal and that the option is available to use theskin of the prepuce in the repair. Therefore, circumcision generally is delayed until the surgicalrepair.

Hypospadias is increasing in frequency in the population and is second only tocryptorchidism as the most frequent urinary tract malformation. Environmental estrogens orantiestrogens, including phthalates, may be contributing to this phenomenon.

Hypospadias usually does not predispose to urinary tract infection unless it is accompaniedby other urinary tract anomalies, which is rare because embryogenesis of the urethra occursafter that of the kidneys. Renal ultrasonography is not recommended for glanular hypospadiasbut may be indicated for associated anomalies, including undescended testes. Karyotyping isrecommended for infants who have both hypospadias and bilateral cryptorchidism.

References:

American Academy of Pediatrics Committee on Genetics, Section on Endocrinology, and Sectionon Urology. Evaluation of the newborn with developmental anomalies of the external genitalia.Pediatrics. 2000;106:138-142. Available at:http://pediatrics.aappublications.org/cgi/content/full/106/1/138

Bellinger MF. Urologic disorders. In: Zitelli BJ, Davis HW, eds. Atlas of Pediatric PhysicalDiagnosis. 5th ed. Philadelphia, Pa: Mosby Elsevier; 2007:535-562

Farhat W, McLorie G. Urethral syndromes in children. Pediatr Rev. 2001;22:17-21. Available at:http://pedsinreview.aappublications.org/cgi/content/full/22/1/17

Porter MP, Faizan MK, Grady RW, Mueller BA. Hypospadias in Washington state: maternal riskfactors and prevalence trends Pediatrics. 2005;115:e495-e499. Available at:http://pediatrics.aappublications.org/cgi/content/full/115/4/e495

Snodgrass WT. Consultation with the specialist: hypospadias. Pediatr Rev. 2004;25:63-67.Available at: http://pedsinreview.aappublications.org/cgi/content/full/25/2/63

Copyright © 2010 by the American Academy of Pediatrics page 407

Page 408: AAP PREP 2010

2010 PREP SA on CD-ROM

Question: 165

The dentist in your community health center’s clinic calls you with a question about a patient thathe is seeing later that day. The child is 14 years old and underwent surgical repair of hiscongenital heart disease 5 years ago. The dentist wants to know if this patient’s cardiaccondition warrants antibiotic prophylaxis for a routine dental cleaning.

Of the following, the condition for which antibiotic prophylaxis is MOST appropriate when thepatient is at risk for bacteremia is

A. atrial septal defect transcatheter device closure with no residual shunt

B. complete atrioventricular septal defect repair with moderate mitral regurgitation

C. prosthetic aortic valve with no residual stenosis or regurgitation

D. tetralogy of Fallot repair with mild pulmonary stenosis and regurgitation

E. ventricular septal defect repair with aortic insufficiency

Copyright © 2010 by the American Academy of Pediatrics page 408

Page 409: AAP PREP 2010

2010 PREP SA on CD-ROM

Preferred Response: CCritique: 165

Infective endocarditis is an infection that involves the endocardial surface of the heart. Mostcommonly, the infection occurs on the cardiac valves, but it also may be seen on the margins ofa ventricular septal defect, along the chordae supporting the atrioventricular valves, or alongvascular grafts or foreign material such as a prosthetic valve. The infection is diagnosed byisolation of the pathogen from blood cultures. Streptococcus viridans and S bovis as well asStaphylococcus aureus are the most common pathogens in children. Other potential organismsinclude Enterococcus, coagulase-negative Staphylococcus, fungi, and a group of bacteriareferred to as the HACEK organisms (Haemophilus, Actinobacillus actinomycetemcomitans,Cardiobacterium hominis, Eikenella corrodens, and Kingella kingae). These gram-negativeoral and pharyngeal flora are fastidious, slow-growing, and often require growth factors andcarbon dioxide to provide positive test results in isolation.

Endocarditis treatment depends on the organism isolated from the blood or tissue of theinfected patient. In general, long-term antibiotic treatment (4 to 6 weeks) is undertaken in aneffort at complete eradication of the bacteria that have been sequestered in a nonvascularvegetation. Surgery is reserved for patients who develop severe congestive heart failure fromsevere valve regurgitation, fungal infection, and annular abscess that is not responding toantibiotic therapy.

Most cases of endocarditis cannot be prevented because clinicians do not always knowwhen bacteremia occurs. For many years, the American Heart Association (AHA)recommended that children and adults at increased risk for endocarditis, such as those who hadundergone surgical repair of congenital heart disease, take antibiotics before certain dental andsurgical procedures. The hypothesis was that because dental or surgical procedures cancause bacteremia, prophylactic administration of antibiotics (endocarditis prophylaxis) couldprevent the bacteria from surviving in the bloodstream. However, the AHA made major changesto their guidelines in 2007. After reviewing all of the scientific evidence on the causes andprevention of endocarditis, they concluded that the prophylactic dose of antibiotics given beforedental and surgical procedures prevents very few, if any, cases of endocarditis and that thereis some risk involved with taking antibiotics. Therefore, the new guidelines state that antibioticprophylaxis should be reserved for those children and adults who have the greatest risk for apoor outcome from endocarditis.

Patients who have congenital heart disease and are at the greatest danger of pooroutcomes from endocarditis and for whom preventive antibiotics prior to a dental procedure are

worth the risks include those who have:•Unrepaired or incompletely repaired cyanotic congenital heart disease, including those

who have palliative shunts and conduits•A completely repaired congenital heart defect involving prosthetic material or device,

whether placed by surgery or by catheter intervention, during the first 6 months after theprocedure

•Any repaired congenital heart defect with residual defect at the site or adjacent to the

site of a prosthetic patch or a prosthetic deviceAntibiotic prophylaxis is no longer recommended arbitrarily for any other form of congenital

heart disease.

Copyright © 2010 by the American Academy of Pediatrics page 409

Page 410: AAP PREP 2010

2010 PREP SA on CD-ROM

Other people who should receive antibiotics before most dental procedures include thosewho have prosthetic heart valves, those who have had endocarditis previously, and those whohave received cardiac transplants in whom a problem develops in a heart valve.

Because the boy described in the vignette underwent repair of a ventricular septal defect 5years ago, he no longer requires antibiotic prophylaxis. Endocarditis prophylaxis also is notindicated for the child who has a repaired atrioventricular septal defect with or without residualmitral regurgitation, tetralogy of Fallot with or without residual pulmonary valve disease, an atrialseptal defect that has been closed by a transcatheter device, or a ventricular septal defect thathas been repaired but has aortic insufficiency.

For typical dental and other upper airway procedures, oral amoxicillin (50 mg/kg) can beadministered approximately 30 to 60 minutes prior to the procedure for patients who requireprophylaxis. The AHA recommends a variety of antibiotic choices for dental procedures (ItemC165).

As a result of reviewing this information, do you intend to make a change in practiceto provide better patient care?Yes No

References:

Hoyer A, Silberbach M. Infective endocarditis. Pediatr Rev. 2005;26:394-400. Available at:http://pedsinreview.aappublications.org/cgi/content/full/26/11/394

Nishimura R, Carabello BA, Faxon DP, et al. ACC/AHA 2008 Guideline update on valvular heartdisease: focused update on infective endocarditis: a report of the American College ofCardiology/American Heart Association Task Force on Practice Guidelines endorsed by theSociety of Cardiovascular Anesthesiologists, Society for Cardiovascular Angiography andInterventions, and Society of Thoracic Surgeons. J Am Coll Cardiol. 2008;52:676-685

Taubert KA, Dajani AS. Infective endocarditis. In: Moller JH, Hoffman JIE, eds. PediatricCardiovascular Medicine. Philadelphia, Pa: Churchill Livingstone; 2000:768-779

Wilson W, Taubert KA, Gewitz M, et al. Prevention of infective endocarditis: guidelines from theAmerican Heart Association: a guideline from the American Heart Association Rheumatic Fever,Endocarditis, and Kawasaki Disease Committee, Council on Cardiovascular Disease in theYoung, and the Council on Clinical Cardiology, Council on Cardiovascular Surgery andAnesthesia, and the Quality of Care and Outcomes Research Interdisciplinary Working Group.Circulation. 2007;116:1736-1754. Available at:http://circ.ahajournals.org/cgi/content/full/116/15/1736

Copyright © 2010 by the American Academy of Pediatrics page 410

Page 411: AAP PREP 2010

2010 PREP SA on CD-ROM

Question: 166

A 5-month-old girl presents with a 1-week history of exaggerated startle movements anddecreased motor and social activity. According to her parents, the child bends her head andtrunk forward, extends her arms out quickly, and cries (Item Q166). Such actions may occur 10to 20 times consecutively and have become more frequent over the past several days. She alsois much less physically active and less interactive with her parents. Electroencephalographyshows a diffuse, severely abnormal pattern (hypsarrhythmia).

Of the following, the MOST effective treatment for this child is

A. adrenocorticotropic hormone

B. carbamazepine

C. phenobarbital

D. phenytoin

E. prednisone

Copyright © 2010 by the American Academy of Pediatrics page 411

Page 412: AAP PREP 2010

2010 PREP SA on CD-ROM

Preferred Response: ACritique: 166

The child described in the vignette is experiencing infantile spasms. This epilepticencephalopathy is characterized by myoclonic-like seizures that can be subtle, such as a headdrop (Item C166), or more dramatic, such as a "clasp-knife" spasm of the body with arm flexionor extension. The clustering is common, particularly at times of sleep-wake transition.Importantly, the ictal spasms are accompanied by changes in cognitive and motor function,manifested by reduced social interaction and activity, that are related to the underlying diffuseelectrical brain dysfunction. Such dysfunction produces a severely chaotic, cortical electricalpattern identified on electroencephalography as "hypsarrhythmia." Outcome is almost uniformlypoor for infants who have pre-existing neurologic problems and develop infantile spasms.Neurologic outcomes sometimes are good for those who have previously normalneurodevelopment. Infantile spasms should prompt a careful physical examination as well asneuroimaging because tuberous sclerosis may present with infantile spasms.

The treatment of choice for infantile spasms, recommended in current United States practiceguidelines, is adrenocorticotropic hormone. Prednisone is generally less effective. Otherantiseizure medications may be useful when other seizure types develop, but these have notbeen shown to be effective for infantile spasms. Phenobarbital is inexpensive, but it is sedating,and the drug often is not recommended after the neonatal period due to adverse behavioraleffects. Neither phenytoin nor carbamazepine is effective against infantile spasms. Vigabatrin isunavailable in the United States but is the drug of choice in Europe and Canada, particularly forchildren who have infantile spasms due to tuberous sclerosis.

References:

Mackay MT, Weiss SK, Adams-Webber T, et al; American Academy of Neurology; ChildNeurology Society. Practice parameter: medical treatment of infantile spasms: report of theAmerican Academy of Neurology and the Child Neurology Society. Neurology. 2004;62:1668-1681. Available at: http://www.neurology.org/cgi/content/full/62/10/1668

Major P, Thiele EA. Seizures in children: determining the variation. Pediatr Rev. 2007;28:363-371.Available at: http://pedsinreview.aappublications.org/cgi/content/full/28/10/363

Overby PJ, Kossoff EH. Treatment of infantile spasms. Curr Treat Options Neurol. 2006;8:457-464. Abstract available at: http://www.ncbi.nlm.nih.gov/pubmed/17032566

Riikonen R. Infantile spasms: therapy and outcome. J Child Neurol. 2004;19:401-404. Abstractavailable at: http://www.ncbi.nlm.nih.gov/pubmed/15446386

Copyright © 2010 by the American Academy of Pediatrics page 412

Page 413: AAP PREP 2010

2010 PREP SA on CD-ROM

Question: 167

You are called urgently to the nursery to evaluate a newborn who exhibits possible seizures.The baby is a 2-day-old boy who has been healthy and breastfeeding well. Over the past 12hours, he has become increasingly difficult to arouse and now is refusing to feed. Physicalexamination reveals a normally formed baby who has hypertonia and obtundation and respondsweakly to painful stimuli. A bedside glucose determination is 60 mg/dL (3.3 mmol/L), and vitalsigns are stable. While arranging for further laboratory testing and transfer to the neonatalintensive care unit, you observe a generalized seizure.

Of the following, this presentation is MOST suggestive of a

A. fatty acid oxidation defect

B. glycogen storage disease

C. lipid storage disease

D. lysosomal storage disease

E. urea cycle defect

Copyright © 2010 by the American Academy of Pediatrics page 413

Page 414: AAP PREP 2010

2010 PREP SA on CD-ROM

Preferred Response: ECritique: 167

The signs of a progressive encephalopathy within days after birth described for the termnewborn in the vignette could indicate the presence of a urea cycle defect (ornithinetranscarbamylase deficiency, citrullinemia, carbamyl phosphate synthetase deficiency,argininosuccinic aciduria, or argininemia). The urea cycle, which is the primary pathway for theexcretion of nitrogenous waste, contains five enzymes, any of which may be deficient. The firstsymptoms of this group of inborn errors of metabolism include poor feeding and lethargy that, ifuntreated, progress to coma. At the first signs of obtundation, it is important to measure plasmaammonia concentrations as part of a metabolic evaluation. If the infant does not display acidosisbut does exhibit hyperammonemia, a urea cycle defect is likely. Amino acids should be measuredfor infants who have plasma ammonia concentrations greater than 210 mcg/dL (150 mcmol/L) toaid in diagnosis. Strong evidence suggests that the extent of neurologic damage in survivors isrelated directly to the duration of hyperammonemic coma, so treatment should be initiatedpromptly. Treatment is aimed at removing ammonia from the blood and may include hemodialysisand arginine infusion. Initially, protein is removed from the diet, but it must be replaced slowly andlimited thereafter.

Fatty acid oxidation defects typically present with hypoglycemia and metabolic acidosis withincreased anion gap. If left untreated, hyperammonemia can occur. The normal bedside glucosedetermination reported for the infant in the vignette makes this diagnosis unlikely, although serumglucose also should be measured.

Glycogen storage diseases can present from days to years after birth. The major presentingfeatures include hypoglycemia and hepatomegaly in type I and hepatomegaly in type III. Type II(Pompe disease) is a lysosomal storage disorder and may present with poor feeding and failureto thrive followed by progressive cardiac failure.

The lipid and lysosomal storage diseases typically do not present with early-onsetobtundation. Clinical features include neurodegeneration and organomegaly. The findings ofcoarsening of the facial features or cherry red macular spots (as seen in GM1 gangliosidosisand Tay-Sachs disease, for example) may be helpful in diagnosing these conditions.

References:

Burton BK. Inborn errors of metabolism in infancy: a guide to diagnosis. Pediatrics.1998;102:e69. Available at: http://pediatrics.aappublications.org/cgi/content/full/102/6/e69

Burton BK. Urea cycle disorders. Clin Liver Dis. 2000;4:815-830. Abstract available at:http://www.ncbi.nlm.nih.gov/pubmed/11232359

Nyhan WL, Barshop BA, Ozand PT. Glycogen storage disease: introduction. In: Atlas ofMetabolic Diseases. 2nd ed. London, England: Hodder Arnold; 2005:382-408

Nyhan WL, Barshop BA, Ozand PT. GM1 gangliosidosis/beta-galactosidase deficiency. In: Atlasof Metabolic Diseases. 2nd ed. London, England: Hodder Arnold; 2005:599-615

Copyright © 2010 by the American Academy of Pediatrics page 414

Page 415: AAP PREP 2010

2010 PREP SA on CD-ROM

Nyhan WL, Barshop BA, Ozand PT. Introduction to hyperammonemia and disorders of the ureacycle. In: Atlas of Metabolic Diseases. 2nd ed. London, England: Hodder Arnold; 2005:193-227

Copyright © 2010 by the American Academy of Pediatrics page 415

Page 416: AAP PREP 2010

2010 PREP SA on CD-ROM

Question: 168

A 15-year-old boy comes to the emergency department because of cramping abdominal pain,diarrhea, and body aches. Physical examination reveals no icterus or organomegaly, although hehas increased bowel sounds and mild diffuse abdominal tenderness. His genitalia are at Sexual

Maturity Rating 4. Among the results of laboratory tests obtained are:

•Total bilirubin, 0.6 mg/dL (10.3 mcmol/L)

•Alanine aminotransferase, 18 units/L

•Aspartate aminotransferase, 22 units/L•Alkaline phosphatase, 360 international units/L

Of the following, the MOST likely explanation for the results of these laboratory tests is

A. bone malignancy

B. infectious hepatitis

C. inflammatory bowel disease

D. physiologic growth spurt

E. viral gastroenteritis

Copyright © 2010 by the American Academy of Pediatrics page 416

Page 417: AAP PREP 2010

2010 PREP SA on CD-ROM

Preferred Response: DCritique: 168

An increased serum alkaline phosphatase (SAP) value on a liver function test panel in anadolescent, as reported for the boy in the vignette, often is the result of rapid bone growthduring the pubertal growth spurt. Therefore, it is important to correlate the value with the SexualMaturity Rating (SMR) rather than with chronologic age. The highest mean SAP concentrations ingirls occur at SMR 2 and in boys at SMR 3, coinciding in each instance with peak height velocityand presumed maximum osteoblastic activity. Osteoblasts, by creating a local environment ofalkalinity via alkaline phosphatase, help build bone. With increasing SMR or age, the SAP valuesin both sexes decrease markedly. During some growth spurts, concentrations can be as high as500 international units/L. Although the normal range for alkaline phosphatase in adults isapproximately 20 to 140 international units/L, the normal range for children and adolescentsvaries with age, sex, sexual maturity, and reference laboratory. Thus, it is important to consultwith your laboratory regarding normal values. The isoenzyme test can reveal whether anelevation of SAP is from bone or liver, but this test is not widely available.

Pathologic causes of increased SAP include diseases of the liver or bile duct, gallbladderdysfunction, pregnancy, drugs (eg, phenytoin), skeletal disease, or endocrine disorders such ashyperparathyroidism. Normal concentrations of liver enzymes suggest a nonhepatic cause ofSAP elevation and rule out infectious hepatitis and inflammatory bowel disease with hepaticinvolvement. Bone pathology (eg, osteosarcoma) presents with higher SAP values than reportedfor this boy along with other symptoms (eg, limb pain) and signs (eg, swelling). Elevated SAPconcentrations do not occur in viral gastroenteritis.

References:

Bennett DL, Ward MS, Daniel WA Jr. The relationship of serum alkaline phosphataseconcentrations to sex maturity ratings in adolescents. J Pediatr. 1976;88:633-636. Abstractavailable at: http://www.ncbi.nlm.nih.gov/pubmed/1255325

Carswell JM, Stafford DEJ. Normal physical growth and development. In: Neinstein L, ed.Adolescent Health Care: A Practical Guide. 5th ed. Philadelphia Pa: Lippincott Williams & Wilkins;2008:3-26

Corathers SD. Focus on diagnosis: the alkaline phosphatase level: nuances of a familiar test.Pediatr Rev. 2006;27:382-384. Available at:http://pedsinreview.aappublications.org/cgi/content/full/27/10/382

Copyright © 2010 by the American Academy of Pediatrics page 417

Page 418: AAP PREP 2010

2010 PREP SA on CD-ROM

Question: 169

A 4-year-old child who has complex congenital heart disease comes to your clinic because hismother is concerned about his decreased activity and increased cyanosis. According to hisparents, he had been doing well until about 3 months ago, when he became more easilyfatigued, complained of shortness of breath after playing, and stopped gaining weight. They addthat he has had no recent illnesses. On physical examination, the child appears cyanotic but isalert and cooperative. He has a temperature of 37.0°C, heart rate of 120 beats/min, respiratoryrate of 25 breaths/min, and blood pressure of 90/50 mm Hg. You note a normal S1 and single S2without murmur. His lungs are clear to auscultation bilaterally, and his liver is 1 cm below theright costal margin. He has moderate clubbing of the fingers.

Of the following, the BEST test to assess this boy’s degree of hypoxemia accurately is

A. arterial blood gas

B. change in oxygen saturation in response to 100% oxygen administration

C. echocardiography

D. pulse oximetry

E. venous blood gas

Copyright © 2010 by the American Academy of Pediatrics page 418

Page 419: AAP PREP 2010

2010 PREP SA on CD-ROM

Preferred Response: ACritique: 169

Cyanosis may indicate severe hypoxemia, its prompt evaluation is critical, and rapid

treatment is indicated if hypoxemia (Pao2 <60 mm Hg) is present. The relationship between

hemoglobin saturation and Pao2 depends on many physiologic factors, including types ofhemoglobin present (fetal versus adult), temperature, and pH. Therefore, a child who hascyanosis, such as the boy described in the vignette, should undergo a complete evaluation, notonly to establish the cause of cyanosis but to determine the degree of hypoxemia.

The best test to measure the degree of cyanosis and hypoxemia is an arterial blood gas. Co-oximetry should be added if there is any suspicion of the presence of abnormal hemoglobins,such as carboxyhemoglobin or methemoglobin. Oxygen measurements from a venous blood gasdo not correlate well with values from arterial blood gases. Administration of 100% oxygen andassessment of the response frequently is used in the neonatal period to distinguish pulmonaryfrom cardiac disease, but such action is of little value in a 4-year-old child. Echocardiography isan important component of the assessment of children who have congenital heart disease, but ithas no value in assessing the degree of hypoxemia. Pulse oximetry is a valuable tool in theassessment of patients who have suspected hypoxemia, but it is not as accurate as arterialblood gases, especially at lower saturations (<70%).

As a result of reviewing this information, do you intend to make a change in practiceto provide better patient care?Yes No

References:

Kamat V. Pulse oximetry. Indian J Anaesth. 2002;46:261-268

Marcum J, Newth CJL. Respiratory monitoring. In: Wheeler DS, Wong HR, Shanley TP, eds.Pediatric Critical Care Medicine: Basic Science and Clinical Evidence. New York, NY:Springer-Verlag London Limited; 2007:376-389

Silberbach M, Hannon D. Presentation of congenital heart disease in the neonate and younginfant. Pediatr Rev. 2007;28:123-131. Available at:http://pedsinreview.aappublications.org/cgi/content/full/28/4/123

Copyright © 2010 by the American Academy of Pediatrics page 419

Page 420: AAP PREP 2010

2010 PREP SA on CD-ROM

Question: 170

You are called to see a newborn who appears to be a male and has a well-developed rugatedscrotum without palpable testes. The phallus is 3.5 cm in length, but there is hypospadiasextending to the base of the phallus and what seems to be an open urogenital sinus. It is hard todetermine whether there is a separate urethral opening. On reviewing the maternal records, youdiscover that prenatal amniocentesis obtained because of advanced maternal age showed anXX chromosome pattern. You arrange a complete initial evaluation for a potential disorder ofsexual differentiation.

Of the following, the MOST important initial test to obtain for this child is

A. fluorescence in situ hybridization for sex determining region of the Y chromosome (SRY)

B. measurement of serum 17-hydroxyprogesterone

C. measurement of serum testosterone

D. pelvic ultrasonography

E. repeat confirmatory chromosome study

Copyright © 2010 by the American Academy of Pediatrics page 420

Page 421: AAP PREP 2010

2010 PREP SA on CD-ROM

Preferred Response: BCritique: 170

The most common underlying diagnosis in genetic female infants who have genital ambiguity,such as the infant described in the vignette, is congenital adrenal hyperplasia (CAH) due toCYP21-alpha-1 (21 alpha-hydroxylase) deficiency, a disorder that affects 1 in 12,000 to 1 in15,000 infants in the general population. This is the most common of the gene defects that causeCAH in the United States and western Europe and is associated with aldosterone deficiency inmore than 50% of affected children, leading to salt loss and adrenal insufficiency within 1 to 2weeks after birth. Therefore, the diagnosis must be made early to prevent serious illness in theinfant.

An elevated serum 17-hydroxyprogesterone value can help to make the diagnosis of CAH.Although fluorescence in situ (FISH) hybridization for sex determining region of the Ychromosome (SRY), measurements of serum testosterone, pelvic ultrasonography, andconfirmatory chromosome study all might be indicated in the evaluation of this child,measurement of serum 17-hydroxyprogesterone is the most important initial test. In those areasthat offer newborn screening for CAH, this adrenal steroid is measured routinely in heel stickblood, and the results may be obtained from the screening laboratory. Modestly elevated 17-hydroxyprogesterone concentrations also may be found in some of the other less common CAHenzyme defects associated with genital ambiguity, including 3-beta-hydroxysteroiddehydrogenase and 11-beta hydroxylase deficiencies.

A FISH study to look for SRY should be obtained to document Y chromosomal material in thegenome of the baby who has an XX chromosome constitution. Such a finding diagnoses atesticular disorder of sexual differentiation (DSD) (formerly termed XX sex reversal) orovotesticular DSD (formerly termed true hermaphroditism). Serum testosterone concentrationsmight be elevated if there is SRY expression, depending upon the age of the baby, or incongenital adrenal hyperplasia, and the study is indicated in the evaluation. Pelvicultrasonography should be part of the evaluation because it might help to determine the nature ofthe internal urogenital structures, but similarly would not confirm the diagnosis of the only life-threatening disorder that presents in this manner. A repeat chromosome study might be indicatedto complete the evaluation of this child, if the diagnosis remains in doubt after determining the 17-hydroxyprogesterone value.

References:

Houk CP, Levitsky LL. Evaluation of the infant with ambiguous genitalia. UpToDate Online 16.3.2008. Available for subscription at:http://www.utdol.com/online/content/topic.do?topicKey=pediendo/11554

Lee PA, Houk CP, Ahmed SF, Hughes IA in collaboration with the participants in the InternationalConsensus Conference on Intersex organized by the Lawson Wilkins Pediatric EndocrineSociety and the European Society for Pediatric Endocrinology. Consensus statement onmanagement of intersex disorders. Pediatrics. 2006;118:e488-e500. Available at:http://pediatrics.aappublications.org/cgi/content/full/118/2/e488

Copyright © 2010 by the American Academy of Pediatrics page 421

Page 422: AAP PREP 2010

2010 PREP SA on CD-ROM

Levine LS. Congenital adrenal hyperplasia. Pediatr Rev. 2000;21:159-171. Available at:http://pedsinreview.aappublications.org/cgi/content/full/21/5/159

Copyright © 2010 by the American Academy of Pediatrics page 422

Page 423: AAP PREP 2010

2010 PREP SA on CD-ROM

Question: 171

The parents of a 3-month-old child are concerned that he is not getting enough sleep and iswaking frequently during the night. Every time they check on him at night, he is making suckingmovements and his limbs twitch. His past medical history and physical examination findings arewithin normal parameters. He is being fed a cow milk-based formula.

Of the following, the BEST next step in the management of this infant is to

A. add rice cereal to the infant’s formula

B. change the infant’s formula

C. obtain polysomnography (sleep study)

D. reassure the parents that this sleep pattern is normal

E. refer the infant for neurologic evaluation

Copyright © 2010 by the American Academy of Pediatrics page 423

Page 424: AAP PREP 2010

2010 PREP SA on CD-ROM

Preferred Response: DCritique: 171

Newborns can sleep 16 to 20 hours in a 24-hour period, alternating between 1- to 4-hourperiods of sleep and 1 to 2 hours of being awake. Newborns cycle between rapid eyemovement (REM) and non-REM sleep every 50 minutes. At the end of each cycle, the newbornmay experience an arousal that is not true awakening. Parents may misinterpret this as the babybeing awake and pick up the infant, causing a true arousal. During REM sleep (active sleep in thenewborn period), associated movements may occur, which may include facial movements,sucking, and limb movements, as described for the infant in the vignette. By 2 months of age,infants are able to establish a day-night cycle. By 4 months, many infants can sleepuninterrupted through the night. A child of 1 year should be sleeping 13 to 14 hours, primarilyduring the night.

The parents of the infant described in the vignette should be counseled that their baby is notexperiencing a true awakening and should be left to sleep undisturbed. There is no reason toobtain polysomnography or refer the child for a neurologic evaluation. Thickened feedings mayreduce the severity and amount of regurgitation in a child who has gastroesophageal reflux, butthis child is not having feeding difficulties, failure to thrive, or irritability. In the absence of signsof reflux or adverse reaction to the formula, there is no indication to change it.

References:

Kass LJ. Sleep problems. Pediatr Rev. 2006;27:455-462. Available at:http://pedsinreview.aappublications.org/cgi/content/full/27/12/455

Kumar Y, Sarvananthan R. GORD in children. BMJ Clinical Evidence. 2008. Available forsubscription at: http://clinicalevidence.bmj.com/ceweb/conditions/chd/0310/0310-get.pdf

Copyright © 2010 by the American Academy of Pediatrics page 424

Page 425: AAP PREP 2010

2010 PREP SA on CD-ROM

Question: 172

A 5-year-old boy develops fever, abdominal pain, diarrhea, and malaise 5 days after drinkingfresh cow’s milk on a school field trip. His stool culture is positive for Campylobacter jejuni.

Of the following, the MOST appropriate treatment for this child is administration of

A. amoxicillin

B. azithromycin

C. cephalexin

D. clindamycin

E. metronidazole

Copyright © 2010 by the American Academy of Pediatrics page 425

Page 426: AAP PREP 2010

2010 PREP SA on CD-ROM

Preferred Response: BCritique: 172

Campylobacter jejuni is one of the most common agents associated with bacterialgastroenteritis, and infection is characterized clinically by fever, abdominal pain, and bloodydiarrhea. The acute diarrheal illness cannot be differentiated clinically from other bacterialgastrointestinal infections. Postinfectious Guillain-Barré syndrome and reactive arthritis alsohave been associated with Campylobacter infection.

Handling uncooked poultry, cross-contamination (eg, preparing foods on uncleaned cookingboards), and unpasteurized milk are the most common sources of C jejuni infection. Thediagnosis can be confirmed by stool culture. Isolation of Campylobacter from feces requires theuse of selective media and incubation in a gas mixture of 5% to 10% oxygen and 1% to 10%carbon dioxide (microaerophilic conditions) at a temperature of 42°C.

As with most gastrointestinal infections, rehydration is the primary goal of therapy.Antibiotics can help shorten the course of the illness. Macrolides (eg, azithromycin) are thedrugs of choice for the treatment of Campylobacter enteritis. Erythromycin, with its narrowerspectrum, decreased absorption (leaving increased drug in the gastrointestinal tract), and lowercost, is an equivalent choice, although the decreased frequency of dosing and duration oftreatment and the improved palatability of azithromycin make it an attractive option.Fluoroquinolones such as ciprofloxacin are generally active against Campylobacter, but theyare not approved as first-line therapy in children. In addition, increasing resistance to quinoloneshas been reported in Campylobacter. Extended-spectrum cephalosporins may be active againstCampylobacter, but first-generation cephalosporins, such as cephalexin, and penicillins, suchas amoxicillin, are not effective. Clindamycin and metronidazole are not active against gram-negative organisms, including Campylobacter.

A different species of Campylobacter, C fetus, has been associated with systemic infectionin neonates and immunocompromised hosts. Bacteremia caused by C fetus can be treated withextended-spectrum cephalosporins or a carbapenem. The organism also is susceptible togentamicin and other aminoglycosides.

References:

Allos BM. Microbiology, pathogenesis and epidemiology of Campylobacter infection. UpToDateOnline 16.3. 2007. Available for subscription at:www.utdol.com/online/content/topic.do?topicKey=gi_infec/6417&view=print

American Academy of Pediatrics. Campylobacter infections. In: Pickering LK, Baker CJ,Kimberlin DW, Long SS, eds. Red Book: 2009 Report of the Committee on Infectious Diseases.28th ed. Elk Grove Village, Ill: American Academy of Pediatrics; 2009:241-242

Copyright © 2010 by the American Academy of Pediatrics page 426

Page 427: AAP PREP 2010

2010 PREP SA on CD-ROM

Question: 173

A 2-year-old girl is brought in to your office by her mother because she failed her hearing testupon entry into preschool. The child had congenital cytomegalovirus infection that was relativelyasymptomatic at birth except for mild elevation of her liver enzymes. Her mother has noticed thatthe girl rarely has wanted to vocalize over the past several months. She does not seem tostartle to loud sounds or respond to commands unless spoken to in a very loud voice. Shepassed her infant hearing screen and a repeat hearing test at 9 months of age. She has beenotherwise healthy. Physical examination findings include normal external ears, ear canals, andtympanic membranes.

Of the following, the MOST likely explanation of this child’s failed hearing test is

A. conductive hearing loss

B. ossicular fixation

C. otitis media with effusion

D. otosclerosis

E. sensorineural hearing loss

Copyright © 2010 by the American Academy of Pediatrics page 427

Page 428: AAP PREP 2010

2010 PREP SA on CD-ROM

Preferred Response: ECritique: 173

There are two major types of hearing loss: conductive and sensorineurol. Conductivehearing loss results from middle ear dysfunction or blockage of the external auditory canal.Common causes are fluid in the middle ear, ruptured tympanic membrane, cholesteatoma, andblockage of the outer ear canal by cerumen. Sensorineural hearing loss can result from nervedamage, malformation of the inner ear, and dysfunction of the auditory nerve. A number ofbacteria, viruses, and parasites can cause a sensorineural hearing loss as a sequela ofinfection with the pathogen. The most common viruses associated with a sensorineural hearingloss are cytomegalovirus (CMV), measles virus, mumps virus, and rubella virus. Studies haveshown that late-onset hearing loss develops in 17% of all congenital and 14% of all subclinicalcongenital CMV infections. In some patients, CMV has been found in the cells of the organ ofCorti, in the neurons of the spiral ganglia, and in the cochlea.

Sensorineural hearing loss is one of the most common sequelae of bacterial meningitis andmay be seen with meningitis caused by any bacteria. The bacteria with which hearing loss isseen commonly include Streptococcus pneumoniae, Haemophilus influenzae type b, andNeisseria meningitidis. Sensorineural hearing loss also may occur with Toxoplasma gondii (aprotozoan parasite) encephalitis and congenital infections.

The patient described in the vignette has progressive sensorineural hearing loss related toher congenital CMV infection. Unfortunately, newborn hearing screening fails to identify asignificant number of children who subsequently develop CMV-related sensorineural hearingloss. Available evidence indicates that such children may present with hearing deficits as late as6 years of age. The normal physical examination findings of her external ears, ear canals, andtympanic membranes make a conductive hearing loss, otitis media with effusion, and ossicularfixation (a form of conductive hearing loss that may develop in persons who have chronic otitismedia in which fibrous tissue, collagen, or new bone formation occurs around the middle earbones) unlikely. Otosclerosis is a form of conductive hearing loss that is an inherited disorder inwhich the bones of the middle and inner ear sclerose together into an immovable mass. Thehearing loss in this disorder manifests between the ages of 10 and 30 years.

References:

American Academy of Pediatrics. Cytomegalovirus infection. In: Pickering LK, Baker CJ, KimberlinDW, Long SS, eds. Red Book: 2009 Report of the Committee on Infectious Diseases. 28th ed.Elk Grove Village, Ill: American Academy of Pediatrics; 2009:275-280

American Academy of Pediatrics. Measles. In: Pickering LK, Baker CJ, Kimberlin DW, Long SS,eds. Red Book: 2009 Report of the Committee on Infectious Diseases. 28th ed. Elk GroveVillage, Ill: American Academy of Pediatrics; 2009:444-455

American Academy of Pediatrics. Rubella. In: Pickering LK, Baker CJ, Kimberlin DW, Long SS,eds. Red Book: 2009 Report of the Committee on Infectious Diseases. 28th ed. Elk GroveVillage, Ill: American Academy of Pediatrics; 2009:579-584

Copyright © 2010 by the American Academy of Pediatrics page 428

Page 429: AAP PREP 2010

2010 PREP SA on CD-ROM

Lazoff M. Meningitis. eMedicine Specialties, Emergency Medicine, Infectious Diseases. 2007.Available at: http://www.emedicine.com/EMERG/topic309.htm

Report and Recommendations: NIDCD Workshop on Congenital Cytomegalovirus Infection andHearing Loss. Available at: http://www.nidcd.nih.gov/funding/programs/hb/cmvwrkshop.asp

Stagno S, Reynolds DW, Amos CS, et al. Auditory and visual defects resulting from symptomaticand subclinical congenital cytomegaloviral and Toxoplasma infections. Pediatrics. 1977;59:669-678. Abstract available at: http://pediatrics.aappublications.org/cgi/content/abstract/59/5/669

Wubbel L, McCracken GH. Management of bacterial meningitis: 1998. Pediatr Rev. 1998;19:78-84. Available at: http://pedsinreview.aappublications.org/cgi/content/full/19/3/78

Copyright © 2010 by the American Academy of Pediatrics page 429

Page 430: AAP PREP 2010

2010 PREP SA on CD-ROM

Question: 174

A 4-year-old girl presents with a 10-day history of increased urinary frequency but noassociated dysuria or fever. She often voids a few times per hour during the day, but does notawaken at night to void. She typically sleeps 9 hours per night and is dry on awakening eachmorning. She was toilet trained at 2½ years of age. Her parents report that her older brotherrecently started school, and she has seemed a bit restless in recent weeks. Findings on herphysical examination are unremarkable. A urinalysis shows a specific gravity of 1.020; pH of6.0; and negative findings for blood, protein, leukocyte esterase, and nitrite. Urine culture resultsare negative.

Of the following, the MOST appropriate next step in treating this patient is to

A. order voiding cystourethrography

B. place the child on a timed voiding program

C. prescribe a 10-day course of antibiotics

D. reassure the parents that the problem should be short-lived

E. start the child on a laxative to treat any component of constipation

Copyright © 2010 by the American Academy of Pediatrics page 430

Page 431: AAP PREP 2010

2010 PREP SA on CD-ROM

Preferred Response: DCritique: 174

The child described in the vignette has the clinical picture of pollakiuria. This condition ofextraordinary urinary frequency typically occurs suddenly in toilet-trained children, causing themto need to void small urine volumes every 5 to 20 minutes without associated dysuria, abdominalpain, or fever. Affected children are typically 4 to 6 years old. Another characteristic feature ismarked symptoms during the day that usually resolve completely during sleep and the lack ofnocturnal enuresis. The urinary tract is structurally normal, and, therefore, imaging such asultrasonography and voiding cystourethrography generally is not needed. Because urinecultures are negative, there is no role for antibiotic treatment. Pollakiuria may be triggered bypsychosocial stress such as a death in the family or parental divorce. The prognosis isexcellent, with anticipated resolution of symptoms within 2 to 6 months.

A more significant type of voiding dysfunction that should be considered in a child who hasurinary frequency is due to an unstable (overactive) bladder. Affected children often experienceurgency due to uninhibited bladder contractions and frequently have daytime and nighttimeenuresis. The presence of nocturnal enuresis distinguishes the child who has an unstablebladder from one who has pollakiuria. Children who have unstable bladders compensate fortheir uninhibited bladder contractions by learning to contract the external urinary sphinctervoluntarily to avoid incontinence, often assuming postures such as squatting, leg crossing, orVincent curtsy (using the heel to provide pressure at the perineal region). Because this conditionis not short-lived and may be associated with urinary tract infections from urinary retention, itsprognosis is not as favorable as that of pollakiuria. Therefore, timed voiding is recommended,and for those who are unable to void often enough, anticholinergic agents are recommended.Treatment of constipation is useful in children who have dysfunctional elimination or recurrenturinary tract infections, but it does not appear to have a role in children who have pollakiuria orbladder instability (overactive bladder).

As a result of reviewing this information, do you intend to make a change in practiceto provide better patient care?Yes No

References:

Hellerstein S, Linebarger JS. Voiding dysfunction in pediatric patients. Clin Pediatr (Phila).2003;42:43-49. Abstract available at: http://www.ncbi.nlm.nih.gov/pubmed/12635981

Hood BS. Index of suspicion: case 2. Pediatr Rev. 2003;24;207-212. Available at:http://pedsinreview.aappublications.org/cgi/content/full/24/6/207

Issenman RM, Filmer RB, Gorski PA. A review of bowel and bladder control development inchildren: how gastrointestinal and urologic conditions relate to problems in toilet training.Pediatrics. 1999;103 (suppl):1346-1352. Available at:

http://pediatrics.aappublications.org/cgi/content/full/103/6/S1/1346

Copyright © 2010 by the American Academy of Pediatrics page 431

Page 432: AAP PREP 2010

2010 PREP SA on CD-ROM

Pohl HG, Henderson CG. Voiding disorders. In: Kher KK, Schnaper HW, Makker SP, eds. ClinicalPediatric Nephrology. 2nd ed. London, England: Informa Healthcare; 2007:519-537

Copyright © 2010 by the American Academy of Pediatrics page 432

Page 433: AAP PREP 2010

2010 PREP SA on CD-ROM

Question: 175

A 14-year-old patient of yours who has exercise-induced asthma (EIA) will be attending asoccer camp this summer. The camp involves an intensive regimen, with exercise starting assoon as he wakes up and continuing through the day. The boy would like to know if there issomething he can take to decrease his EIA symptoms that not only works quickly but lasts for 8to 12 hours.

Of the following, the BEST choice is

A. albuterol

B. formoterol

C. ipratropium

D. montelukast

E. salmeterol

Copyright © 2010 by the American Academy of Pediatrics page 433

Page 434: AAP PREP 2010

2010 PREP SA on CD-ROM

Preferred Response: BCritique: 175

Exercise-induced asthma (EIA) is not only a problem for children and adolescents who haveunderlying persistent asthma but also can affect 10% to 50% of elite athletes. In addition toensuring control for underlying persistent asthma, a warm-up period prior to maximal exertion as

well as administration of an inhaled beta2 agonist are recommended to prevent EIA. Short-acting

beta2 agonists, such as albuterol or levalbuterol, are the most common bronchodilators used.

Typically administered 20 minutes prior to activity, short-acting beta2 agonists provide 2 to 4hours of bronchodilation. However, such treatment may not be effective when anticipatedactivity is longer, such as the 8 to 12 hours described for the boy in the vignette. Long-acting

beta2 agonists that are approved for prevention of EIA in children older than 4 and 5 years ofage include salmeterol and formoterol, respectively. However, these agents should not be usedas monotherapy for persistent asthma. Formoterol has a faster onset of action than salmeterol(10 minutes versus 30 minutes to reach an increase of 15% in forced expiratory volume in 1second, and is appropriate for the boy in the vignette.

In December 2008, the Pulmonary-Allergy Drugs Advisory Committee recommended that theUnited States Food and Drug Administration (FDA) consider a ban on the use of both salmeteroland formoterol in children and adolescents. However, the Committee focused on the use of long-

acting beta2 agonists for persistent asthma, citing a meta-analysis in adults showing increasedrisk for asthma-related hospitalization, asthma-related intubation, and asthma-related deaths. Atthe time of this writing (March 2009), the FDA has not rendered a decision on this issue.

Leukotriene receptor antagonists (LTRAs) such as montelukast can be used for EIA. Studieshave demonstrated that LTRAs reduce the immediate and late phase of exercisebronchoconstriction. Interestingly, the response to LTRAs varies from poor to excellent.Pharmacogenetic studies are ongoing to determine which patient group exhibits the bestresponse. Anticholinergic inhalers such as ipratropium are not recommended for the treatment ofEIA.

References:

Fitch KD, Sue-Chu M, Anderson SD, et al. Asthma and the elite athlete: summary of theInternational Olympic Committee’s consensus conference, Lausanne, Switzerland, January 22-24, 2008. J Allergy Clin Immunol. 2008;122:254-260.e1-7. Abstract available at:http://www.ncbi.nlm.nih.gov/pubmed/18678340

Schwartz LB, Delgago L, Craig T, et al. Exercise-induced hypersensitivity syndromes inrecreational and competitive athletes: a PRACTALL consensus report (what the generalpractitioner should know about sports and allergy). Allergy. 2008;63:953-961. Abstract availableat: http://www.ncbi.nlm.nih.gov/pubmed/18691297

Weinberger M. Long-acting beta-agonists and exercise. J Allergy Clin Immunol. 2008;122:251-253

Copyright © 2010 by the American Academy of Pediatrics page 434

Page 435: AAP PREP 2010

2010 PREP SA on CD-ROM

Question: 176

A 6-year-old boy who has attention-deficit/hyperactivity disorder is found by his mother holdingan empty bottle of methylphenidate. He tells her that he took "all of his medicine," which shebelieves was ten 10-mg tablets. She brings the boy to the emergency department, where he isagitated and screaming that there are "bugs on him." Physical examination reveals a heart rate of180 beats/min, respiratory rate of 28 breaths/min, and blood pressure of 140/95 mm Hg. Hispupils are 6 mm and briskly reactive. No other findings on physical examination are of note. Youobtain electrocardiography (Item Q176). His toxicology screen is positive for amphetamines. Youadminister activated charcoal and place him in a quiet room, but he continues to be inconsolable.

Of the following, the next MOST appropriate step is to administer

A. adenosine

B. haloperidol

C. labetalol

D. lorazepam

E. naloxone

Copyright © 2010 by the American Academy of Pediatrics page 435

Page 436: AAP PREP 2010

2010 PREP SA on CD-ROM

Preferred Response: DCritique: 176

The patient described in the vignette is exhibiting signs and symptoms consistent with anacute amphetamine overdose, including tachycardia, hypertension, agitation, formication(sensation of insects crawling on or under the skin), and mydriasis. Among the other findingsthat may be observed are nausea, vomiting or diarrhea, palpitations, diaphoresis, hyperthermia,and seizures, all related to the central and peripheral nervous system stimulant effects ofamphetamine-induced release of dopamine, serotonin, and catecholamines.

Management of an acute amphetamine overdose is directed at stabilization of vital signs,decontamination, and relief of symptoms. Tachycardia and hypertension often are related toagitation, and sedation with benzodiazepines frequently obviates the need for further treatment.Patients who have hyperthermia should be cooled aggressively, and seizures should be treatedwith benzodiazepines. Decontamination should be performed with oral activated charcoal priorto sedation or with a protected airway in the sedated patient.

The patient described in the vignette should be given lorazepam and monitored fordecreasing agitation. Because electrocardiography shows sinus tachycardia, adenosine, thedrug of choice for supraventricular tachycardia, is not indicated. Haloperidol is not recommendedfor agitation related to amphetamine overdose because it lowers the seizure threshold.Antihypertensives such as labetalol rarely are needed once agitation has been treatedeffectively. Naloxone is indicated for treatment of an opiate overdose.

References:

Handly N. Toxicity, amphetamine. eMedicine Specialties, Emergency Medicine, Toxicology.2007. Available at: http://www.emedicine.com/emerg/topic23.htm

Krull KR. Pharmacotherapy for attention deficit hyperactivity disorder in children andadolescents. UpToDate Online 16.3. 2008. Available at:http://www.utdol.com/online/content/topic.do?topicKey=behavior/7213&selectedTitle=8~150&source=search_result

Copyright © 2010 by the American Academy of Pediatrics page 436

Page 437: AAP PREP 2010

2010 PREP SA on CD-ROM

Question: 177

A 16-year-old girl presents with a complaint of constipation. She passes two to three small,pellet-like stools per week and claims that she has not experienced a "normal bowel movement"in 2 months. She usually skips breakfast and buys lunch at school. Both parents are employed,but the family tries to eat dinner together, usually at 8 pm during the week. Physical examinationdemonstrates a well-developed, well-nourished adolescent who has no unusual findings. Rectalexamination reveals normal anal sphincter tone and an empty rectal vault.

Of the following, the MOST appropriate treatment of this patient’s constipation includes

A. lactulose

B. methylcellulose

C. milk of magnesia

D. mineral oil

E. polyethylene glycol

Copyright © 2010 by the American Academy of Pediatrics page 437

Page 438: AAP PREP 2010

2010 PREP SA on CD-ROM

Preferred Response: BCritique: 177

The adolescent described in the vignette has erratic eating habits and presents with therecent onset of a small-volume, infrequent stooling pattern. Physical examination reveals noobvious abnormalities, and rectal examination demonstrates normal anal sphincter tone and arectal vault devoid of feces. This young woman is exhibiting nonretentive (nonwithholding)constipation that should be treated with a diet that includes increased fluid intake and acommercially available fiber supplement such as methylcellulose.

The dietary history and absence of feces in the rectal vault reported for the girl in thevignette suggest that a more conservative approach that does not encompass medications maybe attempted initially. Recent evidence indicates that fiber supplements for constipationmanagement are an important addition to the therapeutic armamentarium, particularly for those inwhom dietary history suggests poor fiber intake. Multiple fiber supplements are available overthe counter, and two of the most frequently used are psyllium and methylcellulose. No availableevidence suggests superiority of one formulation over another; patients should be encouragedto use whichever product is the most palatable to them, thus assuring the greatest likelihood ofcompliance. Depending on the patient’s response to this nonpharmacologic approach, therapyusing an osmotic agent may be added after an adequate trial of the fiber supplement.

Constipation comprises nearly 5% of all primary care pediatric and nearly 25% of allpediatric gastroenterology visits in the United States. Although many factors, includingphysiologic, anatomic, and psychological disorders may be involved, most affected childrenexhibit no identifiable pathologic condition, and the term "functional constipation" is applied totheir condition. In the young child, constipation most commonly is the consequence of hard,painful bowel movements that lead to voluntary stool withholding. Stress, dietary changes, toilettraining, and being "too busy" to use the bathroom are among the many precipitating factors.Prolonged withholding results in fecal stasis and fluid reabsorption in the colon, with an increasein stool size and consistency. Physical findings often include mild abdominal distention and apalpable stool mass in the left lower quadrant. Rectal examination typically demonstrates a rectalvault filled with firm stool, often extending to the anal verge.

A careful history and physical examination should direct the clinician to prescribing anappropriate course of therapy. For children who have stool withholding (with or withoutencopresis), behavior modification may be beneficial as an adjunct to medical therapy,particularly for the patient older than 5 years of age. However, treatment should include someform of pharmacologic intervention.

Oral medications used in the treatment of constipation fall into three defined categories:osmotic agents, lubricants, and stimulants. Each drug class possesses a different mechanism ofaction. In general, stimulants (eg, senna, bisacodyl), which promote enhanced colonic transit,should be avoided as long-term therapy, although they may have some efficacy during an initial"clean-out" period. Lubricants such as mineral oil have long been used as stool-softening agentsand to reduce withholding while promoting rectal evacuation. Although mineral oil-containinglaxatives theoretically may interfere with vitamin absorption, no available evidence supports thisassertion. Currently, the most frequently used medications for treatment of functionalconstipation are the osmotic agents, including lactulose (a synthetic disaccharide), sorbitol, andpolyethylene glycol (PEG 3350). These agents increase intestinal intraluminal osmolality, thereby

Copyright © 2010 by the American Academy of Pediatrics page 438

Page 439: AAP PREP 2010

2010 PREP SA on CD-ROM

inducing water movement. A softer, larger volume of stool is produced, with gradual dissolutionof hard fecal matter. These medications are well tolerated long-term, and their doses may betitrated to achieve maximal effectiveness. Recent evidence indicates that PEG 3350 has greaterpalatability and acceptance over all age ranges, and preliminary clinical data suggest thatpolyethylene glycol is safe and effective in infants. However, recommendations regarding itsuse in children younger than 1 year of age must await further clinical trials. Magnesiumhydroxide also acts as an osmotic agent, but abdominal cramping and the risk of overdosing,leading to hypermagnesemia, hypophosphatemia, and secondary hypocalcemia, make this a lessdesirable alternative for long-term use.

References:

Abi-Hanna A, Lake AM. Constipation and encopresis in childhood. Pediatr Rev. 1998;19:23-30.Available at: http://pedsinreview.aappublications.org/cgi/content/full/19/1/23

Chao HC, Lai MW, Kong MS, Chen SY, Chen CC, Chiu CH. Cutoff volume of dietary fiber toameliorate constipation in children. J Pediatr. 2008;153:45-49. Abstract available at:http://www.ncbi.nlm.nih.gov/pubmed/18571534

Constipation Guidelines Committee of the North American Society for Pediatric Gastroenterology,Hepatology and Nutrition. Evaluation and treatment of constipation in infants and children:recommendations of the North American Society for Pediatric Gastroenterology, Hepatology andNutrition. J Pediatr Gastroenterol Nutr. 2006;43:e1-e13. Abstract available at:http://www.ncbi.nlm.nih.gov/pubmed/16954945

Loening-Baucke V, Miele E, Staiano A. Fiber (glucomannan) is beneficial in the treatment ofchildhood constipation. Pediatrics. 2004;113:e259-e264. Available at:http://pediatrics.aappublications.org/cgi/content/full/113/3/e259

Pashankar DS, Bishop WP, Loening-Baucke V. Long-term efficacy of polyethylene glycol 3350for the treatment of chronic constipation in children with and without encopresis. Clin Pediatr(Phila). 2003;42:815-819. Abstract available at: http://www.ncbi.nlm.nih.gov/pubmed/14686553

Roma E, Adamidis D, Nikolara R, Constantopoulos A, Messaritakis J. Diet and chronicconstipation in children: the role of fiber. J Pediatr Gastroenterol Nutr. 1999; 28:169-174.Abstract available at: http://www.ncbi.nlm.nih.gov/pubmed/9932850

Copyright © 2010 by the American Academy of Pediatrics page 439

Page 440: AAP PREP 2010

2010 PREP SA on CD-ROM

Question: 178

A 28-year-old woman who is positive for human immunodeficiency virus and has a history ofintravenous drug use delivers a 2,300-g term infant. She had only two prenatal visits, and shewas being treated for Chlamydia infection at the time of delivery. Physical examination of theinfant reveals facial edema, erythema and scaling of the palms and soles (Item Q178), clearrhinorrhea, and hepatosplenomegaly.

Of the following, the MOST likely cause of this infant’s signs and symptoms is infection with

A. Candida albicans

B. Chlamydia trachomatis

C. Pneumocystis jiroveci

D. Streptococcus agalactiae

E. Treponema pallidum

Copyright © 2010 by the American Academy of Pediatrics page 440

Page 441: AAP PREP 2010

2010 PREP SA on CD-ROM

Preferred Response: ECritique: 178

The newborn described in the vignette has signs of congenital syphilis due to the organismTreponema pallidum. The risk for maternal syphilis is increased in women who are humanimmunodeficiency virus (HIV)-positive, and the infection may be transmitted vertically in utero tothe fetus at any point in the gestation. It also can be transmitted perinatally at delivery. Earlyphysical signs of congenital syphilis include hydrops fetalis, intrauterine growth restriction,hepatosplenomegaly, hemolytic anemia, jaundice, and a maculopapular rash (Item C178A).Radiographic findings include lines of arrested growth, metaphyseal destruction, and periostealchanges (periosteitis) in the long bones. Finally, sensorineural hearing loss, detectable usingautomated auditory brainstem response technology, may be present.

Maternal HIV infection is associated with a 2% to 25% vertical transmission risk, dependingupon maternal disease stage and highly active antiretroviral therapy, infant gestational age,rupture of membranes, and mode of delivery. However, other than potential growth restriction,HIV infection does not manifest acutely in the immediate neonatal period. Appropriate testing forHIV viral particles and early chemoprophylaxis with zidovudine is important in reducing the riskfor HIV disease in infancy. The use of trimethoprim-sulfamethoxazole as prophylaxis againstopportunistic infection with Pneumocystis jiroveci (previously known as Pneumocystis carinii),an important cause of pneumonia in those infected with HIV, is recommended beginning at thepostnatal age of 6 weeks. However, other than potential growth restriction, HIV infection doesnot manifest acutely in the immediate neonatal period and does not produce the cutaneousfindings exhibited by the infant in the vignette.

Candida albicans infection may be transmitted to the newborn from the mother’s vagina andpresents with a diffuse eruption composed of erythematous papules, pustules, and scaling (ItemC178B). In extremely low-birthweight newborns (<1,000 g), it may cause significant systemicmorbidity and high mortality. Maternal Chlamydia trachomatis infection that is partially treatedmay confer risk to the newborn because vertical transmission of this pathogen may occur in25% to 60% of pregnancies. However, neonatal chlamydial disease manifests primarily as arespiratory (pneumonia) or ocular (conjunctivitis) problem.

Neonatal infection with Streptococcus agalactiae (group B Streptococcus) is a serious andlife-threatening condition that may have an early onset (first week after birth) or late onset (1 to12 weeks after birth). Early-onset disease typically becomes apparent in the first 48 hours afterbirth and sometimes at delivery. The major clinical signs are those of septicemia, pneumonia, andmeningitis; a rash is not typically present. The most fulminant form presents with the newborn inhemodynamic collapse (shock) and hypoxic respiratory failure.

References:

American Academy of Pediatrics. Candidiasis (moniliasis, thrush). In: Pickering LK, Baker CJ,Kimberlin DW, Long SS, eds. Red Book: 2009 Report of the Committee on Infectious Diseases.28th ed. Elk Grove Village, Ill: American Academy of Pediatrics; 2009:245-249

American Academy of Pediatrics. Chlamydia trachomatis. In: Pickering LK, Baker CJ, KimberlinDW, Long SS, eds. Red Book: 2009 Report of the Committee on Infectious Diseases. 28th ed.

Copyright © 2010 by the American Academy of Pediatrics page 441

Page 442: AAP PREP 2010

2010 PREP SA on CD-ROM

Elk Grove Village, Ill: American Academy of Pediatrics; 2009:255-259

American Academy of Pediatrics. Group B streptococcal infections. In: Pickering LK, Baker CJ,Kimberlin DW, Long SS, eds. Red Book: 2009 Report of the Committee on Infectious Diseases.28th ed. Elk Grove Village, Ill: American Academy of Pediatrics; 2009:628-634

American Academy of Pediatrics. Pneumocystis jiroveci infections. In: Pickering LK, Baker CJ,Kimberlin DW, Long SS, eds. Red Book: 2009 Report of the Committee on Infectious Diseases.28th ed. Elk Grove Village, Ill: American Academy of Pediatrics; 2009:536-540

American Academy of Pediatrics. Syphilis. In: Pickering LK, Baker CJ, Kimberlin DW, Long SS,eds. Red Book: 2009 Report of the Committee on Infectious Diseases. 28th ed. Elk GroveVillage, Ill: American Academy of Pediatrics; 2009:638-651

Centers for Disease Control and Prevention, Workowski KA, Berman SM. Congenital syphilis.Sexually transmitted diseases treatment guidelines 2006. MMWR Morb Mortal Wkly Rep.2006;55(RR-11):30-33. Available at:http://www.guideline.gov/summary/summary.aspx?doc_id=9676&nbr=5185&ss=6&xl=999

Yudin MH, Gonik B. Perinatal infections. In: Martin RJ, Fanaroff AA, Walsh MC, eds. Fanaroff andMartin's Neonatal-Perinatal Medicine: Diseases of the Fetus and Infant. 8th ed. Philadelphia, Pa:Mosby Elsevier; 2006:429-454

Copyright © 2010 by the American Academy of Pediatrics page 442

Page 443: AAP PREP 2010

2010 PREP SA on CD-ROM

Question: 179

A 14-year-old girl comes to your office because her mother is concerned about scoliosis. Themother had mild scoliosis as an adolescent, but she did not require treatment. The girl ispreviously healthy and participates in sports at school. She denies back pain, weakness, orabnormal sensations. On physical examination, lumbar asymmetry is apparent when she bendsforward with her arms hanging down. There are no cutaneous findings, and her neurologicexamination results are normal. You order posteroanterior and lateral radiographs of the spine toevaluate the degree of scoliosis.

Of the following, the radiographic finding that is MOST suggestive of a nonidiopathic cause forthis girl’s scoliosis is a

A. Cobb angle of 30 degrees

B. lack of thoracic kyphosis

C. lumbar curve to the left

D. thoracic curve to the right

E. widening of the pedicles

Copyright © 2010 by the American Academy of Pediatrics page 443

Page 444: AAP PREP 2010

2010 PREP SA on CD-ROM

Preferred Response: ECritique: 179

One of the primary considerations for the clinician evaluating an adolescent who hasscoliosis is determining whether the scoliosis is idiopathic or due to another abnormality, such asvertebral anomaly, leg length discrepancy, spinal cord lesion, or neuromuscular disorder.Scoliosis is defined as a lateral curvature of the spine greater than 10 degrees on a posterior-anterior spine radiograph, and most cases in adolescent patients are idiopathic. Clinical featuressuggesting a secondary cause for the curvature include nighttime or severe back pain andneurologic symptoms, such as bowel/bladder incontinence and weakness. Radiologic findingscan add to the suspicion for a secondary cause. In idiopathic scoliosis, the curve is usually tothe left in the lumbar region and to the right in the thoracic region (Remember Left-Lumbar). Acurve to the opposite side in either of these areas is suggestive of a nonidiopathic cause. Otherradiologic features suggesting secondary scoliosis are significant thoracic kyphosis andwidening of the pedicles, which can be seen with spinal canal anomalies. The degree ofcurvature does not distinguish between primary and secondary causes.

When a patient seeks medical attention for concerns about scoliosis, it is important to take acomplete history, including pubertal development, and perform a complete physical examination,including forward bending test, neurologic evaluation, and skin examination. However, routinescreening of all adolescents, as has been performed previously in schools, is controversial. Areport by the United States Preventive Services Task Force in 2004 recommended againstscreening all adolescents regardless of symptoms because of the risk of unnecessary bracingand no evidence that routine screening decreased the need for surgical intervention in affectedpatients. The Task Force suggested that most cases of scoliosis discovered at screening wouldnot progress to clinical significance, and those that would require surgical treatment would beevident without screening. Another study by the Netherlands Evaluation Study on Screening forScoliosis in 2008 concluded that screening programs could be abolished due to similar findings.However, an information statement endorsed by the American Academy of Pediatrics andorthopedic societies, such as the American Academy of Orthopaedic Surgeons, states thatbenefits of early bracing could be substantial in some patients and that if screening programswere put in place, screening should be performed in girls at ages 10 and 12 years and in boysat ages 13 and 14 years. Screeners should be well trained at detecting abnormalities on theforward bending test, and adolescents who have abnormal test results should be referred totheir primary physicians for evaluation. Abnormal findings on evaluation by the primary physicianshould prompt radiographic confirmation, but those who have normal findings can be observed.

References:

Bunge EM, Juttmann RE, van Biezen FC, et al for the Netherlands Evaluation Study on Screeningfor Scoliosis (NESCIO) Group. Estimating the effectiveness of screening for scoliosis: a case-control study. Pediatrics. 2008;121:9-14. Available at:http://pediatrics.aappublications.org/cgi/content/full/121/1/9

Richards BS, Vitale MG. Screening for idiopathic scoliosis in adolescents: an informationstatement. J Bone Joint Surg. 2008;90:195-198. Available at:

Copyright © 2010 by the American Academy of Pediatrics page 444

Page 445: AAP PREP 2010

2010 PREP SA on CD-ROM

http://www.ejbjs.org/cgi/content/full/90/1/195

Stewart DG Jr, Skaggs DL. Consultation with the specialist: adolescent idiopathic scoliosis.Pediatr Rev. 2006;27:299-306. Available at:http://pedsinreview.aappublications.org/cgi/content/full/27/8/299

U.S. Preventive Services Task Force. Screening for Idiopathic Scoliosis in Adolescents:Recommendation Statement. Rockville, Md: Agency for Healthcare Research and Quality; June2004. Available at: http://www.ahrq.gov/clinic/3rduspstf/scoliosis/scoliors.htm

Copyright © 2010 by the American Academy of Pediatrics page 445

Page 446: AAP PREP 2010

2010 PREP SA on CD-ROM

Question: 180

A large-for-gestational age newborn girl who was born via caesarean section for breechpresentation is ready for discharge from the newborn nursery at 72 hours of age. She has hadan entirely normal newborn course.

Of the following, you are MOST likely to advise the mother that her infant is at increased risk for

A. developmental dysplasia of the hip

B. positional deformity of the foot

C. positional plagiocephaly

D. recurrent hypocalcemia

E. recurrent hypoglycemia

Copyright © 2010 by the American Academy of Pediatrics page 446

Page 447: AAP PREP 2010

2010 PREP SA on CD-ROM

Preferred Response: ACritique: 180

Female sex, breech presentation, and family history have been suggested as contributing tothe epidemiology of developmental dysplasia of the hip. Accordingly, the infant described in thevignette should be monitored for this condition. Initially believed to be a congenital dysplasia, itnow is understood that multiple factors are involved in developmental dysplasia of the hip, whichmay evolve and be detected after the immediate newborn period. A careful history and attentivephysical examination are important to detect an abnormal hip in the young infant. Imaging studiesmay be indicated in infants who are at high risk for the disease. Careful follow-up and physicalexamination of infants at increased risk, especially females who had nonvertex presentationsand those who have concerning findings on hip examination over the first 2 months after birth, isthe standard of care, with imaging reserved for those at highest risk. Once concern for hipdysplasia has been established, highly repetitious hip examinations should be avoided becausethe Ortolani and Barlow maneuvers may injure the hip capsule. Prompt imaging and referralshould ensue.

Recent outcome-based epidemiologic studies suggest that imaging studies should beperformed at 1 to 2 months of age rather than in the immediate postnatal period and should belimited primarily to ultrasonography in early infancy. Routine imaging of all children based on riskis not effective from either a cost or detection standpoint. Minor anomalies seen onultrasonography early in infancy often resolve. Later in infancy, anteroposterior and "frog-leg"position plain radiographs are more likely to be helpful in the child who has abnormal results onhip examination.

The large-for-gestational age infant described in the vignette, who is asymptomatic and hasnormal findings on physical examination prior to discharge, is unlikely to have an increased riskfor recurrent hypocalcemia or hypoglycemia. Positional deformity of the foot is, by definition,transient, in contrast to talipes equinovarus deformity. Positional plagiocephaly is a risk forinfants placed in the supine sleep position; perinatal risk factors such as prematurity or abnormalmuscle tone may predispose to dolichocephaly.

References:

Goldberg MJ. Early detection of developmental hip dysplasia: synopsis of the AAP clinicalpractice guideline. Pediatr Rev. 2001;22:131-134. Available at:http://pedsinreview.aappublications.org/cgi/content/full/22/4/131

Schwend RM. A reappraisal of the Ortolani examination for developmental hip dysplasia. AAPGrand Rounds. 2007;17:57-58. Available at:http://aapgrandrounds.aappublications.org/cgi/content/full/17/5/57

Shipman SA, Helfand M, Moyer VA, Yawn BP. Screening for developmental dysplasia of the hip:a systematic literature review for the US Preventive Services Task Force. Pediatrics.2006;117:e557-e576. Available at:http://pediatrics.aappublications.org/cgi/content/full/117/3/e557

Copyright © 2010 by the American Academy of Pediatrics page 447

Page 448: AAP PREP 2010

2010 PREP SA on CD-ROM

Question: 181

You are evaluating a 12-year-old boy who has been fatigued for 2 weeks. His mother reportsthat he had an upper respiratory tract infection 2 weeks ago, and his appetite has beendecreased since then. On physical examination, he is afebrile and has a heart rate at rest of 110beats/min. His respiratory rate is 22 breaths/min. His lungs are clear, and he has a gallop rhythmwithout murmurs on cardiac auscultation. You discern hepatomegaly and mild jugular venousdistention.

Of the following, the MOST likely diagnosis is

A. anemia

B. dilated cardiomyopathy

C. Kawasaki disease

D. primary pulmonary hypertension

E. pulmonary embolism

Copyright © 2010 by the American Academy of Pediatrics page 448

Page 449: AAP PREP 2010

2010 PREP SA on CD-ROM

Preferred Response: BCritique: 181

The gallop rhythm, hepatomegaly, and jugular venous distention described for the boy in thevignette support the diagnosis of congestive heart failure (CHF), most likely due to myocardialdysfunction associated with dilated cardiomyopathy. Generally, CHF is a clinical syndrome thatreflects the inability of the myocardium to meet the metabolic requirements of the body, includingthose for growth. The presentation in the older child differs from that of the young infant. In theformer, CHF usually presents with signs and symptoms of fatigue, particularly with exercise oractivity. In addition, children may present with shortness of breath, palpitations, diaphoresis, andin the most acute cases, extremis. Almost invariably, the left ventricle is affected, and as itssystolic and diastolic function diminishes, its filling pressures increase. Clinically, this maymanifest during auscultation as a gallop rhythm. The increased left ventricular filling pressuresresults in rising pressures in the pulmonary veins, pulmonary capillaries, pulmonary arteries,right ventricle, and right atrium. When the right-sided filling pressures increase, the systemicveins that drain into the right atrium, including those of the hepatic system and the jugularsystem, become congested. Congestion of the former leads to hepatomegaly and that of thelatter may manifest with jugular venous distention discernible on examination.

Laboratory support for the myocardial failure seen in patients who have CHF can bedemonstrated by an elevation in the brain natriuretic peptide value. Results of this test almostalways are abnormal in patients who have significant CHF.

Among the many causes of CHF are large-volume left-to-right shunts with pulmonaryovercirculation, pressure load on the myocardium, inadequate blood flow to the myocardium,infection or infiltration of the myocardium, or genetic or idiopathic diseases of the myocardium.CHF from large-volume shunting lesions is seen almost exclusively during infancy. The othercauses may manifest any time throughout infancy, childhood, or adolescence.

Anemia can lead to a "high-output" state but does not present with the right heart failuredemonstrated by the patient in the vignette. Although Kawasaki disease can present in somecases with CHF due to acute myocarditis, the patient in the vignette has no other physicalfindings to support this diagnosis. Primary pulmonary hypertension is seen more typically infemales during adolescence or adulthood and includes the presence of a loud second heartsound with or without a gallop rhythm. Pulmonary embolism typically presents more acutely withchest pain, hypoxemia, and tachypnea in addition to the findings of acute right heart failure.

References:

Balfour I. Management of chronic congestive heart failure in children. Curr Treat OptionsCardiovasc Med. 2004;6:407-416. Abstract available at:http://www.ncbi.nlm.nih.gov/pubmed/15324616

Dreyer WJ, Fisher DJ. Clinical recognition and management of chronic congestive cardiac failure.In: Garson A Jr, Bricker JT, Fisher DJ, Neish SR, eds. The Science and Practice of PediatricCardiology. 2nd ed. Baltimore, Md: Williams & Wilkins; 1998:2309-2325

Massin MM, Astadicko I, Dessy H. Epidemiology of heart failure in a tertiary pediatric center. Clin

Copyright © 2010 by the American Academy of Pediatrics page 449

Page 450: AAP PREP 2010

2010 PREP SA on CD-ROM

Cardiol. 2008;31:388-391. Available at: http://www3.interscience.wiley.com/cgi-bin/fulltext/121387604/PDFSTART

Talner NS, McGovern JJ, Carboni MP. Congestive heart failure. In: Moller JH, Hoffman JIE, eds.Pediatric Cardiovascular Medicine. Philadelphia, Pa: Churchill Livingstone; 2000:817-829

Copyright © 2010 by the American Academy of Pediatrics page 450

Page 451: AAP PREP 2010

2010 PREP SA on CD-ROM

Question: 182

A 6-year-old boy presents for evaluation due to an episode of screaming and confusion at night.The boy's parents heard him scream in his room, and when they went to him, he exhibited rapidtwitching of his left arm and hand, stiffening of his left leg, rolled back eyes, and some blinking ofboth eyes. He was incoherent and minimally responsive for 5 to 10 minutes. After the episode,he was weak on the left side of his body. By morning, he had returned to a normal status.

Of the following, the MOST likely diagnosis is

A. benign rolandic epilepsy

B. juvenile myoclonic epilepsy

C. night terrors

D. nocturnal frontal lobe epilepsy

E. rapid eye movement sleep behavior disorder

Copyright © 2010 by the American Academy of Pediatrics page 451

Page 452: AAP PREP 2010

2010 PREP SA on CD-ROM

Preferred Response: ACritique: 182

The first unprovoked seizure described for the boy in the vignette occurred out of sleep andwas clearly partial, with vocalization, left-sided motor movements, and transient focal weaknessafterward. He is otherwise completely healthy and normal. It is possible he has had othernocturnal seizures that did not awaken his parents. The most common diagnosis for suchfindings in childhood is benign rolandic epilepsy, a childhood-onset epilepsy that typically occursbetween the ages of 3 and 13 years and resolves before adulthood. Seizures may beinfrequent, and the child usually has no other problems. Inheritance is autosomal dominant.Interictal electroencephalography showing characteristic centrotemporal spikes is confirmatory.Treatment generally is not needed because the nocturnal seizures are infrequent and do notcause any problems the following day. Partial seizure medications such as carbamazepine areeffective, if used.

Juvenile myoclonic epilepsy has a later onset, typically in teenage years, and ischaracterized by one or more of the following: 1) myoclonic jerks, usually in the morning; 2)generalized tonic-clonic seizures, often in the morning, and 3) absence seizures. Unlike benignrolandic epilepsy, juvenile myoclonic epilepsy is a form of generalized epilepsy.

Nocturnal frontal lobe epilepsy has a variable and unusual nocturnal presentation thatinvolves complex, stereotyped dystonic movements and sometimes vocalizations that can leadto confusion with parasomnias. The associated seizures typically last fewer than 2 minutes andmay cluster, occurring many times per night, at any time of the night. The affected child mayhave some partial recall of the events.

Night terrors are a non-rapid eye movement (REM) sleep parasomnia that can be confusedwith nocturnal seizures. Night terrors can start as early as 18 months of age, earlier than benignrolandic epilepsy, peak at age 5 to 7 years, and usually resolve by adolescence. Thephenomenology involves a sudden arousal, vocalization, and confusion, with autonomicchanges such as mydriasis and tachycardia. The movements are not repetitive clonicmovements or twitching, like a seizure. The child is unconscious during the episode and doesnot recall it the next day. Events usually occur, at most, twice per night and during deep slowwave sleep, in the first half of the night, 1 to 2 hours after falling asleep.

REM sleep behavior disorder is an uncommon parasomnia in children. Paralysis normallyoccurs during REM sleep. In REM sleep behavior disorder, there is a partial or full loss of thisparalysis. As a result, the individual may act out dreams that may be vivid, intense, or violent.This disorder tends to occur in adults who have neurodegenerative diseases and is associatedwith the use of psychiatric medications or alcohol withdrawal. However, cases have beendescribed in children, including those who have autism.

References:

Friedman MJ, Sharieff GQ. Seizures in children. Pediatr Clin North Am. 2006;53:257-277.Available at: http://www.ncbi.nlm.nih.gov/pubmed/16574525

Major P, Thiele EA. Seizures in children: determining the variation. Pediatr Rev. 2007;28:363-371.Available at: http://pedsinreview.aappublications.org/cgi/content/full/28/10/363

Copyright © 2010 by the American Academy of Pediatrics page 452

Page 453: AAP PREP 2010

2010 PREP SA on CD-ROM

Mason TBA 2nd, Pack AI. Sleep terrors in childhood. J Pediatr. 2005;147:388-392

Copyright © 2010 by the American Academy of Pediatrics page 453

Page 454: AAP PREP 2010

2010 PREP SA on CD-ROM

Question: 183

A baby girl for whom you provide care has been referred on her newborn hearing screen.Physical examination results are within normal limits. When she is referred on a follow-uphearing screen at 2 weeks of age, you recommend auditory brainstem response testing, whichsubsequently reveals absent hearing in both ears. A careful family history is negative for anyindividuals who have deafness. The parents ask you what could have caused deafness in theirbaby.

Of the following, the likelihood that this infant has a genetic cause for deafness is CLOSEST to

A. <1%

B. 5%

C. 25%

D. 50%

E. 75%

Copyright © 2010 by the American Academy of Pediatrics page 454

Page 455: AAP PREP 2010

2010 PREP SA on CD-ROM

Preferred Response: DCritique: 183

The incidence of prelingual, moderate-to-profound sensorineural hearing loss (>40 dB) is 1in 500 in developed countries, making it the most common birth defect in these regions.Approximately 1 in 1,000 newborns is deaf (hearing loss in the severe-to-profound range of 71to 90 dB).

Fifty percent of all prelingual, moderate-to-profound hearing loss is genetic, 25% isnongenetic (eg, having bacterial and viral causes), and 25% is idiopathic (Item C183). Of thegenetic forms of deafness, 30% are syndromic, and 70% are nonsyndromic. Syndromic causesof deafness include conditions such as Usher, Waardenburg, and Treacher-Collins syndromes.Nonsyndromic causes can be autosomal recessive (75% to 85%), autosomal dominant (15 % to24%), and X-linked (1% to 2%).

A critical aspect of the evaluation of any child who has prelingual hearing loss is definingthe type of loss as clearly as possible. Careful audiologic evaluation should be undertaken inconcert with examination by an otolaryngologist. Affected individuals should undergo computedtomography scan of the temporal bones to look for malformations of the inner ear. Theinformation gleaned from these studies helps to guide the evaluation because different genemutations are associated with different clinical manifestations. It is important to obtain a careful,three-generation pedigree, with attention to any individuals who have hearing loss, vision loss,dysmorphic features, early death, birth defects, and intellectual disabilities. Referral to ageneticist is important to determine if there is a recognizable pattern of features or any genetictesting that could be helpful.

The gene loci for nonsyndromic deafness are designated "DFN" (for DeaFNess). The lociare categorized as DFNA (autosomal dominant), DFNB (autosomal recessive), and DFN (X-linked). Some genes can be involved in both dominant and recessive forms of deafness,depending on the specific mutation(s), and some mutations cause both sensorineural andconductive hearing loss. Although there are exceptions, most autosomal dominant loci causepostlingual hearing impairment, and most autosomal recessive loci cause severe-to-profoundprelingual deafness. X-linked (DFN) loci are associated with both pre- and postlingual hearingloss.

Because the many molecular genetic testing options can be somewhat confusing,consultation with a clinical geneticist is strongly recommended. Numerous laboratories offerpanels of common mutations. In many populations worldwide, approximately 50% of individualswho have prelingual, autosomal recessive nonsyndromic hearing loss have GJB2 mutations; theremainder may have a mutation in any number of genes that may have been described ascausing deafness in only one or two families.

References:

ACMG Statement. Genetics evaluation guidelines for the etiologic diagnosis of congenital hearingloss. Genetic Evaluation of Congenital Hearing Loss Expert Panel. Genet Med. 2002;4:162-171.Abstract available at: http://www.ncbi.nlm.nih.gov/pubmed/12180152

Declau F, Boudewyns A, Van den Ende J, Peeters A, van den Heyning P. Etiologic and

Copyright © 2010 by the American Academy of Pediatrics page 455

Page 456: AAP PREP 2010

2010 PREP SA on CD-ROM

audiologic evaluations after universal neonatal hearing screening: analysis of 170 referredneonates. Pediatrics. 2008;121:1119-1126. Available at:http://pediatrics.aappublications.org/cgi/content/full/121/6/1119

Smith RJ, Van Camp G. Deafness and hereditary hearing loss overview. GeneReviews. 2008.Available at: http://www.ncbi.nlm.nih.gov/bookshelf/br.fcgi?book=gene&part=deafness-overview

Copyright © 2010 by the American Academy of Pediatrics page 456

Page 457: AAP PREP 2010

2010 PREP SA on CD-ROM

Question: 184

The mother of a 12-year-old white girl is concerned because her daughter has not yetmenstruated. Most of her daughter’s classmates are menstruating. She does not rememberwhen her daughter began breast development but states that the girl is now taller than her 13-year-old brother. The mother asks if you can estimate when her daughter is going to have herfirst menstrual period.

Of the following, the MOST appropriate next step to answer the mother’s question is to

A. determine bone age

B. determine Sexual Maturity Rating

C. measure alkaline phosphatase

D. obtain an endocrinology consultation

E. order pelvic ultrasonography

Copyright © 2010 by the American Academy of Pediatrics page 457

Page 458: AAP PREP 2010

2010 PREP SA on CD-ROM

Preferred Response: BCritique: 184

The National Health and Nutrition Examination Survey (NHANES) III and the National HealthExamination Survey (NHES) indicate that the median age of menarche in United States girls is12.43 years and has not changed over the past 3 decades. Certain ethnic differences haveemerged, with non-Hispanic black girls having menses earlier than white girls and menses forHispanic girls falling between these two groups. The interval between breast development (thefirst sign of puberty in females) and menstruation is 2.3+0.1 years, but the range is 0.5 to 5.75years. Most adolescents (62%) menstruate when they reach Sexual Maturity Rating (SMR)stage 4, with 26% in stage 3 and 11% in stage 5. Only 1% menstruate in stage 2. Menarcheoccurs approximately 3.3 years after the onset of the growth spurt and 1.1 years after the peakheight velocity is attained. The percentage of body fat is critical; according to Frisch’s theory, aminimum body fat value of 17% is required to initiate menses and a value of 22% is required tomaintain regular cycles.

The girl described in the vignette has had a significant growth spurt, which indicates thatshe has entered puberty and is probably in SMR stage 2 or 3. If this is confirmed by clinicalexamination, no further evaluation is necessary and it is likely that the girl will begin menstruatingin the next year or two. A mother’s age of menstruation can be used as a guide for herdaughters’ menarche, but is not as useful as determination of her daughter’s SMR.

If a patient shows no sign of pubertal development by age 13.5 years or if there is noprogression of puberty, endocrinologic consultation and performance of pelvic ultrasonographymay be indicated. The bone age is a useful tool in patients who have delayed puberty becausepubertal events correlate most closely with bone age and not chronologic age. Measuring serumalkaline phosphatase will not be of benefit in predicting menarche.

References:

Carswell JM, Stafford DEJ. Normal physical growth and development. In: Neinstein L, ed.Adolescent Health Care: A Practical Guide. 5th ed. Philadelphia Pa: Lippincott Williams & Wilkins;2008:3-26

Chumlea WC, Schubert CM, Roche AF, et al. Age at menarche and racial comparisons in USgirls. Pediatrics. 2003;111:110-113. Available at:http://pediatrics.aappublications.org/cgi/content/full/111/1/110

Gordon CM, Laufer MR. The physiology of puberty. In: Emans SJ, Laufer MR, Goldstein DP.Pediatric and Adolescent Gynecology. 5th ed. Philadelphia, Pa: Lippincott Williams & Wilkins;2005:120-155

Sun SS, Schubert CM, Liang R, et al. Is sexual maturity occurring earlier among U.S. children? JAdolesc Health. 2005;37:345-355. Abstract available at:http://www.ncbi.nlm.nih.gov/pubmed/16227118

Copyright © 2010 by the American Academy of Pediatrics page 458

Page 459: AAP PREP 2010

2010 PREP SA on CD-ROM

Question: 185

You have administered a benzodiazepine to a 3-year-old boy on the pediatric ward to allow thecompletion of echocardiography. On physical examination, his eyes are closed, his temperatureis 37.0°C, heart rate is 100 beats/min, respiratory rate is 20 breaths/min, blood pressure is 85/55mm Hg, and oxygen saturation on room air is 97% by pulse oximetry. He opens his eyes oncommand and pushes away your hand when you attempt to open his eyelids.

Of the following, the level of sedation achieved for this boy is defined BEST as

A. anxiolysis

B. deep sedation

C. general anesthesia

D. minimal sedation

E. moderate sedation

Copyright © 2010 by the American Academy of Pediatrics page 459

Page 460: AAP PREP 2010

2010 PREP SA on CD-ROM

Preferred Response: ECritique: 185

Sedation should be viewed as a continuum, with patients easily moving between fourdefined levels. Minimal sedation (formerly anxiolysis) is a drug-induced state in which the patientretains the ability to respond normally to verbal commands, and cardiorespiratory functions areunaffected. Moderate sedation (formerly conscious sedation) is a drug-induced depression ofconsciousness in which patients still should respond to verbal and physical stimulation.Cardiovascular status is maintained, but the clinician must be able to recognize and respond topotential airway compromise. Deep sedation is defined as further depression of the level ofconsciousness, with partial or complete loss of protective airway reflexes and the need forassistance with airway maintenance. General anesthesia is the deepest level of sedation that ischaracterized by loss of consciousness and airway protective reflexes. Impairment ofrespiratory and cardiovascular function is common.

The child described in the vignette has been given a benzodiazepine and has reached amoderate level of sedation, as evidenced by his stable cardiovascular and respiratory status. Heopens his eyes with verbal commands and responds appropriately to stimulation. The clinicianmust remain alert to potential airway compromise and respond appropriately.

The goals of procedural sedation are to provide anxiolysis and analgesia and to decreasepatient movement while minimizing risks to the patient. The American Academy of Pediatrics andthe American Academy of Pediatric Dentistry first published guidelines for the monitoring andmanagement of procedural sedation in 1985, with the most recent revision published in 2006.

Safe and effective procedural sedation consists of several key processes that include

evaluation, sedation, and recovery of the patient. Critical requirements include:•Presedation assessment of the patient for underlying medical or surgical conditions that

increase sedation risk.•Assessment for airway abnormalities that might predispose to obstruction. If significant

concerns exist, referral to clinical settings that can support a higher level of sedation and

management of potential complications is indicated. •Appropriate fasting for elective procedures. Emergent procedures require careful

assessment of the balance between aspiration risk and the risk of delaying a procedure.

•Clinicians who possess a clear understanding of the sedative medications to be used.•Appropriate equipment and adequate personnel to perform the procedure, monitor the

patient, and provide rescue if needed.•Appropriate discharge instructions and follow-up after recovery to the presedation level

of consciousness.

References:

American Academy of Pediatrics, American Academy of Pediatric Dentistry, Coté CJ, Wilson S,AAP Work Group on Sedation. Guidelines for monitoring and management of pediatric patientsduring and after sedation for diagnostic and therapeutic procedures: an update. Pediatrics.2006;118:2587-2602. Available at:

Copyright © 2010 by the American Academy of Pediatrics page 460

Page 461: AAP PREP 2010

2010 PREP SA on CD-ROM

http://pediatrics.aappublications.org/cgi/content/full/118/6/2587

Koh JL, Palermo T. Conscious sedation: reality or myth? Pediatr Rev. 2007:28:243-248. Availableat: http://pedsinreview.aappublications.org/cgi/content/full/28/7/243

Wetzel R. Anesthesia and perioperative care. In: Kliegman RM, Behrman RE, Jenson HB, StantonBF, eds. Nelson Textbook of Pediatrics. 18th ed. Philadelphia, Pa: Saunders Elsevier; 2007:460-474

Copyright © 2010 by the American Academy of Pediatrics page 461

Page 462: AAP PREP 2010

2010 PREP SA on CD-ROM

Question: 186

You are examining a 6-year-old boy during a health supervision visit. Physical examinationreveals Sexual Maturity Rating 3 pubic hair and a phallus that appears androgen-stimulated (7cm in stretched length).

Of the following, the MOST important additional step to guide your laboratory investigation is

A. examination of testicular size

B. investigation of possible exposure to androgen

C. investigation of the age of puberty in family members

D. measurement of height and weight

E. skin examination for café au lait macules

Copyright © 2010 by the American Academy of Pediatrics page 462

Page 463: AAP PREP 2010

2010 PREP SA on CD-ROM

Preferred Response: ACritique: 186

The pubic hair and an androgen-stimulated phallus described for the 6-year-old boy in thevignette are characteristic of sexual precocity. An examination for testicular size can direct theevaluation. If the androgen is coming from his testes, they should be greater than the 2 mL (2 cmin length [prepubertal size]). The investigation then would focus on autonomous testicularfunction, human chorionic gonadotropin secretion by a tumor, or central activation of thehypothalamic-pituitary axis. If one testis seems to be larger than the other or appears nodular, aLeydig cell tumor secreting testosterone should be considered. If the testes are 2 mL or less (<2cm in length [prepubertal size]), the evaluation should focus on other sources of endogenousandrogen, perhaps from the adrenal gland, and include evaluation for congenital adrenalhyperplasia and adrenal tumor. Exogenous androgen administration or exposure is anotherpossibility.

Once testicular size has been assessed, other areas may be investigated. Questioningabout exposure to exogenous androgen always should be part of the history of a child who hasthis type of presentation but would be less likely to guide management if there is testicularenlargement. Very early puberty in male family members might point to a diagnosis of male-limitedsexual precocity because of a mutation in the luteinizing hormone receptor of the Leydig cellleading to autonomous activation. A review of the child’s growth charts would help elucidate thetime frame of development of puberty because a growth spurt is expected as a result of thetestosterone exposure, but such a finding would offer little with respect to establishing adiagnosis. The presence of café au lait macules that have irregular outlines might suggestMcCune Albright syndrome, which can be associated with early puberty because of anactivating mutation in GNAS1, the gene encoding Gs alpha, leading to autonomous Leydig cellfunction. Café au lait macules that have smooth borders might point to sexual precocity usuallyassociated with an optic nerve glioma in neurofibromatosis type 1.

References:

Kaplowitz PB. Precocious puberty. eMedicine Specialties, Pediatrics: General Medicine,Endocrinology. 2007. Available at: http://emedicine.com/ped/TOPIC1882.HTM

Lazar L, Pertzelan A, Weintrob N, Phillip M, Kauli R. Sexual precocity in boys: accelerated versusslowly progressive puberty gonadotropin-suppressive therapy and final height. J Clin EdocrinolMetab. 2001;86:4127-4132. Available at: http://jcem.endojournals.org/cgi/content/full/86/9/4127

Muir A. Precocious puberty. Pediatr Rev. 2006;27:373-381. Available at:http://pedsinreview.aappublications.org/cgi/content/full/27/10/373

New MI. Extensive clinical experience: nonclassical 21-hydroxylase deficiency. J ClinEndocrinol Metab. 2006;91:4205-4214. Available at:http://jcem.endojournals.org/cgi/content/full/91/11/4205

Copyright © 2010 by the American Academy of Pediatrics page 463

Page 464: AAP PREP 2010

2010 PREP SA on CD-ROM

Question: 187

A 2½-year-old boy is not yet saying any words. His parents are worried that he is notdeveloping like his peers and are concerned that he is not yet talking. Other than two episodesof otitis media in the past year, he has been in good health. A formal audiology evaluationreveals normal hearing. His head circumference is at the 90th percentile; his weight and heightare at the 75th percentile. His physical examination findings are normal. His father did not talkuntil he was 3 years old.

Of the following, the MOST appropriate next step is to

A. order electroencephalography

B. order head magnetic resonance imaging

C. reassure the parents and have the child return in 6 months

D. refer the child for developmental evaluation

E. refer the child for neurologic evaluation

Copyright © 2010 by the American Academy of Pediatrics page 464

Page 465: AAP PREP 2010

2010 PREP SA on CD-ROM

Preferred Response: DCritique: 187

A 2½-year-old child who is not yet speaking should have his hearing evaluated, and heshould be referred for a developmental evaluation to determine whether he has an isolatedlanguage disorder or a pervasive social impairment. A brain imaging study is not indicated for achild who has a normal head circumference and no neurologic symptoms or findings. If the boyin the vignette had significant language regression or episodes of uninterruptable staring,neurologic evaluation and electroencephalography should be considered. A developmentalevaluation should be considered if there is a strong family history of developmental delays.Reassurance and a return for evaluation in 6 months is not appropriate; not saying words at theage of 2½ years indicates a need for neurodevelopmental evaluation.

As a result of reviewing this information, do you intend to make a change in practiceto provide better patient care?Yes No

References:

Johnson CP Myers SM. Overview of the AAP autism spectrum disorders toolkit and guidelines: apediatrician’s roadmap to the latest ASD guidelines. Contemp Pediatr. 2008 October:43-67

Johnson CP, Myers SM, and the Council on Children With Disabilities. Identification and evaluationof children with autism spectrum disorders. Pediatrics. 2007;120:1183-1251. Available at:http://pediatrics.aappublications.org/cgi/content/full/120/5/1183

Parr J. Autism. BMJ Clinical Evidence. 2008. Available for subscription at:http://clinicalevidence.bmj.com/ceweb/conditions/chd/0322/0322.jsp

Copyright © 2010 by the American Academy of Pediatrics page 465

Page 466: AAP PREP 2010

2010 PREP SA on CD-ROM

Question: 188

A 10-year-old boy presents to the emergency department with a 5-day history of fever, severeheadache, and abdominal pain. His parents report that a papular rash (Item Q188) has appearedover the past 24 hours, beginning on his wrists and ankles and extending centrally. They addthat he appears tired and "out of it." Ten days ago, the family returned from a camping trip to theChesapeake Bay area of Maryland.

Of the following, the MOST likely diagnosis is

A. coxsackievirus infection

B. Lyme disease

C. meningococcemia

D. Rocky Mountain spotted fever

E. West Nile virus infection

Copyright © 2010 by the American Academy of Pediatrics page 466

Page 467: AAP PREP 2010

2010 PREP SA on CD-ROM

Preferred Response: DCritique: 188

Rocky Mountain spotted fever (RMSF) is a tickborne rickettsial disease caused by Rickettsiarickettsii and characterized by a prodrome of fever, malaise, severe headache, abdominal pain,and myalgias. Approximately 3 to 5 days after the appearance of such symptoms, the classicrash emerges. The rash begins as a macular or maculopapular rash on the wrists and anklesand spreads centrally as well as to the palms and soles, as described for the boy in thevignette. As the rash spreads, it becomes petechial and purpuric (Item C188A). The history ofcamping in an endemic area reported for the boy in the vignette heightens the suspicion forRMSF.

The rash of coxsackievirus infection may be petechial at times, but a peripheral onset withcentral spread is not characteristic of this illness. Although Lyme disease also is a tickborneillness, the geographic distribution and tick vector of Lyme disease are different. Also, the rashof Lyme disease, erythema migrans, is not consistent with the lesions described for this boy,and the extent of systemic illness is not consistent with Lyme disease. Meningococcemia maypresent with a rapidly progressive petechial and purpuric rash (Item C188B) and severesystemic illness, but meningococcemia with purpura fulminans tends to evolve more rapidly anddoes not spread centrally. West Nile virus (WNV) infection is not associated with thesymptomatology described for this boy. Severe WNV disease with encephalitis is rare inchildren. A more nonspecific febrile illness may occur with WNV but not with this constellation ofrash or symptoms.

In the United States, RMSF is most prevalent in the southeastern and south central states,with peaks occurring in spring to fall, when the tick vector is active. In these areas, the dog tick(Dermacentor variabilis) is the primary vector (Item C188C). In the mountain states west of theMississippi, the Rocky Mountain wood tick (Dermacentor andersoni) is the primary vector (ItemC188D).

Clinical features consistent with RMSF, especially in an individual who has potential tickexposure in the geographic areas noted previously, merit empiric therapy. RMSF infection isdiagnosed in the laboratory most often by serologic assays that detect immunoglobulin G and Mantibodies to the organism using an indirect fluorescent antibody or enzyme-linkedimmunosorbent assay. However, antibodies may not be detectable until 7 to 10 days after theonset of illness, and acute and convalescent specimens obtained 2 to 3 weeks apartdemonstrating a fourfold or greater rise in antibody are preferred for making the diagnosis.Polymerase chain reaction (PCR) amplification tests for specific R rickettsii DNA are availablethrough the Centers for Disease Control and Prevention. The highest yield from PCR is on atissue sample (eg, skin biopsy or autopsy specimen); blood samples may not contain sufficientorganisms for detection by PCR. Older, nonspecific serologic tests (Weil-Felix reaction), basedon detecting rickettsial antigens that cross-react with certain Proteus antigens, are lesssensitive and specific and no longer are recommended.

References:

American Academy of Pediatrics. Rocky Mountain Spotted fever. In: Pickering LK, Baker CJ,Kimberlin DW, Long SS, eds. Red Book: 2009 Report of the Committee on Infectious Diseases.

Copyright © 2010 by the American Academy of Pediatrics page 467

Page 468: AAP PREP 2010

2010 PREP SA on CD-ROM

28th ed. Elk Grove Village, Ill: American Academy of Pediatrics; 2009:573-575

American Academy of Pediatrics. West Nile virus. In: Pickering LK, Baker CJ, Kimberlin DW, LongSS, eds. Red Book: 2009 Report of the Committee on Infectious Diseases. 28th ed. Elk GroveVillage, Ill: American Academy of Pediatrics; 2009:730-733

Chapman AS, Bakken JS, Folk SM, et al; Tickborn Rickettsial Diseases Working Group; CDC.Diagnosis and management of tickborne rickettsial diseases: Rocky Mountain spotted fever,ehrlichioses, and anaplasmosis—-United States: a practical guide for physicians and otherhealth-care and public health professionals. MMWR Recomm Rep. 2006;55(RR-4):1-27.Available at: http://www.cdc.gov/mmwr/preview/mmwrhtml/rr5504a1.htm

Copyright © 2010 by the American Academy of Pediatrics page 468

Page 469: AAP PREP 2010

2010 PREP SA on CD-ROM

Question: 189

You are evaluating a 4-year-old girl for a 2-day history of progressively worsening mouth soresand fever. Her mother states that several days ago the patient began complaining that her mouthand throat hurt. At that time, the girl’s gums were slightly swollen and red. Over the last 12hours, she has developed a temperature to 39.5°C and multiple small, fluid-filled lesions on hergums (Item Q189), tongue, and sides of her mouth, and she has refused to eat or drink anything.On physical examination, the tired-appearing girl has a temperature of 39.8°C and is drooling. Herlips are mildly swollen with crusted blood, and she has multiple vesicular lesions on her gums,tongue, buccal mucosa, palate, and posterior pharynx. Her gums are swollen and tender, withintermittent areas of bleeding. There are scattered vesicular lesions at the corners of her mouthand on her chin. The rest of her physical examination findings are within normal limits.

Of the following, the MOST rapid method to confirm the etiologic agent of this patient’s conditionis

A. direct fluorescent antibody

B. Giemsa stain

C. polymerase chain reaction

D. serology

E. viral culture

Copyright © 2010 by the American Academy of Pediatrics page 469

Page 470: AAP PREP 2010

2010 PREP SA on CD-ROM

Preferred Response: ACritique: 189

The patient described in the vignette has primary herpesvirus gingivostomatitis. A number oftests may be used to diagnose herpes simplex virus (HSV) disease. Cultures from skin orconjunctiva provide the greatest diagnostic yield, with more than 90% of cultures being positive.HSV infection may be confirmed by isolation of the virus in tissue culture, demonstration of HSVantigens, or demonstration of HSV DNA in scrapings from lesions. HSV grows easily in tissueculture, and cytopathic effects typical of HSV usually are seen 1 to 3 days after inoculation.

Direct fluorescent antibody staining of vesicle scrapings is a rapid method of diagnosingHSV infection that is as specific as viral culture but slightly less sensitive. Results are availablein a matter of hours and frequently are used to type HSV culture isolates. Polymerase chainreaction testing of cerebrospinal fluid (CSF) is a sensitive method for detecting HSV DNA and isespecially useful for evaluating CSF specimens because CSF culture for HSV usually is negativein infants who have localized central nervous system infection and in older patients who haveHSV encephalitis. Results are usually available in 1 to 2 days. HSV serology has little value in thediagnosis of neonatal HSV infection. Type-specific and nonspecific antibodies to HSV developduring the first several weeks after infection and persist indefinitely. In addition, results ofserologic testing typically are not available for days to weeks. Histologic examination of ascraping from lesions using Giemsa staining, looking for the presence of multinucleated giantcells and eosinophilic intranuclear inclusions typical of HSV, has low sensitivity and is notrecommended as a diagnostic test.

References:

American Academy of Pediatrics. Herpes simplex. In: Pickering LK, Baker CJ, Kimberlin DW, LongSS, eds. Red Book: 2009 Report of the Committee on Infectious Diseases. 28th ed. Elk GroveVillage, Ill: American Academy of Pediatrics; 2009:363-373

Corey L. Herpes simplex virus. In: Mandell GL, Bennett JE, Dolan R, eds. Mandell, Douglas, andBennett’s Principles and Practice of Infectious Diseases. 6th ed. Philadelphia, Pa: ElsevierChurchill Livingstone; 2005:1762-1780

Waggoner-Fountain LA, Grossman LB. Herpes simplex virus. Pediatr Rev. 2004;25:86-93.Available at: http://pedsinreview.aappublications.org/cgi/content/full/25/3/86

Copyright © 2010 by the American Academy of Pediatrics page 470

Page 471: AAP PREP 2010

2010 PREP SA on CD-ROM

Question: 190

A 15-year-old girl presents with left-sided flank pain. She denies trauma, fever, frequency, andurgency. Her past medical history is negative for urinary tract infections. Her family history ispositive for kidney failure in her father from polycystic kidney disease. On physical examination,the girl’s temperature is 37°C, respiratory rate is 16 breaths/min, pulse is 78 beats/min, and bloodpressure is 130/86 mm Hg. Her physical examination findings are normal.

Of the following, the BEST study to assess the cause of pain in this patient is

A. computed tomography scan of the abdomen and pelvis

B. diuretic renal scan (mercaptoacetyltriglycine [MAG3] furosemide renal scan)

C. magnetic resonance imaging of the abdomen

D. radiography of the abdomen and posterior rib cage

E. ultrasonography of the abdomen

Copyright © 2010 by the American Academy of Pediatrics page 471

Page 472: AAP PREP 2010

2010 PREP SA on CD-ROM

Preferred Response: ECritique: 190

The adolescent described in the vignette has flank pain and a clinical history inconsistentwith pyelonephritis. Such a presentation might be suspicious for a renal stone, but her borderlinehypertension and positive family history of renal failure from polycystic kidney disease (PKD)raise the possibility of autosomal-dominant PKD (ADPKD), a genetic renal disease that also couldpresent with flank pain.

ADPKD is caused by mutations to PKD1 (chromosome 16, encodes for polycystin 1 andcomprises 85% of cases) or PKD2 (chromosome 4, encodes for polycystin 2 and comprises15% of cases). Individuals afflicted with ADPKD develop renal and extrarenal cysts, which canbe seen best with renal ultrasonography (Item C190). The diagnostic criteria for ADPKD inpatients younger than age 30 is more than one cyst in either kidney or the presence of one cystin both kidneys. Only a few renal cysts may be present up to 30 years of age in affectedindividuals, but by age 50, hundreds to thousands of cysts may be present. Other imagingmodalities to consider when evaluating a patient "at-risk" for ADPKD are computed tomography(CT) scan or magnetic resonance imaging (MRI). The CT scan has two major drawbacks:radiation exposure and exposure to radiocontrast with its potential nephrotoxic effects.Although MRI is not associated with the same risks as CT scan, this imaging modality is morecostly and less well studied in ADPKD. Unless the child has renal insufficiency, using theintravenous MRI contrast agent gadolinium should be safe and allow excellent resolution. AMAG3 furosemide renal scan is a nuclear medicine test used to diagnose obstruction, usually inthe setting of hydronephrosis. This test has no role in the diagnosis of ADPKD. Plain films may behelpful in looking for a rib fracture, but this is unlikely in the absence of trauma.

References:

Nicolau C, Torra R, Badenas C, et al. Autosomal dominant polycystic kidney disease types 1 and2: assessment of US sensitivity for diagnosis. Radiology. 1999;213:273-276. Available at:http://radiology.rsnajnls.org/cgi/content/full/213/1/273

Pei Y. Diagnostic approach in autosomal dominant polycystic kidney disease. Clin J Am SocNephrol. 2006;1:1108-1114. Available at: http://cjasn.asnjournals.org/cgi/content/full/1/5/1108

Copyright © 2010 by the American Academy of Pediatrics page 472

Page 473: AAP PREP 2010

2010 PREP SA on CD-ROM

Question: 191

You are conducting teaching rounds with a group of medical students who are rotating in thepediatric intensive care unit. Their last admission was a 10-year-old boy who experiencedsevere anaphylaxis to a flying insect sting. The boy thought it was a wasp that stung him whenhe was playing in his backyard. Minutes after being stung, he developed diffuse pruritus andflushing. Subsequently, he experienced throat tightness, difficulty breathing, and loss ofconsciousness. Emergency medical services arrived within 5 minutes. They administered threedoses of epinephrine and a fluid bolus and endotracheally intubated him. You suspect animmunoglobulin E-mediated reaction and discuss the different aspects of allergy testing.

Of the following, a TRUE statement regarding allergy testing for Hymenoptera is that

A. Hymenoptera skin or serum testing results rarely are positive in patients who experience onlylocal, "normal" reactions

B. Hymenoptera skin testing is not recommended for children younger than 12 years of age

C. if skin testing results are negative, then serum immunoglobulin E testing does not need to beperformed

D. this patient should be tested to all flying Hymenoptera (ie, wasps, hornets, bees, yellowjackets)

E. this patient should undergo skin testing prior to hospital discharge

Copyright © 2010 by the American Academy of Pediatrics page 473

Page 474: AAP PREP 2010

2010 PREP SA on CD-ROM

Preferred Response: DCritique: 191

The boy described in the vignette experienced a near-fatal reaction to a flying insect, withsigns and symptoms consistent with anaphylaxis. In the United States, stinging insects causingimmunoglobulin (Ig) E-mediated reactions include honeybees, yellow jackets, yellow hornets,white-faced hornets, paper wasps, and fire ants. The identification of an IgE-mediated allergicdisorder involves identifying the allergen, estabishing a causal relationship between the allergenand the reaction, and demonstrating the presence of specific IgE via immediate type skin testingor in vitro testing.

Indications for immediate type skin testing include:

1.Identification of aeroallergen triggers in patients who have asthma2.Allergic rhinitis that is not controlled with usual medications or if specific avoidance (eg,

pet dander) is desired

3.Food allergy

4.Insect sting allergy

5.Vaccine, drug, or latex allergy

6.Evaluation for moderate-to-severe atopic dermatitis7.Other conditions, including allergic fungal sinusitis, allergic bronchopulmonary

aspergillosis, and eosinophilic esophagitis The decision to perform skin or in vitro testing depends on the age of the patient, the desire

to start allergen immunotherapy, time since the reaction, and severity of the reaction. Allergy skin testing can be performed at any age, but infants and toddlers generally do not

tolerate intradermal skin testing well and may not be able to communicate to their parent/clinicianif they are developing a reaction during testing. In addition, clinicians may elect to start with invitro testing in patients who have experienced life-threating reactions, as described in thevignette. Unfortunately, both skin testing and in vitro testing, when used alone for assessment ofinsect sting allergy, may provide false-negative results in up to 25% of patients. In cases ofanaphylaxis, if the results of the initial test (skin or in vitro) are negative, the second testingmodality should be undertaken. If testing is performed too soon (ie, within 2 weeks of thereaction), results may be negative due to depletion of mast cell mediators. Generally, skin testingshould be performed 4 to 6 weeks after a sting reaction.

Current guidelines recommend that patients who experience less severe reactions (eg, localor normal reactions, large local reactions, or cutaneous-only symptoms in children younger thanthan 16 years of age) do not require testing because of their low risk for future anaphylaxis.Epidemiologic studies have demonstrated that 25% to 50% of individuals can develop specificIgE to stinging insects, even after a normal or local reaction. However, such individuals (localreaction only) should not be tested, based on their normal reactions, and no specific avoidancemeasures or immunotherapy are recommended.

Finally, when flying insect testing is performed, a complete set of the five Hymenopteravenoms (honey bee, yellow jacket, white-faced hornet, yellow hornet, wasp) should be usedunless the specific insect is captured and identified by an entomologist.

Copyright © 2010 by the American Academy of Pediatrics page 474

Page 475: AAP PREP 2010

2010 PREP SA on CD-ROM

References:

Cartwright RC, Dolen WK. Consultation with the specialist: who needs allergy testing and how toget it done. Pediatr Rev. 2006;27:140-146. Available at:http://pedsinreview.aappublications.org/cgi/content/full/27/4/140

Golden DBK. Insect allergy. In: Adkinson NF Jr, Yunginger JW, Busse WW, Bochner BS, HolgateST, Simons FER, eds. Middleton’s Allergy Principles and Practice. 6th ed. Philadelphia, Pa:Mosby Elsevier; 2003:1475-1486

Sicherer SH, Leung DYM. Insect allergy. In: Kliegman RM, Behrman RE, Jenson HB, Stanton BF,eds. Nelson Textbook of Pediatrics. 18th ed. Philadelphia, Pa: Saunders Elsevier; 2007:975-977

Waibel KH. Anaphylaxis. Pediatr Rev. 2008;29:255-263. Available at:http://pedsinreview.aappublications.org/cgi/content/full/29/8/255

Copyright © 2010 by the American Academy of Pediatrics page 475

Page 476: AAP PREP 2010

2010 PREP SA on CD-ROM

Question: 192

A 10-year-old boy is brought to the emergency department after he struck a boulder while ridinghis all-terrain vehicle, causing the vehicle to roll over on him. He was not wearing a helmet.Bystanders reported that he had a brief loss of consciousness, but he is now alert andanswers questions appropriately. He reports that he is nauseous, has a severe headache, andis unable to hear with his left ear. On physical examination, his heart rate is 120 beats/min,respiratory rate is 20 breaths/min, and blood pressure is 130/80 mm Hg. He has multiple facialabrasions, a 6-cm laceration on his forehead, and bloody drainage from his left ear. His midfaceis stable to palpation, his extraocular movements are normal, and there is no deformity of hisnasal bone.

Of the following, his clinical presentation is MOST consistent with a(n)

A. diffuse axonal injury

B. occipital skull fracture

C. orbital floor fracture

D. subdural hematoma

E. temporal bone fracture

Copyright © 2010 by the American Academy of Pediatrics page 476

Page 477: AAP PREP 2010

2010 PREP SA on CD-ROM

Preferred Response: ECritique: 192

Nausea, headache, bleeding from the external auditory canal, and hearing loss in the settingof blunt head trauma, as described for the boy in the vignette, are consistent with a temporalbone fracture. Other findings may include facial paralysis, cerebrospinal oto- or rhinorrhea, andvertigo. Because temporal bone fractures result from significant force, many patients havemultiple other intracranial and orthopedic injuries.

Although temporal bone fractures can be associated with other skull fractures, an occipitalfracture usually is characterized by occipital scalp swelling, and an orbital floor fracture ischaracterized by maxillary tenderness, periorbital swelling, and abnormal extraocularmovements. A subdural hematoma or diffuse axonal injury causes altered mental status.

References:

Haddad J. Traumatic injuries of the ear and temporal bone. In: Kliegman RM, Behrman RE, JensonHB, Stanton BF, eds. Nelson Textbook of Pediatrics. 18th ed. Philadelphia, Pa: SaundersElsevier; 2007:2648-2649

Johnson F, Semaan MT, Megerian CA. Temporal bone fracture: evaluation and management inthe modern era. Otolaryngol Clin North Am. 2008;41:597-618. Abstract available at:http://www.ncbi.nlm.nih.gov/pubmed/18436001

Copyright © 2010 by the American Academy of Pediatrics page 477

Page 478: AAP PREP 2010

2010 PREP SA on CD-ROM

Question: 193

You are completing a routine pre-camp examination of a 12-year-old boy. His mother reports thathe recently has been experiencing some difficulties in school, and the boy admits to problemsconcentrating and to frequent daydreaming in class. On physical examination, his height is 160cm and weight is 70 kg. The only other finding of note is a firm liver edge palpated 2.5 cm belowthe right costal margin. Diagnostic studies demonstrate aspartate aminotransferase of 65 units/Land alanine aminotransferase of 80 units/L. Findings on computed tomography scan of theabdomen are consistent with fatty infiltration of the liver (Item Q193).

Of the following, the single MOST important additional laboratory test to obtain is serum

A. alpha-fetoprotein

B. antismooth muscle antibody

C. ceruloplasmin

D. hepatitis B serology

E. lipids

Copyright © 2010 by the American Academy of Pediatrics page 478

Page 479: AAP PREP 2010

2010 PREP SA on CD-ROM

Preferred Response: CCritique: 193

The physical finding of hepatomegaly may be associated with myriad clinical conditions,reflecting either intrinsic liver disease or a more generalized disorder. The obesity of the patient

described in the vignette (calculated body mass index = weight (kg)/height2(m) = 27.3), coupledwith the findings of a liver edge palpable 2.5 cm below the right costal margin and mild elevationsin liver enzymes, suggest the presence of nonalcoholic fatty liver disease (NAFLD). However, adefinitive diagnosis of NAFLD may be established only after careful consideration of otherpossible causes of hepatomegaly.

The most important diagnosis to rule out immediately for this boy is Wilson disease becausethe history of recent problems at school suggests the potential for new-onset Wilson-associatedneurobehavioral dysfunction. This disorder may be discerned by measuring serumceruloplasmin. A low value (<20 mg/dL [200 mg/L] in most, but not all cases) indicates the needfor immediate further evaluation, including an ophthalmologic exam, 24-hour urine copperexcretion measurement, and a liver biopsy for histology and assessment of hepatic coppercontent.

A palpable liver edge below the right costal margin does not necessarily indicateenlargement. Normative data for liver size are based on assessment by percussion, and imagingstudies generally are not performed in the routine assessment of hepatomegaly. Liver sizeincreases linearly with both body weight and height. In a 12-year-old child, the mean normal liverspan is 7 to 8 cm in boys and 6 to 7 cm in girls. However, whenever a palpable liver edge isfound on physical examination, a distance below the right costal margin of more than 2 cm inchildren and more than 3.5 cm in newborns should be considered abnormal. The finding ofhepatomegaly, by itself, suggests neither the diagnosis nor the prognosis of liver disease.Rather, depending on patient age, this observation, in conjunction with a careful history and thepresence of other physical findings, should determine the course of diagnostic evaluation.

A careful history and physical examination are required to direct the initial evaluation towardthe likely pathophysiologic mechanism for hepatic dysfunction: inflammatory, excess storage(fat, glycogen, metals, abnormal proteins), infiltrative, congestive, or obstructive. In this case, thepatient’s obesity predisposes him to manifest the hepatic steatosis associated with NAFLD, andthe liver enzyme elevations also indicate secondary hepatic inflammation (a commonconsequence of storage diseases). Questioning about travel history, sick contacts, drug orenvironmental toxin exposure, and exposure to blood products can indicate risk factors foracute hepatitis. Systemic symptoms in the older child may herald the presence of chronicinflammatory or autoimmune conditions; a history of jaundice, dark urine, or acholic stoolsindicates cholestasis due to inflammatory or obstructive causes. In the older child, findings ofnew-onset behavioral or neurologic dysfunction, especially in the setting of chronic liverdisease, may indicate hepatic decompensation. In the case of Wilson disease, such findings arethe consequence of copper accumulation in the central nervous system, which may beconfirmed by identifying Kayser-Fleisher rings on ophthalmologic evaluation.

The most important task for the clinician who is evaluating hepatomegaly in the pediatricpatient is to identify serious liver disease at the time of initial presentation. Hepatomegaly, as amarker of chronic liver disease, is associated with many findings that suggest an inflammatory,obstructive, or infiltrative process (Item C193A). It may represent a transient observation

Copyright © 2010 by the American Academy of Pediatrics page 479

Page 480: AAP PREP 2010

2010 PREP SA on CD-ROM

accompanying a systemic viral illness, and the finding of hepatic tenderness indicates acuteinflammation. Persistent liver enlargement warrants further evaluation. When the liver edge isfirm or hard, a storage or infiltrative disorder must be considered, and this finding often isassociated with splenomegaly. A hard, thin, or nodular liver edge suggests cirrhosis, andconsequent portal hypertension is accompanied by splenomegaly.

Laboratory studies performed in the assessment of hepatomegaly should be directed initiallyat determining the extent of hepatic inflammation or dysfunction (Item C193B). Albumin andprothrombin time are used to monitor hepatic synthetic capacity; aminotransferases can assessinflammation; and fractionated bilirubin, alkaline phosphatase, and gamma-glutamyltranspeptidase are used to evaluate cholestasis. Once abnormal test results demonstrate liverdisease, secondary studies are directed at establishing a specific diagnosis.

In the absence of a history suggesting systemic involvement, a high-risk infectious or toxicexposure, or abnormal physical findings (eg, jaundice, splenomegaly), a logical approach is toperform the initial screening laboratory tests, which in this case demonstrated only mildaminotransferase elevations, prior to proceeding with a more extensive evaluation. Because theboy in the vignette demonstrates no hepatic decompensation, he can be monitored for 2 to 3months. At the end of this monitoring period, if liver enzyme values remain abnormal, furthertesting should be performed. Such secondary evaluation includes alpha-fetoprotein, antismoothmuscle antibody, hepatitis B serology, and lipids.

References:

Alfire ME, Treem WR. Nonalcoholic fatty liver disease. Pediatr Ann. 2006;35:290-299. Abstractavailable at: http://www.ncbi.nlm.nih.gov/pubmed/16637558

D’Agata ID, Balistreri WF. Evaluation of liver disease in the pediatric patient. Pediatr Rev.1999;20:376-390. Available at: http://pedsinreview.aappublications.org/cgi/content/full/20/11/376

Nazer H, Ede RJ, Mowat AP, Williams R. Wilson's disease: clinical presentation and use ofprognostic index. Gut. 1986;27:1377-1381. Abstract available at:http://www.ncbi.nlm.nih.gov/pubmed/3792921

Squires R, Teitelbaum JE. Approach to the patient with hepatobiliary symptoms or signs. In:Rudolph CD, Rudolph AM, Hostetter MK, Lister G, Siegel NJ, eds. Rudolph’s Pediatrics. 21st ed.New York, NY: McGraw-Hill Medical Publishing Division; 2003:1477

Wolf AD, Lavine JE. Hepatomegaly in neonates and children. Pediatr Rev. 2000;21:303-310.Available at: http://pedsinreview.aappublications.org/cgi/content/full/21/9/303

Copyright © 2010 by the American Academy of Pediatrics page 480

Page 481: AAP PREP 2010

2010 PREP SA on CD-ROM

Question: 194

You are discharging an African American infant at 48 hours of postnatal age. His mother’s bloodtype is A-positive and so is his own. He is breastfeeding and has jaundice in the face and chest.A serum total bilirubin obtained 8 hours ago was 12.5 mg/dL with a conjugated fraction of 0.5mg/dL.

Of the following, the BEST recommendation to give his mother about managing his jaundice is to

A. breastfeed the infant every 4 to 6 hours

B. give the baby glucose water if he does not nurse well

C. obtain a follow-up visit with the primary care pediatrician in 24 hours

D. start phenobarbital to reduce the risk of conjugated hyperbilirubinemia

E. switch to formula feeding

Copyright © 2010 by the American Academy of Pediatrics page 481

Page 482: AAP PREP 2010

2010 PREP SA on CD-ROM

Preferred Response: CCritique: 194

The term newborn described in the vignette exhibits mild jaundice, but he has no signs ofillness or feeding intolerance and does not have major blood-group incompatibility that mightincite hemolysis. Nevertheless, his bilirubin value falls within the "high intermediate risk zone" ofa commonly used hour-specific bilirubin nomogram first reported in 1999 by Bhutani (ItemC194A). The newborn is breastfeeding and does not have evidence of conjugatedhyperbilirubinemia that might suggest cholestasis or hepatic dysfunction. Accordingly, he shouldhave a follow-up visit with his primary care pediatrician in 24 hours.

Hyperbilirubinemia remains a prevalent and potentially concerning condition in newbornsaround the world. The greatest concern is for those who are ill, have elevated free bilirubinconcentrations, and are at risk for bilirubin-induced neurologic disability or kernicterus. Thecontributing risk factors for such concern are elaborated in Item C194B.

When assessing such risks, it is important to note the maternal history, prior success orfailure in breastfeeding other children, history of jaundice in primary family members or aprevious newborn, and how well breastfeeding currently is going. Among East Asian races,jaundice is known to be more frequent, and bilirubin concentrations reach higher peak values.The potential contribution of glucose-6-phosphate dehydrogenase deficiency to jaundice inprimary family members also may be important.

Frequent breastfeeding (every 2 to 3 hours) generally improves breastfeeding results byincreasing let-down, stimulating production, and increasing the baby’s efficiency andeffectiveness at feeding. These combine to improve the newborn’s gastrointestinal motility,stimulating bile flow and reducing enterohepatic recirculation of bilirubin, thereby reducing serumbilirubin concentrations. Feeding the newborn glucose water has no effect in reducing serumbilirubin. An infant who fails to breastfeed successfully should be provided cow milk-basedinfant formula or a protein hydrolysate formula, but neither of these is indicated for this infant atthis time. Phenobarbital is not indicated in the healthy newborn who has jaundice.

References:

American Academy of Pediatrics Subcommittee on Hyperbilirubinemia. Management ofhyperbilirubinemia in the newborn infant 35 or more weeks of gestation. Pediatrics.2004;114:297-316. Available at: http://pediatrics.aappublications.org/cgi/content/full/114/1/297

Bhutani VK, Johnson L, Sivieri EM. Predictive ability of a predischarge hour-specific serumbilirubin for subsequent significant hyperbilirubinemia in healthy term and near-term newborns.Pediatrics. 1999;103:6—14. Available at:http://pediatrics.aappublications.org/cgi/content/full/103/1/6

Bhutani VK, Johnson L. Kernicterus in late preterm infants cared for as term healthy infants.Semin Perinatol. 2006;30:89—97. Abstract available at:http://www.ncbi.nlm.nih.gov/pubmed/16731283

Keren R, Bhutani VK. Predischarge risk assessment for severe neonatal hyperbilirubinemia.

Copyright © 2010 by the American Academy of Pediatrics page 482

Page 483: AAP PREP 2010

2010 PREP SA on CD-ROM

NeoReviews. 2007;8:e68-e76. Available for subscription at:http://neoreviews.aappublications.org/cgi/content/full/8/2/e68

Keren R, Luan X, Friedman S, Saddlemire S, Cnaan A, Bhutani VK. A comparison of alternativerisk-assessment strategies for predicting significant neonatal hyperbilirubinemia in term and near-term infants. Pediatrics. 2008;121:e170-e179. Available at:http://pediatrics.aappublications.org/cgi/content/full/121/1/e170

Copyright © 2010 by the American Academy of Pediatrics page 483

Page 484: AAP PREP 2010

2010 PREP SA on CD-ROM

Question: 195

A 14-year-old boy comes to your office complaining of a "rash" on his feet for the past 2 weeks.His medical history is negative except for mild atopic dermatitis, for which he uses a topicalsteroid as needed. He complains of itching, especially between his toes. Examination of his skinreveals dry skin on his arms and legs but no discrete lesions or erythema. The interdigital areasof his feet are macerated, erythematous, and scaling (Item Q195), with peeling of thesurrounding skin. The dorsal aspects of his feet are dry but free of lesions.

Of the following, the MOST likely condition causing these findings is

A. atopic dermatitis

B. psoriasis

C. scabies

D. seborrheic dermatitis

E. tinea pedis

Copyright © 2010 by the American Academy of Pediatrics page 484

Page 485: AAP PREP 2010

2010 PREP SA on CD-ROM

Preferred Response: ECritique: 195

The boy described in the vignette has clinical features suggestive of tinea pedis. This fungalinfection of the feet typically is caused by the dermatophytes Trichophyton, Epidermophyton,and Microsporum. Vesicopustules, maceration, and scaling of the interdigital web spaces (ItemC195A) of the feet are the most characteristic features, and significant pruritus is common.Scaling and peeling also may be present on the plantar and lateral aspects of the feet, but thedorsal aspects generally are spared. Therapy consists of topical antifungal medication and goodfoot hygiene. Resistant cases may be treated with oral griseofulvin. Concomitant involvement ofthe toenails (tinea unguium) is difficult to treat and generally requires oral therapy with one of thenewer antifungals (e.g., terbinafine or itraconazole).

Atopic dermatitis affecting the feet is characterized by scaling, dryness, and lichenificationof the dorsal aspects (Item C195B). Skin findings associated with psoriasis are silver-scaledplaques that usually affect the extensor surfaces of the arms and legs. If the feet are affected,erythema and scaling occur on the plantar or lateral aspects (Item C195C). Scabies might be adiagnostic possibility for the boy in the vignette, but in adolescents, papules are concentrated inskin flexures, such as the interdigital areas of the fingers (Item C195D); involvement limited to thefeet would be rare. Seborrheic dermatitis does not involve the feet and in the adolescent usuallyis characterized by scaling of the scalp.

References:

American Academy of Pediatrics. Tinea pedis and tinea unguium (athlete’s foot ringworm of thefeet). In: Pickering LK, Baker CJ, Kimberlin DW, Long SS, eds. Red Book: 2009 Report of theCommittee on Infectious Diseases. 28th ed. Elk Grove Village, Ill: American Academy ofPediatrics; 2009:665-666

Morelli JG. Eczematous disorders. In: Kliegman RM, Behrman RE, Jenson HB, Stanton BF, eds.Nelson Textbook of Pediatrics. 18th ed. Philadelphia, Pa: Saunders Elsevier; 2007:2693-2696

Shy R. Tinea corporis and tinea capitis. Pediatr Rev. 2007;28:164-173. Available at:http://pedsinreview.aappublications.org/cgi/content/full/28/5/164

Copyright © 2010 by the American Academy of Pediatrics page 485

Page 486: AAP PREP 2010

2010 PREP SA on CD-ROM

Question: 196

A 9-year-old boy presents to your office with purple spots (Item Q196) on his legs and mildswelling of his scrotum of 1 day’s duration. He has had no vomiting, diarrhea, or constipation. Heis afebrile, alert, and active. On palpation, he reports mild abdominal discomfort. He has noedema of the lower extremities or presacral area. His weight is 1 kg more than his weight at hishealth supervision visit 6 months ago.

Of the following, the MOST likely abnormal laboratory finding to expect for this boy is

A. anemia

B. hypoalbuminemia

C. microscopic hematuria

D. prolonged partial thromboplastin time

E. thrombocytopenia

Copyright © 2010 by the American Academy of Pediatrics page 486

Page 487: AAP PREP 2010

2010 PREP SA on CD-ROM

Preferred Response: CCritique: 196

The purpuric lesions on the lower extremities described for the previously healthy school-age boy in the vignette are suggestive of Henoch-Schönlein purpura (HSP), a systemic vasculitisof unknown cause that is diagnosed clinically. Purpuric lesions are palpable, nonblanchinghemorrhages in the skin that are 5 to 10 mm in diameter (Item C196).

Although multiple systems may be involved, the long-term morbidity of HSP is related directlyto the severity of renal involvement. Because renal manifestations may follow the developmentof the rash by up to 3 months, urinalysis should be performed monthly, looking for microscopichematuria, the most common manifestation of renal disease. Blood urea nitrogen and creatinineshould be measured if hematuria or proteinuria is present. Proteinuria may occur in patients whohave renal involvement, but it is not a frequent finding at initial presentation.

In children who have HSP, serum albumin and partial thromboplastin time usually are normalat presentation. Anemia is not a typical finding unless chronic renal disease has developed. Theplatelet count is normal and helps differentiate HSP from thrombocytopenic purpura. For patientswho have abdominal pain, a stool occult blood test may be positive if the bowel wall is involvedin the vasculitic process.

References:

Davin JC, Weening JJ. Henoch-Schönlein purpura nephritis: an update. Eur J Pediatr.2001;160:689-695. Abstract available at: http://www.ncbi.nlm.nih.gov/pubmed/11795675

Kraft DM, McKee D, Scott C. Henoch-Schönlein purpura: a review. Am Fam Physician.1998;58:405-408, 411. Available at: http://www.aafp.org/afp/980800ap/kraft.html

Lanzkowsky S, Lanzkowsky L, Lanzkowsky P. Henoch-Schönlein purpura. Pediatr Rev.1992;13:130-137. Abstract available at:http://pedsinreview.aappublications.org/cgi/content/abstract/13/4/130

Li SC. Risk factors for the development of nephritis in Henoch-Schönlein purpura. AAP GrandRounds. 2006;16:14-15. Available at:http://aapgrandrounds.aappublications.org/cgi/content/full/16/2/14

Oner A, Erdogan ø, Eren T, et al. long-term prognosis of Henoch-Schönlein nephritis in children.Pediatrics. 2008;121:S144-S145. Abstract available at:http://pediatrics.aappublications.org/cgi/content/abstract/121/Supplement_2/S144-a

Ronkainen J, Koskimies O, Ala-Houhala M, et al. Early prednisone therapy in Henoch-Schönleinpurpura: a randomized, double-blind, placebo-controlled trial. J Pediatr. 2006;149:241-247.Abstract available at: http://www.ncbi.nlm.nih.gov/pubmed/16887443

Shin JI, Park JM, Shin YH, Hwang DH, Kim JH, Lee JS. Predictive factors for nephritis, relapse,and significant proteinuria in childhood Henoch-Schönlein purpura. Scand J Rheumatol.

Copyright © 2010 by the American Academy of Pediatrics page 487

Page 488: AAP PREP 2010

2010 PREP SA on CD-ROM

2006;35:56—60. Abstract available at: http://www.ncbi.nlm.nih.gov/pubmed/16467044

Weiss PF, Feinstein JA, Luan X, Burnham JM, Feudtner C. Effects of corticosteroid on Henoch-Schönlein purpura: a systematic review. Pediatrics. 2007;120:1079-1087. Available at:http://pediatrics.aappublications.org/cgi/content/full/120/5/1079

Copyright © 2010 by the American Academy of Pediatrics page 488

Page 489: AAP PREP 2010

2010 PREP SA on CD-ROM

Question: 197

An 18-month-old boy is brought to the clinic with fever and irritability. His mother explains that hehas had a fever for the past week and a red rash on his extremities. On physical examination,he has a temperature of 39.2°C; he is irritable; his eyes are injected without discharge (ItemQ197A); and his lips are dry, red, and cracked (Item Q197B). All other findings are within normallimits.

Of the following, the MOST appropriate next step in this patient’s care is to

A. administer intravenous antibiotics

B. administer intravenous gamma globulin

C. obtain blood cultures

D. obtain electrocardiography

E. perform a lumbar puncture and culture the cerebrospinal fluid

Copyright © 2010 by the American Academy of Pediatrics page 489

Page 490: AAP PREP 2010

2010 PREP SA on CD-ROM

Preferred Response: BCritique: 197

The child described in the vignette is in the acute phase of Kawasaki disease (KD), amultisystem illness characterized by vasculitis of small- and medium-sized blood vessels,including the coronary arteries. Although the precise cause remains unclear, recent researchsuggests that at least some of the cases are related to an infectious organism. The median ageof affected patients is 2.3 years, and nearly 80% of cases occur in children younger than 4years of age. More cases are reported in the winter and spring than during the summer and fall,and males are affected approximately 1.5 times more frequently than females. KD can be seen inpatients of all ethnicities, races, and cultures, but it is most prevalent in those of Asian descent.

KD is diagnosed using a set of diagnostic criteria that include the presence of fever for atleast 5 days and at least four of the five following clinical findings: changes in the extremities(edema, erythema, desquamation) (Item C197); polymorphous exanthem that is usually truncal;conjunctival injection; erythema or fissuring of the lips, oral cavity, and tongue; and cervicallymphadenopathy. In addition, the constellation of findings with which the patient presentscannot be explained by any other known disease process. Other features associated with KDbut not part of the diagnostic criteria may include migratory polyarthritis, aseptic meningitis,hydrops of the gallbladder, and sterile pyuria.

The acute phase of KD lasts 1 to 2 weeks after the onset of fever and is when thediagnostic criteria typically are present. The subacute phase follows from 2 to 8 weeks afterdisease onset, and patients may demonstrate desquamation of the fingers and toes. It is duringthis phase when coronary aneurysms may develop, particularly in those who have not beentreated with intravenous gamma globulin during the acute phase. The convalescent phase lastsfor months following the illness.

Patients who have KD may have involvement of the cardiovascular system, includingmyocarditis, valvulitis, or arteritis. The latter is diagnosed by the presence of edematouscoronary walls on echocardiography, usually between days 5 and 8 of the illness. Coronaryartery aneurysms develop after the acute phase, typically in the subacute phase. Amonguntreated patients, the incidence of coronary aneurysms may be as high as 22% to 25%, but forthose who receive intravenous gamma globulin therapy in the acute phase, the incidence ofcoronary involvement is 3% to 5%.

Because the child in the vignette is in the acute phase of KD, he should be treated withintravenous gamma globulin at a dose of 2 g/kg. If his fever persists, a second dose of gammaglobulin may be administered. In addition, high-dose aspirin therapy (80 to 100 mg/kg per day) isadministered until the patient is afebrile for 48 hours, at which time the dose is decreased to 3 to5 mg/kg per day for 6 to 8 weeks or until the platelet values normalize.

Antibiotic administration is not indicated for treatment of KD. There is no clinical indication toobtain blood cultures or cerebrospinal fluid in the patient who has otherwise uncomplicated KD.Although cardiovascular involvement can occur in affected patients, electrocardiography is notpart of the initial management.

References:

Newburger JW, Fulton DR. Kawasaki disease. Curr Treat Options Cardiovasc Med. 2007;9:148-

Copyright © 2010 by the American Academy of Pediatrics page 490

Page 491: AAP PREP 2010

2010 PREP SA on CD-ROM

158. Abstract available at: http://www.ncbi.nlm.nih.gov/pubmed/17484817

Newburger JW, Takahashi M, Gerger MA, et al. Diagnosis, treatment, and long-term managementof Kawasaki disease: a statement for health professionals from the Committee on RheumaticFever, Endocarditis, and Kawasaki Disease, Council on Cardiovascular Disease in the Young,American Heart Association. Pediatrics. 2004;114:1708-1733. Available at:http://pediatrics.aappublications.org/cgi/content/full/114/6/1708

Satou GM, Giamelli J, Gewitz MH. Kawasaki disease: diagnosis, management, and long-termimplications. Cardiol Rev. 2007;15:163-169. Abstract available at:http://www.ncbi.nlm.nih.gov/pubmed/17575479

Copyright © 2010 by the American Academy of Pediatrics page 491

Page 492: AAP PREP 2010

2010 PREP SA on CD-ROM

Question: 198

A 12-year-old girl presents with a 2-day history of progressive back pain and difficulty walking,especially up stairs. This morning she had trouble dressing and combing her hair. On physicalexamination, the girl’s vital signs are stable and she is not short of breath. Cardiac, pulmonary,and joint evaluation results are normal. Neurologic examination reveals normal mental status andcranial nerves. Strength is diffusely reduced in the arms and legs, and reflexes are absent. Hergait is slow, and she requires assistance to arise from the floor. Noncontrast head computedtomography scan results are normal. Forced vital capacity is normal. Lumbar puncture reveals acerebrospinal fluid protein concentration of 110 mg/dL. Cerebrospinal fluid white blood cellcount, red blood cell count, and glucose concentration are normal. You admit the girl forcardiorespiratory monitoring and treatment.

Of the following, the treatment that is MOST likely to be beneficial in hastening this child’srecovery is

A. immune globulin intravenously

B. methotrexate intrathecally

C. methylprednisolone intravenously

D. neostigmine intravenously

E. tissue plasminogen activator intravenously

Copyright © 2010 by the American Academy of Pediatrics page 492

Page 493: AAP PREP 2010

2010 PREP SA on CD-ROM

Preferred Response: ACritique: 198

The evaluation of subacute generalized weakness is a medical emergency becausesymptoms may progress to respiratory insufficiency or dysautonomia and death. The evaluationshould be systematic, considering possible causes at the level of brain, brainstem, spinal cord,anterior horn cell, root, nerve, junction, and muscle. The girl described in the vignette hasgeneralized weakness, with normal mentation and sparing, at present, of muscles of the faceand swallowing. Therefore, the problem is less likely to involve the brain or brainstem. Similarly,the absence of sensory loss makes spinal cord involvement unlikely. The back pain and absentreflexes localize the problem to root and nerves, and the classic cerebrospinal fluid findings(high protein concentration, normal cells) confirm the diagnosis of acute inflammatorydemyelinating polyneuropathy (AIDP), also known as Guillain-Barré syndrome (GBS).

The treatment of choice for AIDP/GBS in adults and children is intravenous immune globulin,administered at 2 g/kg total dose. Respiratory status, with forced vital capacity or negativeinspiratory force, and cardiovascular status should be monitored in the hospital due to the risk ofdeath. Neuropathic pain also should be treated, and both gabapentin and carbamazepine areeffective.

Intravenous methylprednisolone alone is not effective in this disorder. Neostigmine is anacetylcholinesterase inhibitor used for diseases at the neuromuscular junction such asmyasthenia gravis. Intravenous tissue plasminogen activator is used in acute treatment ofthrombotic stroke in adults. Intrathecal methotrexate is used as a chemotherapeutic agent forcentral nervous system malignancies.

References:

Hughes RAC, Raphaël JC, Swan AV, van Doorn PA. Intravenous immunoglobulin for Guillain-Barré syndrome. Cochrane Database Syst Rev. 2006;1: CD002063. Available at:http://www.mrw.interscience.wiley.com/cochrane/clsysrev/articles/CD002063/frame.html

Ryan MM. Guillain-Barré syndrome in childhood. J Paediatr Child Health. 2005;41:237-241.Abstract available at: http://www.ncbi.nlm.nih.gov/pubmed/15953319

Sarnat HB. Guillain-Barré syndrome. In: Kliegman RM, Behrman RE, Jenson HB, Stanton BF, eds.Nelson Textbook of Pediatrics. 18th ed. Philadelphia, Pa: Saunders Elsevier; 2007:2565

Copyright © 2010 by the American Academy of Pediatrics page 493

Page 494: AAP PREP 2010

2010 PREP SA on CD-ROM

Question: 199

A 7-year-old boy who is new to your practice comes in for evaluation of developmental delayand poor school performance. He began speaking in sentences at age 4. He repeatedkindergarten and is struggling in first grade. On physical examination, you note that he has fairhair and light skin compared with his brown-haired, olive-skinned younger brother and mother.He is wearing thick glasses, and his mother says that he was diagnosed as being near-sightedwhen he was 2 years old. He has a lanky build with long fingers, and on forward bending, thereis a curve in the thoracolumbar spine.

Of the following, the condition that is MOST consistent with this presentation is

A. alkaptonuria

B. homocystinuria

C. nonketotic hyperglycinemia

D. oculocutaneous tyrosinemia

E. phenylketonuria

Copyright © 2010 by the American Academy of Pediatrics page 494

Page 495: AAP PREP 2010

2010 PREP SA on CD-ROM

Preferred Response: BCritique: 199

Inborn errors of amino acid metabolism (aminoacidopathies) result from the abnormalbreakdown of amino acids in the cytosol. The symptoms associated with this group of disordersare due to the accumulation of toxic intermediates, such as phenylalanine, that cause organdamage. Disorders of amino acid metabolism are diagnosed with the aid of plasma and urineamino acid quantitation; sometimes, measuring urine organic acids also is helpful. The treatmentof aminoacidopathies involves generally limiting protein intake, specifically limiting intake of theoffending protein, and avoiding catabolic states. These conditions are typically autosomalrecessive.

Individuals who have homocystinuria are deficient in the enzyme cystathionine synthase,which leads to increased methionine in the blood. There are two subtypes of homocystinuria: B6-responsive and B6-nonresponsive. Excess methionine may be associated with a noticeable,unpleasant odor. Affected individuals often have more lightly pigmented eyes, skin, and hair thantheir unaffected family members, as described for the boy in the vignette. Dislocation of thelens(es) of the eye(s) is usually apparent by 10 years of age, and the lenses typically subluxdownward. Other eye anomalies include myopia, optic atrophy, cataracts, and retinaldetachment. Skeletal abnormalities are a prominent feature of homocystinuria and overlap withthose seen in Marfan syndrome; they include tall stature with thin body habitus, pectusexcavatum or carinatum, narrow palatal contour, and scoliosis. Some individuals havearachnodactyly (long fingers), such as the boy in the vignette. Thromboembolism is the mostcommon cause of morbidity and premature death. The average intelligence quotient (IQ) forindividuals who have the B6-responsive form of the disease is 79, whereas the average IQ forthose who have the B6-nonresponsive form is 57. It is important to identify those individualswho are B6-responsive to mitigate poor intellectual outcome. Newborn screening forhomocystinuria is required by law in most states in the United States.

Individuals who have alkaptonuria are deficient in the enzyme homogentisate 1,2-dioxygenase, which is involved in the tyrosine degradation pathway. Affected persons aretypically asymptomatic in childhood. With age, they develop dark gray or black pigmentation ofthe sclerae or ear cartilage. Sweat may be dark, and cerumen may be almost black. At all ages,the urine of an affected individual, when left to stand, turns dark. Arthritis starts to develop inearly adult life and progresses to marked limitation of movement and ankylosis of the spine.Individuals who have alkaptonuria have a high incidence of heart disease, and myocardialinfarction is a common cause of death. Intelligence is typically normal.

Nonketotic hyperglycinemia results from deficient activity of proteins in the glycine cleavagesystem. The classic form of the disease presents in the first few days after birth, coincidingwith intake of protein-containing feedings. Affected children develop anorexia and lethargy,which progresses to coma. Most affected individuals die at this time. Those who survive theacute neonatal crisis subsequently develop spastic cerebral palsy, with no evidence ofpsychomotor development.

Oculocutaneous tyrosinemia (tyrosinemia type II) is caused by hepatic deficiency of theenzyme tyrosine aminotransferase in the cytosol. The most important clinical manifestations ofthis disorder involve the eye due to the accumulation of tyrosine. Corneal erosions, ulcers, andplaques can occur and ultimately lead to corneal clouding and visual impairment. Affected

Copyright © 2010 by the American Academy of Pediatrics page 495

Page 496: AAP PREP 2010

2010 PREP SA on CD-ROM

children may present with tearing, photophobia, and eye redness and pain. Cutaneousmanifestations include painful keratoses occurring most often on pressure-bearing regions,such as the palms and the soles. Treatment of oculocutaneous tyrosinemia includes theinstitution of a diet low in tyrosine and phenylalanine. It is not entirely clear whetheroculocutaneous tyrosinemia is associated with an increased incidence of intellectual disability; apreviously described positive association may be due to ascertainment bias.

Phenylketonuria (PKU) is caused by complete or near-complete deficiency of the enzymephenylalanine hydroxylase, which converts phenylalanine to tyrosine. Sometimes, the onlymanifestation of untreated PKU is intellectual disability. However, vomiting can be an earlysymptom, and irritability, an eczematous rash, and an unusual odor (described as mousy, barny,wolflike, or musty) also may be present. More than 90% of affected individuals are light-eyed,fair-skinned, and light-haired compared with their unaffected family members. Developmentaldelays usually are apparent in untreated individuals within the first 6 postnatal months; if leftuntreated, affected individuals suffer severe-to-profound intellectual disability. Spasticity andseizures also may occur. PKU is treated with a diet that is low in phenylalanine in conjunctionwith tyrosine supplementation. Early treatment can prevent the symptoms and signs of disease.Every state in the United States has required neonatal screening for PKU.

References:

Introne WJ, Kayser MA, Gahl WA. Alkaptonuria. GeneReviews. 2007. Available at:http://www.ncbi.nlm.nih.gov/bookshelf/br.fcgi?book=gene&part=alkap

Kaye CI and Committee on Genetics. Technical report: introduction to the newborn screeningfact sheets. Pediatrics. 2006;118:1304-1312. Available at:http://pediatrics.aappublications.org/cgi/content/full/118/3/1304

Mitchell JJ, Scriver CR. Phenylalanine hydroxylase deficiency. GeneReviews. 2007. Available at:http://www.ncbi.nlm.nih.gov/bookshelf/br.fcgi?book=gene&part=pku

Nyhan WL, Barshop BA, Ozand PT. Alkaptonuria. In: Atlas of Metabolic Diseases. 2nd ed.London, England: Hodder Arnold; 2005:121-135

Nyhan WL, Barshop BA, Ozand PT. Homocystinuria. In: Atlas of Metabolic Diseases. 2nd ed.London, England: Hodder Arnold; 2005:146-152

Nyhan WL, Barshop BA, Ozand PT. Nonketotic hyperglycinemia. In: Atlas of MetabolicDiseases. 2nd ed. London, England: Hodder Arnold; 2005:183-189

Nyhan WL, Barshop BA, Ozand PT. Oculocutaneous tyrosinemia/tyrosine aminotransferasedeficiency. In: Atlas of Metabolic Diseases. 2nd ed. London, England: Hodder Arnold; 2005:168-174

Copyright © 2010 by the American Academy of Pediatrics page 496

Page 497: AAP PREP 2010

2010 PREP SA on CD-ROM

Picker JD, Levy HL. Homocystinuria caused by cystathionine beta-synthase deficiency.GeneReviews. 2006. Available at:http://www.ncbi.nlm.nih.gov/bookshelf/br.fcgi?book=gene&part=homocystinuria

Copyright © 2010 by the American Academy of Pediatrics page 497

Page 498: AAP PREP 2010

2010 PREP SA on CD-ROM

Question: 200

A 13-year-old girl is upset that her periods "have never come on time." Menarche occurred atage 11 years and 9 months. She keeps a menstrual calendar, which indicates that the timebetween her periods ranges from 15 to 45 days. She reports that she uses up to five menstrualpads a day and that her periods can last up to 2 weeks. She occasionally has minor cramping

pain. Her body mass index is 22.8 kg/m2. She has mild comedonal acne, no hirsutism, and noacanthosis nigricans. Her hemoglobin concentration is 11.1 mg/dL (111.0 g/L). Her aunt had ahysterectomy at age 35 years for heavy vaginal bleeding.

Of the following, the MOST appropriate next step is to

A. obtain prothrombin time and partial thromboplastin time

B. obtain thyroid-stimulating hormone and free thyroxine

C. order pelvic ultrasonography

D. provide reassurance with follow-up in 3 to 6 months

E. refer to a gynecologist

Copyright © 2010 by the American Academy of Pediatrics page 498

Page 499: AAP PREP 2010

2010 PREP SA on CD-ROM

Preferred Response: DCritique: 200

Understanding the average values for the duration of a menstrual cycle, the number of daysof bleeding, and the amount of bleeding can aid in determining the need for further evaluation ofindividual patients. A menstrual cycle is the time period from the first day of menstrual bleedingof one cycle to the first day of bleeding of the next cycle. The duration can range from 3 to 6weeks in an adolescent. Cycles that are fewer than 21 days apart are termed polymenorrhea.The duration of bleeding varies from 2 to 7 days. Although blood loss of greater than 80 mL oruse of eight or more pads/tampons per day indicates heavy flow, this is not a very accurateassessment. The potential impact of heavy menses should be evaluated by measuringhemoglobin or hematocrit. Flow lasting 8 or more days is excessive; the term menorrhagia isused for menstrual bleeding that is excessive in duration or quantity. Prolonged or heavy periodsthat occur at irregular intervals is termed menometrorrhagia.

The differential diagnosis of abnormal bleeding during adolescence includes local andsystemic disorders, but dysfunctional uterine bleeding (DUB), pregnancy (either intrauterine orectopic), complications of pregnancy, and local infections (pelvic inflammatory disease) shouldbe highest on the list. Other local causes include trauma, foreign bodies, and rarely, benign ormalignant tumors. Among systemic causes are bleeding disorders, endocrine disorders, andmedications. DUB refers to bleeding that is painless, profuse, and irregular; of endometrial origin;and unrelated to any structural or systemic disease. Most cases (90%) are the result ofphysiologic anovulation from delayed maturation of the hypothalamic pituitary axis; only a smallnumber (10%) result from a dysfunctional corpus luteum or atrophic endometrium.

The history of bleeding for the girl described in the vignette is consistent with DUB. Becausethe bleeding severity is minimal, as gauged by the lack of anemia, she can be observed withfollow-up appointments at 3-month intervals to ascertain menstrual regulation. She also shouldbe screened for contraceptive needs. Further evaluation, including prothrombin and partialthromboplastin times, thyroid-stimulating hormone, free thyroxine, and pelvic ultrasonography,are indicated only if this bleeding pattern continues, the girl develops other symptoms or signs,or she needs contraception. Her current presentation does not warrant referral to agynecologist.

References:

Gray SH, Emans SJ. Abnormal vaginal bleeding in adolescents. Pediatr Rev. 2007;28:175-182.Available at: http://pedsinreview.aappublications.org/cgi/content/full/28/5/175

Sharathkumar AA, Pipe SW. Bleeding disorders. Pediatr Rev. 2008;29:121-130. Available at:http://pedsinreview.aappublications.org/cgi/content/full/29/4/121

Journeycake JM, Buchanan GR. Coagulation disorders. Pediatr Rev. 2003;24:83-91. Available at:http://pedsinreview.aappublications.org/cgi/content/full/24/3/83

Mitan LAP, Slap BG. Dysfunctional uterine bleeding. In: Neinstein, L, ed. Adolescent Health Care:A Practical Guide. 5th edition. Philadelphia Pa: Lippincott Williams & Wilkins; 2008:687-690

Copyright © 2010 by the American Academy of Pediatrics page 499

Page 500: AAP PREP 2010

2010 PREP SA on CD-ROM

Rimsza ME. Dysfunctional uterine bleeding. Pediatr Rev. 2002;23:227-233. Available at:http://pedsinreview.aappublications.org/cgi/content/full/23/7/227

Copyright © 2010 by the American Academy of Pediatrics page 500

Page 501: AAP PREP 2010

2010 PREP SA on CD-ROM

Question: 201

You are evaluating an 18-month-old boy in the emergency department who appears "toxic" andis sitting uncomfortably and leaning forward in his mother’s lap. His temperature is 40.0°C, heartrate is 140 beats/min, respiratory rate is 35 breaths/min, blood pressure is 90/60 mm Hg, andoxygen saturation on room air is 94% by pulse oximetry. He is drooling from the corners of hismouth, and his cry appears muffled. The nurse shows you the lateral neck radiograph that wasjust obtained (Item Q201).

Of the following, the MOST appropriate next step in the treatment of this patient is

A. administration of intramuscular penicillin

B. blood cultures and a complete blood count

C. emergent otolaryngology and anesthesia consultation

D. intramuscular administration of dexamethasone

E. throat culture

Copyright © 2010 by the American Academy of Pediatrics page 501

Page 502: AAP PREP 2010

2010 PREP SA on CD-ROM

Preferred Response: CCritique: 201

Epiglottitis is a medical emergency. The incidence of this disease has significantly decreasedsince the introduction of the Haemophilus influenzae type b vaccine, and the most commoninfectious pathogens now are Streptococcus pneumoniae, group A beta-hemolyticStreptococcus, and Staphylococcus aureus. Often, the infection involves the entire supraglotticarea, not just the epiglottis, and is called supraglottitis. The current incidence in children youngerthan 5 years of age is approximately 1.3 cases per 100,000, a decrease from the 40 cases per100,000 seen 20 years ago. Affected patients, typically between 2 and 8 years of age, usuallypresent with the rapid onset of fever, sore throat, and the "four Ds" (drooling, dysphagia,dysphonia, and dyspnea), as described for the boy in the vignette. Patients may assume aposition of comfort by sitting upright, leaning forward, and bracing themselves with their arms,known as the "tripod position." Clinicians must be vigilant for atypical presentations, especiallyamong children younger than 2 years of age.

Airway management for this disease is paramount. Direct examination of the airway underanesthesia (with the availability of personnel who can perform a tracheostomy if needed) is thepreferred management for suspected cases that involve signs of respiratory compromise (ItemC201A). Therefore, an emergent otolaryngology and anesthesia consultation is the mostappropriate next step for the boy in the vignette. An endotracheal tube that is 0.5 to 1 mm smallerthan usual for age generally is needed. Cultures of the supraglottic area can be obtained at thetime of intubation and broad-spectrum antibiotics effective against beta-lactamase-producingorganisms can be initiated. Lateral radiographs are diagnostic (Item C201B), as in this case, butshould be deferred in children strongly suspected of having epiglottitis until the proper personneland equipment are available to secure the airway if needed.

Invasive procedures, including phlebotomy, obtaining vascular access or a throat culture, orintramuscular administration of medications, should be deferred until the airway is securedproperly. In addition, neither penicillin nor dexamethasone is indicated for the initial treatment ofepiglottitis.

As a result of reviewing this information, do you intend to make a change in practiceto provide better patient care?Yes No

References:

Mehta R, Hariprakash SP, Cox PN, Wheeler DS. Diseases of the upper respiratory tract. In:Wheeler DS, Wong HR, Shanley TP, eds. Pediatric Critical Care Medicine: Basic Science andClinical Evidence. New York, NY: Springer-Verlag London Limited; 2007:485-505

Roosevelt GE. Acute inflammatory upper airway obstruction (croup, epiglottitis, laryngitis, andbacterial tracheitis). In: Kliegman RM, Behrman RE, Jenson HB, Stanton BF, eds. NelsonTextbook of Pediatrics. 18th ed. Philadelphia, Pa: Saunders Elsevier: 2007:1762-1766

Copyright © 2010 by the American Academy of Pediatrics page 502

Page 503: AAP PREP 2010

2010 PREP SA on CD-ROM

Question: 202

During the health supervision visit for a 14-year-old girl, you note that her thyroid gland issymmetric, somewhat firmer than normal, and about twice normal size. Thyroid testing shows afree thyroxine value of 1.3 ng/dL (16.7 pmol/L) (normal, 0.9 to 1.8 ng/dL [11.6 to 23.2 pmol/L])and a thyroid-stimulating hormone value of 2.4 mIU/L (normal, 0.5 to 5.0 mIU/L).

Of the following, the MOST likely cause of this child’s thyroid enlargement is

A. adolescent goiter

B. chronic lymphocytic thyroiditis

C. Graves disease

D. iodine deficiency

E. thyroid carcinoma

Copyright © 2010 by the American Academy of Pediatrics page 503

Page 504: AAP PREP 2010

2010 PREP SA on CD-ROM

Preferred Response: BCritique: 202

The girl described in the vignette has a symmetrically enlarged, firm thyroid gland sometimesreferred to as a goiter. The most common cause of thyroid enlargement in adolescents is chroniclymphocytic thyroiditis, or Hashimoto thyroiditis. This autoimmune disorder can be diagnosed inmost cases by measuring concentrations of antithyroid antibodies such as those directedagainst thyroperoxidase (antimicrosomal or anti-TPO antibodies) or against thyroglobulin(antithyroglobulin antibodies).

Abnormal thyroid function is not required to have chronic lymphocytic thyroiditis, althoughmany people who have this disorder develop hypothyroidism. The thyroid may enlarge duringperiods of rapid growth of adolescence (ie, adolescent goiter) or increased need for thyroidhormone, as during pregnancy, but it does not develop the firm consistency seen with chroniclymphocytic thyroiditis. The girl described in the vignette has normal thyroid hormone and thyroid-stimulating hormone (TSH) values, indicating that she could not have active Graves disease,which is autoimmune hyperthyroidism. Iodine deficiency causes thyroid enlargement andelevated TSH concentrations, but such deficiency is very uncommon in the United States, unlessthe child eats a very restricted iodine-deficient diet. Thyroid cancer is rare in children andadolescents and usually presents as a nodule within the thyroid or with cervicallymphadenopathy rather than symmetric, smooth, firm thyroid enlargement.

References:

Gopalakrishnan S, Chugh PK, Chhillar M, Ambardar VK, Sahoo M, Sankar R. Goitrousautoimmune thyroiditis in a pediatric population: a longitudinal study. Pediatrics. 2008;122:e670-e674. Available at: http://pediatrics.aappublications.org/cgi/content/full/122/3/e670

Hoffman RP. Thyroiditis. eMedicine Specialties, Pediatrics: General Medicine, Endocrinology.2007. Available at: http://www.emedicine.com/ped/topic2248.htm

Hopwood NJ, Kelch RP. Thyroid masses: approach to diagnosis and management in childhoodand adolescence. Pediatr Rev. 1993;14:481-487. Abstract available at:http://pedsinreview.aappublications.org/cgi/content/abstract/14/12/481

Copyright © 2010 by the American Academy of Pediatrics page 504

Page 505: AAP PREP 2010

2010 PREP SA on CD-ROM

Question: 203

An infant born at 34 weeks’ gestation comes in for her 1-month-old evaluation. Her neonatalcourse was uncomplicated. Her parents ask if she will have delayed development due to herprematurity.

Of the following, the MOST appropriate response is that healthy preterm infants

A. have age-appropriate language skills by the time they are 12 months of age

B. have an increased risk of mild motor impairment

C. born at 32 to 36 weeks’ gestation have a fourfold increase in intellectual disabilities

D. should have their developmental age corrected for the degree of prematurity until 4 years ofage

E. show hand preference at an earlier age than term infants

Copyright © 2010 by the American Academy of Pediatrics page 505

Page 506: AAP PREP 2010

2010 PREP SA on CD-ROM

Preferred Response: BCritique: 203

Some factors that lead to greater risk of neurological/behavioral problems among preterminfants (those born at <37 weeks’ gestation) include low birthweight (<2,500 g), intrauterinegrowth restriction, and a history of maternal prenatal drug use. Medical complications that couldaffect the infant’s development include intraventricular hemorrhage (IVH), cardiac lesions,bronchopulmonary dysplasia, feeding problems, and retinopathy of prematurity. A preterm infantwho has either a grade 3 or 4 IVH has a 35% to 90% increased risk of a neurodevelopmentaldisability.

A 34 weeks’ gestation infant who has an uncomplicated neonatal course, such as the childdescribed in the vignette, has an increased risk of mild motor impairment, referred to as adevelopmental coordination disorder. Most experts agree that preterm infants should have theirdevelopmental ages corrected for the degree of prematurity at least for the first 12 months afterbirth; many clinicians continue to use a full correction until 24 months of age. However, agecorrection up to 4 years of age is not necessary. Preterm infants born at 32 to 36 weeks’gestation have a 1.4-fold increased risk of intellectual disabilities over term infants, although therisk increases to 7-fold for children born prior to 32 weeks’ gestation. Healthy preterm infantsmay not develop age-appropriate language skills until they are older than 12 months of age,which is why their developmental level must be corrected for their level of prematurity. Apreterm infant should not routinely have early hand preference; such a finding may be indicativeof cerebral palsy.

References:

Allen M. Preterm development. In: Accardo PJ, ed. Capute & Accardo’s NeurodevelopmentalDisabilities in Infancy and Childhood. Volume II: The Spectrum of NeurodevelopmentalDisabilities. 3rd ed. Baltimore, Md: Paul H. Brookes Publishing Co; 2008:29-46

Vanderbilt D, Wang CJ, Parker S. The do’s in preemie neurodevelopment. Contemp Pediatr. 2007September:84-97

Copyright © 2010 by the American Academy of Pediatrics page 506

Page 507: AAP PREP 2010

2010 PREP SA on CD-ROM

Question: 204

A 15-year-old boy presents to the emergency department with a 5-day history of fever, nausea,and severe abdominal pain. He also reports malaise and diffuse myalgias. He is admitted to thesurgical service for evaluation of possible appendicitis. You are called for consultation becauseon the day after admission, a papular rash appeared, beginning on his wrists and ankles andextending centrally (Item Q204). Five days ago he had returned from a Boy Scout camping trip tothe Ozark Mountains in Arkansas.

Of the following, the MOST appropriate choice for initial antibiotic therapy pending results oflaboratory studies is

A. azithromycin

B. cefotaxime

C. ciprofloxacin

D. doxycycline

E. penicillin

Copyright © 2010 by the American Academy of Pediatrics page 507

Page 508: AAP PREP 2010

2010 PREP SA on CD-ROM

Preferred Response: DCritique: 204

The prodromal symptoms of fever, nausea, malaise, myalgias, and severe abdominal painfollowed by development of a distal rash that spreads centrally in the boy described in thevignette, who has been camping in the Ozark Mountains in Arkansas, are most consistent withthe diagnosis of Rocky Mountain spotted fever (RMSF). Tetracyclines, particularly doxycycline,are the drugs of choice for treating RMSF, even in children (doxycycline 100 mg bid or 2.2 mg/kgper dose bid orally or intravenously). The optimal duration of therapy is not known, but generalguidelines are to treat for a minimum of 5 to 7 days and at least 3 days after the fever resolves.Treatment within the first 5 days of illness is associated with a good outcome. Later treatmentmay be associated with progressive disseminated intravascular coagulation. Althoughtetracyclines generally are not recommended in children younger than 8 years of age, the risk ofdental staining from a short course of doxycycline is very low, and there are no goodalternatives for treatment of RMSF.

Chloramphenicol is considered an alternative agent for the treatment of RMSF, but it is notavailable orally in the United States and has potentially significant hematologic toxicities. Inaddition, it may not be as effective as tetracyclines in the treatment of RMSF.

The Red Book also lists fluoroquinolones, which include ciprofloxacin, as a possiblealternative choice of therapy of RMSF. Although in vitro data suggest sensitivity for theseagents, clinical data in RMSF are lacking. Azithromycin, cephalosporins, and penicillins are notactive against Rickettsia rickettsii.

References:

American Academy of Pediatrics. Rocky Mountain spotted fever. In: Pickering LK, Baker CJ,Kimberlin DW, Long SS, eds. Red Book: 2009 Report of the Committee on Infectious Diseases.28th ed. Elk Grove Village, Ill: American Academy of Pediatrics; 2009:573-575

Chapman AS, Bakken JS, Folk SM, et al; Tickborn Rickettsial Diseases Working Group; CDC.Diagnosis and management of tickborne rickettsial diseases: Rocky Mountain spotted fever,ehrlichioses, and anaplasmosis—-United States: a practical guide for physicians and otherhealth-care and public health professionals. MMWR Recomm Rep. 2006; 55(RR-4):1-27.Available at: http://www.cdc.gov/mmwr/preview/mmwrhtml/rr5504a1.htm

Copyright © 2010 by the American Academy of Pediatrics page 508

Page 509: AAP PREP 2010

2010 PREP SA on CD-ROM

Question: 205

A 16-year-old football player presents for evaluation of a 1-week history of fever, progressivelyworsening fatigue, and a sore throat. Physical examination shows a tired-appearing teenagerwho has a temperature of 38.9°C, moderate tonsillar enlargement with exudates, a liver edgethat is palpable 3 cm below the right costal margin, and a spleen tip that is easily palpable 2 cmbelow the left costal margin. Results of the spot test for infectious mononucleosis are positive.

Of the following, the MOST appropriate management for this patient includes

A. avoidance of contact sports

B. bed rest for 1 week

C. oral acyclovir

D. oral amoxicillin

E. oral steroids

Copyright © 2010 by the American Academy of Pediatrics page 509

Page 510: AAP PREP 2010

2010 PREP SA on CD-ROM

Preferred Response: ACritique: 205

The patient described in the vignette has acute infectious mononucleosis (IM), the mostcommon cause of which is Epstein-Barr virus (EBV), a herpesvirus. Humans are the onlysource of EBV, and close personal contact is needed for transmission. The spectrum of diseasemanifestations is wide, ranging from asymptomatic to fatal infection. Infections frequently areunrecognized in infants and young children. Classic IM is an acute illness characterized by theclinical findings of exudative tonsillopharyngitis, fever, lymphadenopathy, andhepatosplenomegaly. Anorexia, malaise, myalgias, and fatigue are common associatedsymptoms.

Nonspecific tests for heterophile antibodies frequently are used for diagnosis. Theheterophile antibody response is a transient immunoglobulin M response that is present in about85% to 90% of cases of EBV IM in adolescents and adults. The antibody appears during the first2 weeks of illness and gradually disappears over a 6-month period. The results of heterophileantibody tests are often negative in children 4 years of age and younger.

Serologic testing is used routinely for the detection and diagnosis of EBV infection.Interpretation of the serologic tests is shown in Item C205.

Treatment of IM is primarily supportive. The patient’s level of activity is tailored to what he orshe can tolerate comfortably. Bed rest may be needed, but there is no specific recommendationfor its use and little evidence to support that this shortens the course of disease or preventscomplications. Contact sports should be avoided until the patient is fully recovered and thespleen no longer is palpable. Neither ampicillin nor amoxicillin should be given to patients whohave suspected mononucleosis because their use is associated with the development of anonallergic morbilliform rash in a large proportion of patients, leading to an incorrect suspicion ofallergy. Most importantly, antibiotics do not treat the infection.

The routine use of corticosteroids in patients who have IM is not recommended. A short-course of corticosteroids may be considered only for those who have marked tonsillarinflammation with impending airway obstruction, massive splenomegaly, myocarditis, hemolyticanemia, or hemophagocytic syndrome. Even though acyclovir has in vitro antiviral activityagainst EBV, this agent has no proven value in the treatment of IM.

As a result of reviewing this information, do you intend to make a change in practiceto provide better patient care?Yes No

References:

American Academy of Pediatrics. Epstein-Barr virus infections (infectious mononucleosis). In:Pickering LK, Baker CJ, Kimberlin DW, Long SS, eds. Red Book: 2009 Report of the Committeeon Infectious Diseases. 28th ed. Elk Grove Village, Ill: American Academy of Pediatrics;2009:289-292

Johannsen EC, Schooley RT, Kaye KM. Epstein-Barr virus (infectious mononucleosis). In:Mandell GL, Bennett JE, Dolan R, eds. Mandell, Douglas, and Bennett’s Principles and Practice

Copyright © 2010 by the American Academy of Pediatrics page 510

Page 511: AAP PREP 2010

2010 PREP SA on CD-ROM

of Infectious Diseases. 6th ed. Philadelphia, Pa: Elsevier Churchill Livingstone; 2005:1801-1820

Junker AK. Epstein-Barr virus. Pediatr Rev. 2005;26:79-85. Available at:http://pedsinreview.aappublications.org/cgi/content/full/26/3/79

Sumaya CV, Ench Y. Epstein-Barr virus infectious mononucleosis in children: I. Clinical andgeneral laboratory findings. Pediatrics. 1985;75:1003-1010. Abstract available at:http://pediatrics.aappublications.org/cgi/content/abstract/75/6/1003

Copyright © 2010 by the American Academy of Pediatrics page 511

Page 512: AAP PREP 2010

2010 PREP SA on CD-ROM

Question: 206

On rounds, you ask a resident to discuss diuretic use in the pediatric patient. After relating theindications for their use, he describes the various adverse effects, including acid/basedisturbances.

Of the following, the diuretic that is MOST likely to induce metabolic acidosis is

A. acetazolamide

B. furosemide

C. hydrochlorothiazide

D. mannitol

E. metolazone

Copyright © 2010 by the American Academy of Pediatrics page 512

Page 513: AAP PREP 2010

2010 PREP SA on CD-ROM

Preferred Response: ACritique: 206

Diuretics are used in a variety of clinical settings to induce sodium and water excretion.Their effectiveness and, in some circumstances, adverse effect profile can be predicted basedon their site of action in the nephron.

Acid-base disturbances can be seen in children treated with diuretics. Diuretics that result inchloride and potassium losses (including loop diuretics such as furosemide and thiazide diureticssuch as metolazone and hydrochlorothiazide) usually cause a metabolic alkalosis. Metabolicacidosis can be seen with the diuretic acetazolamide that, because of its carbonic anhydraseinhibitory properties, results in bicarbonate losses at the level of the proximal convoluted tubule.This creates a proximal renal tubular acidosis (type II) with a normal anion gap. Another class ofdiuretics that can cause metabolic acidosis is the potassium-sparing agents such asspironolactone because of its inhibition of aldosterone, which regulates both sodium/potassiumexchange and sodium/hydrogen exchange. This creates a clinical picture of distal renal tubularacidosis with hyperkalemia (type IV renal tubular acidosis).

Mannitol is an osmotic diuretic that causes volume loss but tends not to affect acid-basebalance.

References:

Greenberg A. Diuretic complications. Am J Med Sci. 2000;319:10-24. Abstract available at:http://www.ncbi.nlm.nih.gov/pubmed/10653441

Rose BD, Post TW. Clinical use of diuretics. In: Clinical Physiology of Acid-base and ElectrolyteDisorders. 5th ed. New York, NY: McGraw-Hill Medical Publishing Division; 2001:447-477

Copyright © 2010 by the American Academy of Pediatrics page 513

Page 514: AAP PREP 2010

2010 PREP SA on CD-ROM

Question: 207

A 12-year-old girl presents with a 6-month history of increasing cough, dyspnea, and wheezing.She has a history of "asthma" but reports only using a short-acting beta agonist (SABA).Despite using her SABA three to four times per day, she reports nightly asthma symptoms,significant limitations in physical activity, and frequent asthma symptoms throughout the day.Spirometry performed in the clinic shows significant obstruction, with a forced expiratoryvolume in 1 second of 58%.

Of the following, the MOST appropriate asthma medication to consider starting in this child is

A. a high-dose inhaled steroid and long-acting beta agonist

B. a medium-dose inhaled steroid and oral leukotriene antagonist

C. an oral leukotriene antagonist only

D. omalizumab

E. oral theophylline and daily SABA

Copyright © 2010 by the American Academy of Pediatrics page 514

Page 515: AAP PREP 2010

2010 PREP SA on CD-ROM

Preferred Response: ACritique: 207

Based on the 2007 National Asthma Education and Prevention Program Expert PanelReport 3 Guidelines for the Diagnosis and Management of Asthma, the 12-year-old girldescribed in the vignette has severe persistent asthma. A stepwise approach is used to identifythe severity of asthma in a new patient and determine the level of control. This stepwiseapproach accounts for patient’s treatment responses and changes that often occur over time.The choice and schedule of medications is determined by the level of asthma severity andasthma control. Severe persistent asthma in children 12 years of age or older requires theinitiation of step 4 or step 5 therapy (Item C207).

The preferred treatment includes a medium- to high-dose inhaled corticosteroid plus a long-acting beta agonist (LABA). Omalizumab is an additional therapy that could be considered forthis girl if she had allergic asthma, with evidence of specific sensitization to associated allergensand an appropriate total serum immunoglobulin E concentration.

A medium-dose inhaled corticosteroid plus an oral leukotriene antagonist is an alternative tothe previously noted regimen but is not the first choice. An oral leukotriene antagonist or oraltheophylline as monotherapy are only appropriate for step 2 therapy.

In asthma, exposure to allergens leads to an early airway response that develops fullywithin 10 to 20 minutes. In addition, a late airway response can occur and develop fully 3 to 8hours after exposure. Oral and inhaled glucocorticoids inhibit the late-phase reaction but not theearly-phase reaction. Inhaled short-acting and long-acting beta-adrenergic agents are potentinhibitors of the early-phase reaction. Leukotriene receptor antagonists (montelukast, zafirlukast)and leukotriene synthesis inhibitors (zileuton) also block the early-phase response and mayhave a mild impact on the late-phase response.

References:

Fish JE, Peters SP. Bronchial challenge testing. In: Adkinson NF Jr, Yunginger JW, Busse WW,Bochner BS, Holgate ST, Simons FER, eds. Middleton’s Allergy Principles and Practice. 6th ed.Philadelphia, Pa: Mosby Elsevier; 2003:657-670

Liu AH, Covar RA, Spahn JD, Leung DYM. Childhood asthma. In: Kliegman RM, Behrman RE,Jenson HB, Stanton BF, eds. Nelson Textbook of Pediatrics. 18th ed. Philadelphia, Pa: SaundersElsevier; 2007:953-970

McFadden ER Jr. Exercise-induced airway narrowing. In: Adkinson NF Jr, Yunginger JW, BusseWW, Bochner BS, Holgate ST, Simons FER, eds. Middleton’s Allergy Principles and Practice.6th ed. Philadelphia, Pa: Mosby Elsevier; 2003: 1323-1332

National Heart, Lung, and Blood Institute and the National Asthma Education and PreventionProgram. Expert Panel Report 3 (EPR3): Guidelines for the Diagnosis and Management ofAsthma. Bethesda, Md: U.S. Department of Health and Human Services. 2007. Available athttp://www.nhlbi.nih.gov/guidelines/asthma/asthgdln.htm

Copyright © 2010 by the American Academy of Pediatrics page 515

Page 516: AAP PREP 2010

2010 PREP SA on CD-ROM

Stempel DA. Asthma and the athlete. In: Leung DYM, Sampson HA, Geha RS, Szefler SJ, eds.Pediatric Allergy Principles and Practice. St. Louis, Mo: Mosby Elsevier; 2003:435-443

Copyright © 2010 by the American Academy of Pediatrics page 516

Page 517: AAP PREP 2010

2010 PREP SA on CD-ROM

Question: 208

A 17-year-old football player is brought to the emergency department because of an upper leginjury he sustained while being tackled. He recalls that when he was tackled, an opposingplayer’s knee "rammed into" his right thigh. He has been unable to walk because of pain.Paramedics were called, his thigh was splinted, and he was transported to the emergencydepartment. On physical examination, he is alert and in moderate pain, his thigh is markedlyswollen, but there is no other obvious deformity. His heart rate is 100 beats/min, respiratory rateis 20 breaths/min, and blood pressure is 130/60 mm Hg. His lower leg is neurovascularly intact.A plain radiograph of his right thigh reveals no fracture or dislocation.

Of the following, the MOST likely potential complication of this injury is

A. myositis ossificans

B. Osgood-Schlatter disease

C. rhabdomyolysis

D. slipped capital femoral epiphysis

E. unicameral bone cyst

Copyright © 2010 by the American Academy of Pediatrics page 517

Page 518: AAP PREP 2010

2010 PREP SA on CD-ROM

Preferred Response: ACritique: 208

The athlete described in the vignette is suffering from a quadriceps contusion related todirect blunt trauma to his upper thigh. Because of the size of the quadriceps muscle, asignificant amount of blood can extravasate into the muscle, although rarely is it enough tocause hypovolemia or vital sign changes. Treatment of such a contusion includes ice,nonsteroidal anti-inflammatory medication, and early mobilization.

Complications are rare, although myositis ossificans is reported to occur in as many as 9%of quadriceps hematomas. This condition, which results in bone formation in injured muscle,develops 3 to 4 weeks after a soft-tissue injury and presents as a firm, nontender swelling atthe site of a previous injury. Peripheral calcification is evident on plain radiography (Item C208A)and progresses over 3 to 6 months. In most cases, the lesion resolves or decreases in sizeover time without medical intervention. Rarely, surgical excision is required. Myositis ossificanscan be confused with osteosarcoma, but the presence of calcification in the muscle alone andthe absence of a "sunburst" or "hair-on-end" appearance of calcification distinguishes it fromosteosarcoma (Item C208B).

Osgood-Schlatter disease is an overuse syndrome that is characterized by knee pain overthe tibial tuberosity. Rhabdomyolysis is the condition in which muscle injury leads to muscle celldeath and the release of intracellular contents. Rhabdomyolysis develops more commonly aftersignificant muscular crush injuries and less so after isolated blunt trauma. Slipped capital femoralepiphysis and unicameral bone cyst are not complications of blunt muscular trauma.

References:

Beiner JM, Jokl P. Muscle contusion injuries: current treatment options. J Am Acad Orthop Surg.2001;9:227-237. Abstract available at: http://www.ncbi.nlm.nih.gov/pubmed/11476532

Harner CD, Donatelli R, Mauro CS. Contusions. In: Griffin LY, ed. Essentials of MusculoskeletalCare. 3rd ed. Rosemont, Ill: American Academy of Orthopaedic Surgeons; 2005:511-512

Pizzutillo PD, Chandler JB, Maxwell T. Osgood-Schlatter syndrome. In: Griffin LY, ed. Essentialsof Musculoskeletal Care. 3rd ed. Rosemont, Ill: American Academy of Orthopaedic Surgeons;2005:913-914

Walter KD, Lin DY, Schwartz E. Slipped capital femoral epiphysis. eMedicine Specialties, SportsMedicine, Lower Limb. 2008. Available at: http://www.emedicine.com/sports/TOPIC122.HTM

Wheeless CR III. Myositis ossificans (Sterner’s tumor). In: Wheeless’ Textbook of Orthopaedics.2008. Available at: http://www.wheelessonline.com/ortho/myositis_ossificans_sterners_tumor

Wheeless CR III. Simple bone cyst (unicameral). In: Wheeless’ Textbook of Orthopaedics. 2008.Available at http://www.wheelessonline.com/ortho/simple_bone_cyst_unicameral

Copyright © 2010 by the American Academy of Pediatrics page 518

Page 519: AAP PREP 2010

2010 PREP SA on CD-ROM

Question: 209

A 19-year-old previously healthy boy who is a sophomore at a local college began experiencingan intermittent "burning feeling" in the epigastric region 2 weeks ago. Early this morning, heawoke feeling nauseated and vomited last evening’s dinner, along with a few "dark specks."Now he complains of a headache, epigastric discomfort, and a feeling of fullness. He reportsthat his grades have declined recently and that he has been receiving mostly "C’s" this year. Heattended a party last night and admits to "having a few beers." Physical examinationdemonstrates an alert, well-hydrated young man. Abdominal examination reveals epigastrictenderness without rebound, and the rectal examination produces a small amount of brown stoolthat is positive for occult blood. A hematocrit in your office is 42% (0.42).

Of the following, the MOST important next step is to

A. obtain serum Helicobacter pylori antibodies

B. prescribe lansoprazole 60 mg bid for 6 weeks

C. prescribe sucralfate 1 g four times a day for 1 month

D. recommend that he stop drinking alcohol

E. refer for immediate upper gastrointestinal endoscopy

Copyright © 2010 by the American Academy of Pediatrics page 519

Page 520: AAP PREP 2010

2010 PREP SA on CD-ROM

Preferred Response: DCritique: 209

The young college student described in the vignette presents with typical symptoms ofupper gastrointestinal tract damage that have increased in severity after attending a party theprevious evening. Physical examination reveals epigastric tenderness and evidence of occultgastrointestinal tract bleeding. These signs point to an alcohol-induced problem, in this case,alcoholic gastritis. Although specific medical therapy aimed at gastric acid blockade may beprescribed, the most important course of action is to obtain a careful history, including recentexposure to drugs or alcohol, and to urge the young man to refrain from alcohol consumption.Although abstinence for at least 1 month would be the recommendation for adults beforeresuming alcohol consumption in moderation, this young man should be counseled to resumeconsumption only upon achievement of legal age.

Physiologically, ethanol is a well-described esophageal and gastric mucosal toxin, inducingboth erosive lesions and frank hemorrhage. Chronic alcohol abuse has been associated with thedevelopment of atrophic gastritis as well as with esophageal cancer. Studies in humansdemonstrate that adults who consume alcohol, even in moderation, may develop subepithelialgastric mucosal hemorrhage and edema but show little evidence of a mucosal or submucosalinflammatory response. Follow-up endoscopies demonstrate that the gastric lesions disappearrapidly following abstinence from alcohol intake.

In addition to recommending that alcohol intake be curtailed, standard acid-blocking therapymay be prescribed for this patient to reduce the likelihood of acid injury to an already damagedmucosa. Depending on clinical severity, once- or twice-daily therapy with a proton pump inhibitormay be prescribed. The maximum dose for lansoprazole is 30 mg bid, although once-dailytherapy usually is adequate in uncomplicated cases. Sucralfate is an aluminum-containingsulfated derivative of the disaccharide sucrose. When ingested orally, it forms a gel thatadheres to inflamed esophageal and gastric mucosa as well as to gastric and duodenal ulcerbeds, thus creating a protective barrier against further acid-mediated damage. Sucralfate maybe used in the acute, primary management of peptic ulcer disease, but it is administered morecommonly as adjunctive therapy. Endoscopic evaluation should not be required for the youngman in the vignette unless he fails to respond to conservative medical therapy.

Determination of H pylori serum antibodies is of limited utility because bacterial colonization,with the appearance of positive antibody titers, occurs in many healthy individuals by the end ofthe second decade of life. In the United States and Canada, the overall prevalence of positive Hpylori antibodies is 35% to 45%. Furthermore, although a positive antibody titer indicatesbacterial colonization, it does not predict a cause-and-effect relationship between infection and

inflammation. Other diagnostic modalities (13C-urea breath testing, fecal H pylori antigen) areunder evaluation, but upper endoscopy with biopsy remains the gold standard for diagnosing Hpylori gastritis in pediatrics. Although several published reports have attempted to link alcoholicgastritis to an association with H pylori infection, other studies note that ethanol (particularlywhen consumed in red wine) actually may inhibit bacterial colonization.

Approximately 40% of college students are binge drinkers, but the incidence of alcoholicgastritis among college students is unknown.

References:

Copyright © 2010 by the American Academy of Pediatrics page 520

Page 521: AAP PREP 2010

2010 PREP SA on CD-ROM

American Academy of Pediatrics, Committee on Substance Abuse. Alcohol use and abuse: apediatric concern. Pediatrics. 1995;95:439-442. Abstract available at:http://pediatrics.aappublications.org/cgi/content/abstract/95/3/439

Bujanda L. The effects of alcohol consumption upon the gastrointestinal tract. Am JGastroenterol. 2000;95:3374-3382. Abstract available at:http://www.ncbi.nlm.nih.gov/pubmed/11151864

Cleary C, Klein JD. In brief: alcoholic gastritis. Pediatr Rev. 1997;18:282. Available at:http://pedsinreview.aappublications.org/cgi/content/full/18/8/282

Comerci GD. Office assessment of substance abuse and addiction. Adolesc Med. 1993;4:277-294. Abstract available at: http://www.ncbi.nlm.nih.gov/pubmed/10356215

Kato S, Sherman PM. What is new related to Helicobacter pylori infection in children andteenagers? Arch Pediatr Adolesc Med. 2005;159:415-421. Available at: http://archpedi.ama-assn.org/cgi/content/full/159/5/415

Laine L, Peterson WL. Bleeding peptic ulcer. N Engl J Med. 1994;331:717-727. Extract availableat: http://content.nejm.org/cgi/content/extract/331/11/717

Valencia-Parparcén J. Alcoholic gastritis. Clin Gastroenterol. 1981;10:389-399

Copyright © 2010 by the American Academy of Pediatrics page 521

Page 522: AAP PREP 2010

2010 PREP SA on CD-ROM

Question: 210

You are meeting with a pregnant woman who has received a liver transplant and is takingchronic immunosuppression therapy. She asks you if the drugs she takes will precludebreastfeeding her infant.

Of the following, the immunosuppressive drug that has the BEST safety profile for lactatingwomen is

A. cyclophosphamide

B. cyclosporin A

C. methotrexate

D. prednisone

E. tacrolimus

Copyright © 2010 by the American Academy of Pediatrics page 522

Page 523: AAP PREP 2010

2010 PREP SA on CD-ROM

Preferred Response: DCritique: 210

Many medications prescribed to lactating mothers pose no risk for the newborn. Amongthose for which there are concerns, antineoplastic agents and immunosuppressant drugs arebest known. The woman in the vignette, who has a history of solid organ transplantation, isreceiving immunosuppressant therapies. One of the safest immunosuppressive drugs isprednisone (Item C210A).

When a physician or other health-care professional is uncertain of the potentialcontraindication of a medication in a breastfeeding mother, he or she should contact thepharmacist because many resources available to clinicians may, in fact, be out of date.

Cyclophosphamide and cyclosporin A both enter human milk and are transferred to thenursing infant, but they have unknown effects on infant growth. Methotrexate is known tocause neutropenia. Tacrolimus may enter the milk, and data are insufficient to declare it safe inall instances. Other medications pose a high risk for breastfeeding infants (Item C210B).

As a result of reviewing this information, do you intend to make a change in practiceto provide better patient care?Yes No

References:

Akus M, Bartick M. Lactation safety recommendations and reliability compared in 10 medicationresources. Ann Pharmacother. 2007;41:1352-1360. Abstract available at:http://www.ncbi.nlm.nih.gov/pubmed/17623757

American Academy of Pediatrics Committee on Drugs. The transfer of drugs and otherchemicals into human milk. Pediatrics. 2001;108:776-789. Available at:http://pediatrics.aappublications.org/cgi/content/full/108/3/776

Coady NT. Maternal transplantation medications during breastfeeding. J Hum Lact. 2002;18:66-68. Abstract available at: http://www.ncbi.nlm.nih.gov/pubmed/11845741

Gardiner SJ, Begg EJ. Breastfeeding during tacrolimus therapy. Obstet Gynecol. 2006;107:453-455. Abstract available at: http://www.ncbi.nlm.nih.gov/pubmed/16449146

Hale TW. Pharmacology review: drug therapy and breastfeeding: antibiotics, analgesics, andother medications. NeoReviews. 2005;6:e233-e240. Available for subscription at:http://neoreviews.aappublications.org/cgi/content/full/6/5/e233

Hale TW. Pharmacology review: drug therapy and breastfeeding: antidepressants,antipsychotics, antimanics, and sedatives. NeoReviews. 2004;5:e451-e456. Available forsubscription at: http://neoreviews.aappublications.org/cgi/content/full/5/10/e451

Mastrobattista JM, Gomez-Lobo V; Society for Maternal-Fetal Medicine. Pregnancy after solid

Copyright © 2010 by the American Academy of Pediatrics page 523

Page 524: AAP PREP 2010

2010 PREP SA on CD-ROM

organ transplantation. Obstet Gynecol. 2008;112:919-932. Abstract available at:http://www.ncbi.nlm.nih.gov/pubmed/18827137

Copyright © 2010 by the American Academy of Pediatrics page 524

Page 525: AAP PREP 2010

2010 PREP SA on CD-ROM

Question: 211

You are precepting a group of residents in continuity clinic. Several of them have seen patientswho have earaches and have diagnosed acute otitis media. One resident says that he read thatantibiotic treatment is not always necessary in these cases, and he wants to observe one of hispatients with pain medication only. You tell him that this is only a reasonable option for certainchildren, and proper follow-up must be ensured.

Of the following, the patient who would be treated MOST appropriately with observation andpain management rather than antibiotic therapy is

A. a 4-month-old who has a temperature of 38.4°C and an air-fluid level behind a mobile,erythematous tympanic membrane

B. a 6-month-old who has a temperature of 38.9°C and fluid behind an erythematous, immobiletympanic membrane

C. a 9-month-old who has a temperature of 38.0°C and a shiny, pink tympanic membrane thatmoves well with insufflation

D. a 15-month-old who has a temperature of 39.5°C and purulent drainage from the externalauditory canal

E. a 21-month-old who has a temperature of 38.1°C and an erythematous, bulging tympanicmembrane

Copyright © 2010 by the American Academy of Pediatrics page 525

Page 526: AAP PREP 2010

2010 PREP SA on CD-ROM

Preferred Response: CCritique: 211

Acute otitis media, acute inflammation of the middle ear, has been treated routinely withantibiotic therapy in the United States. The most common bacterial causes are Streptococcuspneumoniae, Haemophilus influenzae, and Moraxella catarrhalis, although many cases actuallyare viral in origin and resolve without antibiotic therapy. The growing problem of antibioticresistance and the overall cost of antibiotic therapy for this very common problem have led toconsideration of "watchful waiting" as a management option for some children. A clinicalpractice guideline has been published by the American Academy of Pediatrics and the AmericanAcademy of Family Physicians to assist clinicians in making decisions about whether to initiateantibiotic therapy for children who have acute otitis media.

Acute otitis media should be diagnosed by the rapid onset of symptoms, evidence of middleear effusion, and signs or symptoms of inflammation, including otalgia and erythema of thetympanic membrane. The diagnosis may be considered "certain" if all of these features arepresent. If middle ear effusion cannot be documented, the diagnosis may be considered"uncertain." Pain always should be assessed. The option of observation or "watchful waiting"versus antibiotic therapy is detailed in Item C211, which can be found in the clinical practiceguideline. Severe illness is defined by the presence of moderate-to-severe otalgia or atemperature of at least 39°C.

Observation should be considered only if adequate follow-up can be ensured. Parents maybe given a prescription for antibiotics that can be filled if symptoms persist beyond 48 to 72hours or follow-up visits can be scheduled within that time to ensure improvement. Painmanagement always should be offered to children, and parents should be cautioned to seekmedical attention if symptoms worsen or if evidence of more severe disease, such asmastoiditis, becomes apparent.

The 9-month-old infant has a low-grade fever and an uncertain diagnosis of acute otitismedia because the tympanic membrane moves well with insufflation and, therefore, middle eareffusion cannot be confirmed. Accordingly, he may be observed with pain medication alone aslong as follow-up can be ensured. The 4-month-old, 6-month-old, and 21-month-old childrenhave diagnoses indicating the need to treat with antibiotic therapy. The 15-month-old child hasevidence of severe disease (temperature to 39.5°C) and otorrhea, necessitating the use ofantibiotics.

As a result of reviewing this information, do you intend to make a change in practiceto provide better patient care?Yes No

References:

American Academy of Pediatrics and American Academy of Family Physicians. Subcommittee onManagement of Acute Otitis Media. Clinical practice guideline. Diagnosis and management ofacute otitis media. Pediatrics. 2004;113:1451-1465. Available at:http://pediatrics.aappublications.org/cgi/content/full/113/5/1451

Copyright © 2010 by the American Academy of Pediatrics page 526

Page 527: AAP PREP 2010

2010 PREP SA on CD-ROM

Siegel RM, Bien JP. Acute otitis media in children: a continuing story. Pediatr Rev. 2004;25:187-193. Available at: http://pedsinreview.aappublications.org/cgi/content/full/25/6/187

Copyright © 2010 by the American Academy of Pediatrics page 527

Page 528: AAP PREP 2010

2010 PREP SA on CD-ROM

Question: 212

A 2-year-old boy presents to your office with a 2-day history of swelling of the right eye. Hehas been otherwise well. There are scattered insect bites on his body, including one about 2 cmlateral to the affected eye. There is no discharge, and the bite appears to be healing. The boy’sright eyelids are swollen and seem tender to palpation.

Of the following, the MOST concerning additional ophthalmologic finding for this boy is

A. decreased extraocular movements

B. hyperemia of the palpebral conjunctiva

C. photophobia

D. purulent exudates

E. subconjunctival hemorrhage

Copyright © 2010 by the American Academy of Pediatrics page 528

Page 529: AAP PREP 2010

2010 PREP SA on CD-ROM

Preferred Response: ACritique: 212

The child described in the vignette has developed swelling of the soft tissues of the eye.The differential diagnosis includes periorbital (preseptal) cellulitis and orbital cellulitis.Distinguishing between these two conditions is critical because orbital cellulitis is a much moreserious condition. Generally, the conditions are differentiated on clinical grounds and confirmedwith imaging of the orbit.

High fever, toxicity, limitation of eye movement due to involvement of extraocular muscles,proptosis, and pain are signs of the potentially more serious diagnosis of orbital cellulitis (ItemC212A). Lack of pain, full range of movement of the extraocular muscles, and lack of toxicityand fever are common with periorbital cellulitis.

Photophobia and hyperemia of the palpebral conjunctivae may accompany infection of theeye and surrounding tissue, but these are nonspecific findings. Periorbital cellulitis (Item C212B)may be associated with purulent exudates from the eye, but this finding is not indicative of moreserious orbital involvement. Subconjunctival hemorrhage frequently is related to abruptincreases in intraocular pressure from a variety of causes (eg, forceful coughing, vomiting). Itdoes not suggest a more serious disorder in the patient who has swelling of the eye.

References:

Nield LS, Kam D. Visual diagnosis: a 9-year-old girl who has fever, headache, and right eyepain. Pediatr Rev. 2005;26:337-340. Available at:http://pedsinreview.aappublications.org/cgi/content/full/26/9/337

Wald ER. Periorbital and orbital infections. Pediatr Rev. 2004;25:312-320. Available at:http://pedsinreview.aappublications.org/cgi/content/full/25/9/312

Yellon RF, McBride TP, Davis HW. Otolaryngology. In: Zitelli BJ, Davis HW. Atlas of PediatricPhysical Diagnosis. 5th ed. Philadelphia, Pa: Mosby Elsevier; 2007:889932

Copyright © 2010 by the American Academy of Pediatrics page 529

Page 530: AAP PREP 2010

2010 PREP SA on CD-ROM

Question: 213

You treat a 15-year-old girl in your practice who has juvenile idiopathic arthritis (JIA). She isbrought in by her mother today with complaints of a low-grade fever and diffuse pain. Onphysical examination, she has a temperature of 38.0°C and a heart rate of 100 beats/minute. Asshe sits on the examination table, she leans forward. During auscultation of her lungs, shecomplains of pain with deep inspiration.

Of the following, the MOST likely explanation for her symptoms is

A. costochondritis

B. gastritis

C. pericarditis

D. pneumonia

E. pulmonary embolism

Copyright © 2010 by the American Academy of Pediatrics page 530

Page 531: AAP PREP 2010

2010 PREP SA on CD-ROM

Preferred Response: CCritique: 213

Chest pain in children and adolescents is a common problem that frequently leads patientsand parents to seek medical care. The causes of chest pain in the pediatric population are variedand can be considered by organ systems: musculoskeletal, respiratory, gastrointestinal,psychological, and cardiac. Among the musculoskeletal causes are chest wall strain, trauma,costochondritis, and the precordial catch syndrome. Respiratory causes include asthma,pneumonia, pneumothorax, pneumomediastinum, and chronic cough. Chest pain may be asymptom of gastritis, esophagitis, or indigestion. Chest pain also may result from or beexacerbated by psychogenic processes, including anxiety, fear, and attention-seekingbehaviors. Perhaps the most common causes of chest pain in pediatrics are those referred to asidiopathic. This diagnosis often is given to the patient who presents with a 1- to 2-week historyof intermittent, brief, sharp or stabbing pain that is not associated with exercise or exertion.

The cardiac causes of chest pain are important to recognize because they can beassociated with significant morbidity and mortality. The pain can be due to pericardial problems,angina and myocardial ischemia, arrhythmias, and aortic dissection. Pericardial pain is caused byinflammation and frequently is associated with pericarditis. The pain often is substernal,positional, and possibly severe. Affected patients frequently prefer to sit and may refuse to liedown. Pain that results from angina and myocardial ischemia is rare in children but can occur inthose who have anomalies of the coronary arteries. Such pain often is referred to as apressure sensation with burning, and there may be radiation to the neck, shoulder, or arm.Typically, the pain occurs during or following exercise or activity, and it may improve with rest.The clinician should strongly consider this cause in the patient who has had Kawasaki diseaseor cardiac surgery. It is also important to consider use of illicit drugs such as cocaine and otheradrenergic stimulators as a potential cause of coronary vasospasm. Some of thetachyarrhythmias, such as supraventricular tachycardia, may present with chest pain, althoughthis usually is described as a discomfort associated with palpitation and other symptoms. Thepain may be associated with exercise, but this is not the typical presentation. Pain from aorticdissection usually is acute and sharp and may present in the anterior chest or the back,depending on the area of the aorta that is affected. A history of Marfan syndrome or Ehlers-Danlos syndrome should be pursued in affected patients and their families.

For the patient described in the vignette, the positional pain exacerbated by inspirationsuggests a pericardial source. Coupled with her concurrent fever, the most likely diagnosis ispericarditis, which is a known complication of juvenile idiopathic arthritis. The pain described inthe vignette is not consistent with costochondritis, gastritis, pneumonia, or pulmonary embolism.

References:

Cava JR, Sayger PL. Chest pain in children and adolescents. Pediatr Clin North Am.2004;51:1553-1568. Abstract available at: http://www.ncbi.nlm.nih.gov/pubmed/15561173

Fahey J. Chest pain. In: Rudolph CD, Rudolph AM, Hostetter MK, Lister G, Siegel NJ, eds.Rudolph’s Pediatrics. 21st ed. New York, NY: McGraw Hill; 2003:1894-1897

Copyright © 2010 by the American Academy of Pediatrics page 531

Page 532: AAP PREP 2010

2010 PREP SA on CD-ROM

Question: 214

At a routine health supervision visit, the parents of a 12-year-old boy report that their son’swalking and running seem to have become clumsy over the past few years. He reports no painin the legs or back, and there is no family history of neurologic problems. Physical examinationreveals no abnormalities of joints, muscles, or skin. Mental status, speech, and cranial nervesare normal on neurologic examination. Arm strength, reflexes, and coordination are normal.Strength in his feet is slightly diminished, and reflexes at the knees and ankles are absent. Nomuscle wasting is apparent. He has a slightly broad-based, unsteady gait and has greatdifficulty with tandem gait. He exhibits significant postural sway with his eyes closed and feettogether (positive Romberg sign).

Of the following, the MOST likely explanation for these symptoms is

A. ataxia telangiectasia

B. Dandy Walker syndrome

C. Friedreich ataxia

D. hydrocephalus

E. multiple sclerosis

Copyright © 2010 by the American Academy of Pediatrics page 532

Page 533: AAP PREP 2010

2010 PREP SA on CD-ROM

Preferred Response: CCritique: 214

Ataxia refers to incoordination of motor actions due to disease or disorders of thecerebellum or its connections. Most children who present for evaluation of clumsiness or poorcoordination have a static developmental problem that can be monitored clinically withoutneuroimaging or laboratory testing. Progressive ataxias can be categorized as acute/subacute,episodic/paroxysmal, and chronic progressive. The boy described in the vignette has a historyof chronic progressive clumsiness with very worrisome signs demonstrated on carefulneurologic examination. The combination of progressive symptoms and findings of tandem gaitdifficulty and areflexia should prompt a referral to a neurologist.

When assessing children who have progressive incoordination, four key steps can narrow

the broad differential diagnosis:1.Localization of the problem to the cerebellum alone or cerebellum with other levels of

the nervous system (brain, basal ganglia, brainstem, spinal cord, and nerves). Forexample, the boy in the vignette appears to have spinal cord and peripheral nerve

disease.2.Clarification of the time course, such as chronic over a specific time period or

intermittent worsening.3.Identification of pattern of inheritance as autosomal dominant (it is advisable to examine

both parents, if possible), X-linked, autosomal recessive, or maternal/mitochondrial. Theboy in the vignette has a negative family history. Absence of an autosomal dominantdisease pattern reduces the likelihood of a number of spinocerebellar ataxias. The

inheritance may be either spontaneous or autosomal recessive. 4.Identification, if possible, of a highly characteristic phenotype. For example, are there

suggestive skin or eye findings? Combining these findings in a search using OnlineMendelian Inheritance in Man (OMIM) at http://www.ncbi.nlm.nih.gov/omim can helpnarrow the differential diagnosis. Useful reviews also are available, along with contactinformation for laboratories for testing, at www.geneclinics.org. Given the expense ofgenetic testing, a referral to a neurologist or movement disorders specialist may be

cost effective before genetic testing.Chronic ataxias in children are rare, but the most common diagnoses are Friedreich ataxia

and ataxia telangiectasia (both autosomal recessive) and spinocerebellar ataxias 1, 2, 3, and 7(all autosomal dominant). The child in the vignette has symptoms consistent with Friedreichataxia, which can be confirmed genetically by identifying a more than 90 GAA repeat expansionin the Frataxin gene. A recent clinical trial showed high-dose idebenone to be a beneficialtreatment.

Ataxia telangiectasia usually presents earlier than 12 years of age with chorea/athetosis orataxia, and although neurologic symptoms precede dermatologic ones, by this age, the oculartelangiectasias (Item C214) should be apparent. Hydrocephalus and Dandy Walker syndromeboth can cause chronic gait problems. However, progressive hydrocephalus should causeheadaches and distal hyperreflexia, and Dandy Walker syndrome should not be progressive orbe associated with distal wasting. Multiple sclerosis can have a chronic progressive course in

Copyright © 2010 by the American Academy of Pediatrics page 533

Page 534: AAP PREP 2010

2010 PREP SA on CD-ROM

some cases, but a relapsing remitting course is much more common. Also, ocular findings suchas an afferent pupillary defect should be present, the motor findings should not be so symmetric,and the gait should be spastic.

References:

Fogel BL, Perlman S. Clinical features and molecular genetics of autosomal recessive cerebellarataxias. Lancet Neurol. 2007;6:245-257. Abstract available at:http://www.ncbi.nlm.nih.gov/pubmed/17303531

Gilbert DL. Ataxia. In: Singer HS, Kossoff EH, Hartman AL, Crawford TO, eds. Treatment ofPediatric Neurologic Disorders. Boca Raton, Fla: Taylor & Francis Group; 2005:415-422

Johnston MV. Movement disorders. In: Kliegman RM, Behrman RE, Jenson HB, Stanton BF, eds.Nelson Textbook of Pediatrics. 18th ed. Philadelphia, Pa: Saunders Elsevier; 2007:2488-2493

Schöls L, Bauer P, Schmidt T, Schulte T, Riess O. Autosomal dominant cerebellar ataxias: clinicalfeatures, genetics, and pathogenesis. Lancet Neurol. 2004;3:291-304. (OMIM) available throughhttp://www.ncbi.nlm.nih.gov

Copyright © 2010 by the American Academy of Pediatrics page 534

Page 535: AAP PREP 2010

2010 PREP SA on CD-ROM

Question: 215

You are seeing a 4-year-old boy who is new to your practice for a health supervision visit.While reviewing the family history, you learn that his father has neurofibromatosis type 1 (NF1).Careful examination of the boy’s skin using a Woods lamp reveals six café au lait maculesmeasuring at least 5 mm, but no other lesions.

Of the following, the MOST likely feature(s) of NF-1 to develop next in this boy is (are)

A. cutaneous neurofibromas

B. Lisch nodules

C. plexiform neurofibroma

D. pseudarthrosis of the tibia

E. skinfold freckling

Copyright © 2010 by the American Academy of Pediatrics page 535

Page 536: AAP PREP 2010

2010 PREP SA on CD-ROM

Preferred Response: ECritique: 215

Neurofibromatosis type 1 (NF1) is among the most common genetic disorders in humans,occurring in approximately 1 in 3,000 individuals worldwide. The diagnosis is based on clinicalcriteria established by the National Institutes of Health Consensus Development Conference in1987 (Item C215A). To meet the diagnostic criteria for NF1, a person must have at least two ofthe criteria listed.

Typically, the first feature to appear is café au lait macules (Item C215B), as described forthe boy in the vignette. The diagnosis should be considered in any prepubertal child who has sixor more café au lait macules measuring at least 5 mm at their greatest dimension. The maculesmay be present at birth or appear in infancy. Approximately 80% of individuals who have NF1have at least 6 café au lait macules by age 5 years. These may be difficult to see initially, and aWoods lamp may be helpful. Sun exposure causes them to darken.

It is important to recognize that the appearance of the diagnostic features of NF1 is largelyage-dependent. This fact, together with the extreme variability of the condition at any age, canmake the diagnosis challenging. Most commonly, the second feature to appear is skinfoldfreckling (Item C215C), usually in the axillae and groin; this commonly occurs between ages 3and 5 years. Cutaneous or subcutaneous neurofibromas are often the third feature to appear,and they typically herald the onset of puberty. Lisch nodules, frecklelike hamartomas (ItemC215D) in the iris stroma, may not be evident until early adulthood.

Plexiform neurofibromas occur in approximately 25% of individuals who have NF1 and areoften congenital (Item C215E). They can remain quiescent for indefinite periods of time, growsteadily, or grow intermittently. These lesions can be disfiguring.

Tibial pseudarthrosis is either congenital or occurs following fracture due to congenital tibialbowing (Item C215F). Children who have tibial dysplasia should be referred promptly fororthopedic care.

Although genetic testing is available for NF1, implementation is not often necessary. By theage of 11 years, 95% of affected individuals meet clinical diagnostic criteria.

References:

Diagnosis of NF1. Children’s Tumor Foundation. Available at:http://www.ctf.org/neurofibromatosis-type-1/diagnosis-of-nf1.html

Friedman JM. Neurofibromatosis 1. GeneReviews. 2007. Available at:http://www.ncbi.nlm.nih.gov/bookshelf/br.fcgi?book=gene&part=nf1

Hersh JH and Committee on Genetics. Health supervision for children with neurofibromatosis.Pediatrics. 2008;121:633-642. Available at:http://pediatrics.aappublications.org/cgi/content/full/121/3/633

Viskochil D. Neurofibromatosis type 1. In: Cassidy SB, Allanson JE, eds. Management of GeneticSyndromes. 2nd ed. Hoboken, NJ: Wiley-Liss; 2005:369-384

Copyright © 2010 by the American Academy of Pediatrics page 536

Page 537: AAP PREP 2010

2010 PREP SA on CD-ROM

Question: 216

A 15-year-old girl presents for treatment of menstrual cramps. She had menarche 3 years ago,and over the last year, she has begun having pain on the first day of her cycle. She says that ifshe can get past the first day, the pain decreases and goes away in the next day or two. Theherbal tea her mother was giving her no longer works.

Of the following, the MOST effective initial treatment for this girl’s symptoms is

A. acetaminophen

B. calcium channel blocker

C. combined oral contraceptive

D. ibuprofen

E. omega-3 fatty acids

Copyright © 2010 by the American Academy of Pediatrics page 537

Page 538: AAP PREP 2010

2010 PREP SA on CD-ROM

Preferred Response: DCritique: 216

Dysmenorrhea or painful menstrual bleeding is the leading cause of recurrent, short-termschool absenteeism and lost work time among adolescent girls. Primary dysmenorrhea refers topainful menses with normal pelvic anatomy and starts when menstrual cycles becomeovulatory. A presumptive diagnosis can be based on a typical history of lower abdominal painthat begins shortly before or with the onset of menses and lasts 1 to 3 days with normalfindings on physical examination. Potential associated symptoms include headaches, back pain,vomiting, and diarrhea.

The pathogenesis of primary dysmenorrhea is believed to be related to increasedendometrial prostaglandin production, which causes myometrial contractility that, if excessive,leads to uterine ischemia and pain and the other associated symptoms. Elevated vasopressinconcentrations also are observed in affected patients. Risk factors for primary dysmenorrheainclude nulliparity, heavy menstrual flow, smoking, and poor mental health and social supports. Ifthe history or examination findings are inconsistent with primary dysmenorrhea, furtherevaluation for secondary causes is necessary.

Initial therapy for dysmenorrhea includes drugs that inhibit the synthesis of prostaglandinthrough the prostaglandin synthetase system and antagonize their action at the cell receptorlevel. The most established medications are the nonsteroidal anti-inflammatory drugs (NSAIDs),such as ibuprofen. They have been found to be significantly more effective for pain relief thanplacebo. Evidence is insufficient to determine which NSAID is the safest and most effective forthe treatment of dysmenorrhea. However, clinical trials have shown that all NSAIDs are moreeffective than acetaminophen. These medications are available over the counter and, thus,should be the first line of treatment. Specific cyclooxygenase-2 inhibitors also may relievedysmenorrhea symptoms.

In as many as 20% to 25% of patients, NSAIDs are not effective and alternate treatmentsuch as combined hormonal contraceptives is indicated. Combined hormonal contraceptives actby suppressing ovulation and lessening the endometrial lining of the uterus. Therefore, bothmenstrual fluid volume and the amount of prostaglandins produced decreases, which effectivelyreduce dysmenorrhea by decreasing uterine motility and uterine cramping. Other hormonalcontraceptive products have been used for treatment of dysmenorrhea, including depo-medroxyprogesterone acetate and levonorgestrel intrauterine device. All hormonal methods ofcontraception are prescription medications and require a doctor’s visit. This increases their costin time and money. Therefore, they are recommended for use only if NSAIDs are not effective.Several medications that are effective in inducing uterine relaxation have been proposed astreatment for dysmenorrhea. For example, early uncontrolled trials of calcium channel blockershave shown promise but require further investigation.

Many consumers are seeking alternatives to conventional medicine, and research into themenstrual cycle suggests that nutritional intake and metabolism may play an important role in thecause and treatment of menstrual disorders. One small trial showed fish oil (omega-3 fattyacids) to be more effective than placebo for pain relief. However, evidence is not yet strongenough to recommend the use of any herbal and dietary therapies for the treatment of primary orsecondary dysmenorrhea. Pelvic nerve interruption has been suggested for recalcitrant casesof dysmenorrhea, but this procedure is not recommended in the management, regardless of

Copyright © 2010 by the American Academy of Pediatrics page 538

Page 539: AAP PREP 2010

2010 PREP SA on CD-ROM

cause.

As a result of reviewing this information, do you intend to make a change in practiceto provide better patient care?Yes No

References:

French L. Dysmenorrhea in adolescents: diagnosis and treatment. Paediatr Drugs. 2008;10:1-7.Abstract available at: http://www.ncbi.nlm.nih.gov/pubmed/18162003

Hillard PJA. Consultation with the specialist: dysmenorrhea. Pediatr Rev. 2006;27:64-71.Available at: http://pedsinreview.aappublications.org/cgi/content/full/27/2/64

Marjoribanks J, Proctor ML, Farquhar, C. Nonsteroidal anti-inflammatory drugs for primarydysmenorrhoea. Cochrane Database Syst Rev. 2003;4:CD001751. Available at:http://www.mrw.interscience.wiley.com/cochrane/clsysrev/articles/CD001751/frame.html

Proctor ML, Murphy PA. Herbal and dietary therapies for primary and secondarydysmenorrhoea. Cochrane Database Syst Rev. 2001;2:CD002124. Available at:http://www.mrw.interscience.wiley.com/cochrane/clsysrev/articles/CD002124/frame.html

Proctor ML, Roberts H, Farquhar CM. Combined oral contraceptive pill (OCP) as treatment forprimary dysmenorrhoea. Cochrane Database Syst Rev. 2001;2:CD002120. Available at:http://www.mrw.interscience.wiley.com/cochrane/clsysrev/articles/CD002120/frame.html

Sanfilippo J, Erb T. Evaluation and management of dysmenorrhea in adolescents. Clin ObstetGynecol. 2008;51:257-267. Abstract available at:http://www.ncbi.nlm.nih.gov/pubmed/18463457

Copyright © 2010 by the American Academy of Pediatrics page 539

Page 540: AAP PREP 2010

2010 PREP SA on CD-ROM

Question: 217

You are evaluating a 2-year-old girl whose parents report that she choked while eating peanuts1 hour ago. The choking was relieved by her father pounding on her back. Her vital signs are:temperature of 37.5°C, heart rate of 120 beats/min, respiratory rate of 30 breaths/min, and bloodpressure of 80/50 mm Hg. Her oxygen saturation is 94% by pulse oximetry on room air. The childis in mild respiratory distress and has bilateral wheezing. Her parents state that she has nohistory of asthma.

Of the following, the BEST test to confirm a foreign body aspiration is

A. arterial blood gas

B. bronchoscopy

C. computed tomography scan of the chest

D. fluoroscopy of the chest

E. inspiratory and expiratory chest radiographs

Copyright © 2010 by the American Academy of Pediatrics page 540

Page 541: AAP PREP 2010

2010 PREP SA on CD-ROM

Preferred Response: BCritique: 217

All children who have a witnessed choking episode followed by coughing, wheezing, orinspiratory stridor should undergo evaluation for a foreign body aspiration. Physical findings caninclude tachypnea, retractions, wheezing, stridor, decreased unilateral breath sounds, orcyanosis. Bronchoscopy remains the gold standard for both diagnosis and management.Removal of the foreign body using rigid bronchoscopy is safe (complication rates of <1%) andhas helped reduce mortality. Flexible bronchoscopy may be helpful for diagnosis in equivocalcases but generally should be not used for removal of the object.

Because only 20% of foreign bodies are radiopaque, standard radiography and fluoroscopyare used only to evaluate for indirect signs of foreign body aspiration, such as unilateralhyperinflation, atelectasis, mediastinal shift, or pneumomediastinum. Expiratory chestradiographs are more sensitive for detection than inspiratory chest radiographs, but 50% ofradiographs of patients who have tracheal foreign bodies appear normal within the first 24hours of the event (Item C217). Fluoroscopy may increase the sensitivity due to its ability toassess diaphragmatic and mediastinal motion.

Computed tomography scan of the chest is of little additional benefit for acute aspirations.However, such imaging may prove useful for objects that have been present for a sufficientperiod of time to induce granulation and can provide useful information such as location, size ofobject, and degree of granulation prior to attempted extraction. Arterial blood gas analysis,although useful for judging the adequacy of oxygenation and ventilation, is not helpful indetermining the presence of a foreign body.

Mortality associated with foreign body aspiration has been reduced, likely due to betterpublic awareness and improved safety standards. However, the overall incidence of suchaspiration has not decreased, necessitating prompt recognition and management. Childrenyounger than 3 years of age are at the highest risk and account for almost 75% of pediatricforeign body aspirations. Although a variety of objects are aspirated, one third are nuts, withpeanuts predominating.

References:

Holinger LD. Foreign bodies of the airway. In: Kliegman RM, Behrman RE, Jenson HB, Stanton BF,eds. Nelson Textbook of Pediatrics. 18th ed. Philadelphia, Pa: Saunders Elsevier; 2007:1769

Rovin JD, Rogers BM. Pediatric foreign body aspiration. Pediatr Rev. 2000;21:86-90. Available at:http://pedsinreview.aappublications.org/cgi/content/full/21/3/86

Copyright © 2010 by the American Academy of Pediatrics page 541

Page 542: AAP PREP 2010

2010 PREP SA on CD-ROM

Question: 218

You are examining a 9-year-old boy who has a soft, but distinctly palpable 2-cm nodule on theleft lobe of his thyroid. It moves with swallowing. You arrange for thyroid fine-needle aspirationbiopsy with ultrasonographic guidance.

Of the following, the MOST appropriate information to share with the family is that

A. all thyroid nodules in boys should be removed because they have a higher risk of malignancythan nodules in girls

B. no further follow-up is necessary if the pathology report suggests a benign thyroid adenoma

C. there is a 50% chance that the thyroid nodule will be malignant

D. the biopsy offers a greater than 90% chance of determining whether a thyroid nodule isbenign or malignant

E. thyroid nodules in girls are more likely to be malignant than nodules in boys

Copyright © 2010 by the American Academy of Pediatrics page 542

Page 543: AAP PREP 2010

2010 PREP SA on CD-ROM

Preferred Response: DCritique: 218

Thyroid fine-needle aspiration (FNA) biopsy, usually conducted under ultrasonographicguidance, has revolutionized the management of thyroid nodules in adults. Depending on theseries, almost all malignancies are identified by aspiration biopsy (more than 95%), althoughsome malignancies cannot be diagnosed easily on FNA smear, and an area of malignancy maybe missed in a complex nodule. Nodules may be simple and cystic, simple and composed offollicular or papillary tissue, or complex and composed of some areas that are cystic and otherareas with follicular or components. Calcitonin-secreting medullary carcinoma of the thyroid alsomay present as a nodule and is most worrisome because of its resistance to therapy. Less than10% of thyroid cancers in children are medullary carcinomas. The risk of malignancy in an adultwho has a thyroid nodule is less than 15%.

Because most thyroid carcinomas progress slowly, watchful waiting and carefulobservation after biopsy may be all that is needed in the average adult. The results of FNA seemsimilar in children, but the greater likelihood of a malignant lesion (a little less than 25%) and thelonger life span of children make many endocrinologists uncomfortable with observationalmanagement after a negative biopsy. The risk of malignancy is higher in boys who have thyroidnodules, but the general risk still is slightly less than 25% of all nodules in children. Any nodulethat is not removed should be monitored because an area of malignancy in a complex nodulecould have been missed.

References:

Amrikachi M, Ponder TB, Wheeler TM, Sith D, Ramzy I. Thyroid fine-needle aspiration biopsy inchildren and adolescents: experience with 218 aspirates. Diagn Cytopathol. 2005;32:189-192.Abstract available at: http://www.ncbi.nlm.nih.gov/pubmed/15754376

Hebra A, Miller M, Thomas PB. Solitary thyroid nodule. eMedicine Specialties, Pediatrics:General Medicine, Endocrinology. 2008. Available at:http://www.emedicine.com/ped/topic2120.htm

Hopwood NJ, Kelch RP. Thyroid masses: approach to diagnosis and management in childhoodand adolescence. Pediatr Rev. 1993;14:481-487. Abstract available at:http://pedsinreview.aappublications.org/cgi/content/abstract/14/12/481

Copyright © 2010 by the American Academy of Pediatrics page 543

Page 544: AAP PREP 2010

2010 PREP SA on CD-ROM

Question: 219

A healthy term newborn fails the universal hearing screening. Follow-up testing reveals that hehas a profound hearing loss. His parents inquire about the development of young children whohave congenital hearing loss.

Of the following, the BEST response is that

A. early amplification and developmental services improve overall language outcome

B. hearing aids promote language development only for children who have mild hearing loss

C. lip reading encourages the child to speak and understand language faster

D. prelingual deafness is associated with better cognitive outcome than postlingual deafness

E. severe or profound hearing loss is associated with impaired cognitive skills

Copyright © 2010 by the American Academy of Pediatrics page 544

Page 545: AAP PREP 2010

2010 PREP SA on CD-ROM

Preferred Response: ACritique: 219

Among newborns who have hearing loss, those who receive early amplification anddevelopmental services fare better than those who receive such services at a later date.Children who have postlingual deafness have a better cognitive outcome than those who haveprelingual deafness. Quickly developing spoken language is a challenge for the child who solelyuses lip reading. Children who have severe or profound hearing loss do not necessarily haveimpaired cognitive skills. The use of hearing aids promotes language development for childrenwho have varying degrees of hearing loss. Almost all children who have mild and moderatehearing loss and some who have severe hearing loss can develop spoken language with theearly and consistent use of personal hearing aids.

Children who have a congenital hearing loss may have delays in speech and languagedevelopment. One of the early signs of this is a delay in prespeech babble. Children who havecongenital hearing loss also may have decreased attention span and other behavioral difficultiesdue to their frustration with their inability to communicate.

References:

Feldman HM. Evaluation and management of language and speech disorders in preschoolchildren. Pediatr Rev. 2005;26:131-141. Available at:http://pedsinreview.aappublications.org/cgi/content/full/26/4/131

Gravel JS, O’Gara J. Communication options for children with hearing loss. Ment Retard DevDisabil Res Rev. 2003;9:243-251. Available at: http://www.ncbi.nlm.nih.gov/pubmed/14648817

Copyright © 2010 by the American Academy of Pediatrics page 545

Page 546: AAP PREP 2010

2010 PREP SA on CD-ROM

Question: 220

You are examining a 7-day-old infant in your office because his mother noticed a vesicular rashon his forehead (Item Q220) and on the scalp at the site of an external electrode placed duringlabor. The baby is feeding well, and he appears vigorous and afebrile. The mother denies ahistory of sexually transmitted infections.

Of the following, the MOST appropriate choice for initial therapy is

A. acyclovir intravenously

B. ampicillin and gentamicin intravenously

C. ganciclovir intravenously

D. valacyclovir orally

E. vancomycin and cefotaxime intravenously

Copyright © 2010 by the American Academy of Pediatrics page 546

Page 547: AAP PREP 2010

2010 PREP SA on CD-ROM

Preferred Response: ACritique: 220

Neonatal herpes simplex virus (HSV) infection may present as a sepsislike syndrome;localized central nervous system disease; or vesicular lesions localized to the skin, eyes, ormucous membranes. The virus typically is acquired during passage through an HSV-infectedbirth canal, although the mother is unaware of being infected in up to 70% of cases. Localizedvesicular lesions at a scalp electrode site several days to a week after birth, as described forthe infant in the vignette, are suggestive of localized HSV disease (Item C220). Treatment forHSV infection in neonates, even in a well-appearing child who has localized skin, eye, mucousmembrane disease, is parenteral (intravenous) acyclovir (60 mg/kg per day divided q 8 hours).Following intravenous acyclovir for neonatal disease, oral acyclovir may be considered forsuppressive therapy over the next 6 months to prevent skin and possibly central nervoussystem recurrences.

Although scalp lesions also might suggest bacterial infection, purulence (versus a clearervesicular appearance) is expected with bacterial infection. If there is any question about theappearance, a lesion can be unroofed and analyzed for bacteria (Gram stain, culture) and HSV(direct fluorescence antibody stain, viral culture). Even if bacterial infection is a consideration,however, ampicillin and gentamicin are not appropriate antibiotics because the combination doesnot provide adequate coverage for Staphylococcus aureus. Vancomycin and cefotaxime is agood combination to treat a bacterial infection, but the clinical description for the infant in thevignette is inconsistent with such an infection. Ganciclovir is an antiviral drug used for treatmentof cytomegalovirus infections in immunocompromised hosts. It does have activity against HSV,but it has increased toxicity compared with acyclovir. Valacyclovir is an oral prodrug ofacyclovir that has greater oral bioavailability and is converted to acyclovir in vivo afterabsorption. It is only available in an oral preparation and is not indicated for neonates. It has beenused in older children and adolescents as an alternative to oral acyclovir for treatment orsuppressive therapy of genital HSV, oral HSV, and herpes zoster infections. The increasedbioavailability allows for less frequent dosing compared with acyclovir.

Acyclovir has clinically useful activity against HSV type 1, HSV type 2, and varicella zostervirus (VZV). Beyond the neonatal period, acyclovir is indicated for treatment of primary genitaland possibly primary mucocutaneous HSV infections in immunocompetent children and adults. Italso has a role as suppressive therapy to prevent recurrences of these infections in childrenwho have frequent episodes. For immunosuppressed children, acyclovir is effective in thetreatment and prevention of mucocutaneous HSV disease. Intravenous acyclovir is the drug ofchoice for HSV encephalitis. Acyclovir also has activity against VZV, although at higher dosesthan for HSV. Intravenous acyclovir is indicated in immunosuppressed children who havevaricella or disseminated herpes zoster. If begun within 24 hours of onset of symptoms, oralacyclovir may be of benefit in select immunocompetent children who have varicella (eg, at least12 years of age, underlying skin or pulmonary disease, using intermittent or aerosolized steroids,unvaccinated household contact).

Acyclovir is generally well tolerated. Gastrointestinal symptoms (nausea, diarrhea, andvomiting) and malaise are the most commonly reported adverse events. Acute renal failure dueto precipitation of acyclovir in renal tubules has been reported rarely with intravenous acyclovir.Proper hydration and slow infusion over 1 to 2 hours can minimize this complication.

Copyright © 2010 by the American Academy of Pediatrics page 547

Page 548: AAP PREP 2010

2010 PREP SA on CD-ROM

References:

American Academy of Pediatrics. Herpes simplex. In: Pickering LK, Baker CJ, Kimberlin DW, LongSS, eds. Red Book: 2009 Report of the Committee on Infectious Diseases. 28th ed. Elk GroveVillage, Ill: American Academy of Pediatrics; 2009:363-373

American Academy of Pediatrics. Varicella-zoster infections. In: Pickering LK, Baker CJ,Kimberlin DW, Long SS, eds. Red Book: 2009 Report of the Committee on Infectious Diseases.28th ed. Elk Grove Village, Ill: American Academy of Pediatrics; 2009:714-727

Gutierrez K, Arvin AM. Long term antiviral suppression after treatment for neonatal herpesinfection. Pediatr Infect Dis J. 2003;22:371-372

Zachary KC. Acyclovir: an overview. UpToDate Online 16.3. 2007. Available for subscription at:www.utdol.com/online/content/topic.do?topicKey=viral_in/4905&view=print

Zachary KC. Valacyclovir: an overview. UpToDate Online16.3. 2007. Available for subscriptionat: www.utdol.com/online/content/topic.do?topicKey=viral_in/6580&view=print

Copyright © 2010 by the American Academy of Pediatrics page 548

Page 549: AAP PREP 2010

2010 PREP SA on CD-ROM

Question: 221

You are speaking to a group of medical students about infections that can result in outbreaks inthe community. A student asks you about the ways that a person can acquire a hepatitis Ainfection.

Of the following, the MOST common mode of hepatitis A transmission is

A. blood transfusion

B. casual contact

C. fecal-oral route

D. sexual contact

E. vertical transmission

Copyright © 2010 by the American Academy of Pediatrics page 549

Page 550: AAP PREP 2010

2010 PREP SA on CD-ROM

Preferred Response: CCritique: 221

Hepatitis A virus (HAV) infection has a worldwide distribution, and the age at infectionvaries with socioeconomic status and associated living conditions (eg, crowded livingconditions, poor hygiene). Among children younger than 6 years of age, the infection typically isasymptomatic or characterized by mild, nonspecific symptoms. For older children and adults,acute infection is symptomatic, and clinical symptoms may last from 2 weeks to several months,with jaundice occurring in 70% or more of patients.

HAV is an acute, self-limited infection that rarely has been associated with fulminantdisease, but not with chronic hepatitis. Common symptoms include fever, malaise, headache,nausea, vomiting, diarrhea, anorexia, and jaundice. It is highly contagious and spreads fromperson to person, most commonly by fecal contamination and oral ingestion. The most commonsource of infection is contact with an infected person or ingestion of contaminated water, food,or ice. Usually such exposure occurs under conditions of poor sanitation or poor hand washing,with travel to endemic areas of the world, or after ingestion of raw or undercooked shellfish(eg, oysters, clams, mussels) harvested from contaminated waters. Other important sources ofinfection are exposure to children in child care centers (especially in large centers and in thosethat have many children in diapers), foodborne or waterborne outbreaks, male homosexualactivity, and intravenous drug use. Transmission via sexual contact may occur but is lesscommon, and transmission by blood transfusion, casual contact, or from an infected mother to anewborn (vertical transmission) is rare. In about 50% of the cases, the source of infection isunknown. The incubation period ranges from 15 to 50 days, with an average of about 30 days.The highest titers of HAV in stool occur during the 1 to 2 weeks before the onset of illness, andthe highest period of communicability is 1 week before through 1 week after the onset ofsymptoms.

References:

American Academy of Pediatrics. Hepatitis A. In: Pickering LK, Baker CJ, Kimberlin DW, Long SS,eds. Red Book: 2009 Report of the Committee on Infectious Diseases. 28th ed. Elk GroveVillage, Ill: American Academy of Pediatrics; 2009:329-337

Bell BP, Anderson DA, Feinstone SM. Hepatitis A virus. In: Mandell GL, Bennett JE, Dolan R, eds.Mandell, Douglas, and Bennett’s Principles and Practice of Infectious Diseases. 6th ed.Philadelphia, Pa: Elsevier Churchill Livingstone; 2005:2162-2185

El-Kamary SS, Cheng TL. In brief: hepatitis A. Pediatr Rev. 2005;26:75-76. Available at:http://pedsinreview.aappublications.org/cgi/content/full/26/2/75

Kemmer NM, Miskovsky EP. Hepatitis A. Infect Dis Clin North Am. 2000;14:605-615. Abstractavailable at: http://www.ncbi.nlm.nih.gov/pubmed/10987112

Copyright © 2010 by the American Academy of Pediatrics page 550

Page 551: AAP PREP 2010

2010 PREP SA on CD-ROM

Question: 222

A 3-week-old infant presents for a health supervision visit. His mother is concerned that he onlyhas two wet diapers per day. On further questioning, the mother says that this infant has areduced urinary stream compared with her other two sons when they were infants. Shereports that he has been eating well without vomiting or diarrhea, and he has had no fever. Theinfant is breastfed. Findings on physical examination include a temperature of 37.0°C, respiratoryrate of 32 breaths/min, pulse of 110 beats/min, and blood pressure of 90/50 mm Hg. He hasfullness over the suprapubic region up to the umbilicus. Abdominal ultrasonography shows

bilateral hydronephrosis with bladder wall thickening. Laboratory findings reveal:

Serum values:

•Sodium, 126 mEq/L (126 mmol/L)

•Potassium, 5.6 mEq/L (5.6 mmol/L)

•Chloride, 100 mEq/L (100 mmol/L)

•Bicarbonate, 20 mEq/L (20 mmol/L)

•Blood urea nitrogen, 14.0 mg/dL (5.0 mmol/L)

•Creatinine, 0.4 mg/dL (35.4 mcmol/L)

Urine electrolytes:

•Sodium, 56 mEq/L (56 mmol/L)

•Potassium, 10 mEq/L (10 mmol/L)•Chloride, 80 mEq/L (80 mmol/L)

Of the following, the MOST likely explanation for hyponatremia in this infant is

A. excessive maternal water intake

B. maternal breast milk that is too low in sodium content

C. renal impairment in free water excretion

D. syndrome of inappropriate antidiuretic hormone secretion

E. urinary sodium losses from renal dysplasia

Copyright © 2010 by the American Academy of Pediatrics page 551

Page 552: AAP PREP 2010

2010 PREP SA on CD-ROM

Preferred Response: ECritique: 222

Hyponatremia is defined as a plasma sodium concentration less than 135 mEq/L (135mmol/L). In general, hyponatremia is characterized as a disproportionate concentration of waterto sodium, which can arise from either water retention (most cases) or sodium losses. It iscategorized as hypervolemic, euvolemic, and hypovolemic, depending on the total body sodiumcontent (Item C222).

Determination of the volume status in the patient who has hyponatremia allows the clinicianto delineate its cause and provide the most appropriate treatment. Measurement of urineelectrolytes, with special attention to urine sodium, can be helpful. In general, patients who havea hypervolemic cause (excluding those who have renal failure) have decreased effectivecirculating blood volume and, thus, have avid sodium and water retention, with a low urinesodium concentration. Those who have euvolemia (especially in the presence of the syndromeof inappropriate antidiuretic hormone secretion [SIADH]) tend to have urine sodiumconcentrations greater than 20 mEq/L (20 mmol/L). Hypovolemia is associated with extrarenalsodium losses and low urine sodium values (<20 mEq/L [20 mmol/L]) due to physiologiccompensatory mechanisms targeting the restoration of blood volume to normal.

The male infant described in the vignette has hyponatremia, with fullness on palpation in thesuprapubic region and bilateral hydronephrosis with bladder wall thickening on ultrasonography.The working diagnosis is posterior urethral valves, a condition associated with renal dysplasia.Moreover, the urine sodium concentration of 56 mEq/L (56 mmol/L) points toward urinary lossesof sodium, most likely from the renal dysplasia. Children who have marked urinary sodium lossesneed to receive sodium supplements to allow sufficient sodium for growth and avoidhyponatremia. Excessive maternal water intake, human milk that is too low in sodium content,renal impairment in free water excretion, and SIADH are not likely in this clinical setting.

References:

Farrell C, Del Rio M. In brief: hyponatremia. Pediatr. Rev. 2007;28:426-428. Available at:http://pedsinreview.aappublications.org/cgi/content/full/28/11/426

Rose BD, Post TW. Hypoosmolal states—hyponatremia. In: Clinical Physiology of Acid-base andElectrolyte Disorders. 5th ed. New York, NY: McGraw-Hill Medical Publishing Division; 2001:696-745

Copyright © 2010 by the American Academy of Pediatrics page 552

Page 553: AAP PREP 2010

2010 PREP SA on CD-ROM

Question: 223

You are evaluating a 12-year-old boy for a routine assessment prior to starting school. He isgenerally healthy, except for mild seasonal allergic rhinitis that is controlled with an as-neededsecond-generation antihistamine. However, his mother has noted that over the past 6 months hehas awakened coughing 1 to 2 nights a month. In addition, once or twice a month she hasnoticed that he "wheezes" during physical exertion, although he is still able to continue hisactivity. A pulmonary function test in the office reveals age-appropriate lung function with a

normal forced expiratory volume in 1 second (FEV1) (>80%) and normal FEV1/forced vitalcapacity ratio. You suspect asthma.

Of the following, the MOST appropriate classification for this boy’s asthma is

A. exercise-induced

B. intermittent

C. mild persistent

D. moderate persistent

E. severe persistent

Copyright © 2010 by the American Academy of Pediatrics page 553

Page 554: AAP PREP 2010

2010 PREP SA on CD-ROM

Preferred Response: BCritique: 223

The 2007 National Asthma Education and Prevention Program Expert Panel Report 3 forthe Diagnosis and Management of Asthma (ERP3) continues to provide a framework for thedetermination of asthma severity and the initiation of appropriate therapy. EPR3 advisesassessment of baseline asthma severity when initiating therapy, then monitoring asthma controlwhile adjusting stepwise therapy to prevent functional impairment and minimize the risk of anexacerbation (Item C223).

The new guidelines separate children into three groups: 0 through 4 years, 5 through 11years, and 12 years and older. Based on an assessment of the impairment and risk, the severityof asthma experienced by the 12-year-old boy described in the vignette is categorized asintermittent. The nomenclature has changed from mild intermittent to intermittent as cliniciansneed to recognize that intermittent symptoms may be mild, moderate, or severe. Characteristicsof intermittent asthma, as seen in the child in the vignette, include normal pulmonary function testresults, daytime symptoms fewer than 2 days per week, and nighttime symptoms fewer than 2days per month. Step 1 therapy for intermittent asthma involves using an as-needed short-actingbronchodilator. Once a child is diagnosed with asthma, the frequency of follow-up can be basedon asthma severity and level of control.

Exercise is a common trigger of asthma for children. Approximately 70% to 90% of childrenwho have asthma are believed to have symptoms of exercise-induced asthma (EIA). Althoughthe child in the vignette may have exercise-induced symptoms, a classification of EIA is lessaccurate than intermittent asthma because the child has symptoms outside of exercise (ie, atnight). Because up to 40% of children who have allergic rhinitis have some degree of asthmaand 60% to 80% of children who have asthma have allergic rhinitis, asking about other atopicdiseases is recommended.

References:

Liu AH, Covar RA, Spahn JD, Leung DYM. Childhood asthma. In: Kliegman RM, Behrman RE,Jenson HB, Stanton BF, eds. Nelson Textbook of Pediatrics. 18th ed. Philadelphia, Pa: SaundersElsevier; 2007:953-970

McFadden ER Jr. Exercise-induced airway narrowing. In: Adkinson NF Jr, Yunginger JW, BusseWW, Bochner BS, Holgate ST, Simons FER, eds. Middleton’s Allergy Principles and Practice.6th ed. Philadelphia, Pa: Mosby Elsevier; 2003:1323-1332

National Heart, Lung, and Blood Institute and the National Asthma Education and PreventionProgram. Expert Panel Report 3 (EPR3): Guidelines for the Diagnosis and Management ofAsthma. Bethesda, Md: U.S. Department of Health and Human Services; 2007. Available athttp://www.nhlbi.nih.gov/guidelines/asthma/asthgdln.htm

Stempel DA. Asthma and the athlete. In: Leung DYM, Sampson HA, Geha RS, Szefler SJ, eds.Pediatric Allergy Principles and Practice. St. Louis, Mo: Mosby Elsevier; 2003:435-443

Copyright © 2010 by the American Academy of Pediatrics page 554

Page 555: AAP PREP 2010

2010 PREP SA on CD-ROM

Question: 224

A 12-year-old gymnast presents to the office with a complaint of 2 months of lower back painthat has been worsening and now is interfering with her practice. She denies any trauma to herback and has no other complaints. She localizes the pain to the L5-S1 area of her lower backand is unable to arch backward comfortably because of pain. Findings on her neurologicexamination are normal.

Of the following, the MOST appropriate next step is to

A. obtain anteroposterior, oblique, and lateral radiographs of the lumbosacral spine

B. obtain magnetic resonance imaging of the lumbosacral spine

C. recommend bedrest for 2 weeks

D. recommend nonsteroidal anti-inflammatory drugs and continued practice

E. refer the girl to a physical therapist for back-strengthening exercises

Copyright © 2010 by the American Academy of Pediatrics page 555

Page 556: AAP PREP 2010

2010 PREP SA on CD-ROM

Preferred Response: ACritique: 224

Although most back pain in children and adolescents results from minor musculoskeletalinjuries, patients in these age groups are more likely than adults to have serious causes thatrequire specific treatment, including neoplasms, infection, or rheumatologic disease. For thisreason, a child presenting with back pain deserves careful evaluation to identify features thatmight indicate a significant underlying condition. The history should focus on the presence ofsystemic signs, location and quality of the pain, sports activities, and factors that exacerbate orrelieve the pain. Physical examination should include inspection and palpation of the spine;observation of spinal range of motion and gait; motor and sensory evaluations; and generalexamination of the abdomen, rectum, and lower extremities. The presence of systemic orneurologic symptoms or constant or worsening pain should prompt an in-depth evaluation thatmay involve laboratory or radiologic studies, as indicated by the initial data gathering.

The patient described in the vignette is exhibiting typical signs and symptoms ofspondylolisthesis. Spondylolisthesis is the anterior slippage of one lumbar vertebra on anotherdue to bilateral defects in the lumbar vertebral pars interarticularis (spondylolysis). Commonlyseen in young adolescent athletes who participate in sports that involve frequent lumbarhyperextension (eg, gymnastics, diving, butterfly stroke in swimming), signs and symptomsinclude vague lower back pain that is worsened with lumbar hyperextension, hamstringtightness, and limited forward bending. Neurologic impairment is rare.

Radiologic evaluation of spondylolisthesis should begin with anteroposterior, lateral, andoblique plain radiographs of the lumbar spine. In most cases, the pars articularis defects areevident, specifically on the oblique views. If plain films are negative, a bone scan or singlephoton emission computed tomography scan (Item C224A) and (Item C224B), which are moresensitive than magnetic resonance imaging in this setting, should be obtained. Treatment iscontroversial. Recommended interventions include limitation of vigorous aggravating activity(relative rest), bracing, and in severe cases, spinal fusion. Anti-inflammatory medication, bedrest, and back-strengthening exercises are not helpful because they do not alleviate theunderlying pathology.

References:

Masci L, Pike J, Malara F, Phillips B, Bennell K, Brukner P. Use of the one-legged hyperextensiontest and magnetic resonance imaging in the diagnosis of active spondylolysis. Br J Sports Med.2006;40:940-946. Abstract available at: http://www.ncbi.nlm.nih.gov/pubmed/16980534

Nigrovic PA. Evaluation of the child with back pain. UpToDate Online 16.3. 2008. Available at:http://www.utdol.com/online/content/topic.do?topicKey=gen_pedi/18246

Nigrovic PA. Overview of the causes of back pain in children and adolescents. UpToDateOnline 16.3. 2008. Available at:http://www.utdol.com/online/content/topic.do?topicKey=gen_pedi/18106

Spiegel DA, Hosalkar HS, Dormans JP. The spine. In: In: Kliegman RM, Behrman RE, Jenson HB,

Copyright © 2010 by the American Academy of Pediatrics page 556

Page 557: AAP PREP 2010

2010 PREP SA on CD-ROM

Stanton BF, eds. Nelson Textbook of Pediatrics. 18th ed. Philadelphia, Pa: Saunders Elsevier;2007:2811-2821

Copyright © 2010 by the American Academy of Pediatrics page 557

Page 558: AAP PREP 2010

2010 PREP SA on CD-ROM

Question: 225

You are seeing a 2-year-old girl for constipation of a few weeks’ duration. On physicalexamination, her height is at the 25th percentile for age, her weight is at the 5th percentile, shehas mild abdominal distention, and you palpate a tubular mass in the left lower quadrant. Yourecommend treatment with a polyethylene glycol-containing laxative and arrange for a follow-upappointment in 2 weeks. After 1 week, her mother calls to tell you that her stools are softer, butthat the girl has experienced three episodes of "something like her intestines" protruding fromher rectum after a bowel movement. Further, although her stools are softer, she continues topass large-caliber stools with some "straining."

Of the following, the MOST important test to obtain in further evaluating this child is

A. abdominal ultrasonography

B. barium enema

C. stool for ova and parasites

D. sweat chloride

E. tissue transglutaminase antibody measurement

Copyright © 2010 by the American Academy of Pediatrics page 558

Page 559: AAP PREP 2010

2010 PREP SA on CD-ROM

Preferred Response: DCritique: 225

Rectal prolapse involves the intussusception and exteriorization of rectal tissue. Prolapsedtissue may contain only the rectal mucosa or all tissue layers (mucosa, submucosa, andmuscularis) may be exposed. Overall, rectal prolapse is most common between 1 and 5 years ofage, with a mean incidence age of 3 years. It is a well-described complication in young childrenbeing treated for constipation. In fact, available evidence indicates that functional constipation, inthe absence of any underlying anatomic or metabolic disorder, is the most common cause ofrectal prolapse in childhood, accounting for up to 30% of cases. Although the presentationdescribed for the girl in the vignette appears to be relatively typical, two "red flags" should pointthe astute clinician to an alternate explanation for this problem. First, the child’s growthpercentiles demonstrate a significant height/weight discrepancy. By itself, in the absence ofprior growth data and family history, this is not necessarily an alarming finding, but the secondfactor is that several episodes of prolapse have occurred following the production a soft stool.Accordingly, a high index of suspicion in this and in all cases of recurrent rectal prolapse inchildhood must be given to cystic fibrosis (CF). Therefore, a sweat chloride test is the mostimportant study to obtain for this patient. Ruling out CF in this setting is of particular importancebecause it represents a common heritable disorder in the United States, with a gene frequencyof approximately 1:29 in the white population. Overall incidence figures indicate that CF affectsapproximately 1 in 3,500 children of Caucasian descent, with the incidence in other populationsof approximately 1 in 11,500 (Hispanics), 1 in 20,000 (African Americans) and 1 in 90,000(Asians).

CF accounts for approximately 10% of all reported cases of rectal prolapse during infancyand early childhood. Collected case series suggest that rectal prolapse occurs in 20% ofpatients who have CF between the ages of 6 months and 3 years, and it may be the presentingsign of disease for many patients. When prolapse occurs in infants younger than 1 year of age,the likelihood of an underlying CF diagnosis is even higher. Other diagnostic possibilities forrectal prolapse include acute diarrhea and neuromotor and connective tissue disorders(especially Ehlers-Danlos syndrome); up to 20% of patients have no identifiable cause.

Rectal prolapse is a potential complication for any child who has an acute or chronicdiarrheal illness, including intestinal infections and malabsorptive disorders. Heavy parasiticinfestations of enterobiasis (pinworm), amebiasis, and trichuriasis (whipworm, a rare infectionin the United States) have been associated with rectal prolapse. Therefore, examining stool forova and parasites may be indicated in selected cases. Celiac disease should be considered(tissue transglutaminase antibody measurement) in any child who exhibits poor weight gain andbulky stools, but rectal prolapse has not been reported to be a common complication in affectedpatients. Hirschsprung disease rarely has been reported as a cause of prolapse, but theconstipation of only several weeks’ duration and the spontaneous passage of soft stoolsfollowing oral polyethylene glycol therapy make such a diagnosis unlikely for this child.Therefore, a barium enema examination is not indicated. Finally, abdominal ultrasonographygenerally is not helpful in the evaluation of constipation or rectal prolapse.

References:

Copyright © 2010 by the American Academy of Pediatrics page 559

Page 560: AAP PREP 2010

2010 PREP SA on CD-ROM

American Lung Association. Lung Disease Data at a Glance: Cystic Fibrosis (CF). Available at:http://www.lungusa.org/site/pp.asp?c=dvLUK9O0E&b=320330

Davis PB. Cystic fibrosis. Pediatr Rev. 2001;22:257-264. Available at:http://pedsinreview.aappublications.org/cgi/content/full/22/8/257

Pietzak MM, Thomas DW. Childhood malabsorption. Pediatr Rev. 2003;24:195-206. Available at:http://pedsinreview.aappublications.org/cgi/content/full/24/6/195

Siafakas C, Vottler TP, Andersen JM. Rectal prolapse in pediatrics. Clin Pediatr (Phila).1999;38:63-72. Abstract available at: http://www.ncbi.nlm.nih.gov/pubmed/10047938

Sondheimer JM. Other anorectal disorders. In: Rudolph CD, Rudolph AM, Hostetter MK, Lister G,Siegel NJ, eds. Rudolph’s Pediatrics. 21st ed. New York, NY: McGraw-Hill Medical PublishingDivision; 2003:1459-1461

Copyright © 2010 by the American Academy of Pediatrics page 560

Page 561: AAP PREP 2010

2010 PREP SA on CD-ROM

Question: 226

A 30-weeks’ gestation very low-birthweight (VLBW) 1,400-g infant has respiratory distresssyndrome (RDS). He is receiving assisted ventilation following administration of three doses ofexogenous pulmonary surfactant. On his second postnatal day, his mother asks if she canbreastfeed her infant. You explain that she will not be able to breastfeed until he is extubatedand able to suckle. She asks you whether she should pump (express) her breast milk.

Of the following, the BEST response is that expressed human milk feedings

A. are contraindicated in VLBW infants who have RDS

B. are too difficult for VLBW infants to digest

C. can be fed by nasogastric tube

D. have no net benefit for VLBW infants once frozen

E. have too much protein for VLBW infants

Copyright © 2010 by the American Academy of Pediatrics page 561

Page 562: AAP PREP 2010

2010 PREP SA on CD-ROM

Preferred Response: CCritique: 226

Preterm and very low-birthweight (VLBW) infants who have respiratory distress syndrome(RDS) require medically assisted nutrition due not only to their illness but also because of theirneurologic immaturity. Oral suckling and coordinated respiration are developmentally challenginguntil most preterm infants attain a postconceptual age of 32 to 34 weeks. Until that time, mothersof such infants should be encouraged to express and store their milk for future use. Whendeemed appropriate, generally by the third postnatal day, most VLBW preterm infants (includingthose who have RDS) can be offered a small volume of milk feedings via nasogastric tube tostimulate gastrointestinal function. Such small trophic or priming volumes (generally only 10mL/kg per day total) are not relied upon for nutrition, which is provided parenterally.Nonetheless, infants ultimately benefit from receiving human milk (even if frozen) throughenhanced immune system function, improved tolerance of feedings and ease of digestion, areduction in the incidence of necrotizing enterocolitis, and improved neurodevelopmentaloutcomes and intelligence quotients.

Many preterm infants require supplementation to their mother’s milk because preterm humanmilk is low in total protein and certain minerals. In addition, those who have increased caloricneeds, due to RDS or chronic lung disease, may need supplementation of the expressed humanmilk with fortifiers (liquid or solid). Human milk fortifiers provide additional protein, calories,calcium, phosphorous and other minerals, and vitamins to meet the specific needs of preterminfants. Iron supplements should be provided to preterm infants receiving only human milk oncethey attain the goal volume of enteral feeding.

References:

Amaizu N, Shulman R, Schanler R, Lau C. Maturation of oral feeding skills in preterm infants. ActaPaediatr. 2008;97:61-67. Abstract available at: http://www.ncbi.nlm.nih.gov/pubmed/18052999

Gewolb IH, Vice FL. Abnormalities in the coordination of respiration and swallow in preterminfants with bronchopulmonary dysplasia. Dev Med Child Neurol. 2006;48:595-599. Abstractavailable at: http://www.ncbi.nlm.nih.gov/pubmed/16780630

Nyqvist KH. Early attainment of breastfeeding competence in very preterm infants. ActaPaediatr. 2008;97:776-781. Abstract available at:http://www.ncbi.nlm.nih.gov/pubmed/18460108

Raab EL. The resuscitation & care of the newborn at risk. In: DeCherney AH, Nathan L, eds.CURRENT Diagnosis & Treatment Obstetrics & Gynecology. 10th ed. New York, NY: TheMcGraw-Hill Company, Inc; 2007:Chapter 31. Available for subscription at: http://www.accessmedicine.com/content.aspx?aID=2387621

Thilo EH, Rosenberg AA. The newborn infant. In: Hay WW Jr, Levin MJ, Sondheimer JM,Deterding RR, eds. CURRENT Diagnosis & Treatment: Pediatrics. 19th ed. New York, NY: TheMcGraw-Hill Companies; 2009:Chapter 1. Available for subscription at:

Copyright © 2010 by the American Academy of Pediatrics page 562

Page 563: AAP PREP 2010

2010 PREP SA on CD-ROM

http://www.accessmedicine.com/content.aspx?aID=3396500

Vice FL, Gewolb IH. Respiratory patterns and strategies during feeding in preterm infants. DevMed Child Neurol. 2008;50:467-472. Abstract available at:http://www.ncbi.nlm.nih.gov/pubmed/18422676

Copyright © 2010 by the American Academy of Pediatrics page 563

Page 564: AAP PREP 2010

2010 PREP SA on CD-ROM

Question: 227

You are working in the emergency department when a mother brings in her 8-month-old son.She is concerned because he has had diarrhea for 2 months that has worsened over the lastday. She explains that the stool is greasy, but there is no blood. He has had two episodes ofsinusitis, but no hospitalizations. They are new to town and he has not seen a physician sincehis 2-month health supervision visit. On physical examination, the boy appears pale, cachectic,and mildly dehydrated, but alert. He has nasal congestion, his lungs are clear, and findings onhis abdominal examination are normal. His weight is below the 5th percentile, length is at the 10thpercentile, and head circumference is at the 25th percentile. He has a foul-smelling, greasy stoolin his diaper.

Of the following, the test MOST likely to reveal the diagnosis is

A. enzyme-linked immunosorbent assay for Giardia

B. serology for antigliadin antibodies

C. stool evaluation for alpha-1-antitrypsin

D. stool evaluation for ova, cysts, and parasites

E. sweat test

Copyright © 2010 by the American Academy of Pediatrics page 564

Page 565: AAP PREP 2010

2010 PREP SA on CD-ROM

Preferred Response: ECritique: 227

Cystic fibrosis is an autosomal recessive disorder characterized primarily by chronicpulmonary disease and pancreatic insufficiency. A defect in chromosome 7 causes abnormalregulation of the cystic fibrosis transmembrane conductance regulator (CFTR), which transportschloride in luminal areas such as airways, intestines, sweat ducts, and pancreatic ducts. Fluidsecretions become thickened, leading to multisystem mucus plugging. Varying degrees of clinicalfeatures may be seen because of the many mutations that can occur in the CFTR gene. Theclinical features that can be present include recurrent upper and lower respiratory tractinfections (especially due to Pseudomonas aeruginosa, Staphylococcus aureus, andHaemophilus influenzae), steatorrhea (from fat malabsorption) and meconium ileus,cholelithiasis, and male infertility. Deficiencies of vitamins A, D, E, and K are present because offat malabsorption. Failure to thrive is a common presenting feature because of chronicmalabsorption and frequent infections. Laboratory testing may reveal hypoproteinemia, anemia,and hypochloremic metabolic alkalosis. Because the boy described in the vignette has featuresthat are most consistent with cystic fibrosis, a sweat chloride determination (sweat test) is themost appropriate test to make the diagnosis.

A sweat test involves the application of pilocarpine to the skin to stimulate sweat production,followed by measurement of the chloride concentration in the sweat. A positive result is greaterthan 60 mEq/L. False-positive results may be seen in Addison disease, hypothyroidism, andsome glycogen storage diseases. False-negative results may be seen if the infant ismalnourished or if inadequate sweat volume is obtained, as in the neonatal period or withtechnical errors. Treatment of cystic fibrosis includes the administration of pancreatic enzymes,increased caloric intake (especially from fat), intensive antibiotic therapy for respiratoryinfections, and aggressive airway clearance regimens. Inhaled antibiotics, such as tobramycin,used every other month have been shown to reduce the incidence of pulmonary infections.

Giardia infections are caused by a protozoan that can be transmitted by the ingestion ofcysts from contaminated water. Clinical features include watery diarrhea, abdominal pain, andbloating. Steatorrhea and respiratory symptoms are not seen. The diagnosis may be made eitherby viewing the cysts in the stool or by immunosorbent assay. Antigliadin antibodies are helpful inmaking the diagnosis of celiac disease, which causes chronic diarrhea and failure to thrive afterthe introduction of gluten to the diet. Alpha-1-antitrypsin concentrations may be elevated in thestool if a protein-losing enteropathy is present. This may be seen in cystic fibrosis, as well asother diseases such as allergic gastroenteropathy and inflammatory bowel disease, but thefinding is not diagnostic of cystic fibrosis. Stool should be evaluated for ova, cysts, andparasites if infections such as cryptosporidiosis are suspected.

References:

Davis PB. Cystic fibrosis. Pediatr Rev. 2001;22:257-264. Available at:http://pedsinreview.aappublications.org/cgi/content/full/22/8/257

Ghishan FK. Chronic diarrhea. In: Kliegman RM, Behrman RE, Jenson HB, Stanton BF, eds.Nelson Textbook of Pediatrics. 18th ed. Philadelphia, Pa: Saunders Elsevier; 2007:1621-1626

Copyright © 2010 by the American Academy of Pediatrics page 565

Page 566: AAP PREP 2010

2010 PREP SA on CD-ROM

Question: 228

A 15-year-old boy is brought to the emergency department after a crash involving an all-terrainvehicle. His immunization history is not available. Physical examination reveals an open fractureof his tibia and fibula. You stabilize the fracture and ensure hemodynamic stability.

Of the following, the MOST appropriate tetanus prophylaxis to administer is

A. adult-type tetanus toxoid and diphtheria toxoid vaccine (Td) alone

B. tetanus immune globulin (TIG) alone

C. TIG and adult-type tetanus toxoid, reduced diphtheria toxoid, and acellular pertussis vaccine(Tdap)

D. TIG and diphtheria toxoid, tetanus toxoid, and acellular pertussis vaccine (DTaP)

E. TIG and Td

Copyright © 2010 by the American Academy of Pediatrics page 566

Page 567: AAP PREP 2010

2010 PREP SA on CD-ROM

Preferred Response: CCritique: 228

Tetanus prophylaxis for an acute wound varies, depending on the type of wound and theimmunization history of the patient. Because of widespread and effective tetanus immunizationin the United States, clinical tetanus is rare in those who have completed the initial immunizationseries. However, humoral immunity to tetanus does wane with time. Also, because Clostridiumtetani is a ubiquitous soil organism, contaminated wounds in individuals whose immunizationstatus has not been updated recently or is unknown, as described for the boy in the vignette,are of concern.

The most appropriate tetanus prophylaxis to administer to the boy in the vignette, who has aheavily contaminated penetrating wound, is tetanus immunoglobulin (TIG) and adult-type tetanustoxoid, reduced diphtheria toxoid, and acellular pertussis vaccine (Tdap). TIG is indicated whena patient who has a contaminated wound has not yet completed the initial tetanus immunizationseries or if the immunization status is unknown, regardless of age. Using adult-type tetanustoxoid and diphtheria toxoid vaccine (Td) or TIG alone is not recommended. Diphtheria toxoid,tetanus toxoid, and acellular pertussis vaccine (DTaP) or diphtheria toxoid, tetanus toxoid (DT)vaccine should not be used because DT and DTaP have higher concentrations of diphtheriatoxoid and may cause more serious local or systemic vaccine reactions in older children andadults than adult type diphtheria vaccine, which is included in Tdap. TIG alone is not appropriatebecause it provides only passive immunity. Although Td and TIG could be used to protect againsttetanus, the increasing prevalence of pertussis in the adolescent age group makes it desirable toadminister Tdap instead of Td.

As a result of reviewing this information, do you intend to make a change in practiceto provide better patient care?Yes No

References:

American Academy of Pediatrics Committee on Infectious Diseases. Prevention of pertussisamong adolescents: recommendations for use of tetanus toxoid, reduced diphtheria toxoid, andacellular pertussis (Tdap) vaccine. Pediatrics. 2006;117:965-978. Available at:http://pediatrics.aappublications.org/cgi/content/full/117/3/965

American Academy of Pediatrics. Tetanus (lockjaw). In: Pickering LK, Baker CJ, Kimberlin DW,Long SS, eds. Red Book: 2009 Report of the Committee on Infectious Diseases. 28th ed. ElkGrove Village, Ill: American Academy of Pediatrics; 2009:655-660

Sagerman PJ. Wounds. Pediatr Rev. 2005;26:43-49. Available at:http://pedsinreview.aappublications.org/cgi/content/full/26/2/43

Copyright © 2010 by the American Academy of Pediatrics page 567

Page 568: AAP PREP 2010

2010 PREP SA on CD-ROM

Question: 229

You are evaluating a 2-month-old infant who has stridor. His mother states that the stridor isworse during and immediately after feeding. She has thickened his feedings, and he has beenprescribed antireflux medications. On physical examination, he appears comfortable and in nodistress. His respiratory and cardiac examination findings are benign. As you observe hisfeeding, he develops stridor.

Of the following, the MOST appropriate next step in this patient’s evaluation is

A. barium swallow

B. computed tomography scan of the chest

C. electrocardiography

D. radiography of the neck

E. sweat test

Copyright © 2010 by the American Academy of Pediatrics page 568

Page 569: AAP PREP 2010

2010 PREP SA on CD-ROM

Preferred Response: ACritique: 229

A number of clinical entities are responsible for stridor in children, but when stridoraccompanies feeding, as described for the infant in the vignette, vascular ring is an importantpotential cause. Vascular rings occur when the vascular structures of the aorta and pulmonaryartery completely encircle the trachea and the esophagus (Item C229A). Virtually every vascularring can lead to a similar set of clinical findings. For most affected patients, the intracardiacanatomy, physiology, and hemodynamics are normal. When symptoms do occur, they typicallyresult from compression of the trachea or the esophagus. The most common symptoms relatedto tracheal compression are stridor, wheezing, and dyspnea, all of which may be exacerbatedby feeding. The age of onset and type of symptoms depend on the anatomic type and severityof the vascular ring. If the anomaly is unrecognized, patients may have recurrent pneumonia andprogressive difficulty with feeding.

Radiography of the chest may be helpful and often can identify the sidedness of the aorticarch, which is defined by the mainstem bronchus that it crosses. However, barium swallow isthe most useful study (Item C229B). When a vascular ring is suspected, both the posterior-anterior and lateral views should be obtained, and fluoroscopy should be considered.

Computed tomography scan can be a useful diagnostic study for the patient who has stridorand a vascular ring, but it exposes the young patient to a much higher amount of ionizingradiation than does barium swallow (Item C229C). Because most patients who have vascularrings have normal intracardiac anatomy, electrocardiography is not helpful in determining thecause of the stridor. Radiography of the neck and the sweat test are not diagnostic for avascular ring.

References:

Epstein ML. Vascular rings and slings. In: Moller JH, Hoffman JIE, eds. Pediatric CardiovascularMedicine. Philadelphia, Pa: Churchill Livingstone; 2000:642-648

Humphrey C, Duncan K, Fletcher S. Decade of experience with vascular rings at a singleinstitution. Pediatrics. 2006;117:e903-e908. Available at:http://pediatrics.aappublications.org/cgi/content/full/117/5/e903

Seilheimer DK, Fakhoury KF. Pulmonology. In: Garson A Jr, Bricker JT, Fisher DJ, Neish SR, eds.The Science and Practice of Pediatric Cardiology. 2nd ed. Baltimore, Md: Williams & Wilkins;1998:2813-2832

Copyright © 2010 by the American Academy of Pediatrics page 569

Page 570: AAP PREP 2010

2010 PREP SA on CD-ROM

Question: 230

A 25-month-old girl who has neurodevelopmental delay is brought to the emergency departmentvia ambulance. She had appeared well the entire day, with no signs of illness, but atapproximately 8 pm, her mother saw the child start to bend over and then become unconscious,her left arm began jerking, and her eyes rolled back. Her father quickly picked up the limp child,who had some continued jerking of her left arm. The jerking did not involve her legs or face. Hereyes were deviated to the left, her breathing was irregular, and she had some blueness aroundher lips. The twitching lasted less than 1 minute. After about 15 minutes, she began to havesome visual responsiveness, but it was several hours before she started to act normally. Onphysical examination in the emergency department, she has a temperature of 39.5°C and a rightotitis media. Within 4 hours of administration of an antipyretic, she is afebrile and has returned tobaseline status. The father states his brother had "fever seizures" in childhood.

Of the following, the factor that increases this child’s risk of later epilepsy is

A. family history of febrile seizures

B. high temperature at seizure onset

C. neurodevelopmental abnormality

D. simple febrile seizures

E. young age of onset

Copyright © 2010 by the American Academy of Pediatrics page 570

Page 571: AAP PREP 2010

2010 PREP SA on CD-ROM

Preferred Response: CCritique: 230

The 2-year-old child described in the vignette has had a seizure that is characterized byabrupt cessation of normal behavior, jerking movements, eye deviation, loss of consciousnessfor about 1 minute, and subsequent sleepiness. Because the seizure occurred in a childyounger than the age of 6 years, she had no prior history of afebrile seizures or other acuteprovoking factors, and the seizure occurred at the onset of a febrile illness, the diagnosis is afebrile seizure. Febrile seizures are further categorized as simple or complex. If the seizures arebrief (fewer than 15 minutes), isolated, and generalized, they are considered simple febrileseizures. Because this child’s seizure was focal, it is considered a complex febrile seizure.

Only a small percentage of children who have febrile seizures subsequently developrecurrent nonfebrile seizures or epilepsy. Risk factors identified by the National CollaborativePerinatal Project include abnormal neurologic development and a first febrile seizure that iscomplex. A family history of epilepsy may increase the risk. A family history of febrile seizuresand young age of onset increase the risk for future febrile seizures, but not for epilepsy.Children who have simple febrile seizures or a high temperature with a febrile seizure do nothave an increased risk of epilepsy. Electroencephalography results after febrile seizures do notpredict future epilepsy risk.

References:

Duffner PK, Baumann RJ. A synopsis of the American Academy of Pediatrics' practiceparameters on the evaluation and treatment of children with febrile seizures. Pediatr Rev.1999;20:285-287. Available at: http://pedsinreview.aappublications.org/cgi/content/full/20/8/285

Fetveit A. Assessment of febrile seizures in children. Eur J Pediatr. 2008;167:17-27. Abstractavailable at: http://www.ncbi.nlm.nih.gov/pubmed/17768636

Nelson KB, Ellenberg JH. Predictors of epilepsy in children who have experienced febrileseizures. N Engl J Med. 1976;295:1029-1033. Abstract available at:http://www.ncbi.nlm.nih.gov/pubmed/972656

Waruiru C, Appleton R. Febrile seizures: an update. Arch Dis Child. 2004;89:751-756. Abstractavailable at: http://www.ncbi.nlm.nih.gov/pubmed/15269077

Copyright © 2010 by the American Academy of Pediatrics page 571

Page 572: AAP PREP 2010

2010 PREP SA on CD-ROM

Question: 231

A medical student whom you are supervising in clinic presents an 8-year-old girl who has ahistory of unilateral, unifocal retinoblastoma (RB) diagnosed and successfully treated in infancy.The child is in the clinic for a health supervision visit. Her parents recently learned that they areexpecting a baby. No one in their family, other than their daughter, has a history of RB. Thestudent asks about the recurrence risk for RB.

Of the following, the MOST accurate statement regarding RB recurrence for this couple is that

A. if neither parent had RB, recurrence risk is negligible

B. the recurrence risk is higher for boys than girls

C. this couple has a 25% recurrence risk

D. this couple has a 50% recurrence risk

E. unilateral, unifocal RB has a low recurrence risk

Copyright © 2010 by the American Academy of Pediatrics page 572

Page 573: AAP PREP 2010

2010 PREP SA on CD-ROM

Preferred Response: ECritique: 231

Retinoblastoma (RB) is a malignant tumor that occurs in the developing retina, usually beforethe age of 5 years. Germline mutations in the RB gene (RB1), which encodes a tumorsuppressor, are passed on as autosomal dominant traits and predispose to RB. In these cases,RB is caused when a mutation occurs in the second copy of the RB1 gene after conception.Conversely, RB may arise spontaneously when mutations occur in both copies of the RB1 geneafter conception. RB also can be caused by germline mosaicism for an RB1 mutation in theparent of the proband. The recurrence risk for RB is dependent upon multiple variables, includingwhether the proband has a germline mutation in RB1, whether there is gonadal mosaicism, andthe family history and tumor type.

Unifocal RB refers to the presence of a single RB tumor. Often, if the RB is large, it is notpossible to determine if more than one tumor is present in that eye. Unilateral RB sometimescarries a lower recurrence risk than bilateral disease. In the case of unifocal, unilateral RBwithout a positive family history, as for the family described in the vignette, the parents’ empiricrecurrence risk is approximately 1%. Nonetheless, the proband should undergo gene testing,and if a mutation is found, the parents also should be tested to ascertain the recurrence risk.

If parents are unaffected, they may not carry mutations in the RB1 gene, but nonpenetrance(a mutation is present but not expressed) is well described in carriers of germline mutations.This is important to recognize to avoid giving inaccurate information to families. RB occurs withequal frequency in males and females.

References:

Abramson DH, Beaverson K, Sangani P, et al. Screening for retinoblastoma: presenting signs asprognosticators of patient and ocular survival. Pediatrics. 2003;112:1248-1255. Available at:http://pediatrics.aappublications.org/cgi/content/full/112/6/1248

Lohmann DR, Gallie BL. Retinoblastoma. GeneReviews. 2007. Available at:http://www.ncbi.nlm.nih.gov/bookshelf/br.fcgi?book=gene&part=retinoblastoma

Copyright © 2010 by the American Academy of Pediatrics page 573

Page 574: AAP PREP 2010

2010 PREP SA on CD-ROM

Question: 232

You are seeing a 15-year-old girl for her annual health supervision visit. Her menarche occurredat age 12 years, and she had normal monthly menses over the first 2 years. In the last year,however, her periods became progressively more irregular and stopped 4 months ago. Hermother notes that the girl has been very health-conscious since she entered puberty. She hasgained no weight over the last 3 years and is on the cross-country team at school. On physical

examination, her body mass index is 17 kg/m2, her heart rate is 55 beats/min, she has no acneor hirsutism, and she is at Sexual Maturity Rating 5 genital development. The remainder of thephysical examination findings are normal.

Of the following, the MOST likely cause of this girl’s amenorrhea is

A. ergogenic agents

B. exercise regimen

C. heart disease

D. physiologic anovulation

E. school stress

Copyright © 2010 by the American Academy of Pediatrics page 574

Page 575: AAP PREP 2010

2010 PREP SA on CD-ROM

Preferred Response: BCritique: 232

The American College of Sports Medicine coined the term "the female athlete triad" in 1992. Itcomprises three interrelated components: disordered eating, amenorrhea, and osteoporosis. Therisk of the disorder is greatest among those participating in endurance sports. Athletes aredistributed along a spectrum between health and disease, and those at the pathologic end maynot exhibit all of the components simultaneously. The girl described in the vignette hasamenorrhea that is related to her level of exercise and inadequate nutrition.

Nutrition issues underlie most of the pathophysiology of the female athlete triad. Energyavailability is defined as dietary energy intake minus exercise energy expenditure. Exercise-induced amenorrhea can be an indicator of decreased energy availability that may beinadvertent, intentional, or psychopathological. Bone density loss results from the low estrogenenvironment and is concerning because 50% of adult skeletal mass is laid down duringadolescence, with peak bone mass attained between ages 18 and 25 years.

The first aim of treatment for any triad component is to increase energy availability byincreasing energy intake, reducing exercise energy expenditure, or both. Nutrition counselingand monitoring are sufficient for many athletes, but significantly disordered eating warrantsmore intensive intervention.

Education of athletes, parents, coaches, trainers, judges of competitions, and administratorsis a priority for prevention and early intervention. Athletes should be assessed for the triadduring preparticipation physical examination or the annual health screening examination andwhenever an athlete presents with any of the triad's symptoms or signs. Sports administratorsshould consider rule changes to discourage unhealthy weight loss practices. Athletes whohave eating disorders should be required to meet established weight criteria to continueexercising, and their training and competition may need to be modified.

Ergogenic aids are dietary supplements used to enhance athletic performance. The mostcommonly used are those that have supposed anabolic effects because they mimic the effectsof steroids and are legal for use. Creatine is the most widely used such supplement taken byboth professional and recreational athletes. It causes weight gain from muscle hypertrophy andfluid retention. It does not alter vital signs or cause menstrual changes. The girl described in thevignette has no symptoms referable to the cardiovascular system; her low heart rate likely is aresult of her inadequate nutrition and exercise. In the first 2 years following menarche, irregularmenses may be the result of immaturity of the hypothalamic pituitary axis (physiologicanovulation). However, this girl’s regular menses in the first 2 years make physiologicanovulation a less likely cause of the amenorrhea. Psychological stress in an adolescent maycause amenorrhea, but the history and examination findings of this girl are most consistent withamenorrhea resulting from exercise.

References:

Bloomfield D. Secondary amenorrhea. Pediatr Rev. 2006;27:113-114. Available at:http://pedsinreview.aappublications.org/cgi/content/full/27/3/113

Cavanaugh RM Jr. Evaluating adolescents with fatigue: ever get tired of it? Pediatr Rev.

Copyright © 2010 by the American Academy of Pediatrics page 575

Page 576: AAP PREP 2010

2010 PREP SA on CD-ROM

2002;23:337-348. Available at: http://pedsinreview.aappublications.org/cgi/content/full/23/10/337

Lerand SJ, Williams JF. The female athlete triad. Pediatr Rev. 2006;27:e12-e13. Available at:http://pedsinreview.aappublications.org/cgi/content/full/27/1/e12

Metzl JD. Preparticipation examination of the adolescent athlete: part 1. Pediatr Rev. 2001;22:199-204. Available at: http://pedsinreview.aappublications.org/cgi/content/full/22/6/199

Metzl JD. Preparticipation examination of the adolescent athlete: part 2. Pediatr Rev. 2001;22:227-239. Available at: http://pedsinreview.aappublications.org/cgi/content/full/22/7/227

Nattiv A, Loucks AB, Manore MM, Sanborn CF, Sundgot-Borgen J, Warren MP; American Collegeof Sports Medicine. American College of Sports Medicine position stand. The female athlete triad.Med Sci Sports Exerc. 2007;39:1867-1882. Abstract available at:http://www.ncbi.nlm.nih.gov/pubmed/17909417

Copyright © 2010 by the American Academy of Pediatrics page 576

Page 577: AAP PREP 2010

2010 PREP SA on CD-ROM

Question: 233

You have been treating a 5-year-old boy in the pediatric intensive care unit who has beenreceiving mechanical ventilation for 10 days due to pneumonia caused by Staphylococcusaureus. He required chest tube placement for an empyema. Over the past 3 days, he hasdeveloped a persistent air leak, and you are concerned that he may have developed abronchopleural fistula.

In discussions with his parents, a TRUE statement regarding bronchopleural fistulas is that

A. bronchopleural fistulas are not associated with increased mortality

B. bronchoscopy may be useful for both diagnosis and treatment

C. computed tomography scan is of little value in establishing the diagnosis

D. surgical therapy is rarely successful

E. they are likely to heal spontaneously within 12 to 24 hours

Copyright © 2010 by the American Academy of Pediatrics page 577

Page 578: AAP PREP 2010

2010 PREP SA on CD-ROM

Preferred Response: BCritique: 233

Bacterial pneumonia is a significant cause of morbidity and mortality worldwide, especially indeveloping countries. Most cases of pneumonia in the United States are treated on an outpatientbasis. The incidence of complicated pneumonias has increased, due in part to the increasingprevalence of both antibiotic-resistant and more invasive pathogens. Complications ofpneumonia include pleural effusions, empyema, lung abscess, necrotizing pneumonia,pericarditis, pneumothoraces, bronchopleural fistulas, and hematogenous spread to other sites.

Bronchopleural fistulas, the development of a connection between the bronchial tree andpleural space, are rare but are associated with a high rate of morbidity and mortality (rates of30% to 70% have been reported). Most commonly, they follow a pulmonary resection, but theycan occur with necrotizing bacterial pneumonia or as a result of persistent pneumothoraces.Bronchopulmonary fistulas generally are recognized in the intensive care unit patient by thepresence of a persistent air leak, as described for the boy in the vignette. Such fistulas rarelyheal without intervention, and they often prove difficult to manage. Frequently the diagnosis isclinical, but they are localized definitively with bronchoscopy. Chest computed tomography scanhas shown promise in both diagnosing and planning surgical correction of complicated cases.Management strategies include insertion or continuation of chest tubes, decreasing mechanicalventilator support to the lowest pressures needed to allow spontaneous healing, differential lungventilation, application of sealants through a bronchoscope, and surgical closure. Surgicalsuccess rates of 80% to 95% rate have been reported in adults, depending on the cause.

References:

Durbin WJ, Stille C. Pneumonia. Pediatr Rev. 2008;29:147-160. Available at:http://pedsinreview.aappublications.org/cgi/content/full/29/5/147

Lois M, Noppen M. Bronchopleural fistulas: an overview of the problem with special focus onendoscopic management. Chest. 2005;128:3955-3965. Available at:http://www.chestjournal.org/cgi/content/full/128/6/3955

Schultz KD, Fan LL, Pinksy J, et al. The changing face of pleural empyemas in children:epidemiology and management. Pediatrics. 2004;113:1735-1740. Available at:http://pediatrics.aappublications.org/cgi/content/full/113/6/1735

Sectish TC, Prober CG. Pneumonia. In: Kliegman RM, Behrman RE, Jenson HB, Stanton BF, eds.Nelson Textbook of Pediatrics. 18th ed. Philadelphia, Pa: Saunders Elsevier; 2007:1795-1799

Copyright © 2010 by the American Academy of Pediatrics page 578

Page 579: AAP PREP 2010

2010 PREP SA on CD-ROM

Question: 234

The parents of a 10-year-old girl in whom you have just diagnosed type 1 diabetes mellitus andchronic lymphocytic thyroiditis (Hashimoto thyroiditis) tell you that many people in their familyhave these conditions. They wish to know if other autoimmune disorders occur with greaterfrequency in children who have diabetes. You tell them that additional autoimmune disorders inchildren who have type 1 diabetes mellitus can occur.

Of the following, the autoimmune disorders MOST likely to occur in this patient are

A. Addison disease and premature ovarian failure

B. celiac disease and Addison disease

C. Graves disease and alopecia areata

D. pernicious anemia and celiac disease

E. vitiligo and pernicious anemia

Copyright © 2010 by the American Academy of Pediatrics page 579

Page 580: AAP PREP 2010

2010 PREP SA on CD-ROM

Preferred Response: BCritique: 234

Type 1 diabetes mellitus (DM1) may be associated with the development of otherautoimmune disorders, with 10% to 25% of affected children developing chronic lymphocyticthyroiditis, approximately 6% developing celiac disease, and 1% or fewer developing primaryadrenal insufficiency (Addison disease). Premature ovarian failure, vitiligo, alopecia areata, andpernicious anemia also may occur, but are much less common.

The underlying mechanism for the autoimmune destruction that leads to theseendocrinopathies or to gluten enteropathy (celiac disease) is not yet understood. Theassociation only rarely is related to the autosomal recessive monogenic disorder of theautoimmune regulator (AIRE) gene that causes autoimmune polyglandular syndrome (APS)-1,and it is not clear that the disorder in which DM1 is the initial endocrinopathy is the same as APS-2, a possibly dominantly inherited but variably penetrant disorder also seen with otherendocrinopathies.

References:

Anderson MS. Update in endocrine autoimmunity. J Clin Endocrinol Metab. 2008;93:3663-3670.Abstract available at: http://www.ncbi.nlm.nih.gov/pubmed/18842982

Ergun-Longmire B, Kukreja A, Maclaren N. Chapter 6. Autoimmune endocrine disorders.Endotext.org. 2008. Available at: http://www.endotext.org/pediatrics/pediatrics6/pediatrics6.html

Freemark M, Levitsky LL. Screening for celiac disease in children with type 1 diabetes: twoviews of the controversy. Diabetes Care. 2003;26:1932-1939. Available at:http://care.diabetesjournals.org/cgi/content/full/26/6/1932

Levitsky LL, Misra M. Associated autoimmune diseases in children and adolescents with type 1diabetes mellitus. UpToDate Online 16.3. 2008. Available for subscription at:http://www.utdol.com/online/content/topic.do?topicKey=pediendo/19209

Copyright © 2010 by the American Academy of Pediatrics page 580

Page 581: AAP PREP 2010

2010 PREP SA on CD-ROM

Question: 235

A mother and father bring in their infant for a routine health supervision visit. The mother gentlyplaces the child on the examination table, and the child laughs out loud. As you approach thechild, he squeals and reaches for his father.

Of the following, these developmental milestones are MOST typical for a child whose age is

A. 2 months

B. 4 months

C. 6 months

D. 8 months

E. 10 months

Copyright © 2010 by the American Academy of Pediatrics page 581

Page 582: AAP PREP 2010

2010 PREP SA on CD-ROM

Preferred Response: BCritique: 235

The infant described in the vignette exhibits the normal cognitive/behavioral developmentalmilestones for 4 months of age: laughing out loud, squealing, and initiating social interactions.Infants of this age show interest in the environment by looking at or listening to stimuli forapproximately 3 seconds.

The normal cognitive/behavioral developmental milestones for a 2-month-old infant are tosmile on social contact, listen to voice, and coo. A 6-month-old child responds to socialovertures by displaying an emotional response of pleasure or a negative affect. An 8-month-oldchild babbles, enjoys mirrors, and responds to changes in emotional or social content. At 10months of age, a child responds to the sound of his or her name, plays peek-a-boo or patty-cake, and waves bye-bye. In addition, the infant may reach up when the parent bends to pickhim or her up and may use hand movements to request a toy or to be picked up.

References:

Coplan, J. Child development in the first 21 months. In: Parker S, Zuckerman B, Augustyn M, eds.Developmental and Behavioral Pediatrics: A Handbook for Primary Care. 2nd ed. Philadelphia,Pa: Lippincott Williams & Wilkins; 2005:437-438

Feigelman S. The first year. In: Kliegman RM, Behrman RE, Jenson HB, Stanton BF, eds. NelsonTextbook of Pediatrics. 18th ed. Philadelphia, Pa: Saunders Elsevier; 2007:43-47

First Signs web site. Available at: http://www.firstsigns.org

Greenspan S. Socioemotional development of infants and children: themes and behaviors. In:Parker S, Zuckerman B, Augustyn M, eds. Developmental and Behavioral Pediatrics: AHandbook for Primary Care. 2nd ed. Philadelphia, Pa: Lippincott Williams & Wilkins; 2005:447-450

Copyright © 2010 by the American Academy of Pediatrics page 582

Page 583: AAP PREP 2010

2010 PREP SA on CD-ROM

Question: 236

A 17-year-old young man who has acquired immune deficiency syndrome has been diagnosedwith cytomegalovirus retinitis. You are planning to initiate therapy with intravenous ganciclovir.His parents ask about risks associated with this therapy.

Of the following, the MOST likely adverse effect of ganciclovir treatment is

A. anemia

B. lymphopenia

C. mucositis

D. neutropenia

E. peripheral neuropathy

Copyright © 2010 by the American Academy of Pediatrics page 583

Page 584: AAP PREP 2010

2010 PREP SA on CD-ROM

Preferred Response: DCritique: 236

Ganciclovir is indicated for treatment of cytomegalovirus (CMV) retinitis inimmunocompromised hosts, including those who have symptomatic human immunodeficiencyvirus infection. Neutropenia, with a risk for secondary invasive bacterial infection, is the primaryadverse effect of ganciclovir therapy. Anemia and thrombocytopenia also are seen in patientsreceiving ganciclovir but may be related to their underlying conditions as well as the drug.Neutropenia is considered the major hematologic toxicity associated with the drug. Lymphopeniaand mucositis may occur in immunocompromised patients who coincidentally receive ganciclovir,but such findings have not been shown to be related to ganciclovir. Neurologic adverse effectsassociated with ganciclovir include headaches, seizures, encephalopathy, dizziness, andhallucinations but not peripheral neuropathy.

Ganciclovir also is recommended for prevention of CMV infection or reactivation intransplant recipients in conjunction with or without CMV-intravenous immune globulin and hasbeen used to treat CMV colitis and CMV pneumonitis in immunodeficient patients. The drug isadministered primarily intravenously; an oral preparation is available, but absorption is very poor.An intraocular sustained-release formulation may be used for CMV retinitis, but this requiressurgical administration, with replacement every 6 months.

A trial of ganciclovir in infants infected congenitally with CMV suggested potential benefitbased on decreased progression of hearing loss in treated infants, but in view of the need forlong-term intravenous therapy in this setting (6 weeks) and significant potential drug toxicities,the drug is not presently indicated in this disease.

References:

American Academy of Pediatrics. Antiviral drugs. In: Pickering LK, Baker CJ, Kimberlin DW, LongSS, eds. Red Book: 2009 Report of the Committee on Infectious Diseases. 28th ed. Elk GroveVillage, Ill: American Academy of Pediatrics; 2009:777-782

American Academy of Pediatrics. Cytomegalovirus infection. In: Pickering LK, Baker CJ, KimberlinDW, Long SS, eds. Red Book: 2009 Report of the Committee on Infectious Diseases. 28th ed.Elk Grove Village, Ill: American Academy of Pediatrics; 2009:275-280

Kimberlin DW, Lin CY, Sánchez PJ, et al; National Institute of Allergy and Infectious DiseasesCollaborative Antiviral Study Group. Effect of ganciclovir therapy on hearing in symptomaticcongenital cytomegalovirus disease involving the central nervous system: a randomizedcontrolled trial. J Pediatr. 2003;143:16-25. Abstract available at:http://www.ncbi.nlm.nih.gov/pubmed/12915819

Zachary KC. Ganciclovir: an overview. UpToDate Online 16.3. 2006. Available for subscriptionat:http://www.utdol.com/online/content/topic.do?topicKey=viral_in/8159&selectedTitle=4~148&source=search_result

Copyright © 2010 by the American Academy of Pediatrics page 584

Page 585: AAP PREP 2010

2010 PREP SA on CD-ROM

Question: 237

A 6-month-old boy is brought to the emergency department for evaluation of a 2-day history of atemperature to 39.8°C, increasing irritability, constant crying, decreased activity, and emesis. Hismother states that everyone in the house has been ill with colds and that the baby has had arunny nose and nasal congestion for the past week. He has only had his 2-month set ofimmunizations because he has been ill each time he is brought in for his vaccines. On physicalexamination, the infant has a temperature of 40.0°C, appears ill, and is extremely irritable. Hisanterior fontanelle is full, and he has a stiff neck. Studies obtained on his cerebrospinal fluid

show:

•Glucose, 5.0 mg/dL

•Protein, 170.0 mg/dL

•White blood cell count, 550/mm3

•2 red blood cellsGram stain is positive for many white blood cells and gram-positive cocci.

Of the following, the MOST appropriate empiric antibiotic regimen for the treatment of this patientis

A. ampicillin and gentamicin

B. high-dose ceftriaxone alone

C. high-dose ceftriaxone and gentamicin

D. high-dose ceftriaxone and vancomycin

E. high-dose cefuroxime

Copyright © 2010 by the American Academy of Pediatrics page 585

Page 586: AAP PREP 2010

2010 PREP SA on CD-ROM

Preferred Response: DCritique: 237

Over the past several decades, an increasing proportion of Streptococcus pneumoniaestrains worldwide have been shown to be nonsusceptible to penicillin and ampicillin; to the third-generation cephalosporins (cefotaxime and ceftriaxone); and to a number of other agents,including erythromycin, macrolides, clindamycin, and trimethoprim-sulfamethoxazole in someareas of the United States. As many as 50% or more of S pneumoniae strains are penicillin-nonsusceptible, with about 60% of these being intermediately susceptible and 40% being fullyresistant. In general, due to the presence of cross-resistance, the susceptibility of thepneumococcus to other antibiotics decreases in parallel to decreasing penicillin susceptibility.Therefore, penicillin-nonsusceptible strains have increased rates of resistance to the third-generation cephalosporins, erythromycin, macrolides, clindamycin, and trimethoprim-sulfamethoxazole. However, the number of nonsusceptible isolates has been decreasing due tothe administration of the heptavalent pneumococcal conjugate vaccine.

It is important to recognize that beta-lactam antibiotics (penicillins and cephalosporins)generally are clinically effective in treating the nonsusceptible S pneumoniae strains, except inthe case of meningitis, if adequate doses are provided to maintain adequate drug concentrationsat the site of infection. For proven or suspected bacterial meningitis, the initial treatment involvesa combination of vancomycin and a third-generation parenteral cephalosporin (cefotaxime orceftriaxone). For children who cannot receive cephalosporins due to a serious cephalosporin orpenicillin allergy (eg, hives or anaphylaxis), rifampin is added to the vancomycin regimen. Oncesusceptibility testing results are available, the treatment regimen can be tailored accordingly.

Recommendations for therapy of pneumococcal infections based on the site and type ofinfection are shown in Item C237.

The patient described in the vignette has meningitis that most likely is due to S pneumoniae.Appropriate empiric therapy for a person who has suspected meningitis is a combination ofvancomycin and a third-generation parenteral cephalosporin (cefotaxime or ceftriaxone).

As a result of reviewing this information, do you intend to make a change in practiceto provide better patient care?Yes No

References:

American Academy of Pediatrics. Pneumococcal infections. In: Pickering LK, Baker CJ, KimberlinDW, Long SS, eds. Red Book: 2009 Report of the Committee on Infectious Diseases. 28th ed.Elk Grove Village, Ill: American Academy of Pediatrics; 2009:524-535

Durbin WJ. Pneumococcal infections. Pediatr Rev. 2004;25:418-424. Available at:http://pedsinreview.aappublications.org/cgi/content/full/25/12/418

Musher DM. Streptococcus pneumoniae. In: Mandell GL, Bennett JE, Dolan R, eds. Mandell,Douglas, and Bennett’s Principles and Practice of Infectious Diseases. 6th ed. Philadelphia, Pa:Elsevier Churchill Livingstone; 2005:2392-2411

Copyright © 2010 by the American Academy of Pediatrics page 586

Page 587: AAP PREP 2010

2010 PREP SA on CD-ROM

Pilishvili T, Noggle B, Moore MR. Pneumococcal disease.In: Manual for the Surveillance ofVaccine-preventable Diseases. 4th ed. Atlanta, Ga: Centers for Disease Control and Prevention;2008:11-1–11-12. Available at: http://www.cdc.gov/vaccines/pubs/surv-manual/chpt11-pneumo.pdf

Copyright © 2010 by the American Academy of Pediatrics page 587

Page 588: AAP PREP 2010

2010 PREP SA on CD-ROM

Question: 238

A 7-year-old boy who has a history of seizures presents with headaches and increasedconfusion. His complex partial seizures are being treated with carbamazepine. Physicalexamination reveals a weight of 34 kg (50th percentile), with all other findings within normal

limits, including results of the neurologic examination and funduscopy. Laboratory tests reveal:

•Sodium, 126 mEq/L (126 mmol/L)

•Potassium, 4.6 mEq/L (4.6 mmol/L)

•Chloride, 90 mEq/L (90 mmol/L)

•Bicarbonate, 24 mEq/L (24 mmol/L)

•Blood urea nitrogen, 14.0 mg/dL (5.0 mmol/L)

•Creatinine, 0.7 mg/dL (61.9 mcmol/L)Urinalysis shows a specific gravity of 1.030; pH of 5.5; and negative findings for blood, protein,ketones, nitrite, and leukocyte esterase.

Of the following, the MOST appropriate treatment for the patient’s condition is to

A. administer demeclocycline

B. administer 3% sodium chloride solution to raise the sodium to 130 mEq/L (130 mmol/L)

C. induce a water diuresis with chlorothiazide

D. initiate intravenous fluids with 0.9% sodium chloride

E. restrict free water intake

Copyright © 2010 by the American Academy of Pediatrics page 588

Page 589: AAP PREP 2010

2010 PREP SA on CD-ROM

Preferred Response: ECritique: 238

The child described in the vignette presents with confusion, hyponatremia, and a urinalysisfeaturing a high specific gravity, all findings consistent with a diagnosis of the syndrome ofinappropriate antidiuretic hormone secretion (SIADH). His antiepileptic medication,carbamazepine, is associated with the development of SIADH. SIADH results in hyponatremiabecause ADH is released in response to stimuli (eg, central nervous system disease, pulmonarydisease, or medications) instead of the physiologic cues aimed at normalizing serum osmolality(sodium) or plasma volume. The state of hyponatremia is caused by water retention from ADHeffect. The transient volume expansion activates atrial natriuretic peptide, leading to urinarysodium losses and restoring the patient to the euvolemic state. This natriuresis also contributesto the hyponatremia seen in SIADH and prevents the development of clinical edema.

The preferred treatment of the child who has SIADH is free water restriction. Othertreatments include administration of sodium chloride and furosemide. Thiazide diuretics maydecrease serum sodium further due to their effects on the cortical diluting segment. For childrenwho fail to respond to fluid restriction, other therapy may be recommended. Demeclocycline andlithium both directly block the effects of ADH on the collecting tubules. Demeclocycline is atetracycline derivative that is only recommended for children older than 8 years of age. Lithiumgenerally is not recommended because of its adverse effects. The use of 3% sodium chloridegenerally is indicated only for symptomatic hyponatremia. However, it may be used in theabsence of symptoms when the serum sodium is less than 120 mEq/L (120 mmol/L). There is noindication for 0.9% sodium chloride because this patient does not have hypovolemia.

References:

Ellison DH, Berl T. Clinical practice. The syndrome of inappropriate antidiuresis. N Engl J Med.2007;356:2064-2072. Extract available at: http://content.nejm.org/cgi/content/extract/356/20/2064

Rose BD, Post TW. Hypoosmolal states-hyponatremia. In: Clinical Physiology of Acid-base andElectrolyte Disorders. 5th ed. New York, NY: McGraw-Hill Medical Publishing Division; 2001:696-743

Van Amelsvoort T, Bakshi R, Devaux CB, Schwabe S. Hyponatremia associated withcarbamazepine and oxcarbazepine therapy: a review. Epilepsia. 1994;35:181-188. Abstractavailable at: http://www.ncbi.nlm.nih.gov/pubmed/8112243

Copyright © 2010 by the American Academy of Pediatrics page 589

Page 590: AAP PREP 2010

2010 PREP SA on CD-ROM

Question: 239

A 5-year-old girl presents with a 1-week history of coughing, wheezing, and chest tightness.She only has 1 week of school left before summer break, but has missed the last 2 days ofschool because of her symptoms. Her parents remark that their daughter usually receives oralsteroids for "reactive airway disease" two to three times a year during a viral upper respiratorytract infection. On physical examination, the child appears comfortable but coughs frequently.Her temperature is 37.3°C, respiratory rate is 20 breaths/min, and pulse oximetry result is 96%on room air. Bilateral expiratory wheezes are audible on her lung evaluation.

Of the following, the MOST likely cause of this girl’s symptoms is

A. coronavirus

B. influenza virus

C. parainfluenza virus

D. respiratory syncytial virus

E. rhinovirus

Copyright © 2010 by the American Academy of Pediatrics page 590

Page 591: AAP PREP 2010

2010 PREP SA on CD-ROM

Preferred Response: ECritique: 239

Viral upper respiratory tract infections (URIs) represent the most common trigger for asthmain children. Despite the terminology, URIs not only affect the upper airway but also result inlower airway inflammation. Specifically, rhinovirus, which is the most common URI virus, alsohas been identified as the most common virus in the lower airways of children who haveasthma exacerbations. Moreover, seasonal peaks of rhinovirus have correlated with emergencydepartment visits and hospitalizations for asthma exacerbations. In addition, rhinovirus is themost frequently identified virus in children hospitalized for wheezing episodes outsiderespiratory syncytial virus (RSV) season after age 3 years.

In temperate climates in the northern hemisphere, the yearly epidemic of colds begins inSeptember and continues until spring. The epidemic begins with a sharp increase in thefrequency of rhinovirus infections in September (after children return to school), which isfollowed by the appearance of parainfluenza viruses in October and November. RSV andcoronaviruses circulate during early winter months; infection due to influenza virus peaks in thelate winter. The epidemic finally ends with a small resurgence of rhinovirus infections in thespring. Adenovirus infection occurs at a constant rate throughout the cold season.

Any evaluation of a child who has asthma should include reporting of asthma symptoms,frequency of symptoms, medications, previous hospitalizations or intensive care unitadmissions, and triggers. Asthma triggers include physical exertion, cold or dry air,aeroallergens, poor air quality (eg, ozone, sulfur dioxide), chronic sinusitis, gastroesophagealreflux, and cigarette smoke.

References:

Liu AH, Covar RA, Spahn JD, Leung DYM. Childhood asthma. In: Kliegman RM, Behrman RE,Jenson HB, Stanton BF, eds. Nelson Textbook of Pediatrics. 18th ed. Philadelphia, Pa: SaundersElsevier; 2007:953-970

Mahr TA, Sheth K. Update on allergic rhinitis. Pediatr Rev. 2005;26:284-289. Available at:http://pedsinreview.aappublications.org/cgi/content/full/26/8/284

Mcintosh K. Rhinovirues. In: Kliegman RM, Behrman RE, Jenson HB, Stanton BF, eds. NelsonTextbook of Pediatrics. 18th ed. Philadelphia, Pa: Saunders Elsevier; 2007:1395-1396

Copyright © 2010 by the American Academy of Pediatrics page 591

Page 592: AAP PREP 2010

2010 PREP SA on CD-ROM

Question: 240

A 14-year-old boy is brought to the emergency department by friends after becoming confusedand disoriented while they were "hanging out." On physical examination, he is sleepy but easilyarousable, his heart rate is 60 beats/min, respiratory rate is 18 breaths/min, and blood pressureis 120/60 mm Hg. His pupils are 4 mm and sluggishly reactive to 2 mm. No other findings on hisphysical examination are notable. One hour after arrival at the emergency department, he isawake and alert and has normal vital signs.

Of the following, the MOST likely cause for this adolescent’s altered mental status is

A. cerebral contusion

B. encephalitis

C. gasoline inhalation

D. jimson weed ingestion

E. lysergic acid diethylamide (LSD) ingestion

Copyright © 2010 by the American Academy of Pediatrics page 592

Page 593: AAP PREP 2010

2010 PREP SA on CD-ROM

Preferred Response: CCritique: 240

The patient described in the vignette is exhibiting signs and symptoms consistent with anacute inhalant exposure. Inhalants include volatile hydrocarbons such as gasoline, glue, lighterfluid, and typewriter correction fluid as well as nitrite compounds, such as nitrous oxide andamyl nitrate. They can be inhaled directly from a container, a saturated cloth, or a plastic bag.Because inhalants are highly lipophilic, they are absorbed rapidly into brain tissue, where theyproduce a transient euphoria followed by short-lived depressive symptoms such as lethargy,lightheadedness, sluggish pupillary responses, and bradycardia. At high doses, confusion anddelirium may result. After a single use, symptoms resolve within 15 to 30 minutes, although mostabusers use inhalants repetitively to sustain the effects. Repetitive or chronic use has beenassociated with fatal dysrhythmias due to increased myocardial catecholamine sensitivity,psychosis, dementia, peripheral neuropathy, bone marrow suppression, and a perioral dermatitisknown as "glue-sniffer’s rash," caused by the drying effects of the hydrocarbons around thenose and mouth. Inhalants are among the most commonly abused drugs among adolescents,especially in the southeastern and southwestern United States.

Although encephalitis and cerebral contusions can lead to depressed mental status, suchsymptoms from these conditions generally do not resolve quickly. Jimson weed ingestion causesanticholinergic symptoms such as dilated pupils, hypertension, tachycardia, and dry and flushedskin. Lysergic acid diethylamide ingestion leads to stimulant effects and hallucinations.

References:

Crocetti M, Serwint JR. In brief: inhalants. Pediatr Rev. 2008;29:33-34. Available at:http://pedsinreview.aappublications.org/cgi/content/full/29/1/33

Endom EE. Inhalant abuse in children and adolescents. UpToDate Online 16.3. 2008. Availableat:http://www.utdol.com/online/content/topic.do?topicKey=ped_tox/6962&selectedTitle=1~22&source=search_result

Williams JF, Storck M, and the Committee on Substance Abuse and Committee on NativeAmerican Child Health. Inhalant abuse. Pediatrics. 2007;119:1009—1017. Available at:http://pediatrics.aappublications.org/cgi/content/full/119/5/1009

Copyright © 2010 by the American Academy of Pediatrics page 593

Page 594: AAP PREP 2010

2010 PREP SA on CD-ROM

Question: 241

A 9-year-old boy who has cystic fibrosis (delta-F508 homozygous) presents with a complaint ofabdominal pain. He reports that he has had two to three small bowel movements per week overthe past few weeks. Six months ago, because of problems with weight gain, you had increasedhis dosage of oral pancreatic enzymes to 50,000 lipase units per meal and 10,000 lipase unitsper snack. On physical examination, he weighs 25 kg, appears uncomfortable, and hasmoderate abdominal distention and a palpable right-sided abdominal mass.

Of the following, the MOST likely diagnosis is

A. appendiceal abscess

B. Crohn disease

C. distal intestinal obstruction syndrome

D. fibrosing colonopathy

E. functional constipation

Copyright © 2010 by the American Academy of Pediatrics page 594

Page 595: AAP PREP 2010

2010 PREP SA on CD-ROM

Preferred Response: CCritique: 241

Distal intestinal obstruction syndrome (DIOS), formerly referred to as meconium ileusequivalent, is a common gastrointestinal complication in patients who have cystic fibrosis (CF). Itis of particular concern among those for whom intestinal maldigestion/malabsorption, a problemthat invariably is associated with the delta-F508 homozygous state, remains poorly controlled.DIOS is suggested for the boy in the vignette by the findings of decompensated malabsorption,constipation, and abdominal discomfort and distention. The physical finding of a right-sidedabdominal mass further supports this diagnosis.

CF is the most common heritable disorder among whites in the United States, with a genefrequency of approximately 1 in 29 in this population. Overall incidence figures indicate that CFaffects approximately 1 in 3,500 children of Caucasian descent, with the incidence in otherpopulations of approximately 1 in 11,500 (Hispanics), 1 in 20,000 (African Americans) and 1 in90,000 (Asians). It is transmitted in an autosomal recessive pattern, with heterozygotes beingunaffected. Although more than 2,000 CF gene mutations have been described, approximately70% of CF alleles in the United States demonstrate the delta-F508 mutation on chromosome 7,where a phenylalanine residue is deleted at position 508 of a 1,480-amino acid protein.Approximately 50% of CF patients are homozygous for this mutation, which is associated withsevere disease. The consequent metabolic defect involves mutation in the cystic fibrosistransmembrane conductance regulator (CFTR), resulting in abnormal chloride and watersecretion across epithelial surfaces of all exocrine organs. This causes abnormally thick andviscid secretions, leading to dysfunction in multiple systems, including the lung, liver, pancreas,and gastrointestinal tract.

DIOS results from inspissation of viscous mucus and fecal material in the ileum, cecum, andascending colon. It is a relatively common problem in CF, reported in 10% to 20% of patients.DIOS occurs with greatest frequency in older children and adolescents. Predisposing factors(apart from the primary CFTR defect) include poorly controlled fat malabsorption, previousmeconium ileus, low dietary fiber intake, and possibly prolonged intestinal transit time.Interestingly, although DIOS has been reported in CF patients who retain relatively normalpancreatic exocrine function, the syndrome has not been reported in other forms of pancreaticinsufficiency, likely because of the contribution of abnormal luminal fluid content in CF.

Acute management of DIOS involves the use of a variety of bowel-cleansing agents,depending on the degree of involvement. Intestinal lavage with a balanced electrolyte solutioncontaining polyethylene glycol, use of oral N-acetylcysteine as a mucolytic agent, and high-dosediatrizoate meglumine and diatrizoate sodium (via both oral and rectal instillation) have beenemployed successfully. The particular cleansing protocol often varies with the institution.Chronic management should include optimizing fat absorption plus the use of cathartic agentsand appropriate intake of fiber.

Although DIOS is a relatively common gastrointestinal complication of CF, other problemsmust be considered in any patient who has CF and presents with gastrointestinal complaints.Some of the more prevalent, age-related problems are listed in Item C241. Presenting signs andsymptoms of these disorders are similar to those in patients who do not have CF, and theclinician caring for patients who have CF must be aware of their occurrence in this patientgroup.

Copyright © 2010 by the American Academy of Pediatrics page 595

Page 596: AAP PREP 2010

2010 PREP SA on CD-ROM

A diagnosis of appendiceal abscess or Crohn disease is unlikely in this case because of theabsence of associated symptoms of right lower quadrant abdominal pain, fever, or diarrhea.Functional constipation may exhibit one or more of the clinical findings described for this child,but the relatively recent onset in a child who has no history of stool withholding behavior,coupled with his malabsorptive state and increase in pancreatic enzyme requirement, points toDIOS as the most likely diagnosis, particularly considering its importance and frequency inpatients who have CF.

Fibrosing colonopathy is an uncommon complication of pancreatic enzyme supplementation.This iatrogenic disorder results in colonic stricturing, often requiring surgical resection, and isassociated with pancreatic enzyme supplementation in excess of 24,000 lipase units/kg per day.The risk of fibrosing colonopathy was particularly high when pancreatic enzymes wereprovided in a high-strength microencapsulated form (>20,000 lipase units per capsule), which nolonger is marketed. Due to enzyme reformulation that reduced the lipase content per capsule andcurrent recommendations limiting maximal lipase supplements to 10,000 enzyme units/kg per day,the incidence of fibrosing colonopathy has been reduced dramatically.

References:

American Lung Association. Lung Disease Data at a Glance: Cystic Fibrosis (CF).

Chaudry G, Navarro OM, Levine DS, Oudjhane K. Abdominal manifestations of cystic fibrosis inchildren. Pediatr Radiol. 2006;36:233-240. Abstract available at:http://www.ncbi.nlm.nih.gov/pubmed/16391928

Clifton IJ, Morton AM, Ambrose NS, Peckham DG, Conway SP. Treatment of resistant distalintestinal obstruction syndrome with a modified antegrade continence enema procedure. J CystFibros. 2004;3:273-275. Abstract available at: http://www.ncbi.nlm.nih.gov/pubmed/15698947

Davis PB. Cystic fibrosis. Pediatr Rev. 2001;22:257-264. Available at:http://pedsinreview.aappublications.org/cgi/content/full/22/8/257

FitzSimmons SC, Burkhart GA, Borowitz D, et al. High-dose pancreatic-enzyme supplements andfibrosing colonopathy in children with cystic fibrosis. N Engl J Med. 1997;336:1283-1289.Available at: http://content.nejm.org/cgi/content/full/336/18/1283

Koletzko S, Stringer DA, Gleghorn GJ, Durie PR. Lavage treatment of distal intestinal obstructionsyndrome in children with cystic fibrosis. Pediatrics. 1989;83:727-733. Available at:http://pediatrics.aappublications.org/cgi/content/abstract/83/5/727

Proesmans M, De Boeck K. Evaluation of dietary fiber intake in Belgian children with cysticfibrosis: is there a link with gastrointestinal complaints? J Pediatr Gastroenterol Nutr.2002;35:610-614. Abstract available at: http://www.ncbi.nlm.nih.gov/pubmed/12454573

Copyright © 2010 by the American Academy of Pediatrics page 596

Page 597: AAP PREP 2010

2010 PREP SA on CD-ROM

Rubinstein S, Moss R, Lewiston N. Constipation and meconium ileus equivalent in patients withcystic fibrosis. Pediatrics. 1986;78:473-479. Abstract available at:http://pediatrics.aappublications.org/cgi/content/abstract/78/3/473

Copyright © 2010 by the American Academy of Pediatrics page 597

Page 598: AAP PREP 2010

2010 PREP SA on CD-ROM

Question: 242

The mother of a healthy infant in the newborn nursery notes that her daughter has a good abilityto focus when held close but not while lying in bed. She asks you whether her child really isseeing her.

Of the following, your BEST response is that infants

A. cannot see colors at birth

B. do not have a conjugate gaze at birth

C. focus best on a facial construct

D. focus only on their mothers’ faces

E. have visual acuity equivalent to a 1-year-old child

Copyright © 2010 by the American Academy of Pediatrics page 598

Page 599: AAP PREP 2010

2010 PREP SA on CD-ROM

Preferred Response: CCritique: 242

The visual acuity in a term newborn is approximately 20/200. Throughout the first 4 to 8months of postnatal life, vision improves dramatically and reaches 20/30 by age 1 year. If shownan object or provided a light stimulus, the newborn may demonstrate any of a number of

responses, including:

•Staring

•Cessation of movement of the arms and legs

•Blinking

•Fixation and visually following the object brieflyAlternatively, the infant may have no response.

Infants fixate best on the construct of the human (eg, parent or examiner) face but also mayfixate on a graphically depicted face or a bright red object. The usual gaze is conjugate, andcolor detection is apparent at greater distances than closer distances (18 to 24 inches). Thesharper the contrast of opposing lines or figures, the greater the newborn’s ability to detect it.Gradations of shading are not appreciated well by newborns, but this aspect of vision improvesover the first 2 to 3 postnatal months.

The newborn’s ability to focus and attend to a face or object is best within 12 to 24 inches(30 to 70 cm) of his or her own eyes. The infant’s ability to show signs of attention or "see"objects at both closer and more distant lengths is reduced markedly during the first 2 postnatalmonths. Although memory for the mother’s face develops over time at a rate that is quicker thanfor a stranger’s face, newborn infants do, indeed, focus on a stranger’s face.

References:

Bushnell IWR. Mother's face recognition in newborn infants: learning and memory. Infant andChild Development. 2001;10:67-74. Abstract available at:http://www3.interscience.wiley.com/journal/79502688/abstract

Lissauer T. Physical examination of the newborn. In: Martin RJ, Fanaroff AA, Walsh MC, eds.Fanaroff and Martin's Neonatal-Perinatal Medicine: Diseases of the Fetus and Infant. 8th ed.Philadelphia, Pa: Mosby Elsevier; 2006:513-528

Ricci D, Cesarini L, Groppo M, et al. Early assessment of visual function in full term newborns.Early Hum Dev.2008;84:107-113. Abstract available at:http://www.ncbi.nlm.nih.gov/pubmed/17513071

Copyright © 2010 by the American Academy of Pediatrics page 599

Page 600: AAP PREP 2010

2010 PREP SA on CD-ROM

Question: 243

You are the physician for the high school baseball team. While fielding a ground ball in the firstinning, the shortstop is hit in the left eye with the baseball. He immediately returns to the dugoutfor you to examine him. He has edema of the eyelid and a large subconjunctival hemorrhagelateral to the iris. His extraocular movements are full, but he complains of pain when you assessthem. He says he can see "just fine" and wants to return to the game.

Of the following, the MOST appropriate management of this boy’s injury is

A. immediate referral to the hospital emergency department

B. instillation of fluorescein to check for a corneal abrasion

C. irrigation of the eye with normal saline

D. observation and return to play if vision is normal after 1 hour

E. tight eye patching and ophthalmologic referral in the morning

Copyright © 2010 by the American Academy of Pediatrics page 600

Page 601: AAP PREP 2010

2010 PREP SA on CD-ROM

Preferred Response: ACritique: 243

Eye injuries are common in children and adolescents who play sports, especially baseball,basketball, racquetball, hockey, and boxing, and in athletes who do not wear protectiveeyewear. A number of injuries can occur following ocular trauma, including corneal abrasion,hyphema, globe rupture, foreign body, lens dislocation, retinal detachment, and orbital fracture. Itis critical for the sports physician to evaluate any ocular injury thoroughly as soon as it occurs.

Evaluation of ocular trauma includes an assessment of pain and visual acuity. The externalstructures, such as the orbital bones and eyelids, should be examined, as should the sclera,cornea, and pupils. Extraocular movements should be assessed carefully for fullness of motionand pain on movement. Examination of the cornea using fluorescein stain may be useful if acorneal abrasion is suspected. In general, unless results of a complete eye examination arewithin normal limits, removal from play and further evaluation are indicated. Findings that warrantimmediate emergency department referral include chemical injury, suspected injury to the globe,orbital fracture, hyphema, and intraocular foreign body.

The boy described in the vignette has pain on extraocular movement, which may besuggestive of injuries such as orbital fracture or retrobulbar hemorrhage. Therefore, immediatereferral to the emergency department is indicated. Fluorescein can be instilled to assess for aforeign body but would not be sufficient management for this patient. If a chemical injury issuspected, irrigation of the eye with normal saline is the initial management of choice. If a playerhas a suspected injury to the globe, a loose eye shield should be applied, but tight eye patchingcan put undue pressure on the globe and should be avoided.

References:

Atabaki SM. Pediatric head injury. Pediatr Rev. 2007;28:215-224. Available at:http://pedsinreview.aappublications.org/cgi/content/full/28/6/215

Bord SP, Linden J. Trauma to the globe and orbit. Emerg Med Clin North Am. 2008;26:97-123.Abstract available at: http://www.ncbi.nlm.nih.gov/pubmed/18249259

Luke A, Micheli L. Sports injuries: emergency assessment and field-side care. Pediatr Rev.1999;20:291-302. Available at: http://pedsinreview.aappublications.org/cgi/content/full/20/9/291

Copyright © 2010 by the American Academy of Pediatrics page 601

Page 602: AAP PREP 2010

2010 PREP SA on CD-ROM

Question: 244

A 3-year-old girl in whom Kawasaki disease was diagnosed is being discharged from the short-stay unit of the hospital. Her parents are surprised that her recommended therapy includesaspirin because of lay press reports to avoid aspirin in children younger than 16 years of age.You explain to them that there are some adverse effects to aspirin therapy for which theyshould be alert.

Of the following, the MOST likely adverse effect is

A. gastric toxicity with nausea

B. interstitial nephritis

C. liver toxicity

D. Reye syndrome

E. tinnitus

Copyright © 2010 by the American Academy of Pediatrics page 602

Page 603: AAP PREP 2010

2010 PREP SA on CD-ROM

Preferred Response: ACritique: 244

Salicylates normally are provided to children who have Kawasaki disease. The single mostlikely adverse effect of aspirin is direct irritation of the gastric mucosa. However, adult studiesreveal that patients using low-dose aspirin as prophylaxis for coronary vascular disease,thrombocythemia,, or polycythemia have little risk of a major gastrointestinal hemorrhage. Livertoxicity, interstitial nephritis, and Reye syndrome are theoretical possibilities with even low-to-moderate doses of aspirin. Tinnitus usually is a dose-related response to aspirin.

References:

Baumer JH, Love SJL, Gupta A, Haines LC, Maconochie I, Dua JS. Salicylate for the treatment ofKawasaki disease in children. Cochrane Database Syst Rev. 2006;4:CD004175. Available at:http://www.mrw.interscience.wiley.com/cochrane/clsysrev/articles/CD004175/frame.html

Danford D. Risk of atherosclerosis after Kawasaki disease. AAP Grand Rounds. 2007;18:64-65.Available at: http://aapgrandrounds.aappublications.org/cgi/content/full/18/6/64-a

Hayden M, Pignone M, Phillips C, Mulrow C. Aspirin for the primary prevention of cardiovascularevents: a summary of the evidence for the U.S. Preventive Services Task Force. Ann InternMed. 2002;136:161-172. Available at: http://www.annals.org/cgi/content/full/136/2/161

Hsieh K-S, Weng K-P, Lin C-C, Huang T-C, Lee C-L, Huang S-M. Treatment of acute Kawasakidisease: aspirin’s role in the febrile stage revisited. Pediatrics. 2004;114:e689-e693. Availableat: http://pediatrics.aappublications.org/cgi/content/full/114/6/e689

Squizzato A, Romualdi E, Middeldorp S. Antiplatelet drugs for polycythaemia vera and essentialthrombocythaemia. Cochrane Database Syst Rev. 2008;2:CD006503. Available at:http://www.mrw.interscience.wiley.com/cochrane/clsysrev/articles/CD006503/frame.html

Copyright © 2010 by the American Academy of Pediatrics page 603

Page 604: AAP PREP 2010

2010 PREP SA on CD-ROM

Question: 245

You are evaluating an 11-year-old girl who recently moved to your community. During the history-taking, you learn that her 42-year-old father has extremely high blood cholesterol concentrations(>500.0 mg/dL [12.9 mmol/L]). The girl’s mother states that her husband’s sister also has veryhigh cholesterol values. Findings on the girl’s physical examination are normal.

Of the following, the MOST appropriate next step is to

A. begin therapy with a lipid-lowering medication

B. institute a low-fat diet plan and follow up in 3 months

C. measure fasting cholesterol and triglycerides

D. obtain a blood sample for genetic testing

E. order baseline electrocardiography

Copyright © 2010 by the American Academy of Pediatrics page 604

Page 605: AAP PREP 2010

2010 PREP SA on CD-ROM

Preferred Response: CCritique: 245

Population approaches to lowering cholesterol concentrations in children and adolescentshave focused on encouraging the adoption of diets that are lower in saturated fat, total fat, andcholesterol. However, the young girl in the vignette has a family history that is alarming for apossible genetic disorder of cholesterol metabolism and requires an individualized approach tofacilitate early diagnosis and management. Children and adolescents who have family historiesof premature cardiovascular disease or at least one parent who has high blood cholesterolvalues should be considered for selective screening. Children who have family histories of anyof the genetic hyperlipidemias also should undergo early screening of cholesterol andtriglycerides.

Several genetic syndromes affect the normal metabolic processes of cholesterol-richlipoproteins and are characterized by early onset of frequently severe atherosclerotic changesin the coronary artery system. Among these syndromes are: familial hypercholesterolemia,familial combined hyperlipidemia, familial hypertriglyceridemia, familial dysbetalipoproteinemia, andfamilial decreased high-density lipoprotein. Although the enzyme or protein abnormality varies ineach of these rare entities, the end result is similar in that affected children are at high risk forthe development of coronary artery disease at a young age.

Initiating therapy with lipid-lowering medications is not appropriate for the girl in the vignetteuntil a diagnosis has been made. Similarly, genetic testing may not be of value as a first step.Electrocardiography does not have a diagnostic or prognostic role in the asymptomatic patient.Instituting a low-fat diet plan with 3-month follow-up does not aid in establishing this patient’sdiagnosis.

References:

Cohen MS. Fetal and childhood onset of adult cardiovascular diseases. Pediatr Clin North Am.2004;51:1697-1719. Abstract available at: http://www.ncbi.nlm.nih.gov/pubmed/15561181

Daniels SR, Greer FR and the Committee on Nutrition. Lipid screening and cardiovascular healthin childhood. Pediatrics. 2008;122:198-208.Available at:http://pediatrics.aappublications.org/cgi/content/full/122/1/198

Kwiterovich PO Jr. Recognition and management of dyslipidemia in children and adolescents. JClin Endocrinol Metab. 2008;93:4200-4209. Abstract available at:http://www.ncbi.nlm.nih.gov/pubmed/18697860

Kwiterovich PO. Disorders of lipid and lipoprotein metabolism. In: Rudolph CD, Rudolph AM,Hostetter MK, Lister G, Siegel NJ, eds. Rudolph’s Pediatrics. 21st ed. New York, NY: McGrawHill; 2003:693-711

Lambert M, Lupie P-J, Gagné C, et al. Treatment of Familial hypercholesterolemia in children andadolescents: effect of lovastatin. Pediatrics. 1996;97:619-628. Abstract available at:http://pediatrics.aappublications.org/cgi/content/abstract/97/5/619

Copyright © 2010 by the American Academy of Pediatrics page 605

Page 606: AAP PREP 2010

2010 PREP SA on CD-ROM

Lauer RM, Snetselaar L, Muhonen LF. Hyperlipidemia in children and adolescents. In: Moller JH,Hoffman JIE, eds. Pediatric Cardiovascular Medicine. Philadelphia, Pa: Churchill Livingstone;2000:793-803

Copyright © 2010 by the American Academy of Pediatrics page 606

Page 607: AAP PREP 2010

2010 PREP SA on CD-ROM

Question: 246

You are called to the nursery to evaluate a 12-hour-old infant for episodes of jerking. She hadbeen born following a term pregnancy. Vaginal birth was being attempted after a prior cesareansection. Fetal monitoring had shown an apparently reassuring heart rate and normal status priorto delivery. After replacing the monitor following transport from the labor room to the deliveryroom, the tracing indicated an abrupt decrease in heart rate. A stat cesarean section revealedthat the uterus had ruptured and the infant was out of the uterus and in the abdominal cavity.The baby required endotracheal intubation and chest compression but not epinephrine. Apgarscores were 1 at 1 minute, 1 at 5 minutes, and 5 at 10 minutes. You question the parents andnurse about any possible seizures.

Of the following, the description that MOST likely indicates that the child is having neonatalseizures is

A. episodes of bradycardia with apnea

B. fatiguing and vomiting during nursing

C. focal jerking in both arms simultaneously but asynchronously

D. limb jerking triggered by touching the child

E. spontaneous limb jerking that stops when a hand is placed on the child

Copyright © 2010 by the American Academy of Pediatrics page 607

Page 608: AAP PREP 2010

2010 PREP SA on CD-ROM

Preferred Response: CCritique: 246

The birth history described for the infant in the vignette places her at risk for neurologicinjury, and the staff caring for her should be particularly vigilant for the possibility of neonatalseizures. Such seizures can be difficult to diagnose because they differ from seizures in olderchildren. Neonatal seizures tend to be focal, rather than generalized, and may be clonic(rhythmic jerking) or tonic (sustained), but usually are not tonic-clonic. Focal clonic seizures mayinvolve more than one limb, but if jerking movements are bilateral, they are asynchronous.

Neonatal seizures cannot be provoked by stimulation, such as touching the child, orsuppressed by touching or restraining the limb. Most apneas and bradycardias are not seizures.Vomiting or fatiguing while nursing is not characteristic of neonatal seizures.

References:

Booth D, Evans DJ. Anticonvulsants for neonates with seizures. Cochrane Database Syst Rev.2004;3:CD004218. Available at:http://www.mrw.interscience.wiley.com/cochrane/clsysrev/articles/CD004218/frame.html

Rennie J, Boylan G. Treatment of neonatal seizures. Arch Dis Child Fetal Neonatal Ed.2007;92:F148-F150. Abstract available at: http://www.ncbi.nlm.nih.gov/pubmed/17337664

Silverstein FS, Jensen FE. Neonatal seizures. Ann Neurol. 2007;62:112-120. Abstract availableat: http://www.ncbi.nlm.nih.gov/pubmed/17683087

Zupanc ML. Neonatal seizures. Pediatr Clin North Am. 2004;51:961-978. Abstract available at:http://www.ncbi.nlm.nih.gov/pubmed/15275983

Copyright © 2010 by the American Academy of Pediatrics page 608

Page 609: AAP PREP 2010

2010 PREP SA on CD-ROM

Question: 247

You care for a 7-year-old boy who has moderate intellectual disability and autistic behavior.Molecular genetic testing has confirmed that he has findings consistent with classic fragile Xsyndrome. His pregnant mother has undergone prenatal testing, which revealed that she iscarrying a female fetus that also has fragile X syndrome.

Of the following, the MOST accurate statement regarding fragile X syndrome in females is that

A. their affected sons have more severe intellectual disability than their affected brothers

B. they can have normal intelligence

C. they do not exhibit autistic behaviors

D. they typically are affected as severely as males who have fragile X syndrome

E. they usually are infertile

Copyright © 2010 by the American Academy of Pediatrics page 609

Page 610: AAP PREP 2010

2010 PREP SA on CD-ROM

Preferred Response: BCritique: 247

Fragile X syndrome is an X-linked disorder of dysmorphic features and intellectual disability(usually moderate) with autistic features that is estimated to occur in 1 in 4,000 males. Fragile Xsyndrome is one of a group of disorders caused by progressive expansion, throughgenerations, of a trinucleotide repeat sequence (in this case, CGG) that leads to disruption ofgene function. Other conditions caused by trinucleotide repeat expansion include myotonicdystrophy and Huntington disease. The gene that is disrupted in fragile X syndrome is FMR1.

To predict the result of FMR1 disruption for an individual, it is important to know the averagenumber of CGG repeats in the general population as well as the phenotypes associated withvarious expansion sizes. Typically, humans have 5 to 40 CGG repeats in exon 1 of the FMR1gene. An "intermediate" allele size of approximately 41 to 58 repeats is not associated with anunusual phenotype and is of unclear significance; rarely, alleles of this size have been reportedto expand in subsequent generations. Allele sizes of approximately 59 to 200 repeats arereferred to as "premutation" alleles, and individuals who are premutation carriers usually havenormal intelligence but may have some features of fragile X syndrome; they also may be atincreased risk for fragile X-associated tremor/ataxia syndrome (FXTAS) or premature ovarianfailure. Allele sizes of more than 200 repeats are referred to as "full mutations." Once the allelereaches this size, it is methylated, which effectively silences FMR1 expression, resulting infragile X syndrome.

The clinical presentation of fragile X syndrome in males varies according to age. Prepubertalboys may be large for age (in both height and weight) and have relative macrocephaly. Theyoften exhibit hypotonia and may have gastroesophageal reflux and recurrent otitis media. Theyhave motor and speech delays and display behaviors such as hand-flapping and wrist-biting.They are almost always intellectually disabled, with abilities falling within the moderate range ofintellectual deficit. As they grow, their faces elongate, and the prominence of the forehead,ears, and jaw becomes more noticeable. After puberty, they have striking macroorchidism.

Females who have fragile X syndrome have highly variable presentations. Approximately50% of those who have a full fragile X mutation are intellectually disabled, with features andbehaviors similar to those of affected boys, although they typically are less severely affectedthan males who have equivalent mutations. The other 50% are intellectually normal. The leadingtheory as to the cause of intellectual variability in affected females is that the ratio of active"normal" Xs to active Xs with the FMR1 mutation in brain cells predicts intellectual ability; apredominance of active, "normal" Xs is associated with normal intellectual ability.

The term "anticipation" refers to the increasing severity of a disease as it is passed fromgeneration to generation. Disease anticipation occurs in fragile X syndrome, but it is not possibleto predict an expansion of the CGG sequence each time a conception takes place. Also,because the FMR1 gene is silenced when there are greater than 200 repeats, the person whohas 300 repeats is not expected to be more severely affected than the person who has 200repeats. Therefore, women who have full mutations are not expected to have children who aremore severely affected than their full-mutation brothers.

Approximately 20% of female premutation carriers have premature ovarian failure, butfemale full mutation carriers typically have normal fertility.

Copyright © 2010 by the American Academy of Pediatrics page 610

Page 611: AAP PREP 2010

2010 PREP SA on CD-ROM

As a result of reviewing this information, do you intend to make a change in practiceto provide better patient care?Yes No

References:

Saul RA, Tarleton JC. FMR-1-related disorders. GeneReviews. 2008. Available at:http://www.ncbi.nlm.nih.gov/bookshelf/br.fcgi?book=gene&part=fragilex

Stevenson RE, Schwartz CE, Schroer RJ. Fragile X syndrome. In: X-Linked Mental Retardation.New York, NY: Oxford University Press; 2000:179-182

Copyright © 2010 by the American Academy of Pediatrics page 611

Page 612: AAP PREP 2010

2010 PREP SA on CD-ROM

Question: 248

A 16-year-old sexually active girl presents with lower abdominal pain of 2 days’ duration. Shefinished her last menstrual period a few days ago and notes that it was heavier and morepainful than usual. On physical examination, she is afebrile, has normal vital signs, and exhibitsdiffuse lower abdominal tenderness with no rebound or guarding. Bimanual examination elicitspain on movement of her cervix and palpation of her adnexa, with no palpable masses.

Of the following, the MOST appropriate next step is to obtain a

A. complete blood count and erythrocyte sedimentation rate

B. Gram stain of any cervical discharge

C. pelvic ultrasound

D. test for Neisseria gonorrhoeae and Chlamydia trachomatis

E. urine and blood culture

Copyright © 2010 by the American Academy of Pediatrics page 612

Page 613: AAP PREP 2010

2010 PREP SA on CD-ROM

Preferred Response: DCritique: 248

Pelvic inflammatory disease (PID) is difficult to diagnose because of the spectrum ofsymptoms and signs. Many women have mild or nonspecific symptoms or signs. Laparoscopicexamination is the most precise method to make the diagnosis but is invasive and not indicatedfor most patients. In a woman complaining of lower abdominal pain, the minimum criterion to makethe diagnosis on pelvic examination is cervical motion tenderness, uterine tenderness, oradnexal tenderness. The girl described in the vignette meets the criterion for PID. Additionalcriteria used to support the diagnosis include an oral temperature greater than 38.4°C,mucopurulent cervical discharge, presence of numerous white blood cells on saline microscopyof vaginal secretions, an elevated erythrocyte sedimentation rate, elevated C-reactive proteinvalue, and laboratory documentation of cervical infection with Neisseria gonorrhoeae orChlamydia trachomatis.

To reduce damage to the reproductive tract, empiric treatment for PID should be initiated insexually active young women at risk for sexually transmitted infections if they are experiencingpelvic or lower abdominal pain, no cause of the illness other than PID can be identified, and theyhave the minimum criteria outlined previously.

The identification of N gonorrhoeae or C trachomatis from a cervical specimen wouldsupport the diagnosis of PID in the girl described in the vignette. Newer tests, employing nucleicacid amplification of DNA or RNA from these organisms, are more sensitive than cultures andare specific. In addition to cervical swab specimens, these tests can be performed on urine andvaginal swab specimens. Transcription-mediated amplification also can be used to test self-obtained vaginal swabs for C trachomatis and N gonorrhoeae.

Findings on complete blood count and erythrocyte sedimentation rate are nonspecific. Pelvicultrasonography generally is performed if the diagnosis of PID is in question or a complication issuspected. Ultrasonography may yield normal findings, particularly early in the disease process,or reveal thickened or fluid-filled fallopian tubes, with or without free fluid in the pouch ofDouglas. Pelvic ultrasonography also can identify a tubo-ovarian complex, suggesting pelvicinfection. Gram stain of cervical discharge to detect gonococcal infection provides nonspecificinformation because gram-negative diplococci may represent N vaginalis, part of the normalflora. Urine and blood cultures are only helpful to rule out other diagnoses.

References:

Burstein GR, Murray PJ. Diagnosis and management of sexually transmitted diseases amongadolescents. Pediatr Rev. 2003;24:119-127. Available at:http://pedsinreview.aappublications.org/cgi/content/full/24/4/119

Burstein GR, Murray PJ. Diagnosis and management of sexually transmitted disease pathogensamong adolescents. Pediatr Rev. 2003;24:75-82. Available at:http://pedsinreview.aappublications.org/cgi/content/full/24/3/75

Centers for Disease Control and Prevention, Workowski KA, Berman SM. Sexually transmitteddiseases treatment guidelines, 2006. MMWR Recomm Rep. 2006;55:(RR-11):1-94. Available at:

Copyright © 2010 by the American Academy of Pediatrics page 613

Page 614: AAP PREP 2010

2010 PREP SA on CD-ROM

http://www.cdc.gov/mmwr/preview/mmwrhtml/rr5511a1.htm

Hobbs MM, van der Pol B, Totten P, et al. From the NIH: proceedings of a workshop on theimportance of self-obtained vaginal specimens for detection of sexually transmitted infections.Sex Transm Dis. 2008;35:8-13. Abstract available at:http://www.ncbi.nlm.nih.gov/pubmed/18157061

Shrier LA. Bacterial sexually transmitted infections: gonorrhea, chlamydia, pelvic inflammatorydisease, and syphilis. In: Emans SJ, Laufer MR, Goldstein DP, eds. Pediatric and AdolescentGynecology. 5th ed. Philadelphia, Pa: Lippincott Williams & Wilkins; 2008:565-614

Shrier LA. Pelvic inflammatory disease. In: Neinstein LS, ed. Adolescent Health Care: A PracticalGuide. 5th ed. Philadelphia Pa: Lippincott Williams & Wilkins; 2008:819-824

Copyright © 2010 by the American Academy of Pediatrics page 614

Page 615: AAP PREP 2010

2010 PREP SA on CD-ROM

Question: 249

You are evaluating a 15-month-old boy for cough and fever of 5 days’ duration. His parentsreport that he is producing blood-tinged sputum with coughing. His family is new to the area.Review of his medical record demonstrates three previous episodes of right lower lobepneumonia. In response to your questions, his parents state that he tires easily. On physicalexamination, the boy’s weight is 8.5 kg, temperature is 39.0°C, heart rate is 140 beats/min,respiratory rate is 40 breaths/min, blood pressure is 88/55 mm Hg, and oxygen saturation is 92%by pulse oximetry on room air. He is in mild respiratory distress, and you can hear crackles in theupper right lung and decreased aeration of the right lower lung. Chest radiography demonstratesmultiple large cysts in the right lower lobe. Some of the cysts contain air-fluid levels (Item Q249).

Of the following, the MOST appropriate next step in the treatment in this patient is

A. administration of amoxicillin and follow-up in 2 weeks

B. aortography

C. computed tomography scan of the chest

D. referral for pH probe

E. thoracentesis

Copyright © 2010 by the American Academy of Pediatrics page 615

Page 616: AAP PREP 2010

2010 PREP SA on CD-ROM

Preferred Response: CCritique: 249

Congenital malformations of the lung usually present during the first 6 postnatal months, butsymptoms can be identical to more common pediatric conditions, such as asthma, viralinfections, or bacterial pneumonias. Congenital lung malformations include pulmonary hypoplasia(reduction in bronchiole number and alveoli), pulmonary agenesis (absence of vasculature andlung tissue), pulmonary aplasia (presence of rudimentary bronchus but absence of vasculatureand lung tissue), congenital cystic adenomatoid malformation (dysplastic lung tissue), congenitallobar emphysema (idiopathic hyperinflation of one or more lobes), and pulmonary sequestration(lung tissue that has a systemic arterial supply instead of pulmonary arterial supply andgenerally lacks bronchial communication).

The boy described in the vignette has a classic presentation of pulmonary sequestration,which accounts for 6% of all congenital lung malformations, occurs more commonly in the leftlower lobe, and has a male predominance. Sequestrations are classified as eitherintrapulmonary or extrapulmonary, according to their presence inside or outside the visceral(inner) pleura. Affected infants and children often present with chronic cough and recurrentrespiratory infections in the same anatomic location. Physical examination findings includedullness to percussion, with decreased aeration over the affected area. Crackles frequently areaudible during infection. Patients who have large systemic arterial-to-venous shunts maypresent with a systolic murmur and exercise intolerance. Radiographs demonstrateconsolidation of the affected area, but often show air-filled cystic lesions during activeinfections (Item C249).

Computed tomography scan is used to establish the diagnosis and plan surgical treatment, ifindicated. Aortography may be useful to determine vascular supply, but computed tomographywith angiography is preferable and is the initial step in the evaluation. Administration ofamoxicillin with future follow-up is neither curative nor appropriate. Thoracentesis is not helpfulin establishing the diagnosis. The child does not have symptoms consistent withgastroesophageal reflux and, therefore, a pH probe is not indicated.

References:

Finder JD, Michelson PH. Congenital disorders of the lung. In: Kliegman RM, Behrman RE, JensonHB, Stanton BF, eds. Nelson Textbook of Pediatrics. 18th ed. Philadelphia, Pa: SaundersElsevier; 2007:1783-1786

Scouten WT, White J. Visual diagnosis: an adolescent female who has persistent cough. PediatrRev. 2002;23:101-105.Available at: http://pedsinreview.aappublications.org/cgi/content/full/23/3/101

Copyright © 2010 by the American Academy of Pediatrics page 616

Page 617: AAP PREP 2010

2010 PREP SA on CD-ROM

Question: 250

You observe a child as he walks into the examination room, accompanied by his parents. He isholding a small ball. When you ask him to let you see the ball, he gives it to you. You hand the ballback to him, and he throws the ball to you. When you throw him the ball, he reaches to catch itbut almost loses his balance. He stoops to the floor but recovers to a standing position whilelaughing and smiling at his parents. You offer him a crayon and a piece of paper, and he drawsa straight line. When he sees a few blocks on the floor, he picks up two and stacks one on topof the other.

Of the following, these developmental milestones are MOST typical for a child whose age is

A. 9 months

B. 12 months

C. 15 months

D. 18 months

E. 24 months

Copyright © 2010 by the American Academy of Pediatrics page 617

Page 618: AAP PREP 2010

2010 PREP SA on CD-ROM

Preferred Response: CCritique: 250

Motor developmental milestones are obtained via parental history and from observationduring neurodevelopmental examination. To determine if a child has delays in motor milestones,the clinician must take into account history, standard neurodevelopmental milestones, and thepresence or absence of primitive reflexes and postural responses. A motor quotient (MQ) canbe determined by the following: MQ = motor age/chronological age x 100. A MQ of above 70 isnormal, between 50 and 70 is suspicious and requires monitoring, and below 50 is abnormal andrequires referral to a subspecialist.

The child described in the vignette shows the normal motor developmental milestones for a15-month-old, which consist of playing ball, giving and taking a toy, walking independently,stooping to the floor and recovering to a standing position, making a line with a crayon, andbuilding a two-cube tower. A 9-month-old child is able to pull to stand and cruise, walk with twohands held, and employ an immature pincer grasp. Most 12-month-old children are able to walkalone, use a mature pincer grasp, and release an object to another person on request. An 18-month-old child is able to run stiffly, walk up steps using a railing, make a tower of four cubes,and imitate scribbling and a vertical stroke. A 24-month-old child can run well, walk up and downsteps one step at a time, jump in place, make a tower of seven cubes, scribble in a circularpattern, and imitate a horizontal stroke.

References:

Blasco P. Motor delays. In Parker S, Zukerman B, Augustyn M, eds. Developmental andBehavioral Pediatrics: A Handbook for Primary Care. 2nd ed. Philadelphia, Pa: LippincottWilliams & Wilkins; 2005:242-247

Feigelman S. The first year. In: Kliegman RM, Behrman RE, Jenson HB, Stanton BF, eds. NelsonTextbook of Pediatrics. 18th ed. Philadelphia, Pa: Saunders Elsevier; 2007:43-47

Feigelman S. The second year. In: Kliegman RM, Behrman RE, Jenson HB, Stanton BF, eds.Nelson Textbook of Pediatrics. 18th ed. Philadelphia, Pa: Saunders Elsevier; 2007:48-53

Copyright © 2010 by the American Academy of Pediatrics page 618

Page 619: AAP PREP 2010

2010 PREP SA on CD-ROM

Question: 251

A 3-year-old boy presents to your office with a 3-day history of a severe sore throat,decreased oral intake (especially solid foods), and pain with swallowing. Nothing in his pastmedical history is noteworthy, and his immunizations are up to date. On physical examination,the boy is uncomfortable but alert and does not appear toxic. He is sitting upright, holding hisneck stiffly, and refusing to open his mouth. His temperature is 38.8°C. He does not haveappreciable lymphadenopathy, his lungs are clear, there is no heart murmur, and no abdominalorganomegaly is evident.

Of the following, the test MOST likely to confirm this child’s diagnosis is

A. cervical lymph node biopsy

B. computed tomography scan of the neck

C. laryngoscopic examination of the airway

D. lumbar puncture

E. sinus radiograph

Copyright © 2010 by the American Academy of Pediatrics page 619

Page 620: AAP PREP 2010

2010 PREP SA on CD-ROM

Preferred Response: BCritique: 251

The 3-day progression of sore throat, fever, dysphagia, and decreased oral intakedescribed for the boy in the vignette, combined with his unwillingness to move his neck, aresuggestive of a deep neck space infection such as a retropharyngeal abscess. Physicalexamination findings for this condition may be minimal; the diagnosis requires a high degree ofclinical suspicion. A lateral neck radiograph demonstrating thickening of the retropharyngealspace (Item C251A) confirms the diagnosis, but interpretation of such films depends onadequate positioning and inspiration. Computed tomography (CT) scan of the neck can confirmthe diagnosis and determine whether a walled-off abscess is present, especially for the childwho appears toxic or fails to respond to antimicrobial therapy (Item C251B).

In the absence of an appreciable fluctuant lymph node, a cervical lymph node biopsy wouldnot be helpful. Although sinus infections may present with some of the symptoms described in achild of this age, the dysphagia and stiff holding of the neck by the child are not suggestive ofacute sinusitis. Thus, a radiograph of this area would not aid in the diagnosis. Laryngoscopywith surgical drainage may be part of the management in some cases of retropharyngealabscess, but the initial approach is radiographic studies for diagnosis. Based on the degree ofsymptoms and clinical response, antibiotic therapy without surgical drainage may be sufficient.The stiff neck in an alert child is not synonymous with meningismus and without other featuresof meningitis does not necessitate a lumbar puncture, especially if the lateral radiographconfirms the diagnosis of retropharyngeal abscess. Some of the features in the clinicalpresentation of retropharyngeal abscess may be suggestive of epiglottitis. However, theprogression of epiglottitis is more rapid (hours), and the affected child appears more toxic andoften is drooling. Fortunately, in the era of Haemophilus influenzae type b vaccination,epiglottitis has become an extremely rare condition.

As with many upper respiratory tract bacterial infections, the peak incidence ofretropharyngeal abscesses is 2 to 5 years of age. Group A streptococci, Staphylococcusaureus, and oral anaerobes are the primary organisms identified when such abscesses aredrained. Therefore, antibiotic therapy with clindamycin or ampicillin-sulbactam is appropriate,although in areas that have high rates of community-acquired methicillin-resistant S aureus,ampicillin-sulbactam may not be optimal pending cultures. Complications of retropharyngealabscesses may include invasion of adjacent structures, airway obstruction, and sepsis.

References:

Craig FW, Schunk JE. Retropharyngeal abscess in children: clinical presentation, utility ofimaging, and current management. Pediatrics. 2003;111:1394-1398. Available at:http://pediatrics.aappublications.org/cgi/content/full/111/6/1394

Dudas R, Serwint JR. In brief: retropharyngeal abscess. Pediatr Rev. 2006;27:e45-e46.Available at: http://pedsinreview.aappublications.org/cgi/content/full/27/6/e45

Copyright © 2010 by the American Academy of Pediatrics page 620

Page 621: AAP PREP 2010

2010 PREP SA on CD-ROM

Question: 252

You are called by a nurse who has sustained a needle stick injury while drawing blood from apatient’s central line. The patient is a 14-year-old male who recently was diagnosed with acutemyelocytic leukemia. He has received several blood product transfusions for anemia andthrombocytopenia. You obtain human immunodeficiency virus and hepatitis C serologies.

Of the following, the additional serologic test that MUST be obtained for the nurse is for

A. cytomegalovirus

B. hepatitis A

C. hepatitis B

D. measles

E. tetanus

Copyright © 2010 by the American Academy of Pediatrics page 621

Page 622: AAP PREP 2010

2010 PREP SA on CD-ROM

Preferred Response: CCritique: 252

The nurse described in the vignette, who has sustained a needlestick, requires a serologictest for hepatitis B. Hepatitis B virus (HBV) is a DNA hepadnavirus that is comprised of an outerlipoprotein envelope containing hepatitis B surface antigen (HBsAg) and an inner nucleocapsidconsisting of hepatitis B core antigen. Only antibody to HBsAg (anti-Hbs) provides protectionfrom HBV infection. The virus is transmitted through blood or body fluids, including woundexudates, semen, cervical secretions, and saliva. Blood and serum contain the highestconcentrations of virus; saliva contains the lowest. The most common modes of transmissionare percutaneous and permucosal exposure to infectious body fluids, sharing or usingnonsterilized needles or syringes, unprotected sexual contact with an infected person, andperinatal exposure to an infected mother. Serology is the most common diagnostic test used todetect and diagnose hepatitis B. The different serologic diagnostic tests for HBV antigens andantibodies are shown in Item C252. In addition, hybridization assays and gene amplificationtechniques (eg, polymerase chain reaction) are available to detect and quantitate HBV DNA.

Cytomegalovirus, hepatitis A, and measles are not transmitted parenterally. Tentanus isacquired through contamination of wounds with soil or excrement that contain the tetanusorganism. Needlestick injury is not a usual pathway for acquiring tetanus.

References:

American Academy of Pediatrics. Hepatitis B. In: Pickering LK, Baker CJ, Kimberlin DW, Long SS,eds. Red Book: 2009 Report of the Committee on Infectious Diseases. 28th ed. Elk GroveVillage, Ill: American Academy of Pediatrics; 2009:337-356

Curry MP, Chopra S. Acute viral hepatitis. In: Mandell GL, Bennett JE, Dolan R, eds. Mandell,Douglas, and Bennett’s Principles and Practice of Infectious Diseases. 6th ed. Philadelphia, Pa:Elsevier Churchill Livingstone; 2005:1426-1441

Hoofnagle JH, Di Bisceglie AM. Serologic diagnosis of acute and chronic viral hepatitis. SeminLiver Dis. 1991;11:73-83

Copyright © 2010 by the American Academy of Pediatrics page 622

Page 623: AAP PREP 2010

2010 PREP SA on CD-ROM

Question: 253

A 6-year-old girl presents with complaints of persistent daytime and nighttime wetting. She hasno dysuria, frequency, urgency, polyuria, polydipsia, abdominal pain, or constipation. Accordingto her mother, the girl has shown signs of toilet training since 3 years of age, using the bathroomwhenever she felt the urge to urinate. Despite these behaviors, however, her mother states thather daughter is "always wet." The child is doing well in school and has no physical limitations.She has had no urinary tract infections. Findings on her physical examination are normal.Urinalysis reveals a urine specific gravity of 1.025; pH of 6; and negative blood, protein,leukocyte esterase, nitrite, and microscopy findings. A urine culture is negative.

Of the following, the MOST likely explanation for this child’s clinical condition is

A. ectopic ureter

B. neurogenic bladder

C. pollakiuria

D. ureterocele

E. viral cystitis

Copyright © 2010 by the American Academy of Pediatrics page 623

Page 624: AAP PREP 2010

2010 PREP SA on CD-ROM

Preferred Response: ACritique: 253

The history described for the child in the vignette strongly suggests the diagnosis of ectopicureter. The normal ureteral insertion site is into the bladder trigone. When insertion occurs caudalto this site, it is defined as ectopic. Ectopic ureters can occur in both males and females, butthey are six times more common in females. The true incidence is unknown, but autopsy studieshave demonstrated their presence in 1 in 1,900. Ectopic ureters often occur in association witha duplicated collecting system. Typically, the ectopic ureter transports urine formed from theupper pole of a duplex collecting system. The renal unit associated with the ectopic uretergenerally is small and dysplastic, making it somewhat difficult to detect with routineultrasonography. Ectopic ureters are bilateral in 10% of cases.

The sites of ectopic ureters in females include the bladder neck, urethra, and vagina; inmales, the locations include the posterior urethra (50%), seminal vesicle, and bladder neck.Because the orifice for insertion of ectopic ureters in males is located proximal to the externalbladder sphincter, males do not present with incontinence.

Diagnosing ectopic ureter can be difficult and frequently delayed when the child ismisdiagnosed as having voiding dysfunction. A history of constant wetness in an otherwisehealthy female who is toilet trained should alert the clinician to this diagnosis. The diagnosticevaluation can be challenging. In one study, ultrasonography was diagnostic in only 2 of 12cases. Furthermore, the ultrasonographic finding in most children ultimately diagnosed withectopic ureters is either a duplex collecting system (on one or both sides) or normal. A normalultrasonography result could falsely reassure the clinician and family. Ultrasonography isnoninvasive and helpful in identifying individuals "at risk" for an ectopic ureter by demonstratinga duplex collecting system. However, the clinician should pursue further diagnostic studiesaggressively when ectopic ureter is suspected.

Until recently, the diagnostic study of choice was excretory urography. The currentrecommendations by most radiologists and urologists is magnetic resonance urography (MRU) todemonstrate the small, dysplastic renal unit and its associated ectopic ureter and the location ofits ureteral orifice. Imaging with MRU avoids the ionizing radiation associated with excretoryurography or computed tomography scan. Voiding cystourethrography has no role in thediagnostic evaluation of this disorder.

Neurogenic bladder usually is associated with overflow incontinence and recurrent urinarytract infections and typically is seen in children who have spina bifida, spinal cord injury, ortethered cord. Pollakiuria is a condition of extraordinary urinary frequency that occurs after toilettraining, is short-lived (weeks to months), and frequently is associated with psychosocialstress. A ureterocele (Item C253), like ectopic ureter, often is seen with a duplex collectingsystem, but the typical presentation is urinary tract infections in the first postnatal year due tourinary stasis. Viral cystitis is a short-lived condition that is associated with symptoms ofdysuria, frequency, and urgency and findings of pyuria on urinalysis but negative bacterialcultures.

References:

Baskin L. Ureteropelvic junction obstruction, congenital megaureter, ureterocele, and ectopic

Copyright © 2010 by the American Academy of Pediatrics page 624

Page 625: AAP PREP 2010

2010 PREP SA on CD-ROM

ureter. UpToDate Online 16.3. 2008. Available for subscription at:http://www.utdol.com/online/content/topic.do?topicKey=pediatri/6078&selectedTitle=1~20&source=search_result630

Carrico C, Lebowitz RL. Incontinence due to an infrasphincteric ectopic ureter: why the delay indiagnosis and what the radiologist can do about it. Pediatr Radiol. 1998;28:942-949. Abstractavailable at: http://www.ncbi.nlm.nih.gov/pubmed/9880638

Krishnan A, Baskin LS. Identification of ectopic ureter in incontinent girl using magneticresonance imaging. Urology. 2005;65:1002. Abstract available at:http://www.ncbi.nlm.nih.gov/pubmed/15882752

Copyright © 2010 by the American Academy of Pediatrics page 625

Page 626: AAP PREP 2010

2010 PREP SA on CD-ROM

Question: 254

You have been asked by a local school to provide recommendations about the use of self-injectable epinephrine for anaphylaxis. The school supervisor is concerned about the increasedincidence of peanut and tree nut food allergy. School officials have requested that each childwho has a diagnosis of "food allergy" have two self-injectable epinephrine devices at the schoolnurse’s office.

Of the following, the BEST response regarding anaphylaxis is that

A. a patient should not receive a second dose of epinephrine unless a clinician is present

B. epinephrine reaches higher peak plasma concentrations if injected into the thigh rather thanthe arm

C. families should keep one epinephrine autoinjector in the car in case a reaction occurs afterschool

D. skin manifestations (eg, flushing, itching, urticaria) are rare in severe anaphylaxis

E. subcutaneous injection of epinephrine is preferable to intramuscular injection

Copyright © 2010 by the American Academy of Pediatrics page 626

Page 627: AAP PREP 2010

2010 PREP SA on CD-ROM

Preferred Response: BCritique: 254

The prevalence of food allergies has continued to increase over the past 3 to 4 decades.Specifically, many children, parents, and school officials have been faced with the need toknow about and understand how to recognize and appropriately treat food anaphylaxis in theschool. Education and counseling of school officials and health-care clinicians is paramount toreduce morbidity and mortality from food anaphylaxis.

The most common antigenic triggers of anaphylaxis are foods, drugs, insect venom,radiocontrast media, and latex. After exposure to an antigenic trigger, symptoms generallydevelop within 5 to 30 minutes, although symptoms can occur up to several hours after theexposure. Severe allergic reactions usually occur after binding of specific immunoglobulin (Ig) Eto the high-affinity IgE receptor, with subsequent cross-linking of receptors and mediator release(eg, histamine, tryptase) from mast cells and basophils.

Cutaneous manifestations such as urticaria, flushing, pruritus, and angioedema are the mostcommon symptoms in anaphylaxis, occurring in 80% to 90% of episodes. Respiratory symptomssuch as dyspnea, wheezing, shortness of breath, and cough are the next most frequentsymptoms. Cardiovascular symptoms include cardiovascular collapse, tachycardia or relativebradycardia, and arrhythmias. Among the gastrointestinal manifestations are nausea, vomiting,diarrhea, abdominal pain, and cramping. Finally, many patients complain of either a metallic tasteor "a sense of impending doom."

Appropriate treatment of anaphylaxis consists of early administration of epinephrine.Because anaphylaxis can occur in the absence of a health-care professional such as at school,home, or a birthday party, children at risk always should have self-injectable epinephrinenearby. Although parents or other adults may be reluctant to inject a child with epinephrine, thisagent, not an antihistamine, is the drug of choice for anaphylaxis. In the past, outpatientadministration of epinephrine was subcutaneous, but research has demonstrated thatintramuscular injection, specifically in the thigh, is the preferred route and location due to higherand faster peak plasma concentration. If epinephrine is administered, parents or schoolpersonnel should follow an emergency action plan. This should involve calling emergencyservices to evaluate the child and transport him or her to the emergency department for furtherevaluation. The effects of a single dose of epinephrine typically last for 5 to 15 minutes; up to20% of individuals experiencing anaphylaxis may require a second epinephrine dose. Whensymptoms persist, a second (or third) dose should be administered, even if the parent or schoolprofessional still is awaiting the ambulance. Although epinephrine always is the drug of choice inanaphylaxis, glucagon may be required in refractory cases for patients using beta blockers.

Self-injectable epinephrine should be available for all locations (ie, the patient usually carriesone to two injectors), but leaving the device in the car is not recommended because extremetemperature changes can decrease the efficacy. Recommended storage temperatures are 20°to 25°C at home and 15 to 30°C during trips outside the home, school, or workplace.

Approximately 5% to 20% of patients who suffer initial anaphylactic events can experiencea "late-phase" response 4 to 24 hours later in which symptoms such as flushing, pruritus, orairway obstruction recur. Such later symptoms result from the recruitment of inflammatory cellsafter the initial hypersensitivity response.

Copyright © 2010 by the American Academy of Pediatrics page 627

Page 628: AAP PREP 2010

2010 PREP SA on CD-ROM

References:

Joint Task Force on Practice Parameters; American Academy of Allergy, Asthma andImmunology; American College of Allergy, Asthma and Immunology; Joint Council of Allergy,Asthma and Immunology. The diagnosis and management of anaphylaxis: an updated practiceparameter. J Allergy Clin Immunol. 2005;115(3 suppl 2):S483-S523

Sampson HA, Leung DYM. Anaphylaxis. In: Kliegman RM, Behrman RE, Jenson HB, Stanton BF,eds. Nelson Textbook of Pediatrics. 18th ed. Philadelphia, Pa: Saunders Elsevier; 2007:983-984

Sampson HA, Mu–oz-Furlong A, Campbell RL, et al. Second symposium on the definition andmanagement of anaphylaxis: summary report—Second National Institute of Allergy andInfectious Disease/Food Allergy and Anaphylaxis Network symposium. J Allergy Clin Immunol.2006;117:391-397. Abstract available at: http://www.ncbi.nlm.nih.gov/pubmed/16461139

Waibel KH. Anaphylaxis. Pediatr Rev. 2008;29:255-263. Available at:http://pedsinreview.aappublications.org/cgi/content/full/29/8/255

Copyright © 2010 by the American Academy of Pediatrics page 628

Page 629: AAP PREP 2010

2010 PREP SA on CD-ROM

Question: 255

A 2-month-old infant is brought to the office because of fussiness, increased sleeping, and poorfeeding. According to her mother, she was doing well until 4 days ago, when her formula intakedecreased from 6 oz every 4 hours to 2 oz every 6 hours and she has had to be awakened forher feedings. She has had no vomiting, diarrhea, or fever. On physical examination, she isdifficult to console, her temperature is 36.8°C, her heart rate is 160 beats/min, and herrespiratory rate is 30 breaths/min. Her anterior fontanelle is full, pupils are 4 mm and equallyreactive, and there is no evidence of corneal abrasions. The remainder of the physicalexamination findings are unremarkable. Results of a complete blood count with differential count,electrolytes, and urinalysis are normal. You obtain computed tomography (CT) scan of the brain(Item Q255).

Of the following, the MOST likely cause of the CT findings is

A. arteriovenous malformation

B. galactosemia

C. meningoencephalitis

D. nonaccidental trauma

E. von Willebrand disease

Copyright © 2010 by the American Academy of Pediatrics page 629

Page 630: AAP PREP 2010

2010 PREP SA on CD-ROM

Preferred Response: DCritique: 255

The computed tomography (CT) scan described for the patient in the vignette demonstratesa subdural hematoma, a collection of blood between the brain and the dura caused by shearingof the veins between the dura and the pia arachnoid (Item C255A). Injury to these veins resultsfrom high-velocity rotational forces, as might occur in a motor vehicle crash or from violentshaking. In a pediatric patient who has no history of a significant traumatic mechanism, such asthis infant, the presence of a subdural hematoma is highly suggestive of nonaccidental injury.

The American Academy of Pediatrics policy on evaluation of suspected child abuse andneglect recommends that when intentional injury is suspected, especially if one seriousintentional injury already is identified, further evaluation should be performed to detect occultinjuries. For an infant who has a subdural hematoma, a skeletal survey always is indicatedbecause rib (Item C255B) and metaphyseal (Item C255C) fractures may not be clinically apparentand previous healing fractures may have escaped medical attention. Examination by anophthalmologist for the presence of retinal hemorrhages should be considered. A platelet countand prothrombin and partial thromboplastin times should be obtained to screen for congenital oracquired coagulopathies. Liver and pancreatic enzymes, urinalysis, stool guaiac test, and a CTscan of the abdomen may be considered if there is a concern for abdominal injury. Urine fromany infant who has an intracranial hematoma should be analyzed for organic acids becauseglutaric aciduria can predispose to spontaneous intracranial bleeding.

Because 95% of serious brain injuries in the first year after birth are caused bynonaccidental trauma, it is unlikely that an arteriovenous malformation caused the hematomaseen for this infant. The initial CT scan may be helpful in eliminating this possibility, and magneticresonance imaging and magnetic resonance angiography/magnetic resonance venography canbe performed if there is ongoing clinical concern. Galactosemia is not a recognized metaboliccause of intracranial bleeding. Brain inflammation from meningoencephalitis may lead to blood inthe cerebrospinal fluid but is not a cause of subdural hematoma. von Willebrand disease, adeficiency of von Willebrand factor that results in platelet dysfunction, typically causes mucosalbleeding and bruising, not deep internal bleeding.

As a result of reviewing this information, do you intend to make a change in practiceto provide better patient care?Yes No

References:

Johnson CF. Abuse and neglect of children. In: Kliegman RM, Behrman RE, Jenson HB, StantonBF, eds. Nelson Textbook of Pediatrics. 18th ed. Philadelphia, Pa: Saunders Elsevier; 2007:171-183

Kellogg ND and the Committee on Child Abuse and Neglect. Evaluation of suspected child abuse.Pediatrics. 2007,119;1232-1241. Available at:http://pediatrics.aappublications.org/cgi/content/full/119/6/1232

Copyright © 2010 by the American Academy of Pediatrics page 630